Download as pdf or txt
Download as pdf or txt
You are on page 1of 237

lOMoARcPSD|39315497

SSC CPO Warrior BOOK FREE

Post Graduation (Patliputra University )

Scan to open on Studocu

Studocu is not sponsored or endorsed by any college or university


Downloaded by Kunal Chauhan (kc6970929@gmail.com)
lOMoARcPSD|39315497

Downloaded by Kunal Chauhan (kc6970929@gmail.com)


lOMoARcPSD|39315497

Downloaded by Kunal Chauhan (kc6970929@gmail.com)


lOMoARcPSD|39315497

Downloaded by Kunal Chauhan (kc6970929@gmail.com)


lOMoARcPSD|39315497

Copyright
I understand that the book is Proprietary &
Copyrighted Material of Rankers Gurukul Publications.
Any reproduction in any form, physical or electronic
mode on public forum etc will lead to infringement of
copyright of Rankers Gurukul Publications and will
attract penal actions including FIR and claim of
damages under Indian Copyright Act 1957.

Price ` 260/-
 Author - Aditya Ranjan
 Published by - Aditya Ranjan

For Distributionship:
Contact us at:-
 E-mail - adityaranjanmathshelp@gmail.com
 Whatsapp - 8130591035

Downloaded by Kunal Chauhan (kc6970929@gmail.com)


lOMoARcPSD|39315497

About Aditya Ranjan


Aditya Ranjan Sir is a renowned Maths Faculty , who has taught lakhs of students
through Rankers Gurukul You Tube channel and its offline centre. His free maths
& guidance videos have millions of views on you tube. He has a unique approach
of solving new TCS questions . He always focuses on relevant and updated content.
His new innovative idea of completing entire maths syllabus for any govt. exam on
you tube through "60 Days 60 Marathon" is a landmark in online education.
He has got selected at his very early age and his words "SELECTED SELECTION
" are very popular among students.

EXAMS QUALIFIED:
CGL , CHSL, CPO , CDS (3 TIMES)
** HE SCORED **
 MATHS – 50/50
 ENGLISH – 50/50
 REASONING – 50/50
and got selected in SSC CHSL 2019 with AIR - 114.
He scored the same in CGL 2019 PRE and 227/200 in MAINS (MATHS) and got
selected in SSC CGL 2019 as an EXCISE INSPECTOR.

CONTACT US
 YOU TUBE – RANKERS GURUKUL
 TELEGRAM– Maths By Aditya Ranjan
 Instagram – aditya_ _ _ranjan
 Facebook – Maths By Aditya Ranjan

 Mail – Mathsbyadityaranjan@gmail.com

Downloaded by Kunal Chauhan (kc6970929@gmail.com)


lOMoARcPSD|39315497

Dedicated to my dear
“ASPIRANTS” & team
“RANKERS' GURUKUL”
Who have inspired me
to write this book.

ACKNOWLEDGMENT

" #$ % &’( )
& * + Content
+ Team
& ,
Co-ordinator Rudra Sir & Mukesh Sir
&-* . /
+ + % $ % ! % %
Proofread Error Free & *
2 3 $ & DTP
3
Operator % +
4& 5 6 7 *
& . 9

Downloaded by Kunal Chauhan (kc6970929@gmail.com)


lOMoARcPSD|39315497

1.
1. Percentage 01 – 13

1.
2. Profit & Loss 14 – 19

1.
3. Discount 20 – 27

1.
4. Simple Interest 28 – 30

1.
5. Compound Interest 31 – 34

1.
5. Ratio & Proportion 35 – 39

1.
5. Partnership 40 – 44

1.
5. Average 45 – 45 – 50

1.
9. Time & Work 51 – 60

1.
10. Pipe & Cistern 61 – 67

1.
11. Time & Distance 68 – 76

1.
12. Train 77 – 80

Downloaded by Kunal Chauhan (kc6970929@gmail.com)


lOMoARcPSD|39315497

1.
13. Boat & Stream 81 – 82

1.
14. Number System 83 – 90

1.
15. LCM & HCF 91 – 97

1.
16. Simplification 98 – 108

1.
17. Algebra 109 – 119

1.
18. Trigonometry01-14 120 – 130

1.
19. Height & Distance 131 – 137

1.
20. Geometry 138 – 154

1.
21. Mensuration-2D 155 – 165

1.
22. Mensuration-3D 166 – 177

1.
23. Data Interpretation 178 – 226

Downloaded by Kunal Chauhan (kc6970929@gmail.com)


lOMoARcPSD|39315497

Percentage

PERCENTAGE
01

1. The ratio of the number of boys 4. Bhavani spent 15% of his monthly 7. The salaries of A, B and C are in
and girls in a school is 3 : 4 income on kid's education and ratio 2 : 3 : 4. If increments of
respectively. If the number of 20% on food. 40% of the 30%, 20% and 10% are allowed,
boys is increased by 10% and the remaining she spent on sports respectively, in their salaries
number of girls is increased by and 30% on transport. She is left then what will be the new ratio
15%, then what will be the new with an amount of `10,257 after of their salaries?
ratio of the number of boys to the all these expenditures. What is
number of girls? Bhavani's monthly income? A, B C $ ,
C $
8 $ !(# -A.

r
"#/ ,"# !"# $% &
B ,"# 8 4 .7
C $ )

si
!"# $% & ;’ "# C
!(# "$#% & $ .7 6 SSC 8 CPO 09/11/2022
. D (Shift-03)
an by )
-
* + ; -.
C
8 $
! " / , ( 0 E F (a) 22 : 11 : 18 (b) 18 : 13 : 22
) +
(c) 13 : 18 : 22 (d) 22 : 18 : 13

n
SSC CPO 09/11/2022 (Shift-01) SSC CPO 09/11/2022 (Shift-02) 8. In an election between two
33 35 (a) `45,800 (b) `48,000 candidates, 65% of the voters
(a) ja (b)
cast their votes, out of which 3%
R s
45 46 (c) `52,600 (d) `81,965
of the votes were decided to be
33 46 5. The monthly income of Ramesh invalid. A candidate got 81965
a th

(c) (d)
46 33 was ` 16,500 and his monthly votes which are 65% of the total
2. In an election between 2 parties expenditure was ` 10,500. Next valid votes. What is the total
year his income increased by 25% number of votes enrolled in that
A and B, A gets 37% of total votes
and his expenditure increased by
election?
ty a

casted and thus lost by 338 19%. Find the percentage increase
votes. The total number of casted in his saving. . $ CK $
votes is (assume none of the
di M

! G / ( " " E F. ’ $ / G(#


votes was declared invalid):
C .7 !"/("" EF ’ / *4 # $ 9
, A B - . . $ / * C K $ 3
$; 7 C ,(# $% & 67
A * 0
C # 1= 2 !?# $% & 6 74 C $ 9 G
3 -. 1 $% & H . 4 $
4 * SSC CPO 09/11/2022 (Shift-02)
+
) + 5 6 7 (a)
8 33.5% $ 9 (b) 34.5% SSC CPO 09/11/2022 (Shift-03)
: ; * ’ < (a) 200000 (b) 190000
(c) 35.8% (d) 35.5%
SSC CPO 09/11/2022 (Shift-01) (c) 215000 (d) 185000
6. The population of a town is
(a) 1300 (b) 1500 1,25,000. If the annual birth rate 9. After deducting 20% from a
(c) 1200 (d) 1800 is 12.7% and the annual death certain sum, and then 10% from
A

3. The number of persons rose by rate is 2.7%, how much will the the remainder, there is ` 7,200
20% to 30,942 in a year. What population increase after 3 years?
left. Find the original sum.
was the population originally a
4 !/,(/"""
year ago? . ,"#
$ ; 7 4I !,F0# $ ; 7
$; 7 = ) ,"# ->
%J ,F0# / $; D; - 4 !"# M
"/? , *67 $; 7 @ 0 , " " E F ;
$% & * +
4 ’ +
SSC CPO 09/11/2022 (Shift-03) SSC CPO 09/11/2022 (Shift-03)
SSC CPO 09/11/2022 (Shift-02)
(a) 43,285 (b) 41,375 (a) ` 9,000 (b) `10,000
(a) 25585 (b) 25785
(c) 25758 (d) 23785 (c) 42,565 (d) 40,275 (c) ` 7,500 (d) ` 7,800

Aditya Ranjan (Excise Inspector) Selected Selection 1

Downloaded by Kunal Chauhan (kc6970929@gmail.com)


lOMoARcPSD|39315497

Percentage

10. In a two-candidate election, 10% AB CJ $ ; 74 - / 4


of the voters did not cast their !"# $% & 67/ * $; 7 4 ,,# ,(#
ballots, 10% of the votes casted
were found invalid. The winning $% & 67 $; 7 6 7 6/ (# 4
candidate received 54% of the $% & 67 CJ 6$7;/ CD C J)
valid votes and a 1620-vote ) *1 $ 7 5# $<% & 6+7 +
majority. Find the number of SSC CPO 10/11/2022 (Shift-02) SSC CPO 11/11/2022 (Shift-01)
people on the voter list who have
(a) 41.5% (b) 40.9% (a) 51.72% (b) 31.67%
registered to vote.
(c) 41.2% (d) 42.8% (c) 22.43% (d) 41.26%
CK $ . $14. The/ ! salary
"# of Ramesh first 18. Due to pandemic, the population
* increase by 35%, then decreases of city reduces at the rate of 2%
!"# $ 9 * $ 4 by 40%,
C K then$ again increases by per annum. What will be its total
$ 9 ( # 1 225%. Find the overall increase or population after 2 years if the
decrease percentage. present population of the city is
$ !G," $4
45,000?
. C * H 4$ (# $% &
/ L "# / L N
4I 4 N
,(# $% & , # $ %1 & $ ; 7 : M

r
1 H 4 $ 7 4 (/"""
SSC CPO 09/11/2022 (Shift-03)
- 6 4 )
SSC CPO 10/11/2022 (Shift-03)

si
(a) 25000 (b) 26000
SSC CPO 11/11/2022 (Shift-01)
(c) 24500 (d) 25500 (a) Increase 1.25%
(b) Decrease 2.25% (a) 40538 (b) 44521
an by
11. If Ram's income exceeds Shyam's
income by 70% and Shyam's
income is less than Sohan's
(c) Decrease 1.25%
(d) Increase 2.25%
(c) 41568 (d) 43218
19. The price of a car is first

n
income by 30%, then find the ratio 15. A man spends 65% of his income. increased by 35% and after that
of the incomes of Ram and Sohan. His income increases by 20% and the price is decreased by 25% due
ja his0expenditure
"# also increases by to reduction in sales. What is the
R s
10%. The percentage of increase net percentage change in final
O price of the car?
in his savings is_______.
a th

"# / (#
= ) G(# .7
H 4
C ,"# $ %$ &% & 4 C
SSC CPO 10/11/2022 (Shift-01) N
C .7 8 !"# $% &
(a) 119 : 100 (b) 121 : 102
ty a

C -. 1 4
(c) 120 : 101 (d) 118 : 99 & 1 $ 7I
$% & 67+
12. In an election, a candidate
di M

SSC CPO 11/11/2022 (Shift-01) SSC CPO 11/11/2022 (Shift-02)


secures 42% of the votes polled
but is defeated by his only (a) 38.5% (b) 35.8% 1
opponent by a majority of 3080 (c) 10% (d) 20% (a) Decrease by 2 %
5
votes, because of 1400 invalid 16. Due to fall in manpower, the
votes. The percentage of invalid production in a company decrease 1
votes is________. by 35%. By what percentage (b) Increase by 2 %
3
. $ / CK $ should the working hours be
increased to restore the original
* ,# 1 2 / 3
production? (c) Decrease by 1 %
! "" $ 9 N / 5
4 ) 67 N /
P 1 Q "3" C -J (#
4 $ 9 1 C J F F F F F -F 1
(d) Increase by 1 %
FFFFFFFFFFF 4
: M 1 ->
A

SSC CPO 10/11/2022 (Shift-01) 20. The marked price of a chair was
4 . +
` 1,000 but it was sold for ` 1,200.
(a) 5% (b) 3% SSC CPO 11/11/2022 (Shift-01) What is the percentage increase
(c) 4% (d) 6% (a) 58.35% (b) 55.83% on the product?
13. The production of AB company
(c) 53.85% (d) 55.38% 7 R EF
increased by 10% after one year,
and in the next year it increased 17. The income of Raju is 25% more 6 EF!/,"" - .
by 22%. In the third year, it than that of Manju. Raju got a 1 $% & ) +
increased by 5%. What overall 25% rise in his income, and
SSC CPO 11/11/2022 (Shift-02)
change (in %) has been there in Manju got a 40% rise is his
income, the percentage increase (a) 10% (b) 15%
the production in three years from
the original production? in their combined income is: (c) 35% (d) 20%

Aditya Ranjan (Excise Inspector) Selected Selection 2

Downloaded by Kunal Chauhan (kc6970929@gmail.com)


lOMoARcPSD|39315497

Percentage

21. In an assembly election, a (a) 1200 (b) 1000 9 7


candidate got 60% of the total (c) 800 (d) 900 5*% < 8 *
valid votes. 2% of the total votes 25. If each side of a rectangle is
were declared invalid. If the total !,# S M 9
decreased by 11%, then its area
number of voters is 1,50,000, then M ) 8 *
will decrease by:
find the number of valid votes = ,/!"" E S
polled in favor of that candidate. 1J 8 4 !!#
M ) 5E <
$9 8 . $ / CK $ / 6 B P LR 67
SSC CPO 23/11/2020 (Shift-2)
$ 9 G"# H1 2 (a) 25,000 (b) 17,500
,# $ 9 : SSC
; CPO 23/11/2020 (Shift-1) (c) 21,000 (d) 18,000
* (a) 24.31% (b) 25% 30. Rice is now being sold at Rs.29
!/("/""" C CK $ (c) 21.13%B (d) 20.79% per kg. During the last month, its
cost was Rs.25 per kg. By how much
* $ 9 26. HThe price 4
of diesel increased by
percentage should a family reduce
SSC CPO 11/11/2022 (Shift-02) 16%. A person wants to increase
its consumption, so as to keep the
his expenditure on diesel by 10%
(a) 90,000 (b) 78,000 expenditure the same as before?
only. By what percentage. correct
(c) 86,400 (d) 88,200 to one decimal place, should he $ 7 4 . $ ,? E
22. The salary of an employee was reduce his consumption? - . 4 / S

r
first increased by 15% and 4 !G# $% & R , ( E F 1 * 1 ’
thereafter it was reduced by 10%.

si
What was the percentage change
= ) 4 = $. $
in his salary? !"# $% & . C * / =

$% &
7.
an by*67
$
C
!(#
- ’ 6 !"# &<
1 5 $
SSC
* + CPO
+ 5 M
23/11/2020
N 7
(Shift-2)

n
(a) 15% (b) 12%
* C $ SSC CPO 23/11/2020 (Shift-1)
(c) 13% (d) 14%
1 $ 7 + (a) 6.5% (b) 5.2% 31. The monthly salary of a person was
ja (c) 4.5% (d) 3.7%
R s
SSC CPO 11/11/2022 (Shift-03) Rs.50,000, He used to spend on
(a) 1.10% (b) 103.5% 27. If decreasing 110 by x% gives the Family expenses (E), Taxes (T),
a th

(c) 3.5% (d) 15% same result as increasing 50 by x%, Charity (C), and the rest were his
then x% of 650 is what percentage savings. E was 60% of the income,
23. In an election between two
more than (x + 20)% of 180? T was 20% of E, and C was 15% of T.
candidates, 85% of the voters
(correct to nearest integer) When his salary got raised by 40%,
cast their votes, out which 4% of
ty a

the votes were declared invalid. 110 x% 1 2he maintained the percentage level
A candidate got 6936 votes which of E, but T becomes 30% of E and C
/ x% - > N 50 1 2 becomesN 20% of T. The difference
were 85% of the valid votes. Find
di M

650
/ x%, 180 (x between the two savings (in Rs.) is:
the total number of voters
enrolled in that election. + 20)% 1 9 * =+ ) $ ("/
CK $ - . . 5 $ M / 3N (# 7 &< 4 $ (E),
$ (T), . 7
$ M / CPO
SSC 4 23/11/2022 (Shift-1) (C) =
# $ 9 : ; (a)* 80% (b) 90% -. E 60%, T, E
CK $ G? G 4
(c) 136% $ 9 (d) 154% 20% C. T 15% 4- C
3(# ’ C . 28.$ If A’s salary is 60% more than B’s 40% $% & E67/ C
H salary,4 then by what percentage 1 $7$ T, E
SSC CPO 11/11/2022 (Shift-03) is B’s salary less than that of A? 30% C, T 20%
(a) 10,500 (b) 10,800 A $ B / $ 60% -. - . 5E <
(c) 10,000 (d) 10,700 O B/ $A $ SSC CPO 23/11/2020 (Shift-2)
A

24. In an election between two 1 + (a) 128 (b) 220


candidates, a candidates secured (c) 130 (d) 250
SSC CPO 23/11/2020 (Shift-1)
60% of the valid votes and is 32. If the area of a square is decreased
elected by a majority of 180 votes. (a) 47.7% (b) 33.3% by 19%, then the diagonal of the
The total number of valid votes is: (c) 37.5% (d) 45% square is decreased by:
CK $ - . 29. .If house
$ tax is/ paid before the due $*7 B P L
CK $ $ 9 Gdate,
" # one1 gets
2 a reduction of 12% 67/ $*7 $ N 7
$ !3" - on the amount
$ 7 of . the bill. By paying SSC CPO 23/11/2020 (Shift-2)
the tax before the due date, a person
$ 9 got) a reduction
* +of Rs. 2,100. The (a) 15% (b) 12%
SSC CPO 11/11/2022 (Shift-03) amount (in Rs.) of house tax was: (c) 10% (d) 5%

Aditya Ranjan (Excise Inspector) Selected Selection 3

Downloaded by Kunal Chauhan (kc6970929@gmail.com)


lOMoARcPSD|39315497

Percentage

33. A person's salary increased from 36. A person’s salary has increased 40. A class has five sections that have
Rs.8,100 to Rs.9,000. What is the from Rs.7,000 to Rs.12,000. What 25, 30, 40, 45 and 60 students,
percentage increase in his salary? is the percentage increase in his respectively. The pass percentage
= ) $ 3/!" " E Fsalary? - > of these sections are 20%, 30%,
?/""" EF 4 C $= ) $6 7 0 / " " " E F 35%, 40%
- > and 100%, respectively.
The pass percentage of the entire
1 $% & H !,/""" EF * C $ class is:
SSC CPO 23/11/2020 (Shift-2) 1 $% & 67+
B5 $*7 / 4 25,
1 7 SSC CPO 24/11/2020 (Shift-1)
30, 40, 4560 S P 6 $
(a) 6 % (b) 13 %
9 9 3 1 CW N 7X 120%, 30%,
1 1 (a) 71 % (b) 61 %
7 7 35%, 40% 100% B
(c) 11 % (d) 9 %
9 9
1 4 CW N 7X 1 H
34. The monthly salary of a person (c) 69 % (d) 76 %
was Rs.75,000. He used to spend 7 7 SSC CPO 24/11/2020 (Shift-2)
on Family Expenses (E), Taxes (T), 37. Ramesh spends 40% of his (a) 87% (b) 63%
Charity (C) and rest were his monthly salary on food, 18% on (c) 53% (d) 79%
savings. E was 60% of the income, house rent, 12% on entertainment, 41. If 25% of 400 + 35% of 1260 + 27%

r
T was 20% of E, and C was 15% of and 5% on conveyance. But due to
of 1800 = 1020 + x, then the value
T. When his salary got raised by a family function, he has to borrow
of x lies between:

si
40%, he maintained the percentage Rs.16,000 from a money lender to
level of E, but T became 30% of E meet the expenses of Rs. 20,000. 400 25% + 1260 35% +
an by
and C became 20% of T. The ratio
of the savings of his earlier salary
His monthly salary is:
"#
1800 27% = 1020 + x
T - . *
/x

n
to that of his present salary is: / !3# / ! , # SSC CPO 424/11/2020 (Shift-2)
= ) $ 0(/""" EF (# $ .7 (a) 6 to 10 (b) 0 to 5
’ / ja4 $ (E),
$ . 7 $ N 0 /to 15
(c) C (d) 16 to 20
R s
(T), (C) = ’ ,"/""" EF .7 42. If decreasing 110 by x% gives the
; -E. ’ 60% ! G / " " " E Fsame results as increasing 50 by
a th

UN CO
’ T,/ E 20% ’ C, T x%, then x% of 650 is what
C $ H
percentage (correct to that nearest
15%’ 4- C 40%
$ SSC CPO 25/11/2020 (Shift-2) integer) more than (x – 10)% of 780?
$% & 6 E7 / 1C (a) Rs.18,000 (b) Rs.16,500
ty a

110 x% 1 2
- T, E 30%
(c) Rs.16,000 (d) Rs.15,000 N50 x% -> 1 2
C, T 20% * C 38. $ If
7 49%
$ of X = Y, then Y% of 50 is:
di M

650
/ x%, 780
-. C $ 7 -.X 49% = Y / 50 Y% (x – 10)% 1 9
H 6 - - * 5 M N 7 &<
SSC CPO 24/11/2020 (Shift-1)
SSC CPO 24/11/2020 (Shift-2) SSC CPO 25/11/2020 (Shift-1)
(a) 655 : 644 (b) 325 : 337
(a) 40% of Y (b) 24.5% of Y (a) 17% (b) 12%
(c) 644 : 655 (d) 337 : 325
(c) 50% of X (d) 24.5% of X (c) 18% (d) 14%
35. A man spends 75% of his income.
39. A number is first increased by 40% 43. If each side of a square is decreased
If his income increases by 28%
and then decreased by 25%, again by 17%, then by what percentage
and his expenditure increases by
increased by 15% and then does its area decrease?
20%, then what is the increase
decreased by 20%. What is the net
or decrease percentage in his $*7 1J 8
increase/decrease percent in the
savings? 4 / 6 B
number?
A

= ) 0(# = 1 4 * +
40% $% & 67
C ,3# $ % 25%
&L 6 715%
/ L SSC CPO 25/11/2020 (Shift-1)
’ C = ,"# $ $%% & & 20% - 67 (a) 30.79% (b) 31.11%
4 / C -. 16 7 & $% &V 1 25% H
(c) (d) 44.31%
$% & 67+
SSC CPO 24/11/2020 (Shift-2) 44. The price of diesel is increased
SSC CPO 24/11/2020 (Shift-1) by 26%. A person wants to increase
(a) 7.2% decrease
(a) 52% increase his expenditure by 15% only. By
(b) 6.4% increase
(b) 13% decrease what percentage, correct to one
(c) 3.4% increase
(c) 13% increase decimal place, should he decrease
(d) 3.4% decrease his consumption?
(d) 52% decrease

Aditya Ranjan (Excise Inspector) Selected Selection 4

Downloaded by Kunal Chauhan (kc6970929@gmail.com)


lOMoARcPSD|39315497

Percentage

4 ,G# 48.
$ % In& an entrance examination at 51. If 60% of (x – y) = 45% of (x + y)
different and y = k% of x, then 21% of k is
= ) 4 = $ centres, a total of 25, 30.
40, 45, 60 and 100 students equal to:
!(# $% & . C
appeared. The pass percentages (x – y) 60% = (x + y) 45%
1 5 of $the different centres are 20%, / y=x k% /k 21%
’ &< *30%,
+ 35%, 40%, 50% and 75%,
respectively. The pass percentage
* +
SSC CPO 25/11/2020 (Shift-1)
of the entrance examination is: SSC CPO 09/12/2019 (Shift-02)
(a) 7.2% (b) 6.5% (correct to the nearest integer) (a) 7 (b) 6
(c) 8.7% (d) 9.5% *X * 1 1$ (c) 3 B (d) 1
45. If A's salary is 30% more than B's 25, 30, 40, 45, 60 100 S P C 52. ’ If A’s income is 40% of B’s income
*X * I 1 and
CW N 7 X 1 B’s income is 24% more than
salary, then by what percentage
C’s income then by what
is B's salary less than that of A? 20%, 30%, 35%, 40% 50% percentage is C’s income more
(correct to one decimal place) 75% C B CW N 7A’s X 1
than income? (Your answer
A $ B / $ 30% 9 5 M N 7 < should
H be correct to one decimal
SSC CPO 25/11/2020 (Shift-2) place.)
/B $A $
A B/ 40%

r
1 * + 5 $ (a) 43% ’ (b) 53%
(c) 50% (d) 59% B C/ 24% 9 /
&<

si
49. If the word PHOTOGRAPH is spelt C A 1
SSC CPO 25/11/2020 (Shift-1) with ‘F’ in place of ‘PH’, then what
9 + 5 CW
(a) 17.5%
(c) 25%
an by (b) 23.1%
(d) 19.7%
would be the percentage reduction
in the number of letters?
PHOTOGRAPH Y ‘PH’

SSC CPO 09/12/2019 (Shift-02)
. <

n
46. If the numerator of a fraction is ’ ‘F’ 4 / B (a) 75.6 (b) 101.6
increased by 60% and the 1 *(c)+104.2 (d) 50.4
ja
R s
denominator is increased by 40%, SSC CPO 09/12/2019 (Shift-01) 53. When the price of sugar increased
(a) 20% (b) 18% by 28%, a family decreases its
16
a th

then the resultant fraction is . (c) 10% (d) 25% consumption per month such
63
50. The monthly salary of a person was that the expenditure on sugar was
The original fraction is: Rs. 50,000. He used to spend on only 12% more than the earlier
three heads personal and family one. If the family consumed 18.4
8 I 60% $ % & expenses (E), taxes (T), philan-
ty a

kg sugar per month earlier, then


40% $% & /
thropy (P), and rest were his sav- what is its new consumption of
ings. E was 50% of the income, T
di M

16 sugar per month?


N 8 I 4 was8 20%
I of E and P was 15% of T.
63 When his salary got raised by 40%, 4- . 28%
R $% & 67
H he maintained the percentage level $ 6
of E, but T became 30% of E and P - .
SSC CPO 25/11/2020 (Shift-2) became 20% of T. By what percent-
age is the new savings more or less = 12% 9$
2 5
(a) (b) than the earlier savings? (correct ’ $ .
9 9 up to one decimal place) 18.4 *1 ’ / .
2 4
= ) $ E F’ ( " / " " " +
(c) (d) $ 6 = ’ X
11 9 SSC CPO 09/12/2019 (Shift-02)
= ) * (E),
$ (T),
=
47. A's salary is 35% more than B's (a) 16.6 kg (b) 16.1 kg
(P), ; C E- . ’
salary. How much percent is B's (c) 15.75 kg (d) 15.8 kg
A

C ( T,
" #E ’ , " # ’ 54. A is 40% less than B, and C is 40%
salary less than that of A's?
P, T !(# ’ 4- C $ of the "sum
# of A and B. the differ-
(correct to the nearest integer)
$ % & E6 7 / 1 C 67 ence between
$ 7 A and B is what per-
A $B $ 35% 9 / T, E " # P, T ,"# centage of C?
B $A $ 1 4 - H 4 A,$ B 7 "- #. C, A B
+ 5 M N 7 1 <8 -. 1 9 * "A # + B - .
5CW $ ’ C . <1 * +
SSC CPO 25/11/2020 (Shift-2)
SSC CPO 09/12/2019 (Shift-01) SSC CPO 11/12/2019 (Shift-01)
(a) 20% (b) 35% (a) 60.5% (b) 64%
(a) 16.4% more (b) 8.2% less
(c) 26% (d) 17.5% (c) 8.2% more (d) 16.4% less (c) 62.5% (d) 60%

Aditya Ranjan (Excise Inspector) Selected Selection 5

Downloaded by Kunal Chauhan (kc6970929@gmail.com)


lOMoARcPSD|39315497

Percentage

2 A, B 3# 9 C, A 2 1
55. A man spends
3
rd of his income. B * G " #A, C / 62. If 66 3 % of 75% of 8 of a certain
If his income increase by 14% and 1 9 + 5 $ ’
1
the expenditure increases by 20%, . < number is 179, then 33 % of
then the percentage increase in 3
SSC CPO 11/12/2019 (Shift-02)
his savings will be three-fourth of that number is:
(a) 50.8 (b) 49.2
2 1
(c) 50.2 (d) 49.8 . 0(#
= ) 8 *
3
.7 8
59. The total numbers of males and
C ! # $ % females
& in4 a town is 70,000. If 2
66 % !0? / C
= ,"# $ % & the number
4 of males
/ increased by 3
= ) -. 1 6% $and
% that
& of *the+ females is 1
X. ’ 336 7
% * +
increased by 4%, then the total 3
SSC CPO 11/12/2019 (Shift-01)
numbers of males and females in SSC CPO 12/12/2019 (Shift-01)
(a) 1% (b) 2% the town would become 73520. (a) 537 (b) 787.6
(c) 4% (d) 6% What is the difference between
(c) 859.2 (d) 716
56. In a test consisting of 140 the number of males and females
in the town, in the beginning? 63. Two persons A and B are paid a
questions, a candidate correctly

r
total of Rs. 2,040 per week by
answered 70% of the first 80 - @; their employer. If B is paid 140 per
questions. What percentage of the

si
0"/""" @ ; cent of the sum paid G #to A, then
remaining questions does the
candidate need to correctly
# how $much
% &is A paid per week?

! " 1
an by
answer to score 60% in the test?
$ B
4
/
/
0 8( , "’ 7
-
4
@;
* -
= A) B
1J @; 2
C )
,/" " EF

n
3" 1 0"# CW 8 ) 4 B + A 8 *
B G"# 1 SSC
2 CPO 11/12/2019 (Shift-02) *67 ! " 1 8
ja (a) 1500 1(b) 1800 / 1 JA 2 8 *
R s
C ; 1 1
(c) 2000 (d) 1400 4 +
CW * +
a th

60. In an office, 70% of the total


SSC CPO 11/12/2019 (Shift-01) SSC CPO 12/12/2019 (Shift-02)
number of employees are females,
80% of the total number of (a) Rs. 750 (b) Rs. 820
1
(a) 40% (b) 45 % (c) Rs. 850
employees, including 85 males, got (d) Rs. 800
3
promotion. If there are 105 female
ty a

64. 2000 employees are assigned to


2 employees, then what percentage of
(c) 46 % (d) 35% complete a project. At the end of the
3 female employees got promotion? first year, 15% of the number of
di M

57. In a school 60% of the number of 7 / 7. employees are decreased and at the
students are boys and the rest
0"# 3( @ ; end of the second 7 . year again 10% of
are girls. If 20% of the number of the number of employees are
boys failed and 65% of the 3"# I
decreased. However, to complete the
number of girls passed the $ !"( 7. /
project in time, the number of
examination, then the 1 7. employees I are increased
+ by 10% at the
percentage of the total number of end of the third year. What was the
SSC CPO 12/12/2019 (Shift-01)
students who passed is:
number of employees working during
$Z $Z ’ 7 (a) 30% " # 33 1 %
G (b) the fourth year?
3
- B ,""" 7.
(c) 40% (d) 35%
,"# W 7N 4 G(# 7 *
61. A is 40% more than B and B is
CW N 7 4 / C W N 7
60% less than C. If C is 60% more / 7. !(#
A

$ $Z ’ 7 than D, then 1 which of the $; 7 7


H 4 following is true?
!"# 4
SSC CPO 11/12/2019 (Shift-02) A, B "# 9 B, C G"#
C, D G"# 9 /
(a) 68 (b) 72 7. !"# -
K X $ R +
(c) 74 (d) 78 . ’ $; 7 $
SSC CPO 12/12/2019 (Shift-01)
58. If A is 48% more than B and C is +
60% less than the sum of A and (a) C is 60% more than B.
SSC CPO 12/12/2019 (Shift-02)
B, then A is what percentage (b) B is 36% less than D.
more than C? (Correct to one (c) D is 10.4% more than A. (a) 1786 (b) 1685
decimal place.) (d) A is 54% less than C. (c) 1683 (d) 1783

Aditya Ranjan (Excise Inspector) Selected Selection 6

Downloaded by Kunal Chauhan (kc6970929@gmail.com)


lOMoARcPSD|39315497

Percentage

65. Mangoes are bought at a rate of (a) 125 (b) 100 B 4 A 4


Rs. 10,000 per ton. If one-third of (c) 75 (d) 90 ""# 9 * A 4$; D
the total mangoes are sold at a
68. In a test consisting of 120 1x% $ ; 7$ % & B
loss of 4%, then at what price
questions, Anuradha answered 4 1 $; 7 C
(per ton) should the remaining
65% of the first 60 questions
mangoes be sold so as to gain 4 , $ ; AD B - /
correctly. What percentage of the
30% on the whole transaction? remaining questions does she 4 - - x 4 /
EF !"/""" 1 M need to answer correctly to score SSC CPO 13/12/2019 (Shift-02)
* 675%
7 in the test?
#
2
- . * / 120 1
- $" # B (a) / 30 % 9
3
8 47 ; G" 1 G(# CW
(b) 40%
R 5 1 M < - . 4 B . +0 ( # 1 2 C ;
1 1 (c)
CW 25% * +
SSC CPO 12/12/2019 (Shift-02)
SSC CPO 13/12/2019 (Shift-01) 1
(a) Rs. 15,000 (b) Rs. 13,500 (d) 33 %
(a) 80 3
(c) Rs. 14,700 (d) Rs. 14,600
(b) 85 71. The difference between the 38%
66. The ratio of expenditure and

r
of a number and 22% of that
savings of a person is 5 : 3. If the (c) 84
number is 3200. What is the
income increases by 20% and the

si
(d) 90
expenditure increases by 10%, 1
69. In a class, there are 54 students. 15 % of that number?
then the person’s savings 2

=
an by
increase by:
) = -.
33
1
3
% of the number of students
are boys and rest are girls. The
3# C

n
- . ,"" C
( C , " average$score
# is 50%
boys % more&in mathematics of
than that of the
C = !"# $ %girls.
& If the4 average /score of all 1
15 % * +
= ) ja -. $% the students is 70, then what is
& *score
+ of the boys? 2
R s
the average
SSC CPO 13/12/2019 (Shift-01) SSC CPO 13/12/2019 (Shift-02)
B ( $Z ’ 7 $Z ’ 7
a th

1 (a) 2800 (b) 3000


2 33 % ;
(a) 36 % 3 (c) 3100 [ (d) 3200
3
* N 72. 1A person
2 /can save 25% of his
1 1 2 income.( "If#his income
9 increases
ty a

(b) 3 % 8 $Z ’ 7 by 20% and 1 2 0"


still he saves the
3
/ 1 2 same H amount as 4 before, the
di M

2 SSC CPO 13/12/2019 (Shift-01) percentage increase in his


(c) 13 % expenditure is.
3 (a) 81
= ) ,(# -.
1 (b) 84
(d) 30 % C ,"# $% &
3 (c) 87
L 8 $ $7 -
67. A is 75% less than B and C is 75% (d) 90
-. / C .7
of the difference between A and 70. The population of town B is 300% 1 +
B, C is what percentage more more than that of town A. For the
than A? next two years, the population of SSC CPO 13/12/2019 (Shift-02)

A, B 0 ( # C, A $B - . A increases by x% per year and that


1
of B decreases by the same (a) 25 (b) 24
0 ( C,
# A 1 percentage per year. After 2 years, 3
9 + if the population of A and B become
A

2
SSC CPO 13/12/2019 (Shift-01) equal, then the value of x is. (c) 25 (d) 26
3

Aditya Ranjan (Excise Inspector) Selected Selection 7

Downloaded by Kunal Chauhan (kc6970929@gmail.com)


lOMoARcPSD|39315497

Percentage

ANSWER KEY
1.(c) 2.(a) 3.(b) 4.(c) 5.(d) 6.(b) 7.(c) 8.(a) 9.(b) 10.(a)

11.(a) 12.(a) 13.(b) 14.(a) 15.(a) 16.(c) 17.(b) 18.(d) 19.(d) 20.(d)

21.(d) 22.(c) 23.(c) 24.(d) 25.(d) 26.(b) 27.(c) 28.(c) 29.(b) 30.(d)

31.(b) 32.(c) 33.(c) 34.(a) 35.(a) 36.(a) 37.(a) 38.(d) 39.(d) 40.(c)

41.(a) 42.(d) 43.(b) 44.(c) 45.(b) 46.(a) 47.(c) 48.(c) 49.(a) 50.(a)

51.(c) 52.(b) 53.(b) 54.(c) 55.(b) 56.(c) 57.(c) 58.(b) 59.(c) 60.(b)

61.(b) 62.(d) 63.(c) 64.(c) 65.(c) 66.(a) 67.(a) 68.(b) 69.(d) 70.(d)

r
71.(c) 72.(d)

si
an by SOLUTIONS
SOLUTIONS

n
ja
R s
1. (c) 6. (b)
SMART APPROACH:-
Annual Birth Rate = 12.7%
a th

Boys : Girls 1 6
Increment of 20% =  Annual Death Rate = 2.7%
5 5
3 : 4 Hence, Actual Growth Rate
(After increment) (original)
 Let  300 400
30,942
= 12.7% – 2.7% = 10%
Original population  5 Population of Town After 3 years
 10%  15%
ty a

6
330 460 = 25785 110 110 110
 125000   
4. (c) 100 100 100
di M

330 33 Income of Bhavani = 1000 units = 166375


Required ratio = =  150 (Kid's education)
460 46 Increase in population
2. (a)  200 (Food) = 166375 – 125000 = 41375
Remaining = 650 units
Let the total number of votes = 7. (c)
100% 2 Ratio of salaries of A, B & C
 × 650 = 260 (Sports)
Candidate A got =37% 5 =2:3:4
Candidate B got = 63% 3 Let, their salaries be 200, 300 & 400.
ATQ, Candidate A lost the  × 650 = 195 (Transport) After increment,
10
election by 338 Votes. New ratio = 260 : 360 : 440
Remaining = 650 – (260 + 195)
Hence, 63% – 37% = 338 = 195 units =13 : 18 : 22
338 We have, 195 units = Rs. 10,257 8. (a)
1%   Income of Bhavani
A

26 Let the voter list = 100x


10,257 Casted vote = 65% of 100x = 65x
100%  13  100  1300  × 1000 = Rs. 52600
195 65x  97
3. (b)
5. (d) Valid vote =
Let the population 1 year ago = x 100
Income Expenditure Savings
ATQ, A.T.Q,
Population is increased by 20% 16500 10500 6000
1 65x  97  65
Hence, 120% of x = 30942 25% = + +19%
4 = 81965
20625 12495 8130 100  100
120  x % Increase in savings
  30942 x = 2000
100 2130
 x = 25785   100 = 35.5% Voter list, 100x = 200000
6000

Aditya Ranjan (Excise Inspector) Selected Selection 8

Downloaded by Kunal Chauhan (kc6970929@gmail.com)


lOMoARcPSD|39315497

Percentage

9. (b) SMART APPROACH:-


 10  22  22  5  5  10   10  22  5 
Let the sum = 100%   
 100   10000  25% 
1
40% 
2
Equivalent successive percentage 4 5
decrement of 20% & 10% = 28%  220  110  50  1100 RAJU MANJU TOTAL
72unit  7200 = 37 +  
 100  10000 Initially 500 400 900
1 unit 100 625  560 
Original sum = 100 unit = 10000 = 37 + 3.80 + 0.11 After
 5  7  1185
= 40.91% Increment 500    400 5 
10. (a)  4  
Let the registered voter = 100 14. (a) 285
Net Increment  100%  31.67%
Initial Final 900

100 Reg. Voter 18. (d)


7
35%  20 27 1
90 Vote casted 20 We know, 2% =
–2 50
81 Valid vote 40%  5 3 Population After Two year
5
Winner 49 49
Loser 1 = 45000   = 43218
25%  4 5 50 50
43.74 37.26 4
Overall 80 81 SMART APPROACH:-
6.48 unit  1620 In the last step complete
1

r
multiplication is not required.
Net Increase% =  100% = 1.25%
Registered voters 80 First check which option is

si
divisible by 9 (sum of digit is
1620 15. (a)
=  100 = 25000 a multiple of 9) you will see
6.48 13 only option (d) satisfied.
an by
SMART APPROACH:-
Let the number of enrolled voters=x
We know, 65% =
20 19. (d)
Let the price of the car = 100

n
x
9

9

8
 1620
Income – Expense  Saving After 35% increment, price of car
10 10 100 20 – 13  7 = 135
 x  25000
ja 20%  10%  After 25% decrement, price of car
R s
= 75% of 135 = 101.25
11. (a) 24 – 14.3  9.7
Net increment percentage in price
Method-1: Using Ratio
a th

Percentage increase in saving of the car–


R SH S 2.7 101.25 – 100
=  100% = 38.5% =  100%
17 10 S 7 100
16. (c) 1.25 1
7 10
ty a

Manpower × Working Hours =  100% = 1 %


100 4
119 70 100 =Production
S M A R T A P P R OACH :-
di M

 R : S = 119 : 100 7
Method-2: Basic Method We know, 35% = Net change
20
Let the income of Sohan be 100.  35 25 
Manpower changes from 20 to 13.   35 – 25 – %
Shyam  100 
Ram Sohan Hence, To restore production
 3
119 70% More 70 30% Less
100 working hours is to be increased  10 – 8  %
 4
 Ram : Sohan = 119 : 100 7
 100% = 53.85% 1
12. (a) by =  1 %
13 4
Looser get = 42% of polled vote. Note:- '+' sign denotes increment.
17. (b)
Remaining vote = 58%,
(this includes 1400 invalid votes) Let the income of Manju = 100 20. (d)
Difference 58 – 42 = 16% = 3080 + income of Raju = 125 Marked Price = ` 1000
1400 = 4480 Combined income of Raju and Manju Selling Price = ` 1200
= 125 + 25% of 125 200
A

4480 %Increment =  100% = 20%


Total vote  × 100 = 28000 =125+31.25 =156.25 1000
16 21. (d)
After 40% rise, Income of Manju
1400 = 100 + 40% of 100 = 140 Total Number of Voter = 150000
% of invalid vote  × 100 After Increment,
2% of total votes were declared
28000 invalid
= 5% Combined Income = 296.25
98
13. (b) 296.25 – 225 Valid Vote = 150000 
 100% 100
Net change %Increase = = 147000
225
 xy  yz  zx   xyz  Number of votes polled in the favor
= (x + y + z) +    71.25
 100   10000 
 100%  31.67% 60
= of candidate = 147000  = 88200
= (10 + 22 + 5) + 225 100

Aditya Ranjan (Excise Inspector) Selected Selection 9

Downloaded by Kunal Chauhan (kc6970929@gmail.com)


lOMoARcPSD|39315497

Percentage

25. (d) 30. (d)


SMART APPROACH:-
Required no. of votes % Decrease in Area Expenditure same
98 3
 1,50,000    88,200 11  11 1
100 5 = –11 – 11 + Price 
In the last step, no. need of 100 consumption
doing complete multiplication.
First check the divisibility of 9 = – 22 + 1.21 Present : Last Month
i n t h e g i ve n o pt i o n s, = – 20.79% Price 29 : 25
only option (b) eliminated now, Cons. 25 : 29
check which option is divisible 26. (b)
by 7. only option (d) satisfied. Price × consumption = expenditure 29 – 25
% Decrease =  100 %
22. (c) Let initial consumption = y 29
And new consumption = y
Net change 4
=  100 %= 13.79% = 14%
100 100  x 29

 15  10  116 110  y 31. (b)
 15 – 10 – %
 100  Let, I = 1000 unit
 (15 – 10 – 1.5)%  3.5% x 110
 I = E + T + C + S (Saving)
y 116 1000 = 600 + 120 + 18 + Savings
SMART APPROACH:-

r
% decrease in consumption Saving = 1000 – 738 = 262
Original Final 1400 = 840 + 252 + 50.4 + Savings

si
6 3
=  100 =  100 Saving = 1400 – 1142.4 = 257.6
20 23 116 58
Difference b/w saving = 262 – 257.6
10
200
an by 9
207
= 5.17% = 5.2%
27. (c)
= 4.4
So, 4.4 = 50000 × 4.4 = 220

n
32. (c)
 100 – x   100  x 
7
110    50   If we decrease the side of square
 100   100 
ja 200
×100 = 3.5% by 10% this will result into 19%
R s
decrement in area, vice-versa.
23. (c) 100 – x 5
 Side  diagonal
100  x 11
a th

Total Enrolled Voters = 100x So, Decrement in diagonal = –10%


Vote Casted = 85x 1100 – 11x = 500 + 5x 33. (c)
Valid Vote = 96% of 85 16x = 600 8100  9000 8 : 9
ATQ,
75
ty a

 85% of (96% of 85x) = 6936 % 1 1


x= Increment % =  11 %
2 9 9
85 96
   85x  6936 Now, x% of 650 : (x + 20%) of 180
di M

34. (a)
100 100
75 115 Let earlier income = 1000
6936  100  100 650 × : 180  I = E + T + C + Saving
 x 200 200
85  96  85 1000 = 600 + 120 + 18 + Savings
 x = 100 325 : 138
Initial saving = 1000 – 738 = 262
Hence, Total enrolled voter 325 – 138 1400 = 840 + 252 + 50.4 + Savings
= 100 × 100 = 10000 % increase = × 100
138 Present saving = 1400 – 1142.4 =
SMART APPROACH:- 257.6
Let Total enrolled voters = 100x 187 Ratio of earlier and present salary-
=  100 = 135.5% = 136%
100x 
85

96

85
= 6936 138
100 100 100 262 257.6
28. (c) 75000  : 75000 
 x = 100 1000 1000
Hence, Total enrolled voter A : B
 100 × 100 = 10000 8 : 5 = 2620 : 2576 = 655 : 644
A

24. (d) % of B salary less than A 35. (a)


Winner candidate = 60% Income – Expenditure = Savings
8–5 3
So, Loser candidate = 40% =  100 =  100 = 37.5% 100 – 75 = 25
8 8 28% 20%
ATQ,
Won by the majority of 180 votes 29. (b) 128 – 90 = 38
 60% – 40% = 180 % Reduction = 12%
38 – 25
 20% = 180 12% = 2100 % increment = × 100%
25
 1% = 9 1% = 175
Thus, Total number of valid votes 100% = 17500 13
=  100 = 52%
= 100% = 900 The amount of house tax = 17500 4

Aditya Ranjan (Excise Inspector) Selected Selection 10

Downloaded by Kunal Chauhan (kc6970929@gmail.com)


lOMoARcPSD|39315497

Percentage

36. (a) 43. (b) E = 70,000 × 50% = 35,000


% Decrement in area of square T = 35000 × 30% = 10,500
 12000 – 7000  P = 10,500 × 20% = 2100
% Increase =   × 100%
 7000  289 Total E = (35,000 + 10,500 + 2100)
= – 17 – 17 +
100 = 47600
5000 3
=  100% = 71 % = – 34 + 2.89 Saving = (70,000 – 47600) = 22400
7000 7 Saving increase in %
37. (c) = – 31.11%
40% + 18% + 12% + 5% = 75% 44. (c) (22400 –19250)
= 100%
Saving = 100% – 75% = 25% 19250
100 100  x
 25% = 20000 – 16000 = 4000  3150
100% = Rs.16000 126 115  y =  100% = (16.4% more)
19250
38. (d)
x 115 51. (c)
49x 
y= y 126 60% of (x – y) = 45% of (x + y)
100
% Decrement = x – y 45 3 x 7
   
49x x  y 60 4 y 1
So, y% of 50 = 50 × % 126 – 115
100  100% = 8.7% y = k% of x
 24.5x % = 24.5% of x 126
39. (d) k
45. (b) 7

r
1=
Initial Final 100
A : B

si
5 7 13 10 100
k=
% of B' salary less than that of A 7
4 3
20
an by 23 =
13 – 10
13
 100% =
300
13
% = 23.1% 21% of k =
100 21
7

100
3

n
5 4
46. (a) 52. (b)
500 483 A B C
x 49.6 124 100
ja Let the fraction = y
R s
(100 – 49.6)
17 Required % = 100%
×100 = 3.4% 49.6
x  160 16 x 2
a th

500
40. (c) ATQ, y  140  63  y  9 50.4
Pass % of whole class = 100% = 101.6%
49.6
47. (c)
5  9  14  18  60 53. (b)
=  100% A : B = 135 : 100
Earlier Now
ty a

25  30  40  45  60
= 27 : 20 Price  25 32
106 % of B' Salary less than that of A Cons.  32 25
=  100 % = 53%
di M

200 7 Earlier family consumed sugar per


41. (a) =  100% = 25.92% = 26% month = 18.4 kg
27 Expenditure on sugar was only 12%
25% of 400 + 35% of 1260 + 27%
of 1800 = 1020 + x 48. (c) more than earlier
100 + 441 + 486 = 1020 + x 5  9  14  18  30  75 18.4
1027 = 1020 + x Pass% = 300
× 100% =  25 × 112% = 16.1 kg
32
x = 7
x lies between 6 to 10. 151 New consumption of sugar per
42. (d) = = 50.3%  50% month = 16.1 kg
3
ATQ,
49. (a) 54. (c)
 100 – x   100  x  Total word in PHOTOGRAPH = 10
110    50   A B C
 100   100  If PH is replaced by F, then the total 60 100 64
75 word in (FOTOGRAF) = 8
 x  % (100 – 60)
A

2 2 % change = × 100%
% Change = × 100% = 20% 64
x% of 650 : (x – 10)% of 780 10
75 55 50. (a) 40
 650  % : 780 × % Salary = 50,000 = × 100% = 62.5%
2 2 64
= 25 : 22 E = 50,000 × 50% = Rs. 25,000 55. (b)
T = 25000 × 20% = 5000
25 – 22 P = 5000 × 15% = 750 Income Expense Saving
% Increase =  100
22 Total E = 25000 + 5000 + 750 = 30750 30 20 10
300 150 Saving = (50,000 – 30,750) = 19250
 %  % When salary got raised by 40% 14% 20%
22 11 New salary = (50,000 × 140%)
= 13.63%  14% = 70,000 34.2 24 10.2

Aditya Ranjan (Excise Inspector) Selected Selection 11

Downloaded by Kunal Chauhan (kc6970929@gmail.com)


lOMoARcPSD|39315497

Percentage

(10.2 – 10) 48.8 2 3 1


% Change = × 100% = × 100% = 49.2% x    179
10 99.2 3 4 8
0.2 59. (c)  x  179  16
= 100% = 2%
10
3
56. (c)  33% of of 179  16  716
Male Female 4
6% 4% 63. (c)
70% x%
A B
352
100 140
70
60% 2040
Sum Paid to A= ×100
240
72 68 = Rs.850
18 : 17 64. (c)
80 : 60 Number of employees working
4 : 3 Given that, during the fourth year,
35 unit = 70,000

r
Then required difference 85 90 110
70%  4  3x = 2000 ×   = 1683
100 100 100

si
 60% = (18 – 17) unit = 2000
(4  3)

 60% =
an by
70%4  3x
7
SMART APPROACH:-
2% M + 4% M + 4% W = 3520
SMART APPROACH:-
Check the applicability

n
420% = 280% + 3x
2% M + 2800 = 3520 of divisibility rule of 9. only
140% 2
x=  46 % option (c) satisfied
3 ja3 2% M = 720
R s
The candidate need to answer  M = 36000 & W = 34000
65. (c)
a th

2 Difference = 2000
correctly = 46 % Let the remaining mangoes be sold
3
at Rs. x.
60. (b) ATQ,
SMART APPROACH:-
ty a

Total number of employee


10,000 24 130
Let the total marks = 140    x  10000 
105 3 25 100
By answering 70% of the first 80 = × 100% = 150
di M

questions correct the has got = 56


70%  3200 + x = 13000
Marks & 60% of total marks = 84 Number of employees who got x = 9800
Now the has to score, 84 – 56 80
= 28 Marks promotion = 150 ×  120 2
100 Price of ton of mangoes= Rs 9800
From the remaining 60 questions. 3
Female employees who got
28 2 3
Req.%   100  46 % promotion = (120 – 85) = 35
60 3 Price of 1–ton mango = 9800 
% of female employees who got 2
57. (c) = Rs. 14700
35 1
100 promotion= × 100% = 33 % 66. (a)
105 3
I E S
61. (b) 80 50 30
Boys Girls
60% 40% A B C D 96 55 41
A

80% Passed 65% Passed 896 640 1600 1000 Per centage increase in savings

Pass 48 26 (Pass) B is % less than D 11 2


=  100 = 36 %
Total students passed = (48 + 26) 30 3
1000 – 640
= 74% = × 100% = 36% 67. (a)
1000
58. (b) A B C
A : B : C 62. (d) 25 100 56.25
148 100 99.2 Let the number be x.
56.25 – 25
% Change =  100%
(148 – 99.2) 1 25
% change = × 100%  66.66%  75%   x  179
99.2 8
= 125%

Aditya Ranjan (Excise Inspector) Selected Selection 12

Downloaded by Kunal Chauhan (kc6970929@gmail.com)


lOMoARcPSD|39315497

Percentage

68. (b) Let, Avg score of girls = 2x  200 – 2x = 100 + x


Let the number of questions to be  100 = 3x
3
answered correctly is x. Avg score of boys = 2x × = 3x 100 1
ATQ, 2 x =  33 %
18 × 3x + 36 × 2x = 54 × 70 3 3
 65  75 54x + 72x = 3780 71. (c)
60 ×    x = 120 × 126x = 3780 Diff. = (38% – 22%) = 16%
 100  100
 x = 30 3200
39 + x = 90 Number = 100% = 20,000
Avg score of Boys = 30 × 3 = 90 16%
x = 51 70. (d)
A : B 1 31
51 15 % of 20,000 = 20,000 ×
Required% = × 100% = 85% 1 4 2 2 100
60
(100  x ) (100  x ) = 3100
Method-2 1 × × 72. (d)
100 100
Income – Exp. = Savings
(65  y) 4(100 – x ) (100 – x )
  75
2 = × 100 – 75 = 25
100 100 +20
65 + y = 150  (100 + x)2 = 4(100 – x)2 120 – 95 = 25
y = 85% 2 20
(100  x ) 4 100%
69. (d)  (100 – x )2  1 % Change in Exp. =

r
75
1 2

si
No. of Boys = 54 × = 18 (100  x ) 2 = 26 %
3  (100 – x )  1 3
No of girls = 54 – 18 = 36
an by
n


ja
R s
a th
ty a
di M
A

Aditya Ranjan (Excise Inspector) Selected Selection 13

Downloaded by Kunal Chauhan (kc6970929@gmail.com)


lOMoARcPSD|39315497

Profit & Loss

PROFIT & LOSS


02
1. A seller uses faulty weight in (a) ` 27 (b) ` 27.04 ’ /% 0!
place of a 2 kg weight and earns (c) ` 28.05 (d) ` 28.15 $ /
a 25% profit. He claims that he
3. A grocer sells rice at 10% profit "* F ! "* F > B
is selling on the cost price in front
and uses weights which are 20%
of the customers but uses a faulty 3
less than the market weight. The ’ !8" # $ %
weight. How much error is there
total gain earned by him is:
in the 2 kg weight to gain 25%?
? 0 .- A % 0 0 &
$ % $ 8 SSC CPO 09/11/2022 (Shift-03)
+ $ - 17%!
(a) (b) 37%

r
!"# $ % ( ’ @ @(c)
@ 47%
@@@@@
(d)@27%
@

si
SSC CPO 09/11/2022 (Shift-02)
7. Swati sold a bag to Ankita at a
! 1 gain of 15%. Aashi bought this
!
an by
SSC CPO 09/11/2022 (Shift-01)
! &(a)
33 %

(c) 40%
3
(d) 37.5%
(b) 20% bag from Ankita for ` 24,840.
Ankita earned a profit of 20%. At
what price would swati have

n
(a) 250g (b) 450g bought the bag?
4. On selling a car to Anuj, Aman
(c) 500g (d) 300g suffered a loss of 10%. Anuj sold
2. ja
A faulty weighing machine reads
$
the car for ` 5,54,400 suffering a
R s
1 kg when 900 gm is actually $ % / 0
loss of 12%. What would be the
weighted on it. The shopkeeper cost price of the car for Aman? A+GA- :@ ! H 0
a th

marked the price of his goods by -


10%, but unfortunately he was ! ’ .- 0
0! H 0 &
caught by the metrology $ %0 ’ + A+A--
department and then they ordered :@ ! . 0 $ %SSC CPO 10/11/2022 (Shift-01)
!
ty a

him to repair his weighing (a)&` 18,500 (b) ` 18,000


machine and gave punishment to !"# B
sell the goods at 10% discount SSC CPO 09/11/2022 (Shift-02)
(c) ` 16,000 (d) ` 20,000
di M

on cost price for a month. If each 8. A man bought two bicycles for `
(a) ` 7,00,000 (b) ` 8,00,000
customer is now paying ` 20 for 3,000 each. If he sells one bicycle
1 kg, then before the (c) ` 6,50,000 (d) ` 7,50,000 at a profit of 10% then for how
raid___amount they have paid for 5. A wrist watch is sold for `1,200 much percentage profit should he
same quantity. (Rounded off to at a profit percent equal to its sell the other bicycle so that he
two places of decimal) cost price. Find the cost price of makes a profit of 20% on the
’ ( ! the wrist watch. whole?
) "* " C
D 420 ! "? # B I+--- 6@ 9
+ ,-- ! $
$ . / .+ -- :@ ! C$ % H 0 0
/ + 0 ’ C D 420 !"# E . - $ %
! !"# .- $12 SSC CPO 09/11/2022 (Shift-03)
" 0 C
+ ’ 3 ! 0 $ % % +
(a) ` 400 (b) ` 250
A

( 42 + $ &
(c) ` 350 (d) ` 300
" 50 4 6 SSC CPO 10/11/2022 (Shift-01)
6. What will be the actual profit
! 0 !"# . - percentage
0 (rounded off to (a) 15% (b) 25%
7"8 ’ ! $ % /
nearest integer) after selling an
9 $article
. at a certain price, while (c) 30% (d) 10%
- : ’ there occurs a loss of 45% on 9. The price of an article is first
increased by 25% and later on,
7 ! ! ; 0 3
selling the article at of the the price was decreased by 30%
< / ’ & = / ! 8 due to a reduction in sales. Find
"* > selling price? the net percentage change in the
SSC CPO 09/11/2022 (Shift-01) final price of the article.

Aditya Ranjan (Excise Inspector) Selected Selection 14

Downloaded by Kunal Chauhan (kc6970929@gmail.com)


lOMoARcPSD|39315497

Profit & Loss

’ 0 0! ! . :N@
0 !H 0 1
16 %
$(a) % (b) 20%
J K 0 0 $ ! $ 0: @.DG 8 ! 3
* 0 0! ! I- . - - -
0 ! $ ,-- !
!0 0 0 ’ ! !0 "# 1 2
(c) 33 % (d) 16 %
! / ’K J K !0 / E / B & 3 3
SSC CPO 10/11/2022 (Shift-02) SSC CPO 11/11/2022 (Shift-01) 17. Swastik purchased a laptop for
(a) 11.% increased `75,000 and sold it to Anju
(a) 23% (b) 20%
suffering a loss of 10%. Anju sold
(b) 14% decreased the same laptop after earning a
11
(c) 13% increased (c) 23 % (d) 25% profit of 20%. At what price (in
18
(d) 12.5% decreased `) did Anju sell the laptop?
10. Ajay buys 18 oranges for ` 90 and 14. A shopkeeper claims to sell his M +--- :@ !
sells 15 oranges for ` 105. Find articles at ` 20 per kg which cost
R 0 C .-
the percentage of his gain. him ` 23 per kg. But while
selling, he uses a false weight $ % " 0 8
,- 6@ ! .G H and0 gives only 800 gm instead of $ %
. .- 6@ ! $ % 1 kg. What is his profit 8 Q 0! =:@ !
/ E percentage?

r
SSC CPO 11/11/2022 (Shift-03)
SSC CPO 10/11/2022 (Shift-02) ’ I 6@ !"#
(a) `78,500 (b) `85,000

si
(a) 35% (b) 30% 0 ’ - 6@ (c) `81,000 (d) `90,000
(c) 25% (d) 40% $ % $ % ! +
an by 18. A person sold an article at a loss
11. A shopkeeper sells his items ! 42$420 of 16%. Had. he sold it for Rs.660
using a faulty balance which $ G-- ! more, he would have gained 8%.

n
measures 25% less. He then What should be the selling price
/ B &
marks up his items 15% above (in Rs.) to gain a profit of 12%?
the cost price. If he also gives a SSC CPO 11/11/2022 (Shift-01)
ja
discount of 10% then find his net C ? B + 0 ’
R s
profit percentage on 1 kg items. 17 16 $ % C ’
(a) 8 % (b) 8 %
23 ) 23
a th

’ 0 ’ "* < ! $ % + G
" $ % . P
14 15
! L 0 (c) 8 ’ % (d) 8 % !"# =:@ ! >
23 23
!"# . < !"# SSC CPO 23/11/2020 (Shift-1)
ty a

.- 0 715.
" 8 A shopkeeper
0 increases the
(a) 3,080 (b) 3,200
selling price of an article by 15%.
+ . ’ / ’K (c) 2,750 (d) 2,970
di M

After increasing the selling price,


/ E 0 he noticed that the profit 19. A man bought an article and sold
SSC CPO 10/11/2022 (Shift-02) percentage changed from 5% to it at a gain of 10%. If he had bought
15%. The percentage increase in the article at 20% less and sold
(a) 32% (b) 41%
the cost price is: it for Rs.1,000 more, he would have
(c) 44% (d) 38% made a profit of 40%. The cost
12. Satish bought 35 notebooks for ’ ’ !"#
price of the article (in Rs.) is:
` 700 and sold 28 of them for ` ! .0 J K !"#
C ? B ’ H 0
600. Find his percentage gain. ! J K + H $
.- $ %
0/ M-- 6@ ! I 8$’ H/ 0 0 ! $
- ! !"#
.
H 0
! G 8$’ ! "@
N-- 6 # !! $ % / J K@@@@@@@@@@@
.+--- :@ < ! $ % +
0 / # E SSC CPO 11/11/2022 (Shift-02)
’ !"#
SSC CPO 10/11/2022 (Shift-03) (a) 20% (b) 10% E
A

1 1 (c) 15% (d) 5% SSC CPO 23/11/2020 (Shift-2)


(a) 7 % (b) 9 %
16. By selling 24 items, a shopkeeper
7 7 (a) 50,000 (b) 60,000
gains the selling price of 4 items.
His gain percentage is: (c) 25,000 (d) 40,000
1 1
(c) 6 % (d) 8 % 20. A man bought an article and sold
7 7 A ’ O $ % + ’
it at a gain of 10%. If he had bought
13. Ravi buys salt at ` 16 per kg and A ’ !"# $ $
the article at 20% less and sold
sells it ` 18 per kg. He also uses P it for / Rs.1,000
B more, he would
the weight of 900 gm instead of & have made a profit of 40%. The
1000 gm. What is Ravi's actual earlier selling price of the article
profit percentage? SSC CPO 11/11/2022 (Shift-02)
(in Rs.) is:

Aditya Ranjan (Excise Inspector) Selected Selection 15

Downloaded by Kunal Chauhan (kc6970929@gmail.com)


lOMoARcPSD|39315497

Profit & Loss

0 ? B ’ H 0(a)0Rs.4,200 (b) Rs.4,800 !0 ’ 0 .I


.- $ % (c) Rs.4,600 ’ (d) Rs.4,400 C :@
N - <M @! " -# $
- ! !"# H 024. I purchase 100 kg of tea and sell + x% ’ 0
.+--- 6@ < ! it for
$ a%profit to the +extent of what !"# IM: @ - +x ! B
I would have paid for 40 kg. What is
A- ! ’ " SSC CPO 11/12/2019 (Shift-02)
my profit percentage?
!"# =:@ ! > E (a) 30 (b) 32
! .-- % H 0 0 (c) 28.4
C ! (d) 29.2
SSC CPO 24/11/2020 (Shift-1)
$ % + ! ( 28. A - profit on selling
The % ’
an article for
(a) 40,000 (b) 50,000 ’ $ $ ! Rs. 1,100
( is equal to three times
(c) 60,000 (d) 55,000 / E 0 the amount of loss on selling it
21. If he selling price of 50 articles for Rs. 700. To gain 12.5% the
SSC CPO 09/12/2019 (Shift-01)
is equal to the cost price of 42 article must be sold for:
articles, the what is the (a) 30% (b) 20%
(c) 25% (d) 40% ’ .+.-- : ! $ %
approximate loss or profit
percentage? 25. By selling an article for Rs. 1,134 + C 0 ’ M--
Anu suffers as much loss as she 0 0 ’
50 42 ! " #’ would have gained by selling it at + ’
!"# $ $ + / ’ profit.
10% K If the sells it for Rs.

r
/ E 1,354.50, then her profit
$ % % &
percentage is: SSC CPO 12/12/2019 (Shift-01)

si
SSC CPO 24/11/2020 (Shift-2)
(a) 8% profit (b) 16% loss
’ :. @
+! . I A$ % + (a) ’ Rs. 956 (b) Rs. 877.50
(c) Rs. 787.50 (d) Rs. 900
(c) 10% lossan by (d) 20% profit
22. A person sold an article at a loss $ %
0 C ’ 29. On. - selling an article for Rs. 115,
$the $gain is 20% more than the
+ loss incurred on selling it for Rs.

n
of 16%. Had he sold it for Rs.660 ’ . +: I@! A @$ - % 0
more, he would have gained 8%. / E 0 104. If the article is sold for Rs.
If the article is sold at Rs.3,080, 130.80, then the profit percentage is:
ja
then how much profit percentage
SSC CPO 09/12/2019 (Shift-02)
’ :@ ! .. $ %
R s
(a) 9
is gained? C 0
’ .-A :@ !
(b) 8
a th

? B 0 ’ .N 0 - <
(c) 8.4
$ % ’ (d) 7.5 NN- 6@ .I-@G- :@ ! $ %
< ! $ % + G P
26. Sujatha sold 75% of her goods at / E 0 &
’ I + a- G - 6of@
profit 24%! and the remaining SSC CPO 13/12/2019 (Shift-01)
ty a

$ % + / at a loss of 40%. P
What is her gain (a) 25 (b) 18
& loss percentage on the whole (c) 30 (d) 20
di M

transaction? 30. An article was sold at a certain


SSC CPO 25/11/2020 (Shift-1)
’ ! M A 4
(a) 20%
/ ) ! A - price.
0 had it been sold at 5 of
(b) 15%
$ % " S ! that price, there
T would have been
(c) 12%
/ E 0 a loss of 10%. At what profit
(d) 10% percentage was the article sold
23. An article was sold at a loss of SSC CPO 11/12/2019 (Shift-01)
initially?
13.5%. If it was sold for Rs.1,104 (a) 8% gain
(b) 10% gain
’ /% !"#
more, then there would have been
a profit of 9.5%. The cost price of (c) 9% loss 4
the article was: (d) 7.5% loss
C ! 5" # $ %
0 13.5%
’ 0 27.
$ % person sold a chair at a profit
A .- 0 0 !
’ . +of. -13%.
A 6Had@ he sold it for Rs. / $ %
A

607.50 more, he would have


< ! $ % 9.5%+ gained x%. If the cost price of the SSC CPO 13/12/2019 (Shift-02)
# ’ ! " #chairsE is Rs. 3750, then the value (a) 10.5 (b) 10
SSC CPO 25/11/2020 (Shift-2) of x is: (c) 12.5 (d) 15

Aditya Ranjan (Excise Inspector) Selected Selection 16

Downloaded by Kunal Chauhan (kc6970929@gmail.com)


lOMoARcPSD|39315497

Profit & Loss

ANSWER KEY
1.(b) 2.(a) 3.(d) 4.(a) 5.(d) 6.(c) 7.(b) 8.(c) 9.(d) 10.(d)

11.(d) 12.(a) 13.(d) 14.(b) 15.(d) 16.(b) 17.(c) 18.(a) 19.(a) 20.(d)

21.(b) 22.(c) 23.(b) 24.(d) 25.(d) 26.(a) 27.(d) 28.(d) 29.(d) 30.(c)

SOLUTIONS
1. (b)
3. (d)

r
Let the error in whole
Let the cost price of 1000g is ` 1000
transactions be x gm. SMART APPROACH:-

si
Selling Price = 110% of 1000
Error CP SP
Profit% = = ` 1100
 True Value  Error 
an by Uses 20% less weight instead of 10 9
x actual weight. 25 22
 25% 

n
2000 – x It means he gave 800 g instead 125 99
of 1000 g.
25 x  Cost price for Aman
  Hence, He sells 800g at ` 1100
ja
100 2000 – x which costs him ` 800 5,54, 400
R s
  125
1 x Profit = 1100 – 800 99
 
a th

4 2000 – x Profit = Rs. 700,000


Profit% =  100%
 2000 – x  4x CP
 x  400 gm 300 5. (d)
  100% = 37.5%
ty a

800 Let the cost price of the article


SMART APPROACH:- = x & profit% = x
Given that, Profit% = 25% SP = CP + P% of CP
di M

Hence, CP : SP = 4 : 5 SMART APPROACH:-


5 Units = 2000 gm  1200 = x + x% of x
CP SP
1 units = 400 gm
Required Error = 400 gm
10 11 x2
4 5  1200 = x 
100
2. (a) 40 55
 120000 = 100x + x2
Before Raid, 15
Actual profit % = 100  37.5%  x2 + 100x – 120000 = 0
Due to Fault in Machine it reads 40
 x2 + 400x – 300x – 120000 = 0
1 kg when 900 gm is actually
weighted and the shopkeeper also 4. (a)  x(x + 400) – 300(x + 400) = 0
marks 10% above Let the cost price of Aman = 100  (x + 400)(x – 300) = 0
900  1000  1100 units  x = –400 or x = 300
 Negative Value can not be
2 Sold Sold
 Profit  Aman Anuj considered as cost price.
A

9 @10% Loss @12% Loss


  Hence, Cost Price = ` 300
After Raid, 6. (c)
100  90  88% of 90
Gave punishment to sell the  79.2 Let the CP of the article = 100%
goods at 10% discount on cost
ATQ,
price for a month
Selling Price of Anuj = 79.2 units There is a loss of 45% while
Cost price = 10
79.2 units = 554400 3
Selling price = 9 selling the article at of SP.
8
Before the Raid Price 554400
1 unit   7000 Initial Selling Price = 8
20 11 2200 79.2 New Selling Price = 3
=  10   = 27.16
9 9 81 100 units = 700000 Hence, 55% = 3

Aditya Ranjan (Excise Inspector) Selected Selection 17

Downloaded by Kunal Chauhan (kc6970929@gmail.com)


lOMoARcPSD|39315497

Profit & Loss

3 60 10. (d)
 100%   100  SMART APPROACH:-
55 11 90
Cost Price of 1 orange = `5 CP SP
60 18
CP  and SP = 8 8 9
11
Selling price of 1 orange 9 10
Profit = SP – CP
60 88 – 60 28 105 72 90
8–   = `7
11 11 11 15 8
Profit % = 100  25%
Profit = SP – CP = 7 – 5 = 2 72
profit
Profit% =  cost price  100% profit
14. (b)
Profit percentage =  100% CP SP
CP
 28  23 20
  2 4 5
11 
  100%  28  100% =  100% = 40%
 60  5 92 100
  60
 11  11. (d) 8 16
= 46.67% = 47% (Approx) Let the price of 1 kg = ` 1000 Profit% = × 100 = 8 %
92 23
7. (b) MP = ` 1150
Method-1 SP = 90% of 1150 = ` 1035 15. (d)

r
CP SP CP of 750 gm = ` 750 Initial profit = 5%
20 23 SP of 750 gm = ` 1035

si
CP SP
5 6 Net profit = 1035 – 750 = ` 285 20 21
100 138

24840
an by
Swati bought at, Profit% =
285
750
 100  38%
New CP = 21×
23 20

20 23
= 21

n
100 = Rs.18000
138 1
SMART APPROACH:- % increase in CP = ×100% = 5%
No need of doing complete 20
ja
calculation as 24840 is a multiple
CP SP
R s
750 1000 16. (b)
of 9, Answer must be a multiple 100 115
of 9 only option (b) satisfied. 100 90 4
a th

600 828 Gain % =  100% = 20%


Method-2 24 – 4
228
Let the Swati's CP = 100 units P%  100  38%
600 17. (c)
Swati
@15% Profit
Ankita
@20% Profit
Aashi
Cost Price of Swastik = 75000
ty a

100 115 138


= 15 = 23(20% of 115) 12. (a) Loss = 10%
 138 units = 24840 700
 ` 20 100 – L% 
CP of 1 Notebook = Selling Price = CP 
di M

24840 35 100
 1 unit = 138
600 3 90
 100 units = 18000 SP of 1 Notebook =  ` 21 = 75000  = 67500
28 7 100
Hence, Swati's cost price = 18000
8. (c) 3 3 10 Cost Price of Anju = 67500
Profit = 21 – 20 = 1 
Profit = 20%
1 Bicycle
st
2 Bicycle
nd 7 7 7
Selling Price of Anju
1 1 10 1 1
Profit% =   100% = 7 %
10% x%
7 20 7 100  P% 
20% = CP 
13. (d) 100
10  x Case-I
 20 120
2 CP : SP = 16 : 18 = 67500  = 81000
100
=8:9
A

10 + x = 40 18. (a)
Case-II
x = 30 –16% and 8%
CP : SP = 900 : 1000
 Profit on other bicycle = 30%
= 9 : 10 Difference = 8 – (–16) = 24%
9. (d)
So, 24% = 660
SP 9 10 90
 25  30  Final,   =
Net change =  25 – 30 – % CP 8 9 72 660
 100  100% = × 100 = 110 × 25
24
90 – 72
= (–5 – 7.5)% Profit% =  100%
72 112
= – 12.5% 12% profit = 110 × 25 ×
Therefore, price will decrease by 18 100
  100% = 25%
12.5% 72 = 110 × 28 = Rs.3080

Aditya Ranjan (Excise Inspector) Selected Selection 18

Downloaded by Kunal Chauhan (kc6970929@gmail.com)


lOMoARcPSD|39315497

Profit & Loss

19. (a) 24. (d) 28. (d)


According to the question Let CP = x
C.P S.P 100 SP – 100 CP = 40 CP ATQ,
100 110 100 SP = 140 CP  (1100 – x) = 3(x – 700)
 1100 – x = 3x – 2100
80 112 CP 100
  3200 = 4x
2 unit  1000 SP 140  x = 800
To gain 12.5% must be sold at
 C.P of article, 100 unit = 500 40
P% = × 100% = 40% 800
× 100 = Rs.50,000 100 =  9 = Rs. 900
8
20. (d) 25. (d)
29. (d)
Selling an article for Rs 1134 and
CP of the article = x
C.P S.P suffers as much loss as she would
100 110 have gained by selling it at 10% 120
ATQ, (115 – x) = (x – 104) ×
80 112 profit. 100
5(115 – x) = 6(x – 104)
2 unit  1000 1134 575 – 5x = 6x – 624
CP = 10 = 1260
 Earlier S.P = 500 × 110 = 55,000 9 11x = 1199
SP = 1354.5 x = 109
21. (b)

r
CP = 109
50 × SP = 42 × CP (1354.5 – 1260) New SP = 130.8
Profit% = × 100%

si
SP 21 1260 (130.8 – 109)
 Profit% = ×100%
CP 25 = 7.5% 109

Loos% =
an by
21 – 25
 100%
26. (a) = 20%
30. (c)
Let SP = 500

n
25 24% 40%
x 4
Had it been sold at of that price
–4 5
= ja
 100 = –16% 75 : 25
R s
25 there would have been loss of 10%
3 : 1
22. (c) 4
a th

500 × = 400 (New SP)


Difference = 8 – (–16) = 24% 5
24  3  (1 (–40))
=x= 1  Loss
 24% = 660 4
Loss = 10% = ,
100% = 110 × 25 = 2750 = 4x = 72 – 40 10  CP
ty a

New Profit% = x = 8% gain SP = 9


27. (d) 400
 3080 – 2750  CP = 10
di M

=    100% CP SP 9
 2750 
3750 113% of 3750 = 4237.5
New SP = 4237.5 + 607.5 = 4845 CP : SP
330
=  100% = 12% 400
2750 (4845 – 3750) 10 : 500
% profit = 100% 9
23. (b) 3750 4000 : 4500
Difference = (9.5) – (13.5) = 23% 5
23% = 1104 1095 Profit % at initially =
40
× 100%
= 100% = 29.2%
100% = 4800 3750 = 12.5%


A

Aditya Ranjan (Excise Inspector) Selected Selection 19

Downloaded by Kunal Chauhan (kc6970929@gmail.com)


lOMoARcPSD|39315497

Discount

DISCOUNT
03
1. A tradesman marks his goods at #6 # 89 6: & + +D E &
26% above the cost price. He ,- 5 ! " # 5)+++ ,-
allows his customer a discount # ! "
6 ) : 6
of 12% on the marked price, His
+ . ! " # 3< ( +
profit per cent is:
# ;< 6 ! " ! " 7 # (
SSC CPO 09/11/2022 (Shift-02) F
0 1 ’ (
(a) ` 23.50 (b) ` 75.00
! " SSC CPO 10/11/2022 (Shift-01)

r
(c) ` 25.00 (d) ` 76.50
# $ % & ’ ( (a) ` 3,150 (b) ` 3,570
5. A computer marked at ` 18,000 (c) ` 3,200 (d) ` 3,650

si
SSC CPO 09/11/2022 (Shift-01) was sold at ` 15,840. The
8. Find the effective discount for the
(a) 10.88% (b) 11.08% percentage of discount is:
an by scheme-Buy 5 get 3 free.
(c) 10.50% (d) 10% =+++ ,- ; > ?( " % ! "
2. A mixer marked at ` 2,300 is sold 5=/+ ,- 0 1 ! " G5 B & # * 6H

n
for ` 2,024. What is the 7 SSC CPO 10/11/2022 (Shift-01)
percentage of discount? (a) 36.5% (b) 37.5%
ja SSC CPO 09/11/2022 (Shift-03)
R s
)*++ ,- (a) 13.6%
. (b) 21.60% (c) 36% (d) 37%
)+ / ,- 0 1 ( ! " 9. The price of one shirt is `700 in
a th

(c) 12% (d) 21%


& 2 a shop, on which 10% discount
6. In a shop, a discount of 8% is is available for all customers. An
SSC CPO 09/11/2022 (Shift-01) provided. If the total payable after additional 20% discount is there
(a) 13% (b) 10% discount is more than `5,000 and for the customers who pay in
ty a

additional discount of 20% is cash. What is the amount a


(c) 12% (d) 11% customer has to pay for a shirt
provided, then determine the final
3. A person offers a 15% discount amount to be paid by customer, if he pays in cash?
di M

on his garments, and he offers if he buys 12 products each of #6 (


a further discount of 10% on the price ` 750.
reduced price to those customers A++ ) ( %
who pay cash. What will a #6 = ! " # + ( ! " $ I # %
customer have to pay in cash for # ! " 0 # % 6 (
a jacket of ` 3,600? ! ") # #
6 & 5)+++ ,-
. 34 5 + ! " . ! " # ( $ (
# ) $ ( # @ A5+ ,- & % 6
% 6 ) ! " 0 # $@ # B # ) $ SSC CPO
C 10/11/2022
% 6 (Shift-02)
+ . ! " # (* ) ++ & ’ (a) ` 432( (b) ` 504
,- ( " #
SSC CPO 09/11/2022 (Shift-03) (c) ` 523 (d) ` 514
% 6 7
A

(a) ` 6,724 (b) ` 6,624 10. A retailer wants to sell an item


SSC CPO 09/11/2022 (Shift-02) for ` 2,700 after granting a 10%
(c) ` 6,424 (d) ` 6,524 discount. What is the marked
(a) ` 3,240 (b) ` 3,366
7. In a showroom the price of a price of the item?
(c) ` 2,754 (d) ` 3,060
washing machine is ` 65,000. The
4. A shopkeeper allows a discount customer gets cash discount of
J6" : 6
of 15% on a text book with the ` 2,000 and gets a scratch card ! " # )A++ ,- 0 1
marked price ` 100. On request, promising percentage discount of : 6 . 7
he further allows a discount of 10% to 15%. Determine the SSC CPO 10/11/2022 (Shift-03)
10% on the new price of the difference between the least and
goods. What is the overall the maximum selling price of the (a) ` 2,850 (b) ` 3,000
discount given to the customer? washing machine. (c) ` 3,050 (d) ` 3,110

Aditya Ranjan (Excise Inspector) Selected Selection 20

Downloaded by Kunal Chauhan (kc6970929@gmail.com)


lOMoARcPSD|39315497

Discount

11. A shopkeeper gives 20% I. Two successive discounts of


commission on the retail price of 25% and 15%
( 0 S J
<1 :"
one article and earns 5% profit. 5 5 # ! ": " 3 " 0 B # $ ;
If his commission is reduced by ! " # $ 5
II. Successive discount of 35%
10%, then what is his )+++ ,-
and 5%
approximate profit percentage?
*5 5 ! " J . S
(Nearest integral value)
I II .A discount of 38% SSC CPO 11/11/2022 (Shift-03)
#6 # : 6 B 6# (a) ` 21,000 (b) ` 20,000
*= ! "
+ & # 5 % (c) ` 18,900 (d) ` 19,200
Which offer is better for the
(< # $ &
customer? 18. After successive discounts of 10%
+ ( ) $ T E J and0 5%, an article
7 was sold for
% % & 7 K " ` 342. What was the original price
SSC CPO 11/11/2022 (Shift-01) of the article?
2 L M
(a) Either I or III (b) Only I + 5 ! "
SSC CPO 10/11/2022 (Shift-03) (c) Only II (d) Only III ;< : 6 */ ,- 0 1
(a) 18% (b) 10% 15. What is the selling price of a % . S 7
(c) 15% (d) 8% dress that has a marked price of
SSC CPO 11/11/2022 (Shift-03)

r
` 500 and is given a 20% discount
12. Thee cost price of an article is ` (a) ` 400 (b) ` 342
and subsequently a 10%
1,200. On buying four such

si
discount? (c) ` 300 (d) ` 442
articles, the customer gets one
for free. If the selling price of & 19. . A merchant marks up the price
an by
each article is ` 1,800, then
determine the net percentage
(
+ ! "
5++ ,
;
- of his goods
0 #
; by 40% more than

the cost price. He then sells


1

n
profit earned by the shop keeper
+ # ! " # ( 7 5
on selling four articles.
SSC CPO 11/11/2022 (Shift-02) of his goods at a discount of 10%,
: 6 ja ) ++ D-
(a) ` 330 (b) ` 360 half of the goods at the marked
R s
1 : 6 N B # price and the rest of the goods
(c) ` 365 (d) ` 345
: 6 O& 6 # @ at a discount of 20%. Find his
a th

16. A retailer offers the following profit percentage.


: 6 ) = + + Ddiscount
- ) shcemes for buyers on
1 : 6 0 1 # 6item. #
an C /+ J
(< & 6P & % ’ U6# ( : 6 U # 1
ty a

SSC CPO 10/11/2022 (Shift-03) > U ! " V# % + !


5
(a) 12% (b) 20% (i) Two successive discount of ) &
di M

(c) 18% (d) 15% 15% + ! " 0 1 $


13. Rajkumar decides to sell his & ’ (
5 # ! "
watch at a discount of 5%. SSC CPO 11/11/2022 (Shift-03)
However, his brother purchase it (ii) A discount of 20% followed by (a) 20.4% (b) 30.2%
from him and receives an 8% a discount of 10%
(c) 28.8% (d) 14.6%
discount. Raj loses ` 45 in profit + ! " 0 # + 20. Three
! " successive discounts 22%,
as a result of this. what was the
watch's marked price? (iii) A discount of 30% 17% and 11% are equivalent to a
single discount of:
( 6 Q 34 5 *+ ! " ! "
0 1 2 < Under
R which
) $ scheme the selling ) A
% ;< $ ; B # price will be
= maximum? ! " 6
! " ? ; 2 : D( SSC CPO 23/11/2020 (Shift-1)
A

( % /5 ,- 7 (a) Approximately 45%


Q 34 . SSCS CPO7 11/11/2022 (Shift-02) (b) Approximately 50%
SSC CPO 11/11/2022 (Shift-01) (a) ii (b) ii and iii
(c) Approximately 42%
(a) ` 1,500 (b) ` 1,410 (c) i and ii (d) i
(d) Approximately 25%
(c) ` 1,430 (d) ` 1,560 17. Elizabeth went to a furniture store
and purchased a study table. She 21. Two successive discounts, each
14. A supermarket offers the
recieved a 20% discount on it. of x% on the marked price of an
following discount schemes for
She would have saved ` 1,000 article, are equal to a single
buyers.
more if she had received a 25% discount of Rs.331.20. If the
6 <" B # discount. >What was
B the marked marked price of the article is 920,
! " ( E J # price of the sofa? then the value of x is:

Aditya Ranjan (Excise Inspector) Selected Selection 21

Downloaded by Kunal Chauhan (kc6970929@gmail.com)


lOMoARcPSD|39315497

Discount

: 6 26. Three
? successive
@ discounts of 12%, #6 # : 6
13% and 11% are equivalent to an + ! " # &1 9 %
x% # ! ") ** - + ,
approximate single discount of:
! " 00 # : 6 (< # : 6
12%, 13% 11%
X + , x ) , ) $ .
! " > % ! "
’ SSC CPO 09/12/2019 (Shift-01)
0 0 7
SSC CPO 23/11/2020 (Shift-1) (a) Rs. 400 (b) Rs. 320
SSC CPO 25/11/2020 (Shift-1)
(a) 20 (b) 18 (c) Rs. 375 (d) Rs. 350
(a) 32% (b) 40%
(c) 15 (d) 25 31. An article was sold for Rs. 98,496
(c) 42% (d) 35%
22. A single discount equivalent to after providing three successive
27. If two successive discounts, each discount of 10%, 5% and 4% re-
three successive discounts of 8%, of 20% on the marked price of an spectively on the marked price.
15% and 12% is: article, are equal to a single What was the marked price?
8%, 15% 12% ! " discount of Rs. 331.20, then the
marked price (in Rs.) of the article : 6 &
6 ! " ’
is: 5 / ! "
SSC CPO 23/11/2020 (Shift-2) 0 # ?; ,X = ) / X 0 1 #
# : 6
(a) 68.816% (b) 31.184%

r
@ + # ! ;" * *: 6- + . S
(c) 35% (d) 17.5%
, ! " 0 0 SSC
) CPO 09/12/2019
: 6 (Shift-01)

si
23. A single discount equivalent to
K, M ’(a) Rs. 1,20,000
three successive discounts of 6%,

) 5
an by
15% and 14% is:
/
SSC CPO 25/11/2020 (Shift-1)
(a) 645
! " (b) 1,200
(b) Rs. 1,10,700
(c) Rs. 1,20,200

n
6 ! " ’ (c) 920 (d) 750 (d) Rs. 1,20,500
28. On the marked price of Rs.1,250 of 32. After allowing 10% discount on
SSC CPO 24/11/2020 (Shift-1)
ja an article, three successive the marked price of an article, a
R s
(a) 68.714% (b) 34.357% discounts of 5%, 15% and 20% were person makes a profit of 16%. If
(c) 31.286% (d) 17.5% offered. The amount (in Rs.) of the cost price of the article is Rs.
a th

24. On the marked price of Rs.1,250 648, then its marked price is:
discount received by a customer is:
of an article, three successive # : 6 , )) 5+ . : 6
discounts of 5%, 15% and 20% are + ! " # 0 #
offered. What will be the selling 5 ) 5 + ! " # ( )
ty a

# : 6 ,
price (in Rs.) after all discounts? 6 ! " K, M 7
) ; .
: 6 ) 5 SSC
+ ,CPO 25/11/2020 (Shift-2)
di M

SSC CPO 09/12/2019 (Shift-02)


) 5 ) 5 + (a)! 450
" # (b) 807.50
(a) Rs. 826.80 (b) Rs. 835.20
( % ! " 0 (c)
# 950.25 (d) 442.50
(c) Rs. 751.68 (d) Rs. 910.40
K, M ’ 29. On a marked price, the difference
of selling prices with a discount 33. The marked price of an article is
SSC CPO 24/11/2020 (Shift-2)
of 35% and two successive Rs. 530. After two successive
(a) 1,000 (b) 807.50 discounts, it is sold for Rs.
discounts of 20% and 15%, is
(c) 975.75 (d) 950.25 Rs.504. The marked price of the 396.44. If the first discount is
25. On the marked price of an article, article (in Rs.) is: 17% and the second discount is
the sum of selling prices with a x%, then what is the value of x?
) *5 ! "
discount of 35% and two : #6 ,- 5 *#+
successive discounts of 20% and S + 5
15%, is Rs.1,995. The marked ! " 5 + / !, " 0 #, - ; 0* X1 -
# K,( # ! "
A

price of the article (in Rs.) is: : 6


# x%! " )x 0 ;
: 6 *5 M ’
! " 0 # ) S SSC+ CPO 25/11/2020 (Shift-2) SSC CPO 09/12/2019 (Shift-02)
5 # ! " 0 (a)
# 18,000 (b) 16,800 (a) 10.5 (b) 10
J )XX5 , :
(c) 15,500 6 (d) 16,000 (c) 12.5 (d) 12
K, M ’ 30. After allowing a 10% discount on 34. An article is sold for Rs. 680 after
SSC CPO 24/11/2020 (Shift-2) the marked price of an article, a two successive discounts of 20%
dealer makes a profit of 5%. What and x% on its marked price. The
(a) 1,550 (b) 1,800 is the marked price, if the cost marked price of the article is Rs.
(c) 1,500 (d) 1,600 price of the article is Rs. 300? 1,000. What is the value of x?

Aditya Ranjan (Excise Inspector) Selected Selection 22

Downloaded by Kunal Chauhan (kc6970929@gmail.com)


lOMoARcPSD|39315497

Discount

: 6 ; 39. A person
+ bought an article at 30% SSC CPO 11/12/2019 (Shift-02)
x% # ! " 0 # =discount
+ , - on its0marked
1 price. The
person then sold it at 30% profit 1
# ( : 6 +++ (a) 10 (b) 10
for Rs. 427.70. What was the 2
, x ’ ( marked price of the article?
1
SSC CPO 11/12/2019 (Shift-01) . : 6 $ (c) 12 (d) 15
(a) 15 (b) 15.5 2
*+ ! " 0 # B # $ .
(c) 12.5 (d) 16 44. An article is marked at 100%
; *+ % ,- / A-A
0 +1 #above its cost price. After
35. The marked price of an article is
: 6 . S 7 allowing two successive discounts
Rs. 800. A retailer buys it for Rs.
SSC CPO 13/12/2019 (Shift-01) of 5% and 20% respectively on the
540 after getting two successive marked price, it is sold at x%
discounts. The first discount is (a) Rs. 470 (b) Rs. 450
(c) Rs. 500 (d) Rs. 480 profit. What is the value of x?
25%. What is the second discount?
40. The marked price of a working : 6 $
: 6 , = ++ machine is Rs. 7,200. If it is sold for
#6 # ; # ! " Rs. ?5,512.50, after two successive
discount
& O 5 +
0 # 5, / + B # ! " of 5 x% each. The value of x is:
E & & ! " A# ) + + , -0x%# %% ; 0 1
# ! " ’ (

r
; x% @ # ! " ( 0 x# ’ (
SSC CPO 12/12/2019 (Shift-01)
5)5 -5+ ,- x 0 1 (’ SSC CPO 11/12/2019 (Shift-02)
(a) 8% (b) 12%

si
( (a) 48 (b) 75
(c) 10% (d) 15%
(c) 72 (d) 52
36. There is a 15% discount on 8 SSC CPO 13/12/2019 (Shift-01)
an by
shirts marked at
many shirts can
Rs. 9,600. How
be bought with
(a) 12
(c) 10.5
(b) 12.5
(d) 15
45. If a discount of 10% is allowed on
the marked price of an article, a
shopkeeper gets a profit of 25%.

n
Rs. 5,100? 41. The marked price of an article is Rs.
250. After allowing two successive If he offers a discount of 25% on
= ( , )X ) +discounts
+ of 20% and x% on the the marked price of the same
( ja 5 ! " # (, 5 ) + +price, it is sold for Rs. article, then his percentage profit
R s
marked
( B # ( 7 is the value of x?
185.60. What loss will be:
# : 6
a th

SSC CPO 12/12/2019 (Shift-02) : 6 , 5+


(a) 4 (b) 5 x%
+ # ! " # ( ) #6 #
(c) 3 (d) 6 ! " 0 # ), ; =05 -1 + ( % ? # #6
37. A shopkeeper has announced x ’ ( 5 ! " #
ty a

40% rebate on the price of TV sets $ % Y &


SSC CPO 13/12/2019 (Shift-02)
at the time of sale. If a purchaser SSC CPO 12/12/2019 (Shift-01)
(a) 8.4% (b) 7.6%
di M

needs to have a rebate of Rs.


(c) 6.8% (d) 7.2% 1
26,400, then how many TV sets
42. Raghu sold an article for Rs.180 (a) 4% Profit (b) 4 % Loss
each costing Rs. 6,000 should he 4
after allowing a 20% discount on
purchase?
its marked price. Had he not al- 1
#6 # " lowed any" discount he would have (c) 4 % Profit (d) 4% Loss
6
/+ ! " # Q gained
W 220%. What is the# cost price 46. A dealer allows 25% discount of
;< B #, ) /! + "+ ? of the article? the marked price of an article and
1 ) $ Q 6" ": 6 gains 20%.
+ If! the
" cost price of the
B # 1 ( @# 0 # ; =+ ,- 0 article
1 # increases # by
$ 20%, how
much discount percentage should
)+
, ++ 7 ;< ! " # ) $ + %
he allow on the marked price so
SSC CPO 12/12/2019 (Shift-02) ? : 6 ’ (
as to earn the same percentage
(a) 9 (b) 11 SSC CPO 11/12/2019 (Shift-01) of profit as before?
A

(c) 8 (d) 12 (a) Rs. 190.40 (b) Rs. 192.80 : 6


38. A single equivalent discount (c) Rs. 188.60 (d) Rs. 187.50 5 ! " # +
equivalent to three successive 43. A trader buys an article at 80% of
its marked price and sells it at
# : 6 Z P
discounts of 10%, 16% and 25%,
is equal to 10% discount on its marked price. ( ) $
His percentage profit is: & ! " # ( 1
10%, 16% 25% ! "
: 6 ; & ;< <
! " 0 0 7
=+ B # SSC CPO 13/12/2019 (Shift-02)
SSC CPO 12/12/2019 (Shift-02)
+ ! " # 0 # 0 1 (a)
# 8.5% $ (b) 10%
(a) 42% (b) 46.5% (c) 12% (d) 7.25%
(c) 41% (d) 43.3%
% & 7

Aditya Ranjan (Excise Inspector) Selected Selection 23

Downloaded by Kunal Chauhan (kc6970929@gmail.com)


lOMoARcPSD|39315497

Discount

ANSWER KEY
1.(a) 2.(c) 3.(c) 4.(a) 5.(c) 6.(c) 7.(a) 8.(b) 9.(b) 10.(b)

11.(a) 12.(b) 13.(a) 14.(c) 15.(b) 16.(a) 17.(b) 18.(a) 19.(c) 20.(c)

21.(a) 22.(b) 23.(c) 24.(b) 25.(c) 26.(a) 27.(c) 28.(d) 29.(b) 30.(d)

31.(a) 32.(b) 33.(d) 34.(a) 35.(c) 36.(b) 37.(b) 38.(d) 39.(a) 40.(b)

41.(d) 42.(d) 43.(c) 44.(d) 45.(c) 46.(b)

SOLUTIONS

r
si
1. (a) an by 4. (a) When customer get highest
Effective Discount percent scratch card, selling price
CP MP SP
will be minimum

n
100 126 88% of 126 = 110.88  15  10 
=  15  10   % = – 23.5% Least Selling Price
Profit = SP – CP  100  = 65000 – (2000 + 15% of 63000)
ja
= 110.88 – 100 = 10.88 Effective Discount (in `) = 65000 – (2000 + 9450)
R s
= 23.5% of 100 = ` 23.50 = 65000 – 11450
Profit 5. (c)
Profit% =  100% = 53550
a th

CP Discount = MP – SP Difference = 56700 – 53550


= 18000 – 15840 =2160 = 3150
10.88
  100%  10.88% Discount
100 Discount% =  100% SMART APPROACH:-
ty a

MP Difference
SMART APPROACH:- = (15 – 10)% of (65000 – 2000)
2160 = 5% of 63000
Apply Successive % change =  100 = 12%
di M

18000 = 3150
formula for 2 variable
26 12 6. (c) 8. (b)
P% =  26 – 12 –  10.88%
100 Total Cost Price of 12 Article Effective Discounts
2. (c) = 12 × ` 750 = ` 9000 3
First discount = 8% of ` 9000 = ×100%
MP of the Mixer = ` 2300 5+3
= ` 720
SP of Mixer = ` 2024 Payable Amount after discount 3
= ×100% = 37.5%
Discount = MP – SP = ` 9000 – ` 720 = 8280 8
= 2300 – 2024 = 276 Since the payable amount is 9. (b)
greater than ` 5000, other
D 9 4
D%   100% discounts are also applicable. Net Price = 700    Rs.504
MP Other discount = 20% of ` 8280 10 5
= ` 1656 10. (b)
A

276 Final payable amount = ` 8280 – 2700


  100%  12% MP of item =  100 = Rs.3000
2300 ` 1656 = 6624 90
3. (c) 7. (a) 11. (a)
When customer get lowest Method-1
Effective Discount
percent scratch card, selling price
 15  10  will be maximum Commission  20%  5 4
= 15  10 –  % = 23.5% Maximum Selling Price Markup X Y
 100 
= 65000 – ( 2000 + 10% of 63000) Overall profit 5% 20 21
Customer has to pay
= 65000 – (2000 + 6300)
3600  76.5 = 65000 – 8300 20 21
= = Rs.2754 X:Y  : = 16 : 21
100 = 56700 5 4

Aditya Ranjan (Excise Inspector) Selected Selection 24

Downloaded by Kunal Chauhan (kc6970929@gmail.com)


lOMoARcPSD|39315497

Discount

Commission 10%  10 9 15. (b) Number of units sold at MRP


Net Discount %
Markup 16 21 1
= 10 × = 5 units
Overall 160 189  20  10  2
=  20  10   % = – 28%
 100  Number of units sold at 20%
29 discount = 10 – 7 = 3 units
Profit% =  100% SP = (100 – 28)% of 500 = `360
160 16. (a) ATQ, Merchant marks up the price
= 18.125% = 18% Case-I of goods by 40%
Method-2 Effective Discount Profit earned when Merchant
gives 10% discount
MP 100  P% 105 21  15 15 
   =  –15 –15  %  40  20 
CP 100 – D% 80 16  100  Profit =  40 – 10 –  %  26%
 100 
Now, commission is 10% reduced = (–30 + 2.25)% = –27.75% Profit earned when Merchant
MP 18.9 Case-II gives No discount
i.e,  Effective Discount Profit = 40%
CP 16
Profit earned when Merchant
 20  10 
2.9 =  20 –10  % gives 20% discount
P%  × 100 = ~ 18%  100 
16  40  20 

r
= (–30 + 2)% = –28% Profit =  40 – 20 –  %  12%
12. (b)  100 
Case-III
Overall profit

si
CP of 5 article = 5 × 1200 = 6000 Discount = –30%
SP of 4 article = 4 × 1800 = 7200 Lower the discount%, Higher will  2  26  5  40  3  12 
= %
an by be the selling price.  10 
Profit = 7200 – 6000 = 1200
SMART APPROACH:-  52  200  36  288
1200 = % = % = 28.8%
 100% = 20%

n
Profit% = In case I & II we have the sum of both  10  10
6000 the given discounts same. Therefore,
In case II difference = 20 – 10 = 10 will 20. (c)
13. (a) give greater discount as compare to case I. Single discount for 17% & 11%
ja
Let the marked price of watch be 17. (b)
R s
17  11
100% Let the marked price of Study = –17 – 11 +
100
Table be 100x.
a th

Difference in Discount = ` 45 =– 28 + 1.87


3% = 45 Difference in discount
 26.13  26%
= (25x – 20x) = 5x Again, single discount for 26%
1% = 15
 5x = 1000 & 22 %
100% = 1500
 x = 200
ty a

14. (c) Thus, The MP of the table 22  26


= – 26% – 22% +
Case-I = 100 × 200 = 20000 100
di M

Effective Discount SMART APPROACH:- = – 48 + 5.72


 Difference in discounts = 1000
= – 42.28%
 25  15   5% = 1000 Thus, single discount for 22%,
=  25  15  %
 100   100% = 20000 17%, 11% = 42.28%.
18. (a) 21. (a)
= (40 – 3.75)% = 36.25%
Net Discount By option (A)
Case-II Single Discount
Effective Discount  10  5 
=  –10 – 5   %  –14.5% 20  20
 100  = – 20 – 20 + = 36%
 35  5  100
=  35  5  % Original Price of the article 36
 100  Price = 920 ×
SP 100
=  100 %
= (– 40 + 1.65)% = – 38.35% 100 – D = 331.2 (satisfied the question)
22. (b)
Case-III 342
A

= 100 – 14.5  100 % Single discount for 8%, 15%


Single Discount= – 38%  
120
Hence, Best offer is only-II 342 = 8 – 15 +
100
=  100 = Rs 400
Note: Since the sum of discount 85.5 = – 23 + 1.2 =– 21.8%
in I & II are same. therefore Again, single discount for 21.8%
19. (c) & 12 %
discount in greater difference will Let the total goods be 10 units.
give bigger discount to the 21.8  12
Number of units sold at a = – 21.8 – 12 +
custmer. Hence, In case II: 35 discount of 10% 100
– 5 = 30 gives bigger discount to = – 33.8 + 2.616 = – 31.184%
the custmer now, compare II & 1 Thus, single discount for 8%, 15%
= 10   2units
III and get the desired answer 5 and 12% = 31.184%

Aditya Ranjan (Excise Inspector) Selected Selection 25

Downloaded by Kunal Chauhan (kc6970929@gmail.com)


lOMoARcPSD|39315497

Discount

23. (c) 28. (d) 34. (a)


Single discount for 6%, 15% Single Discount for 5% & 15% MP SP
1000 680
90 75 Discount% = 32
= – 6 – 15 + = –5 –15 + = – 19.25%
100 100 20  x
= – 21 + 0.9 ATQ, 20 + x – = 32
Again, single discount for 19.25% 100
= – 20.1% & 20%  100 + 5x – x = 160
Again, single discount for 20.1%  4x = 60
& 14 % 19.25  20 x = 15%
= – 19.25 – 20% +
100 35. (c)
20.1  14 ATQ,
= – 20.1% – 14% + = – 39.25% + 3.85 = 35.4%
100 3 (100 – x )
Thus, single discount for 5% ,  800 ×  = 540
= – 34.1 + 2.814 15% & 20% = 35.4% 4 100
= – 31.286%  600 – 6x = 540
35.4  6x = 60
Thus, single discount for 6%, 15% Net Discount=1250× =Rs. 442.50
and 14% = 31.286% 100 x = 10%
29. (b) Thus, second discount = 10%
24. (b)
36. (b)
SP = MP × (–5%) × (–15%) × (– 20%) Single discount for 20% and 15% Given,

r
19 17 4 300 MP of the 8 shirts = 9600
= 1250 ×   = Rs.807.5 = – 20 – 15 + = – 32% MP of the 1 shirt = Rs. 1200

si
20 20 5 100 Discount = 15%
Note: Complete calculation is not MP × 68% – MP × 65% = 504 85
an by
required, just check which option
is divisible by either 17 or 19.
25. (c)
MP × 3% = 504
MP = 16800
SP of 1 shirt = 12000 ×
= Rs. 1020
100

n
30. (d) Shirts can be bought with Rs 5100
Single discount for 20% and 15%
MP (100  P)% 5100
 = =5
ja300 CP (100 – D)% 1020
R s
= – 20 – 15 + = 32%
100 37. (b)
MP = x MP 105 Rebate on each T.V set
 
a th

ATQ, x × 65% + x × 68% = 1995 CP 90 = 40% of 6000 = Rs. 2400


133% x = 1995 105  300 To get the rebate of total Rs.
MP  = Rs 350 26,400 he has bought the number
90
1995 31. (a) 26,400
ty a

x = × 100 = Rs.1500 of T.V set = = 11


133 2400
26. (a) MP SP 38. (d)
di M

Single Discount for 12% & 13% 10 9


MP SP
20 19
156 10 9
= – 12 – 13 + 25 24
100 25 21
5000 4104
= – 23.44% 4 3
Again, single discount for 23.44% SP = Rs.98496 1000 567
& 11%
98496 Required discount %
23.44  11 MP = × 5000= Rs. 1,20,000
= – 23.44 – 11 + 4104 (1000 – 567)
100 32. (b) = × 100% = 43.3%
1000
= – 34.44 + 2.57 = 31.87%
MP 100  P% MP 116 39. (a)
Thus, single discount for 12%,   
CP 100 – D% CP 90 427.70
13%, 11% = 31.87% CP =  100% = 329
A

27. (c) 648 116 130%


MP = = Rs. 835.20
Single Discount of two successive 90 329
MP =  100% = 470
discount of 20% 33. (d) 70%
400 17 (100 – x ) MP of the article = Rs. 470
= – 20 – 20 + = – 36% ATQ, 530 ×  = 396.44 40. (b)
100 20 100 ATQ,
 53 × 17 × (100 – x) = 79288
36 100 – x 100 – x
MP × = 331.20 79288 7200×  = 5512.5
100  (100 – x) = 100 100
53 17
2
331.2  (100 – x) = 88  100 – x  5512.5
MP =  100 = Rs.920
 x = 12%
  
36  100  7200

Aditya Ranjan (Excise Inspector) Selected Selection 26

Downloaded by Kunal Chauhan (kc6970929@gmail.com)


lOMoARcPSD|39315497

Discount

44. (d) Ratio Method


100 – x 55125 Effective discount
  MP SP CP
100 72000
5  20 10 9
100 – x 7 = 5  20 – = 24%
 100 5 4

100 8 MP = 100% 50 45 36
100 MD affter giving 25% discount on MP
x = = 12.5% SP = 50 – 12.5 = 37.5
8 P% = M – D –
100
41. (d) 1.5 1
ATQ, 100  24 New P% = 100 % = 4 %
360 6
= 100 – 24 –
20  x 100
20 + x –  25.76
100 = (76 – 24) = 52% 46. (b)

x SMART APPROACH:- 20  Profit


20 + x –  25.76 Gain = 20% =
5 CP  100 100  CP
MP  200 SP = 120
4x 19 4
  5.76 SP  200    152 CP = 100
5 20 5
25  Discount

r
4x = 28.80 P%  52
Discount, 25% =
x = 7.2% 100  MP
45. (c)

si
42. (d) SP = (100 – 25) = 75
MP = 100
180
MP = 100% = 225 90 120
80%
MP 100  P%

an by SP = 100 ×
100
= 90 75
100 = 160 (MP)

CP = 100 20%

n
CP (100 – l )% 90 New CP = 120
CP = ×100%= 72
225 (100  20)% 125% Profit = 20%
  SP = 144
CP ja
100% (100 – 25)%
R s
225 120 New SP = 100 × = 75 (160 – 144)
100 Discount% = × 100%
 
CP 100 160
a th

 CP = 187.50 (75 – 72)


Profit% = × 100% 16
43. (c) 72 = 100% = 10%
160
MP 3 1
100 = × 100% = 4 % SMART APPROACH:-
ty a

72 6
80% 10% discount MP SP CP
100  P% 4 3
Note: Here applying 6 5
di M

CP = 80 SP = 90 100 – D% 8 6 5
formula can be tricky becuase this If CP is increased by 20% then CP = 6

P%  90 – 80 100 approach invalues lengthy


6
for the same P% SP = 6 = 7 : 2
5
80 calculation you can solve it taking Required D% =
0.8
100  10%
8
ratios as well.
10 1
 100%  12 %
80 2


A

Aditya Ranjan (Excise Inspector) Selected Selection 27

Downloaded by Kunal Chauhan (kc6970929@gmail.com)


lOMoARcPSD|39315497

Simple Interest

SIMPLE INTEREST
04
1. Sunita invested Rs.12,000 on 4. A sum of Rs.27,000 is divided into 7. A certain sum (in Rs.) is invested
simple interest at the rate of 10% two parts A and B such that the at simple interest at y% per an-
p.a. to obtain a total amount of simple interest at the rate of 15%
1
Rs.20,400 after a certain period. p.a. on A and B after two years and num for 3 years. Had it been
four years, respectively, is equal. 2
For how many years did she invest
to obtain the above amount? The total interest (in Rs.) received invested at (y + 4)% per annum at
together from A and B is: simple interest, it would have
fetched Rs. 4,452 more as inter-
0 B
’ A 8 9
est. What is the sum?
! " #$ % 1 4 : % 8 $ % &’ 3y% 1 !4 5+

r
&’( ) * 15% ! A B
’ ;1 <
1
& ! + ! & +’ , ’ ! + " # $ "%3 2 ! +

si
SSC CPO 23/11/2020 (Shift-1)
#$ % A &’B ’ ( 4
#$ % 3 1 45 6 4( $ % &’(

2.
(a) 6
(c) 9
an by(b) 7
(d) 8
If the annual rate of simple
SSC CPO 24/11/2020 (Shift-2)
(a) 9,600 (b) 5,400
(y + 4)% ! +
$ 9$ &
" #$ %

n
#$ % & ( 12
1 (c) 18,000 (d) 10,800
interest from 11% to 17 % SSC CPO 09/12/2019 (Shift-02)
5. A person invested Rs.12,000 on
ja 6
simple interest for 7 years to
(a) Rs. 42,400 (b) Rs. 31,800
R s
increases a person's yearly income (c) Rs. 30,400 (d) Rs. 42,800
obtain a total amount of Rs.20,400
by Rs.1071.20. The principle 8. A person invested a sum of Rs.
a th

on a certain annual rate of interest.


amount invested (in Rs.) is: 18,600 at x% p a and another sum
What was the rate of interest to
$ ! " #$ % that is twice the former at (x + 2) %
obtain the above amount?
p.a. both at simple interest. If the
1 // $
- . 17 6 % & % &’ $ ** " #$ % total interest0 earned on both in-
ty a

! 4+ 1 4 1 $
! $ 0 -. % vestments
! for 3 years is Rs.
2
di M

&’( 1 21 4 5 6 3 #4$( % $ ( ) * 23,110.50, then the rate of interest


SSC CPO 23/11/2020 (Shift-2) #$ % * $ p.a.&on the
9 ,second investment is:
(a) 19,120 (b) 17,250 SSC CPO 25/11/2020 (Shift-1) /$ * @ >
(c) 10,710 (d) 16,480 x% ! + ? " #$ %
(a) 10% (b) 8%
3. If the annual rate of simple $ ’ &
(c) 9% (d) 7%
interest increases from 11% to (x + 2)% ! + " #$ %
6. A sum of Rs.36,000 is divided into
1 two parts, A and B, such that the 1
17 % then a person's yearly $ ( $ 3 ! 4+ 1 4
2 simple interest at the rate of 15% 2
income increases by Rs.1,071.20. p.a. on A and B after two years
%+ #$ % = 7
The simple interest (in Rs.) on and four years, respectively, is
equal. The total interest (in Rs.) #$ % ! +
the same sum at 10% for 5 years
received from A is: SSC CPO 11/12/2019 (Shift-01)
is:
A

(a) 11%
$ " =>
#$ % ! A 8 9 4 (b) 10.5%
(c) 13% (d) 12.5%
1 ’B 1 4 : 8 % $ %
9. A person invested a sum of Rs.
- . 17 2 % & % &’ & ’/ $ * 7 !A ’B 10,500 at x% per annum at simple
! $ 0 ; -1. %< ! + ’ !interest
+ and a sum of Rs. 13,500
&’( ) 7 ! + ? " #$ % A &at’ ( (x + 2)% p.a. at simple
" #$ % 3 1 45 6 # $ 4%( 3 1 45 6 interest.
4( If the total interest
SSC CPO 24/11/2020 (Shift-1)
earned on both the investments
SSC CPO 25/11/2020 (Shift-2)
for 3 years is Rs. 7,650, then the
(a) 16,480 (b) 9,120 (a) 7,200 (b) 5,400 rate of interest on the first
(c) 7,250 (d) 8,240 (c) 3,600 (d) 1,800 investment is:

Aditya Ranjan (Excise Inspector) Selected Selection 28

Downloaded by Kunal Chauhan (kc6970929@gmail.com)


lOMoARcPSD|39315497

Simple Interest

/$ * 7 x% ! + " #$ % $ 4;1
! + " #$ % ’ = 7 1 @ ! + ? " #$ %
(x + 2)% ! + " ! + 1 ’342 @ ! C+ @1 41$ ) $ % &’(
#$ % ( $ 4 & % 4 &’( 2
! + #$ % 6 ! + &
= ! A 1 4 %+ &’ # $ % %0 (> 7 &’ ’ E$ 1 1 1 4
& #$ % 6SSC CPO % ( &’( E$ ? D (
12/12/2019 (Shift-02)
SSC CPO 11/12/2019 (Shift-02) SSC CPO 13/12/2019 (Shift-02)
(a) 12% (b) 10%
(a) 8% (b) 8.5% (a) Rs. 21,500 (b) Rs. 20,500
(c) 9% (d) 15%
(c) Rs. 20,200 (d) Rs. 18,100
(c) 9% (d) 9.5% 12. Two equal sums (in Rs.) are lent
10. A certain sum (in Rs.) is invested at 8% and 4% simple interest p.a. 1
respectively at the same time. The 14. Sudeep invested of a certain
at simple interest at x% p.a. for 8
5 years. Had it been invested at first sum is received 2 years sum at 5% p.a. for two years and
(x + 5)% p.a., the simple interest earlier than the other and the 3
would have been Rs. 9,200 more amount received in each case is of the sum at 6% p.a. for two
5
than the earlier one. What is the Rs. 14,500. Each sum is:
years and the remaining at 10%
sum? $ 4 3 1 4p.a.
5 ;for1 two< years.
@ If the ’ total in-

r
3x% 1 ! 4 +
5 ! + " #$ % terest received
& is Rs. 1,674 then
the total sum invested is:

si
" #$ % 7 ! + 1$ ) % &’( &
$ 9 $ ( (x$ + 5)%: ! + 2 ! + & & & % 1
8
8 9
#$ %
#$ % &
an by B
$ % &’
C
’ E "$
7 &’( E$ 7
&
! + 6 #$ % ! +

n
& ( 12 D , % ( 3
$ ’ ) 5
8 9 >
SSC CPO 12/12/2019 (Shift-01) SSC CPO 13/12/2019 (Shift-01)
ja
(a) Rs. 36,800 (b) Rs. 36,400 (a) Rs. 12,800 (b) Rs. 12,500 ! + #$ % ! +
R s
(c) Rs. 35,800 (d) Rs. 40,000 (c) Rs. 13,200 (d) Rs. 12,000 $ ’ !
13. Two equal sums are lent at 10%
a th

11. A certain sum of money amounts #$ % ! +


to Rs. 8,928 in two years and to and 8% simple interest p.a. & &# ’$ %
respectively, at the same time.
1 The first sum is received 2 years
9:+
Rs. 10,224 in 3 years, both at
2 earlier than the second one and SSC CPO 09/12/2019 (Shift-01)
ty a

simple interest. The rate of the amount received in each case (a) Rs. 13,000 (b) Rs. 12,000
interest per annum is: was Rs. 36,900. Each sum was. (c) Rs. 10,500 (d) Rs. 12,500
di M

ANSWER KEY
1.(b) 2.(d) 3.(d) 4.(d) 5.(a) 6.(a) 7.(b) 8.(d) 9.(d) 10.(a)

11.(a) 12.(b) 13.(b) 14.(b)


A

Aditya Ranjan (Excise Inspector) Selected Selection 29

Downloaded by Kunal Chauhan (kc6970929@gmail.com)


lOMoARcPSD|39315497

Simple Interest

SOLUTIONS
1. (b) 3 unit = 36000 11. (a)
Total SI = 20400 –12000 = Rs. 8400 A= 2 unit = 24000
SI for 1 year = 10% of 12000 3
15  2 SI for years = 10224 – 8928
= Rs. 1200 SI on A =24000 × = 7200 2
100 = 1296
8400
Time = = 7 years 7. (b)
1200 1296
SI for 2 years =  2 2
2. (d) 1 3
Time = 3 year
1 2 = 1728
Rate difference = 17 % – 11% Rate = y% Principal = 8928 – 1728 = 7200
2
Had it been invested at (y + 4)% 100 1728
1 R= = 12%
= 6 % per annum at simple interest 7200  2
2 It fetched Rs more = 4452 as 12. (b)

r
2 interest Time for the 1st sum be T years
Principle, 100% = 1071.2   100
13 Time for the 2nd sum be (T + 2)

si
= Rs. 16480 4452
sum = 100% years
3. (d) 7
4 ATQ,
an by 1
Rate difference = 17 % – 11%
2
2

4452  2 100
8T% of P = 4(T + 2)% of P
8T = 4T + 8
T = 2 years

n
= = Rs. 31800
1 47 Now,
= 6 %
2 8. (d) Amount = 14500, R = 8%,
ja Total interest T = 2 years
R s
2
Principle, 100% = 1071.2   100
13
 7x   7   14500
= Rs. 16480     2 
   (x  2) % of 18600 Principal = ×100%
a th

=   116%
Now,  2   2 
= Rs. 12500
16480  10  5  7x   13. (b)
SI = = Rs. 8240
100 = 


    (7x  14) % of 18600
 R  10% 8%
 2 
ty a

4. (d)
T 8 10
15 4  15  21x + 28  80 80
ATQ, A × 2 × =B× = 
 
 % of 18600
di M

100 100  2 
A 2 36900
  ATQ, (21x + 28) of 93 = 23110.5 Each sum = (100  80)% × 100%
B 1 21x + 28 = 248.5
3 unit = 27000 x = 10.5 = Rs 20500
A, 2 unit = 18000 14. (b)
Rate of second year = (x + 2)%
B, 1 unit = 9000 Let, sum = 40
= 12.5%
Total Interest, Case-I
9. (d)
2  15 4  15 552 1
= 18000 × + 9000 × 7650 SI = 
100 100 100 2
Interest in 1 year = = 2550
= 5400 + 5400 = 10800 3 Case-II
5. (a)
Total SI of 7 years = 20400 – 12000 10500  x  1 13500  (x  2)  1 24  6  2 72
  SI = 
= 8400 100 100 100 25
Case-III
A

8400 = 2550
SI of 1 year = = 1200  105x + 135x + 270 = 2550 11  10  2 11
7 SI = 
 240x = 2280 100 5
12000  1  R
1200 =  x = 9.5%
100  1 72 11  279
R = 10%
10. (a) Total SI =     =
 2 25 5  50
6. (a) 9200
Required Sum = 100% Total interest = Rs 1674 (given)
A  15  2 B  15  4 (5  5)%
ATQ,  50
100 100 Invested sum = 1674 × × 40
9200 279
A 2 = 100% = Rs. 36800
  25% = Rs.12000
B 1

Aditya Ranjan (Excise Inspector) Selected Selection 30

Downloaded by Kunal Chauhan (kc6970929@gmail.com)


lOMoARcPSD|39315497

Compound Interest

COMPOUND INTEREST
05
1. Find the difference between the 4. Find the difference between the (a) 8% (b) 12%
simple interest and the compound interest and simple (c) 20% (d) 15%
compound interest payable interest when a sum of ` 15,625 8. A sum amounts to Rs.7,562 in 4
annually on a sum of ` 6,500 at is invested for 3 years at 4% per years and to Rs.8,469.44 in 5 years,
7% per annum for 3 years. annum. at a certain rate percent per annum
(Correct to two decimal places.) 5
8 6 A ( & is compounded
when the interest
B 7 ) ’ yearly.
) /If Rs.10,000
) at the same
0 "# $ % rate of interest is borrowed for
! "# $ % & ’5 " ( ) * ) two years, then what will be the
&+

r
"# $ % ) * ) SSC CPO & +10/11/2022 (Shift-03) compound interest (in Rs.)?
, ( -. ’ ) (a)/ ` 760 1 5 = ’ D "# $
(b) ` 96

si
&+2 / ) / ) &
(c) ` 56 (d) ` 86
SSC CPO 09/11/2022 (Shift-02) 5. On a certain sum of money lent D 4
an by
(a) ` 94.34
(c) ` 98.73
(b) ` 97.78
(d) ` 95.67
out of 16% per annum, the
differnce between the compound
5

? B E8
6

n
interest for 1 year, payable half- 9 A 3 (& (
2. A man borrowed ` 50,000 from a yearly, and the simple interest for
bank at 10% per annum, 1 year is ` 60. The sum is "# $ % = ’ ,+
ja
compounded annually. At the end SSC CPO
= C25/11/2020 (Shift-1)
R s
of every year, he pays ` 15,000
6 7 ) 9
’ ") (a) 2,544 ! (b) 2,764
as part payment of the loan and
a th

(c) 1,965
interest. How much does he still 6 "# $ %
9.
6 (d) 1,736
If the difference between the compound
owe to the bank after three such 5 " interest and ) simple interest at 17%
installments? 8 / + 0 =C ) ’ of/ money
>
on a sum for 2 years
& 3
4) ’ 5 6 7SSC) (compounded annually) is Rs.433.50,
ty a

CPO 11/11/2022 (Shift-02)


8 9 ( (a) ` 937.5 (b) ` 9,000 then the sum (in Rs.) is:
! "# $ % / )(c) `/ 9,365
+ 7: (d) ` 9,375 ’ 6
di M

) / 9 6. A sum becomes ` 7436 in two 7 F) "# $ %


; <=) ’ ! years
8 when
6 interest is compounded 5 " ) B 8 / ,
annually at the rate of 4% per = ’ D "# $ %
; <=) ’ ) / + & ) ’ )
annum. What was the sum?
5 0 5 )’ , 2 * ) +
&’ B / )
; <=) ’ ’ 5 / > SSC CPO 23/11/2020 (Shift-2)
/ 5 B 7 )
SSC CPO 10/11/2022 (Shift-01) (a) 12,000 (b) 25,000
! ) / ) / + 4
(c) 15,000
. > (d) 20,000
(a) ` 16,900 (b) ` 15,800 SSC CPO 11/11/2022 (Shift-03) 10. If the difference between the
(c) ` 15,200 (d) ` 16,200 (a) ` 7,075 (b) ` 6,275 compound interest and simple
3. Find the amount if a sum of ` (c) ` 6,575 (d) ` 6,875 interest at 17% on a sum of money
7500 is invested on compound 7. A sum amounts to Rs.7,562 in 4
interest at 8% p.a. for 1.5 years for 2 years (compounded annually)
years and to Rs.8,469.44 in 5 years is Rs.433.50, then the compound
A

when the interest is compounded at a certain rate per cent per


half yearly. interest for same time (in Rs.) is:
annum, when the interest is
8 " # $compounded
% yearly. The rate of ’ 6 D
? 7 ) interest is:
68 , D "# $ % 2
(& ’ ) / 5 5 = ’ D " #( &$ " %
# $ %
! "# $ % ) / / ) / ) C ) B 8 / )
) @ ’ * ) &+ ’ ") D B (& "# $ % , 2
SSC CPO 10/11/2022 (Shift-02) A SSC CPO
? B24/11/2020
E8 (Shift-1)
(a) ` 8,436.48 (b) ` 8,736.48 BB / ) / + * (a)) 2,500 + (b) 2,735.50
(c) ` 8,657.48 (d) ` 8,564.48 SSC CPO 23/11/2020 (Shift-1) (c) 5,533.50 (d) 5,100

Aditya Ranjan (Excise Inspector) Selected Selection 31

Downloaded by Kunal Chauhan (kc6970929@gmail.com)


lOMoARcPSD|39315497

Compound Interest

11. The simple interest on a sum of 14. A sum of money becomes Rs. & ’ ") ’ (&
money at 10% per annum for 2 35,680 after 3 years and Rs. ’ ") J
years is Rs.8,100. Compounded 53,520 after 6 years at a certain 7 F) / ’ ( "# $ %
annually, what would be the rate percentage p.a. interest 1
compound interest (in Rs.) on the compounded yearly. What is the 8 22 / + /
same sum for the same period at compound interest on the same
the same rate of interest?
’ 0 )’ /
sum in the first case? (Your
! "# $ %
’ 6 D A
answer should be nearest to
SSC CPO 11/12/2019 (Shift-2)
G integer)
? 6 / + (a) Rs. 37,215 (b) Rs. 36,300
= ’ D " # $ &% ’ " ) ’ ! (c) Rs.
" # $ % (d) Rs. 36,000
38,115
) /<& ’ 17. A sum of Rs. 5,000 amount to Rs.
&’ ’ ( ’ ") 7,200 in 8 years at a certain rate
’ 7 F) "# $ % * ) 5 8 ? percent p.a. interest
5 compounded
+ 8 A / ) / + /( yearly.
( What C will be the
SSC CPO 24/11/2020 (Shift-2) compound interest on a sum of
’ "# $ % * )Rs. 6,550& in
+ 4 years at the same
(a) 8,100 (b) 8,505
, 01 J 5 rate’ ofHinterest?
/ ’
(c) 8,715 (d) 9,000
8 & ’ & ’ ")

r
12. The simple interest on a sum of " /&2
money at 10% per annum for 2
7 ) ) ? 8/ )
SSC CPO 09/12/2019 (Shift-2)

si
years is Rs.8,100. What would be / C B8
(a) Rs. 11,983 (b) Rs. 11,894 ’ )’ "# $ % 7
the compound interest (in Rs.) on
an by
the same sum for the period at
15% p,a, when the interest is
(c) Rs. 10,842
15. What is the compound interest on
(d) Rs. 11,938 SSC CPO 12/12/2019 (Shift-1)
(a) Rs. 1,290 (b) Rs. 1,285

n
compounded yearly? (c) Rs. 1,415 (d) Rs. 1,310
(nearest to Rs.1) 1 18. The compound interest on a
a sum of Rs. 37,500 for 1 years certain sum invested for 2 years
ja ’ 6 D A 3
at 10% per annum is Rs.
R s
G at?a rate
6 of /12%
+ p.a. if the inter- 1,522.50, the interest being
est is compounded 8 monthly?
compounded yearly. The sum is:
a th

= ’ D "# $ %
8 ’ 6A (& 6 "
) /<& ’ ’
(& 6 "# $ % 1 ’ =C & ’ ")
"# $ % 1 )’
3 7 F) / ’ ( "# $ %
, 2 * ) +
ty a

"# $ % 7 F) / = / 5 ? ! "#
, ’ H) F 6 ) 2
"# $ % ) / + J( ’ * ) &+
SSC CPO 25/11/2020 (Shift-2)
di M

SSC CPO 11/12/2019 (Shift-1) SSC CPO 13/12/2019 (Shift-1)


(a) 13,061 (b) 14,671 (a) Rs. 7,250 (b) Rs. 7,200
(a) Rs. 6,440 (b) Rs. 6,240
(c) 8,100 (d) 12,751 (c) Rs. 7,500 (d) Rs. 7,000
13. At what percentage rate compound (c) Rs. 6,420 (d) Rs. 6,448 19. A sum of Rs. x amount to Rs.
interest compounded annually for 16. The compound interest amounts 12,777.60 in 2 years at 15% p.a.,
a sum of Rs. 40,000 will amount on a certain sum at a certain rate when the interest is compounded
to Rs. 44,100 in two year? eight-monthly. The value of x is.
percentage p.a. for the second
B & ’ year
I and
) third
) year are Rs. 3,300
8x & ’ 6
BB 6and Rs.
/ 3,630 respectivley.
&= / A 8 6A 8
! "# $ % What is )the>amount of the same
5 LM
)x / + ’ * ) &+
SSC CPO 09/12/2019 (Shift-1) 1
sum at the same rate in 2 years SSC CPO 13/12/2019 (Shift-2)
(a) 5 (b) 2 2 (a) Rs. 9800 (b) Rs. 10400
A

(c) 4 (d) 7.5 interest compounded yearly? (c) Rs. 10200 (d) Rs. 9600

ANSWER KEY
1.(b) 2.(a) 3.(a) 4.(a) 5.(d) 6.(d) 7.(b) 8.(a) 9.(c) 10.(c)

11.(b) 12.(a) 13.(a) 14.(b) 15.(b) 16.(c) 17.(a) 18.(a) 19.(d)

Aditya Ranjan (Excise Inspector) Selected Selection 32

Downloaded by Kunal Chauhan (kc6970929@gmail.com)


lOMoARcPSD|39315497

Compound Interest

SOLUTIONS
1. (b) 4. (a) 8. (a)
Difference between SI and CI for 2
Difference = 8469.44 – 7562
3 yrs (d)  r   r  = 907.44
D = P  3  
2  100   100 
 r   r  907.44
= P  3   Rate% =  100% = 12%
 100   100   4   4 
2
7562
= 15625   3   Now,
 7  
2
7   100   100 
= 6500   3   P = 10000, R = 12%, T = 2 years
 100   100  16 304
= 15625 ×  = Rs.76
49 307 10000 100  12 
2

= 6500 ×  = 97.78  10,000 1   – 1
100 100 5. (d)  100  
Simple Interest for 1 year = 16%
SMART APPROACH:-

r
Effective Rate for SI for 3 year  88 212 12
Rate for CI   8  8   10,000   = 2544
= 3R% = 3 × 7% = 21% % 100 100
 100 

si
Effective Rate for CI for 3 year 9. (c)
= 3R.3R²|R³ = 16.64%
an by Difference = (16.64 – 16)%= 0.64%
= 21.147 343 17²
 2.89%
 0.64%  60 Difference =
= 22.5043 100
Difference Rate%

n
60 433.5
= (22.5043 – 21)% = 1.5043%  100%   100 = ` 9375 Sum, 100% = × 100
Difference = 6500 × 1.5043% 0.64 2.89
ja
= 97.7795 = Rs.97.78(Appx) Hence, The lent sum = `9375 = Rs.15000
R s
2. (a) 6. (d) 10. (c)
Given, A = 7436
a th

P = 50000 t = 2 year 17²


Difference =  2.89%
T = 3 year r = 4% 100
R = 10% t
I = 15000  r  433.5
A  P 1   Sum, 100% = × 100
ty a

1  100  2.89
We know that, 10% = = Rs.15000
10 2
The amount he owes at the end  4 
di M

 7436  P 1   17 17
of 1st year  100  Net CI = 17  17   36.89%
100
11 2
CI = 15000 × 36.89% = Rs. 5533.50
= 50000 × – 15000  26 
10  7436  P  
 25  11. (b)
= 55000 – 15000 = Rs. 40000 SI for 2 year = 20%
The amont he owes at the end 676P
of 2nd year–  7436  CI for 2 year = 21%
625
11 8100
= 40000 × – 15000 7436  625 CI =  21 = 8505
10 P 20
= 40000 – 15000 = Rs. 29000 676
The amount he owes at the end 12. (a)
 P  6875 P=?
of 3rd year–
11 SMART APPROACH:- P  10  2
= 29000 × – 15000 1 8100 =
A

10 We know, 4%  100
= 319000 – 15000 = Rs.16900 25
3. (a) P = 40500
 25 2 CI of 2 years at the Rate 15%
8 Principle  7436 
 26 

Rate =  4%  
2 15
1 = Rs.6875 I = 40500 × = 6075
Time = 18   3 100
6 7. (b)
Difference = 8469.44 – 7562 II = 6075 + 911.25
1
4%  Total CI = 6075 × 2 + 911.25
25 = 907.44
26 26 26
= 12150 + 911.25
Amount=7500×   907.44 = 13061.25
25 25 25 Rate% =  100% = 12%
= 8436.48 7562  13061

Aditya Ranjan (Excise Inspector) Selected Selection 33

Downloaded by Kunal Chauhan (kc6970929@gmail.com)


lOMoARcPSD|39315497

Compound Interest

SMART APPROACH:- (3630 – 3300) 36  r 


4

Rate =  100%  
1  

SI for 2 year at 10% = 20% 3300
CI for 2 year at 15% = 32.25% 25  100 
8100 4
CI  32.25 = Rs.13061.25 330  r  6
20 =
3300
× 100% = 10%   1  
 
 100  5
13. (a) CI on a sum of Rs 6550 in 4 year
3300
44100 21 CI for first year = × 100 1
 ...(1) 110 at same rate= 6550 × = Rs.1310
In one year = 5
40000 20 = Rs. 3000
SMART APPROACH:-
1 3000 t=8
R% =  100 = 5% Principal =  100% = 30,000 P A
20 10%
5000 7200
14. (b) 1 25 : 36
P = 30,000,R = 10%,T = 2 year t=4
3 yrs 3 yrs 2
Interest compounded yearly 5 6
P 35680 35680 53520 1
10% at 2 year successive = 21% CI  6500   Rs.1310
5
35680 21  5
P = 35680  = Rs. 23786 18. (a)

r
53520 Now, 21 + 5 + = 27.05
100 Using Sucessive Method,
C.I for 3 yrs.= 35680 – 23786

si
27.05 Net CI for 2 Years at10% = 21%
= Rs. 11894 CI = 30,000 × = 8115
100 1522.5
15. (b) an by 16m
2
Amount = 30000 + 8115 = Rs.38115
17. (d)
Sum=

19. (d)
21%
× 100% = Rs. 7250

n
New rate = 8%, cycle =
8m P = 5000, A = 7200, t = 8 years Amount = 12777.6
 r 
t Interest compounded eight
ja 88 1 
A = P  monthly
Net CI = 8 + 8 + = 16.64% 
R s
100  100 
New rate = 10%
8
CI = 37500 × 16.64% = 6240  r  Time = 3 cycle
a th

 7200 = 5000 
1  

16. (c)  100  Net CI for 3 cycle at 10% = 33.1%
8
CI for 3 years = 3630 7200  r  12777.6
 
1  
 Sum = × 100% = Rs 9600
CI for 2 years = 3300 5000  100  133.1
ty a


di M
A

Aditya Ranjan (Excise Inspector) Selected Selection 34

Downloaded by Kunal Chauhan (kc6970929@gmail.com)


lOMoARcPSD|39315497

Ratio & Proportion

RATIO & PROPORTION


06

1. If p : q = 4 : 5, q : r = 3 : 4 and r 5 ( ! (
: s = 2 : 5, then p : s is: 5 1 6
p : q = 4 :5, q : r = 3 : 4 r "# 5 /. 0 ? ; 3(
:s=2:5 p: s
" 7 ( !<B ;C ? 7 (
(
SSC CPO 09/11/2022 (Shift-01)
SSC CPO 09/11/2022 (Shift-02) SSC CPO 10/11/2022 (Shift-03)
(a) 2 : 5 (b) 6 : 25 (a) Rs. 900; Rs. 500 (a) 32 : 24 (b) 32 : 21
(b) Rs. 400; Rs. 500

r
(c) 5 : 2 (d) 25 : 6 (c) 32 : 22 (d) 32 : 23
2. Four numbers are in the ratio 1 (c) Rs. 700; Rs. 900
9. Which of the following is the

si
: 2 : 3 : 5. Two time their sum (d) Rs. 400; Rs. 700 lowest ratio?
is 44. Find the ratio in the 5. Which of the following is the 8 : 7
an by
numbers obtained by substracting
3 from the cube of each number.
smallest ratio
8 3 7 9 %
SSC CPO 11/11/2022 (Shift-02)
(a) 3 : 22
:
(b) 23 : 29

n
SSC CPO 10/11/2022 (Shift-01)
(c) 17 : 5 (d) 5 : 19
(a) 41 : 92 (b) 33 : 41 10. The ratio of first and second sum
!! "#
ja (c) 42 : 49 (d) 35 : 47 of money is 7 : 8. If the first sum
R s
$ $ %6. The "monthly
& incomes of Ravi and is 35, the second sum is:
’ ( ) Shiv are in the ratio 1 : 2 and 3( + ( .
a th

their monthly expenditures are < 3( .


SSC CPO 09/11/2022 (Shift-01)
in the ratio 1 : 3. If each saves
(a) 2 : 31 : 204 : 999 Rs. 4,000 per month, then find
.
the monthly income of Shiv. SSC CPO 11/11/2022 (Shift-03)
(b) 2 : 13 : 208 : 992
ty a

; . ; ( (a) 56 (b) 40
(c) 11 : 67 : 219 : 1003
(c) 45 (d) 44
5
(d) 5 : 61 : 213 : 997
di M

11. The salaries of A, B and C are in


"# "
3. The cost of two varieties of tea ! /// 0 ( 7 3 6 4
ratio of : : . The salary of A
is Rs. 300 and Rs. 375 2 5 3
. ; ( ’
respectively. If both the varieties and B together is Rs. 40,500. By
SSC CPO 10/11/2022 (Shift-02)
of tea are mixed together in the what percentage is the salary of
(a) Rs. 16000 (b) Rs. 18000
ratio 3 : 2, then what should be A more than that of C?
the price of mixed variety of tea (c) Rs. 14000 (d) Rs. 20000
7. The price of a bat and a ball are in 3 6 4
per kg? A, B C ; : :
the ratio 9 : 5. The price of the bat 2 5 3
( * +, -is Rs.
. 380/ more
/ than the price of A B ; 3
0 1 0 the ball.
( Find the price of the ball. !/ // 0 A ; C ;
* 2 7,3 ( ( 6 " . =
7,3 ( SSC( CPO 10/11/2022 (Shift-03)
A

3 ) 4 * (
( ( 0 </ = (a) 12% (b) 13%
+, " "
( ( ’ (c) 11.5% (d) 12.5%
SSC CPO 09/11/2022 (Shift-01) SSC CPO 10/11/2022 (Shift-02) 12. The mean proportional between
(a) Rs. 340 (b) Rs. 330 (a) Rs. 470 (b) Rs. 485 0.04 and 0.36 is:
(c) Rs. 350 (d) Rs. 360 (c) Rs. 475 (d) Rs. 480 /D/! /D B 7( E
4. The monthly income of two 8. The ratio of a father's age to his
persons is the ratio 4 : 5 and their son's age is 3 : 2. The product
SSC CPO 09/11/2022 (Shift-03)
expenditures are in the ratio 7 : of the numbers representing their
age is 486. The ratio of their ages (a) 0.144 (b) 1.2
9. If each saves Rs. 50 per month,
then their monthly incomes are: after 5 years will be: (c) 12 (d) 0.12

Aditya Ranjan (Excise Inspector) Selected Selection 35

Downloaded by Kunal Chauhan (kc6970929@gmail.com)


lOMoARcPSD|39315497

Ratio & Proportion


13. If X, Y, Z and W are in fourth (a) 2,300 (b) 3,100 ( ; L 3 ; L M2
proportion. X = 3, Y = 13 and W
(c) 2,000 (d) 2,800 5
= 39, then the vlaue of Z. K 3NO
19. A sum of Rs.8,200 was divided 12
X, Y, Z W 2 ? among A, B and C in such a way
X = 3, Y = 13 W = 39 Z that A had Rs.500 more than B 4
’ ( ) 3NO 3NO 7
( 3
and C had Rs.300 more than A.
SSC CPO 10/11/2022 (Shift-02) How much was C’s share (in Rs.)? 3NO ( ! ;C ?
(a) 5 (b) 20 < // 0 ( A, B . C 2
(c) 15 (d) 9 J " ;K ) A 3NO ( 3
5
3NO
14. The ratio of the third proportional B // 0 C= A
to 16 and 40 and the mean ( ! ;C ? =
proportional between 10 and 40
// 0 =C 3 * H0 ! ;C ? ; 3 ;
is: I ’ 3 / ; L 3
B !/ F ( ( 2 SSC CPO
/ 24/11/2020
!/ (Shift-1) ( 3 ’
7( E ( (a) 2,300 (b) 2,000 SSC CPO 25/11/2020 (Shift-2)
SSC CPO 11/11/2022 (Shift-01) (c) 2,800 (d) 3,100 (a) 1900 (b) 1820
(a) 5 : 1 (b) 1 : 5 20. The present ages of A and B are in

r
(c) 1290 (d) 1920
(c) 4 : 1 (d) 1 : 4 the ratio 3 : 4. Twelve years ago,
23. The ratio of two numbers is 3 : 5.

si
15. The third proportional of 0.024 their ages were in the ratio 2 : 3.
If eight is added to the first, and
and 0.12 is: The sum of the present ages of A
seven to the second, then the ratio
/D/ ! an by /D F ( and (B (in years) is:
becomes 2 : 3. What will be the ratio
SSC CPO 11/11/2022 (Shift-01) A B ( ; ? 3: become if six is added to each?
(a) 0.060 (b) 6.000 4

n
7 ;C ? +;? (
(c) 0.600 (d) 0.006 2:3 2 A B ( ; ? G + (
16. The mean proportion of 0.03 and A3 H;C ? I ’
0.27 is:
ja )
R s
SSC CPO 24/11/2020 (Shift-2) ) NO )
/D/ /D 1 E ( ’ ( )
(a) 72 (b) 60
a th

SSC CPO 09/12/2019 (Shift-01)


SSC CPO 11/11/2022 (Shift-03)
(a) 0.09 (b) 0.06 (c) 84 (d) 48 (a) 9 : 14 (b) 5 : 7
(c) 0.03 (d) 0.15 21. Seven years ago, the ratio of the (c) 5 : 9 (d) 7 : 9
ages of A and B was 4 : 5. Eight 24. The ratio of the incomes of A and
17. Seven years ago, the ratio of the
ty a

years hence, the ratio of the ages B is 3 : 5, whereas the ratio of their
ages of A and B was 4 : 5. Eight
of A and B will be 9 : 10. What is expenditure is 4 : 7 respectively.
years hence, the ratio of the ages
the difference between their
di M

of A and B will be 9:10. What is If A and B save Rs. 16,000 and Rs.
present ages in years? 26,000 respectively, then what is
the sum of their present ages in
years? ; C ?A 3B ( the difference (in Rs.) between
4:5 2 G A; C B? 7 their expenditure?
; C A? B3 (
! 2 GA ; CB ? ( 7 ( 9 : 10 ( A B (
6 / ( ; ; ?? )
H;C ?7 I5
- .
SSC CPO 25/11/2020 (Shift-1) A - . B
B /// 0D
A3 H;C ? I ’
(a) 4 (b) 2 B /// 0D ( 7
SSC CPO 23/11/2020 (Shift-1)
7( H0D I ’
(a) 41 (b) 82 (c) 3 (d) 6
SSC CPO 11/12/2019 (Shift-01)
(c) 32 (d) 56 5
22. In a school, of the number of (a) 6800 (b) 5400
18. A sum of Rs.8,200 was divided 12
among A, B and C in such a way (c) 5000 (d) 6000
A

students are girls and the rest are


that A has Rs.500 more than B, 25. The total number of students in
and C has Rs.300 more than A. 4 sections A and B of a class is 72.
boys. of the number of boys are
How much was A’s share (in Rs.)? 7 The ratio of the number of stu-
dents in A and B is 7 : 5. The aver-
< // 0 ( A, B . C 2
below 14 years of age, and of age weight (in kg) of the students
7( J " ;K ) ) 5 in section B is 20% more than that
A B // 0 C= the number of girls are 14 years of the students in section A. If the
A // 0 = A
" & or above 14 years of age. If the average weight of the students in
number of students below 14 years the class is 52 kg, then what is
* H0 I ’ of age is 1120, then the total number the average weight (in kg) of the
SSC CPO 23/11/2020 (Shift-2) of students in the school is: students in section B?

Aditya Ranjan (Excise Inspector) Selected Selection 36

Downloaded by Kunal Chauhan (kc6970929@gmail.com)


lOMoARcPSD|39315497

Ratio & Proportion

P A ; R 228. ? Two times


;B Q ( of A’s salary is five (a) 8 (b) 6
3 A
1 9 > B times
( of B’s salary and four times (c) 12 (d) 9
of B’s salary is twice C’s salary.
1 B ; ?
What is A’s salary if C’s salary is 31. The average age of husband, wife
;R 2 ? A; ) H Rs. 1,600?
" I ; ?
and child 7 years ago was 42 years
;R 2 ? ; )A ; / S =B ; and that of wife and child 9 years
P K ( ;R 2 ? 7 B7 ; ago was 36 years. The present age
;) " B ;; R? 2 ? C ; C 7 7 of the husband is:
;) H " I ; B// 0 AD ; 7 ;C ? 3 # ( 7
SSC CPO 11/12/2019 (Shift-01) SSC CPO 12/12/2019 (Shift-02)
(a) 58.2 (b) 57.9 (a) Rs. 2,000 (b) Rs. 2,500 ! ;C ? 2 ( 6 ;C
(c) 57.6 (d) 56.4 (c) Rs. 2,600 (d) Rs. 3,000 7U ( B ;C ?
29. A sum of Rs. 12,000 is divided
26. Seven years ago, the ages (in ( (
between A, B, C and D such that
years) of A and B were in the the ratio of shares of A and B is SSC CPO 12/12/2019 (Shift-02)
ratio 4 : 5 and 7 years hence, 8 : 9, that of B and C is 2 : 3 and
their ages will be in the ratio 5 : (a) 50 (b) 55
that of C and D is 9 : 13.
6. What will be the ratio of their

r
What is the difference between (c) 48 (d) 57
ages 5 years from now? the shares of B and D?
32. A vessel contains a solution of

si
; C A? B3 ( H;C Q / I/ / 0 D ( A, B, = C .D two liquids A and B in the ratio 5
! 2 ) 1 ; C 7?( 7 J ( ;K ) A ( ) : ( 3. When 10 litres of the

7
an by (
B )
C
B7 *; C ?
C
B
D2
< 6solution is taken out and
replaced by the same quantity of

n
SSC CPO 12/12/2019 (Shift-01) 6 B D " & B, the ratio of A and B in the
(a) 34 : 41 (b) 33 : 40 ; 3 * ’ ( ) vessel becomes 10 : 11. The
ja SSC CPO 13/12/2019 (Shift-01)
R s
(c) 33 : 34 (d) 31 : 33 quantity (in litres) of the
(a) Rs. 1,320 (b) Rs. 2,400
27. The salaries of Raju and Peter are solution, in the vessel was.
(c) Rs. 2,520 (d) Rs. 2,760
a th

in the ratio 3 : 5. If the salary of 30. If x is subtracted each of the 7 ? A 3B 2 Q


each is increased by Rs. 2,500, numbers 20, 37, 54 and 105, then
then the new ratio is 11 : 15. ;3 )
the numbers so obtained in the
What is Peter’s salary? / 3(% ;3 3
order are in proportion. What is
ty a

)+ (% ; the mean proportional between ( >B 3 )


"# ; / / (7x
0D– 5) and
( ;(xF+ 1)
T?
di M

A B / )
( ) / 1 ! / "#
7 ? ;3 ( > H3(
(% ; x $ % ) J " &
SSC CPO 13/12/2019 (Shift-02)
SSC CPO 12/12/2019 (Shift-02) (7x – 5) ( (x + 1)
E ’ ( ) (a) 44 (b) 42
(a) Rs. 3,500 (b) Rs. 2,500
(c) Rs. 4,000 (d) Rs. 5,000 SSC CPO 09/12/2019 (Shift-02) (c) 48 (d) 52

ANSWER KEY
A

1.(b) 2.(d) 3.(b) 4.(b) 5.(a) 6.(a) 7.(c) 8.(d) 9.(a) 10.(b)

11.(d) 12.(d) 13.(d) 14.(a) 15.(c) 16.(a) 17.(a) 18.(d) 19.(d) 20.(c)

21.(c) 22.(d) 23.(a) 24.(d) 25.(c) 26.(a) 27.(d) 28.(a) 29.(c) 30.(a)

31.(d) 32.(b)

Aditya Ranjan (Excise Inspector) Selected Selection 37

Downloaded by Kunal Chauhan (kc6970929@gmail.com)


lOMoARcPSD|39315497

Ratio & Proportion

SOLUTIONS
1. (b) 5. (a) 12. (d)
p:q=4:5 With the help of basic division. Mean Proportion of a and b = ab
q:r=3:4 41 33 42 Mean proportion of 0.04 and 0.36
 0.44 ,  0.80 ,  0.85
r:s=2:5 92 41 49 = 0.04  0.36
p : q : r : s = 24 : 30 : 40 : 100
35 = 0.2  0.2  0.6  0.6
Hence, p : s = 6 : 25  0.74
47 = 0.2 × 0.6 = 0.12
2. (d)
Hence, Smallest fraction is 41 : 49. 13. (d)
Let the four number be a, b, c and
d. SMART APPROACH:- X

Z
H igher the difference between Y W
Given that, a : b : c : d = 1 : 2 : antecedent and consequent,
3:5 the smaller the ratio.

r
3 Z
Hence, Answer (a)  
According to question, 2(a + b + 13 39

si
c + d) = 44 6. (a)
Z = 9
2(1 + 2 + 3 + 5) units = 44 Ram Shiv Ram Shiv 14. (a)
1 units = 2 an by
a = 2, b = 4, c =6, d = 10
Income
Expense
1
1
:
:
2
3
2
1
:
:
4
3
Let the third proportion to 16 and
40 is x.

n
Hence, The required ratio 1 unit = 4000 16 40 40  40
 x   100
= 5 : 61 : 213 : 997 Income of Shiv = 4 unit = 4 × 4000 40 x 16
3. (b) ja = Rs.16000 Again, Let mean proportional
R s
7. (c) between 10 and 40 is y.
Let the mixed price of the tea is
4 Unit = Rs. 380 y  10  40 = 20
a th

Rs. x.
Tea-I Tea-II 380 Hence, x : y = 100 : 20 = 5 : 1
300 375 Price of ball = 5  = Rs. 475 15. (c)
4
Let the third proportional of 0.024
8. (d)
ty a

and 0.12 be x.
Let age of father = 3x
x 0.024 0.12
Age of son = 2x  
di M

0.12 x
ATQ, product of their age = 486
 3x × 2x = 486 0.12  0.12
x   0.6
3 : 2  x2 = 81 0.024
Now, x=9 16. (a)
375 – x 3 Age of father after 5 year Mean Proportion  0.03  0.27
  = 3 × 9 + 5 = 32
x – 300 2 = 0.09
Age of son after 5 year 17. (a)
 750 – 2x = 3x – 900
= 2 × 9 + 5 = 23 7 years ago, A : B = 4 : 5
750 + 900 = 3x + 2x
Required ratio = 32 : 23 After 8 years, A : B = 9 : 10
1650 = 5x
9. (a) 5 unit  7 + 8 = 15 year
x = 330
3 : 22 is the lowest ratio. 1 unit  3 year
A

SMART APPROACH:- 10. (b) Sum of present age of A and B


3 300  2 375 = (9 + 10) × 3 – (8 × 2) = 41 Years
Mixed price  8
3 2 Second Sum = 35   40 18. (d)
900  750 1650 7
   Rs.330 Let the share of B = x
5 5 11. (d)
A : B : C = (n + 500): x : (x + 800)
4. (b) 3 6 4
: : ATQ, x + 500 + x + x + 800 = 8200
Income : 4 : 5 8 : 10
A:B:C=
2 5 3 3x + 1300 = 8200
Expense : 7 : 9 7 : 9 = 45 : 36 : 40 3x = 6900
1 unit = Rs.50 5 x = 2300
Required % =  100% = 12.5% A's Share = x + 500 = Rs.2800
Their monthly income be 400, 500 40

Aditya Ranjan (Excise Inspector) Selected Selection 38

Downloaded by Kunal Chauhan (kc6970929@gmail.com)


lOMoARcPSD|39315497

Ratio & Proportion

19. (d) 25. (c) A:B:C=5:2:4


Let the share of B = x Method-1
1600
A : B : C = (x + 500) : x : (x + 800) Using concept of alligation, Salary of A =  5 = Rs. 2000
4
ATQ, x + 500 + x + x + 800 = 8200 A : B
3x + 1300 = 8200 Student 42 : 30 29. (c)
3x = 6900 Avg. 5 x : 6x A : B : C : D
x = 2300 Overall 52 16 : 18 : 27 : 39
C's share = x + 800 = Rs.3100 42 × 5x + 30 × 6x = 52 × 72 Difference b/w the shares of B and
20. (c) 210x + 180x = 52 × 72 12000
12 years ago, A : B = 2 : 3 390x = 52 × 72 D= × (39 – 18)
At present, A : B = 3 : 4 100
52  72
1 unit = 12 years x=  9.6
390 12000
At present, sum of age of A & B = × 21 = Rs. 2520
= (3 + 4) × 12 = 84 year Avg. of students in section B = 6x 100
By option– = 6 × 9.6 = 57.6 kg 30. (a)
Option is division by 7 Method-2 Method -1
Option (c) 84 is right Total student in section A and B = 72
21. (c) Ratio of student, A : B = 7 : 5 20 – x 54 – x
=
37 – x 105 – x

r
7 years ago, A : B = 4 : 5 72
Student in B = × 5 = 30
After 8 year, A : B = 9 : 10 12 x = 3, satisfied the equation.

si
5 unit  7 + 8 = 15 year Ratio of weight of A : B = 5x : 6x Mean proportional between (7x – 5)
1 unit = 3 year ATQ, 42 × 5x + 30 × 6x = 52 × 72

22. (d)
Let,
an by
Difference at present = 3 years  210x + 180x = 3744
390x = 3744
x = 9.6
and (x + 1) = (7 x – 5)(x  1)

n
= (7  3 – 5)(3  1) = 16  4 = 8
Total student = 12x Avg. weight of students in section
B = 9.6 × 6 = 57.6 kg Method -2
Boysja Girls 26. (a)
R s
7x 5x 20 : 37
Seven year ago, A : B = 4 : 5   :: 54 : 105

Seven year hence, A : B = 5 : 6 17 51
a th

1 unit = 14 year 20 :: 54
4x 3x 3x 2x Seven years ago,
Below Above Below Above 1 :: 3
14yrs 14yrs 14yrs 14yrs Age of A = 4 × 14 = 56
After 5 Years, 2 unit = 20 × 3 – 54 × 1
ty a

Given, 7x = 1120 x = 160 Age of A = 56 + 12 = 68


2 unit = 6
Total student = 12x Seven Years Ago,
= 160 × 12 = 1920 Age of B = 5 × 14 = 70 Then, 1 unit = x = 3
di M

23. (a) Seven year hence, Mean proportional between (7x – 5)


Let the no. be 3x and 5x. Age of B = 70 + 12 = 82
Required ratio= 68 : 82 = 34 : 41 and (x + 1)= (7 x – 5)(x  1)
3x  8 2 27. (d)
ATQ, 
5x  7 3 Raju Peter Raju Peter
= (7  3 – 5)(3  1) = 16  4 = 8
 9x + 24 = 10x + 14 Initial 3×2 : 5×2 6 : 10 31. (d)
+5 Unit
 x = 10 Final 11 : 15 11 : 15 7 year ago,H+W+C= 42 × 3 = 126 At
When 6 is added, 5 Unit = 2500
Required ratio = (3x + 6) : (5x + 6) present, H+W+C= 126 + 3 × 7 = 147
1 Unit = 500
=36 : 56 = 9 : 14 Salary of Peter= 10 × 500 = Rs.5000 9 year ago, W+C = 36 × 2 = 72
24. (d) Method-2 At present, W + C = 72 + 2 × 9 = 90
A : B Raju Peter At present, H = (147 – 90) = 57
Income 3x : 5x 3x : 5x 32. (b)
A

Exp 4y : 7y 3x  2500 11 Two liquid A and B = 5 : 3 or 10 : 6


Saving = Income - Expense  
5x  2500 15
3x – 4y = 16000 --------(1)× 5 A : B
5x – 7y = 26000 --------(2)× 3 x = 1000
Peter salary = 5 × 1000 = Rs.5000 10 : 6
15x – 20y  80,000 28. (a) +5
10 : 11
15x – 21y  78,000 A 5
2A = 5B   5 unit – 10 litres
y  2000 B 2
1 unit – 2 litres
Difference b/w their expenditure B 2 1
4B = 2C    Solution in the vesel = 21 unit
= (7y – 4y) = 3y =3×2000 = Rs. 6000 C 4 2 = 21 × 2 litres = 42 litres

Aditya Ranjan (Excise Inspector) Selected Selection 39

Downloaded by Kunal Chauhan (kc6970929@gmail.com)


lOMoARcPSD|39315497

Partnership

PARTNERSHIP
07

1. A, B and C started a business by 4. Three partners shared the profit 7. A, B and C enter into a partner-
investing Rs.13,750, Rs.16,250 in a business in the ratio 8 : 7 : 5. ship with capitals in the ratio
and Rs.18,750, respectively. If B’s They invested their capitals for 7
2 3 5
share in the profit earned by them months, 8 months and 14 months, : : . After 8 months, A in-
is Rs.5,200, what is the total profit respectively. What was the ratio 3 5 6
(in Rs.) earned by them together? of their capitals? creases his share of capital by
A, B C / + 6/% / 25%.
# "If at- the
* +end of the year the
8:7 :5 7 #8 # total
" # $ &earned
profit 9 " #is Rs. 5,820,

r
then the share of C in the profit
! " #$ /
% & : # 5 /’ 7 8
" " is :
14

si
( ) * B+ #" , " " *- . $ & /
- . / # ( ) : # 5 */ * + 2 #$ 2 3 5
A, B C : : #
0 an by # 1 2 #$ SSC CPO 24/11/2020 (Shift-2) 3 5 6

SSC CPO 23/11/2020 (Shift-1) (a) 49 : 64 : 20 (b) 20 : 49 : 64 > % /

n
(a) 15,600 (b) 17,500 (c) 20 : 64 : 49 (d) 64 : 49 : 20 7 A% / :#5/ ? 7@A %
5. A, B and C started a business # # 5 *" * +
(c) 18,200 (d) 16,600
ja investing amounts of Rs.13,750, * + C # ", " 6 =
2. A and B started a business
R s
Rs.16,250, Rs.18,750, respectively.
investing amounts of Rs.92,500 If B's share in the profit earned SSC CPO 09/12/2019 (Shift-01)
and Rs.1,12,500, respectively. If
a th

by them is Rs.5,200, what is the (a) Rs. 2,050 (b) Rs. 2,350
B’s share in the profit earned by difference in the profit (in Rs.) (c) Rs. 2,450 (d) Rs. 2,250
them is Rs.9,000, what is the total earned by A and C?
profit (in Rs.) earned by them 8. A, B and C started a business
A, B C
together? with their capitals in the ratio 4
ty a

A B 3 : 2 : 9. At the end of every quarter,


- / / $ %
A halfs his capital whereas B
di M

& ’ 4 5 B* + " #, doubles his capital and C leaves


" #$ % & ’ ( ) * + #
A" C ’ 45 his capital unchanged. If at the
B ", 3 " & % #
* + # # # 0 ) # 1 of 2a year. / A’s5 profit
end - $ was Rs.
’ ( ) * * + 0 #1 2 #$
SSC CPO 25/11/2020 (Shift-1) 24,000, then what is the total
SSC CPO 23/11/2020 (Shift-2)
(a) 1,800 (b) 1,500 profit (in Rs.)?
(a) 19,000 (b) 21,240 (c) 1,600 (d) 1,200
A, B C < 3
(c) 16,400 (d) 20,000 6. A, B, C subscribe a sum of Rs.75,500
3. A and B started a business for a business. A subscribes
:#5/ *6 - !
investing amounts of Rs.92,500 Rs.3,500 more than B, and B "/ A# / #: # 5 / ;
and Rs.1,12,500, resprectively. If subscribes Rs.4,500 more than C. * " B5 7 / : # 5 / %
B's share in the profit earned by Out of a total profit of Rs.45,300,
them in Rs.9,000 what is the profit how much (in Rs.) does a receive?
* "C / :#5/ #
A

(in Rs.) earned by A? " $ % - AB * +#


A, B, C / #
A B 3 / ; *6 , / A, B "< # $ D . 5
! ; * 6 B, " 0 D #1 2 / 5-$
$ % & ’ B 4 5 * C+ <# ; *6 "SSC$ CPO *09/12/2019 (Shift-02)
", 3 A
’" 45 * + < A # / (a) Rs. 2,16,000
* + 0 #1 2 #$; 0 # #1 * 6/=
(b) Rs. 2,30,400
SSC CPO 24/11/2020 (Shift-1) SSC CPO 25/11/2020 (Shift-2)
(a) 9,000 (b) 7,400 (c) Rs. 2,35,200
(a) 12,600 (b) 17,400
(c) 11,240 (d) 10,000 (c) 15,000 (d) 14,700 (d) Rs. 2,25,600

Aditya Ranjan (Excise Inspector) Selected Selection 40

Downloaded by Kunal Chauhan (kc6970929@gmail.com)


lOMoARcPSD|39315497

Partnership

9. A, B and C enter into a partnership A - B # D A, B C - # A, > %


by investing their capitals in the
5 / :I5/ "/ *- ,
1
2 3 5 ; B $ / 7 2: # 5 /
ratio of : : . After 4 months, 1
5 4 8 " * +7 +# 6 % B,
A increased his capital by 50%, but " *- A / 57 /
B decreased his capital by 20%.
5 / :I5/ "/ *- ,
1
What is the share of B in the total : #B5 / 2 " ; *- 1
profit of Rs. 2,82,100 at the end of 2 * + 5 +# 6 " % C
a year. /$ % < D* * + # 3 "/ # *- < D
2 3 5 B " ,A ", D< " A$ / :I5/
A, B C / : # 55 :/ 4 : 8 " B / :#5/ / . / = SSC CPO 12/12/2019 (Shift-02)
# *6 > % / SSC CPO
" # $11/12/2019
< " / (Shift-02) (a) Rs. 1,800 (b) Rs. 1,650
(c) Rs. 1,600 (d) Rs. 1,850
7 A% / :#5/ # ? / E (a)F Rs. 45,00,000
% " (b) Rs. 42,00,000
14. A, B and C started a business, A
* B / :#5/ # ? / / %
1
" $ B # # ( ) " (c) Rs. *40,00,000 (d) Rs. 48,00,000
D invested 33 % of the total
3

r
* * + B # ", "12. 6A =started a business with a
SSC CPO 11/12/2019 (Shift-01) capital of Rs. 1,12,000. After 2 1
capital, B invest 33 % of the

si
months, B joined the business 3
(a) Rs. 97,500
with a capital of Rs. 80,000, and remaining capital and C, the
(b) Rs. 1,01,400
an by after another 2 months, C joined remaining. If the total profit, at the
(c) Rs. 1,00,750 the business with a capital of Rs. end of a year, was Rs. 20,250, then
72,000. After 10 months from the profit of C exceeds the profit

n
(d) Rs. 83,200
starting of the business, B of B by:
10. A sum (in Rs.) is distributed withdrew Rs. 8,000 and C also
A, B C - A!
ja
between A, B and C in the ratio withdrew Rs. 8,000. If B received
R s
9 : 6 : 11. If A gives Rs. 500 from Rs. 9,800 as his share in the 1
profit at the end of a year, then
* : 33
# 53 /% B
his share to C, the ratio of shares
a th

of A, B and C becomes 4 : 3 : 6. the total profit was:


1
What is the sum of shares (in A D / : I 5 / * 6 B - 33 : #35%/ C
Rs.) of B and C , in the ! $ B" / 7 %
B
ty a

beginning? D / :I5/ *6 # # # * * + C
* " 6 . & * +B " / * + ; "
- ; 0A, D
B C#
7/G 1
di M

7 C% D / :I5/ *6
9 : 6 : 11 # 7 #8/ 5 / " $ % SSC CPO 13/12/2019 (Shift-01)
# * " 6 $ (a) Rs. 5,200/ (b) Rs. 4,500
A " ,500 DC % % " ! B
"/ 7 % D (c) Rs. 6,750 (d) Rs. 2,700
A, B C ", # 4:3: * *C
- + / D
15. A, B and C stared a business with
6 " 5 B " C$ # + " ,* #* - $B % B # # their
* +capitals# in the ratio 1 : 4 :
6 0 D #1 H " = & ", 3 D ( ) 4. At " the
- end of every
* 3 months,
* + . = A double his capital, B halves his
SSC CPO 11/12/2019 (Shift-02)
capital and C leaves his capital
SSC CPO 12/12/2019 (Shift-01)
(a) Rs. 8,500 (b) Rs. 9,100 unchanged. At the end of the
(a) Rs. 30,800 (b) Rs. 35,800 year. If B’s share in the profit was
(c) Rs, 7.800 (d) Rs. 7,500 (c) Rs. 32,400 (d) Rs. 33,600 Rs. 4,50,000, then the total profit
(in Rs. lakhs) was.
11. A invests Rs. 6,00,000 more than 13. A, B and C are partners in a
A

business, A, whose capital has A, B C < <


B in a business. B invests his
been used for 5 months, claims :I5/ *6 - !
1
capital for 7 months. While A 1 " A# /# : I 5 / %
2 of the profit, B whose capital
7 * B" / :I5/ ; *
1 has been used for 7 months, C / :I5/ #
invests his capital for 2 more
2 # # B % * " ,+ < #
1
months than B. Out of the total claims of the profit C has
5 D . * * + 0 D * J
profit of Rs. 12,40,000. If the
share of B is Rs. 2,48,000 less invested Rs. 4,600 for 9 months. SSC CPO 13/12/2019 (Shift-02)
than the share of A, then the How much capital did A (a) 23.1 (b) 32.4
capital of B is: contribute? (c) 34.8 (d) 24.2

Aditya Ranjan (Excise Inspector) Selected Selection 41

Downloaded by Kunal Chauhan (kc6970929@gmail.com)


lOMoARcPSD|39315497

Partnership

ANSWER KEY
1.(a) 2.(c) 3.(b) 4.(d) 5.(c) 6.(b) 7.(d) 8.(c) 9.(b) 10.(a)

11.(d) 12.(d) 13.(a) 14.(b) 15.(a)

r
si
an by
n
ja
R s
a th
ty a
di M
A

Aditya Ranjan (Excise Inspector) Selected Selection 42

Downloaded by Kunal Chauhan (kc6970929@gmail.com)


lOMoARcPSD|39315497

Partnership

SOLUTIONS
1. (a) A : B : C 9. (b)
A : B : C = 13750 : 16250 : 18750 (x + 8000) : (x + 4500) : x 2 3 5 
A:B:C= 
 : : 
 40
= 11 : 13 : 15 ATQ, 5 4 8 
 B(13)  5200
x + 8000 + x + 4500 + x = 75500 = 16 : 30 : 25
5200 3x + 12500 = 75500 We know, P  C × T
Total share, 39 unit = × 39
13
3x = 63000 A : B : C
= Rs.15600
16×4=64 30×4=120
2. (c) x = 21000 25×12=300
24×8=192 24×8=192
A : B = 92500 : 112500 = 37 : 45 A's Share = x + 8000 = 29000 (64+192=256) (120+192=312)

r
45  9000 B's Share = x + 4500 = 25500
Total = 256 + 312 + 300 = 868

si
 Total = (37 + 45) = 82 C's Share = x = 21000
Now, 2,82,100
82 unit 
an by
9000
45
× 82 A : B : C = 29000 : 25500 : 21000
Profit of B =
868
 312

n
= Rs.1,01,400
= Rs. 16400 = 58 : 51 : 42
10. (a)
3. (b) Total ,151 unit  45300
ja A : B : C
R s
A : B = 92500 : 112500 = 37 : 45
45300 9x 6x 11x
A's profit  × 58 = 17400
a th

B (45)  9000 151


7. (d) 9x – 500 4
9000 ATQ, 
A (37)   37 = Rs.7400 A : B : C 11x  500 6
45
ty a

20×12+5×4 : 18×12 : 25×12  54x – 3000 = 44x + 2000


4. (d)
260 : 216 : 300
Let the capital of three partners 10x = 5000
di M

65 : 54 : 75
be x, y, z.  x = 500
Total Profit = Rs. 5820
ATQ,
Profit share of 'C' Sum of share of (B and C) in
x × 7 : y × 8 : z × 14 = 8 : 7 : 5 begining = (6x + 11x)
5820
= (65  54  75 )  75 = 17x = 17 × 500 = Rs. 8500
7x : 8y : 14z = 8 : 7 : 5

5820 11. (d)


8 7 5 =  75  Rs .2250
x:y:z= : : 194
7 8 14 Profit ratio of A & B = 744000 :
8. (c) 496000 = 3 : 2
x : y : z = 64 : 49 : 20
A : B : C Ratio of time of A & B
5. (c)
4 2 9
A

15
A : B : C = 13750 : 16250 : 18750 1st  43 2 3 9 12 = 10 : =4:3
2
= 11 : 13 : 15 2nd  23 4 3
rd
3  1 3 83 1
13 unit  5200 We know, Capital  P ×
4th  0.5  3 16  3 T
1 unit  400 Total  22.5 90 108 Ratio of capital of A & B = 9 : 8

A – C = 4 unit  4 × 400 = 1600 1 unit  6,00000


24000
Total profit =  220.5
6. (b) 22.5 Capital of B, 8 unit = 6,00,000 × 8
Let the share of C = x = Rs.235200 = Rs. 48,00,000

Aditya Ranjan (Excise Inspector) Selected Selection 43

Downloaded by Kunal Chauhan (kc6970929@gmail.com)


lOMoARcPSD|39315497

Partnership
12. (d) 9 unit  Rs. 20,250
1
Profit of B =  35 = 7 unit
5 1 unit  Rs. 2250
A B C
112000  12 80000  8 72000  6 The profit share of C is more than
Profit of C = 35 – 5 – 7 = 23 unit
 1344000  640000  432000 that of B by= (4 – 2) × Rs. 2250
72000  2 64000  2 Capital of A and B are x and y = Rs 4500
 144000  128000 respectively.
15. (a)
We know, Capital × Time = Profit
A B C
Ratio of Total Capital of A, B and C is
5x : 7y : 23 100 400 400
A : B : C 200 200 400
5x 5
1344000 784000 560000 ATQ,  400 100 400
4600  9 23
800 50 400
12 : 7 : 5
On solving, x = 9 × 200 = Rs. 1800 1500 750 1600

9800
Total Profit =  24 = 33600 Ratio of profit,1500 : 750 : 1600
7 14. (b)

r
= 30 : 15 : 32
13. (a) Let the total capital be 9 unit.

si
4,50,000
A B C Total profit =  77
Let the total profit be 35 unit 15
I 3 : 2 : 4
an by
Profit of A =
1
7
× 35 = 5 unit
T 1 : 1 : 1
= 23.1 Lakh

n
Profit 3 : 2 : 4

ja
R s

a th
ty a
di M
A

Aditya Ranjan (Excise Inspector) Selected Selection 44

Downloaded by Kunal Chauhan (kc6970929@gmail.com)


lOMoARcPSD|39315497

Average

AVERAGE
08

1. The average salary of all the 4. The average of a batsman in 16 ’ @ 3 8


employees in a company is Rs. innings is 36, in next innings he # 4 0 + 8
14,000. The average salary of 5 scores 70 runs. What will be his # ( + /
technicians is Rs. 18,000 and the new average? # " 0 /
average salary of the rest is
6 .$ / 8 # 5 1
Rs. 13,200. The total number of
empolyees in the company is: !6 7$ 4 SSC CPO 11/11/2022 (Shift-02)
3 5 7(a)138 years (b) 36 years

r
SSC CPO 10/11/2022 (Shift-03) (c) 37 years (d) 35 years

si
(a) 53 (b) 35 8. The weight of a person was
(c) 106 (d) 38 entered incorrectly as 83 kg
an by ! "
instead of 63 kg. As a result, the
#$ 5. % The &average
& & &age& &of&25
& girls in a average weight of a group of people
class is 11.2 years and that of increased by 500 gm. What is the

n
SSC CPO 09/11/2022 (Shift-01)
(a) 28 (b) 30 the remaining 15 girls is 10 total number of people in the
years. Find the average age of all group?
(c) 29 ja (d) 27
the girls in the class.
R s
2. There are six baskets – Green, A 5 / 7$ 6!
Red, Blue, Yellow, Black and 8 " $9: / !# 7 , 7
a th

White. Each basket has some &" 0 0 $ $ 97: D


rings in it. The average number 7 , E F 7=
of rings in all the six baskets is # 0 8
38. If the blue basket is removed,
#$ % 1
$9: # ; /
ty a

the average number of rings in SSC CPO 11/11/2022 (Shift-02)


SSC CPO 10/11/2022 (Shift-03)
the remaining baskets is 34. How
(a) 44 (b) 48
many rings are there in the blue (a) 10.75 years (b) 11.35 years
di M

basket? (c) 40 (d) 50


(c) 10.45 years (d) 11.05 years
’ ( ) $ The$average $height of three kids 9.
$ 6. The average score in an English
$ * + ,- ( is 56 cm and their# ’ heights are in exam of a class of 45 students
in a coaching centre is 52. A group
’.$ ’ ( ’ . $7 : 8 : 9. Find the
the ratio
of 6 boys having an average score
% ! + heights
$ ( of all the three. of 40 leave the class and another
( + / 0 <( $ = group
6 of eight boys having an
’.$ % ! average score of 43 join the class.
3 / $ = # 4 > >
$ ( ’ . $? 1 What is the new average score
$ = ; of the/ class?
SSC CPO 09/11/2022 (Shift-03)
SSC CPO 11/11/2022 (Shift-01) 7 ( G
(a) 22 (b) 45
(a) 42 cm, 48 cm, 54 cm 8 7, / 8
(c) 37 (d) 58
A

3. The average of 15 results is 21. (b) 56 cm, 64 cm, 72 cm "


The average of the first 7 of those (c) 35 cm, 40 cm, 45 cm 6 $9: D 8
is 21 and the average of the last (d) 49 cm, 56 cm, 63 cm !
7 is 20. What is the 8th result? $9: J D 8
7. The average age of 10 students
2 " and 3their class teacher is 17 $ / 8
$ 4 " years. If 4the age of the class 5 7 1
" 2 5
teacher 1
is excluded, the average SSC CPO 11/11/2022 (Shift-03)
SSC CPO 10/11/2022 (Shift-02) age of the 10 students is reduced
(a) 48 (b) 53
(a) 25 (b) 31 by 2 years. What is the age of the
class teacher? (c) 49.5 (d) 52
(c) 28 (d) 22

Aditya Ranjan (Excise Inspector) Selected Selection 45

Downloaded by Kunal Chauhan (kc6970929@gmail.com)


lOMoARcPSD|39315497

Average

10. Class 10, consisting of 40 A A,5B C $ D + #N


students, took a science test. 30 70
/ 7, D =/ D 3 D.
students had an average score of $ 60 7, ! > > 4 K D.
95. The other students had an
/ EA / 5 D
/ D. L M ;
average score of 85. What is the
/ 5 7, A
K , BH SSC CPO 24/11/2020 (Shift-1)
average score of the whole class?
B, C, D E
8 / G H H ; (a) 3,003 (b) 1,001
59
/ 7, A/
8 + ! G H (c) 21,021 (d) 5,005
/ L 7, M ;
? H J G SSC H CPO 23/11/2020 (Shift-2) 17. The average temperature for
H D(a) 39 8 Monday, Wednesday and Friday
(b) 40
was 41ºC. The average temperature
; / (c) 50 (d) 59 for Wednesday, Friday and
SSC CPO 09/11/2022 (Shift-02) 1 Thursday was 42ºC. If the
(a) 92.3 (b) 92.5 14. If A is of C, and B is twice of temperature on Thursday was
6
43ºC, then the temperature on
(c) 92.4 (d) 92.2 A, and the average of A, B and C
Monday was:
11. The average of twelve numbers is is 30, then the difference between
39. The average of the last five A and C is: #K #O

r
numbers is 35, and that of the 1 41ºC H #K #O
+A, C B, A

si
first four numbers is 40. The fifth 6
+ 7# 42ºC H + 7#
number is 6 less than the sixth 43ºC H
A, B C 30 A
number and 5 more than the
an by C ; H
seventh number. The average of
SSC CPO 23/11/2020 (Shift-2) SSC CPO 24/11/2020 (Shift-2)
the fifth and sixth numbers is:

n
(a) 60 (b) 40 (a) 43ºC (b) 41ºC
" % !?
% ! (c) 80 $ (d) 50 (c) 42ºC (d) 40ºC
ja
R s
% 15. The average weight
% of a group of 18. The average of twelve numbers is
3 people 39. The average of the last five
’I % 6 % A, B and C is 70 kg.
a th

When D joins this group, the numbers is 35, and that of the first
K ’I % four numbers is 40. The fifth number
average becomes 60 kg. A man
; E, whose weight is 5 kg more is 6 less than the sixth number
and 5 more than the seventh
SSC CPO 23/11/2020 (Shift-1) than that of D, replaces A and
ty a

number. The average of the sixth


(a) 39 (b) 50 the average weight of B, C, D and
and seventh number is:
E now becomes 59 kg. What is
(c) 44 (d) 47
di M

the average weight (in kg) of A, 12 % 39


12. The average weight of A, B and C D and E? (correct to the nearest % 35 $
is 65 kg. If the average weight of integer)
A and B is 63.5 kg, and the average % 40 %
weight of Aand C is 67.5 kg. then 3 A 5A, B C $ D ’I %6 %
the weight of A (in kg) is: 70
/ 7, / = D5 K ’I %
A, B C 65/ 7, D $ 60 7, ; 7 1
A
+ B 63.5
/ 7, / EA /5 D
/ SSC CPO 25/11/2020 (Shift-1)
A C 67.5
/ / 5 7, KA BH $(a) 47.5 (b) 44.5
7, A / L 7, M$ ; B, C, D E (c) 39 (d) 50
19. The average weight of A, B and C
59
/ 7, A,
/ D
is 65 kg. If the average weight of
SSC CPO 23/11/2020 (Shift-1) E / L 7, CMand ;B is 61. 5 kg. and the average
A

(a) 65 (b) 67 L ( D2 weight


( Mof A and C is 68.5 kg, then
(c) 60 (d) 68 the weight of C (in kg) is:
SSC CPO 24/11/2020 (Shift-1)
13. The average weight of a group of 3 A, B C 65/ 7,
(a) 39 (b) 35
people A, B and C is 70 kg. When C
+ B 61.5
/ 7,
D joins this group, the average (c) 30 (d) 40 A C 68.5
/ 7,
becomes 60 kg. One man E, whose 16. If the average price of three C / L 7 ,M ;
weight is 5 kg more than that of chairs is Rs.14,014 and their
SSC CPO 25/11/2020 (Shift-1)
D, replace A and the average weight prices are in the ratio of 3 : 4 :
of B, C, D and E now becomes 59 7, then the highest price (in Rs.) (a) 65 (b) 67
kg. A's weight (in kg) is: of chair is: (c) 60 (d) 68

Aditya Ranjan (Excise Inspector) Selected Selection 46

Downloaded by Kunal Chauhan (kc6970929@gmail.com)


lOMoARcPSD|39315497

Average

20. The average age of 40 students of 8 #’ G H 8 / $9: 69.3


a class is 16 years. After admission 7, 3 8
60.5 7, /65 7, /
of 10 new students to the class,
59.4
/ 7, + 8
the average becomes 15 years. If $ G H 8 $
the average age of 5 of the new $9: $9: 63.8
G H 0.9 7, /
students is 11 years, then the 7, 8 $9:
average age (in years) of the E F / 8 G H
, 1
remaining 5 new students is: % H
SSC CPO 11/12/2019 (Shift-02)
8 40 ’ @ #
SSC CPO 09/12/2019 (Shift-02)
5
16 0 10 8 ’ @ ,
(a) 32 (b) 40 (a) 55 (b) 40
9
+ 15 0 / +
(c) 37 (d) 42 4
’ @ 5 11 #0 (c) 45 (d) 44
9
5 0 ’ @ 24. The# average
L 0 of n numbers is 42. 27. The average of n observation is
M ; If 75% of the numbers are 40. If one observation of value 80
SSC CPO 25/11/2020 (Shift-2) increased by 4 each and the is added, then the average of all
(a) 15 (b) 10 remaining numbers are decreased the observations is 41. What is
(c) 11 (d) 16 by 8 each, then what is the the value of n?

r
21. While tabulation of marks scored average of the numbers, so n 8 2
in an examination by the students obtained? 8 2 /

si
of a class, by mistake the marks / 8 2
scored by one student got recorded n % " + 4 R
/ n ; /
an by
as 93 in place of 63, and thereby
the average marks increased by
%
0 %
,- E F /
SSC CPO 11/12/2019 (Shift-02)
(a) 40 , - (b) 39

n
0.5. What was the number of stu-
dents in the class? / = , (c)Q 38 % (d) 43
8 ja ’ @ P 8 ,5 Q 7 1 28. The average of eleven numbers is 68.
The average of the first four numbers
R s
7 $ SSC7 CPO
$ 09/12/2019 (Shift-02) is 78 and that of the next four
’ @ P , Q 7 (a) 43.8 6 (b)
! 43 numbers is 63. The 9th number is
a th

BH ?! $% + (c)742.5 / (d) 44 two times the 11th number and the


2 B C & E numbers
F 10th number is 4 less than the 11th
25. The average of ten is 72. number. What is the average of the
7= 8 ’ @ %
The average H four1 num-
of the first 9th and 11th numbers?
ty a

SSC CPO 09/12/2019 (Shift-01) bers is 69 and that of the next


three numbers is 74. The 8th num-
S % 6
(a) 60 (b) 20 % 4
di M

ber is 6 more than the 9th num-


(c) 15 (d) 30 ber and 12 more then the 10th % 6! ?
22. The average run rate of a cricket number. What is the average of % + 7#
team during the first 20 overs is the 8th and 9th numbers?
4.5. What should be the required % ?
run rate per over for the next 30
+ % 4" $ ; /
overs, if it has to chase of 282 % 6? SSC7 CPO
$ 12/12/2019 (Shift-01)
runs in total? % 4 (a) 70.1% ? (b) 72.2
$ " + O % ( ( 6 K %
(c) 70.5 "(d) 72.6
( & + = K/ $ ? 29.%The average score of 42 students in
#$ " " 7 $5 ! 1 a test is 69. The ratio of the number
of boys to that of girls is 10 : 11.
$ , ( 5 SSC CPO 1
11/12/2019 (Shift-01)
The average score of the boys is
SSC CPO 09/12/2019 (Shift-01) (a) 76 (b) 76.5
A

20% more than that of the girls. The


(a) 6.3 (b) 6.0 (c) 77.5 (d) 77 average score of the boys is:
(c) 6.4 (d) 6.8 8 " G H
26. The average weight of the boys
23. The average weight of some
in a class is 69.3 kg and that of 6? $9: % $
students in a class was 60.5 kg.
the girls in the same class is # $9:
When 5 students, whose average
59.4 kg. If the average weight of $9: , Q
weight was 65 kg. joined the class
all the boys and girls in the $9: , Q
then the average weight of all the
c la ss is 63 .8 k g, the n th e
students increased by 0.9 kg. The SSC CPO 12/12/2019 (Shift-01)
percentage of the number of
number of students in the class (a) 82.8 (b) 75.6
boys in the class is:
initially was: (c) 73.5 (d) 75.2

Aditya Ranjan (Excise Inspector) Selected Selection 47

Downloaded by Kunal Chauhan (kc6970929@gmail.com)


lOMoARcPSD|39315497

Average

30. A batsman scores 92 runs in the ? % " " 33.


& The average$ of+twelve numbers is
15th inning and thus increases 58. The average of the first
% & +
his average by 4. What is his numbers is 56 and the average of
average after the 15th inning? % " &6 + + 7=
the next four numbers is 60. The
% % ( + /
10th number is 4 more than the
.$ / ?" 0 % 115th 7 1the 11th number
number and
/ 3 E F
SSC CPO 13/12/2019 (Shift-01) is one less than the 12th number.
/ + 3 What is the average of the 10th
(a) 21 (b) 23
5 1 and 12th number?
(c) 22 (d) 20
SSC CPO 12/12/2019 (Shift-02) 32. The average age of 24 students is %
15.5 years. The age of the % 6
(a) 40 (b) 32
teachers is 24 years more than the % 6
(c) 35 (d) 36 average age of all the students
and teacher. What is the age (in % K
31. The average of 19 numbers is years) of the teacher? %
22.8. The average of the first ten ;0 /
numbers is 18.4 and that of the
" G H # &

r
last numbers is 28.6. If the 10th 8 # G H 8 (Shift-02)
SSC CPO 13/12/2019
number is excluded from the # " 0 K 8

si
given numbers, then what is the # L 0 T M (a) 59.5
1 (b) 58.5
average of the remaining
numbers? an by SSC CPO 13/12/2019 (Shift-02) (c) 58 (d) 59

(Your answer should be nearest (a) 40 (b) 42

n
to an integer) (c) 41.4 (d) 40.5

ja
R s
a th

ANSWER KEY
ty a

1.(b) 2.(d) 3.(c) 4.(d) 5.(a) 6.(a) 7.(c) 8.(c) 9.(d) 10.(b)
di M

11.(d) 12.(c) 13.(a) 14.(d) 15.(b) 16.(c) 17.(d) 18.(b) 19.(a) 20.(c)

21.(a) 22.(c) 23.(a) 24.(b) 25.(d) 26.(d) 27.(b) 28.(c) 29.(b) 30.(d)

31.(c) 32.(d) 33.(a)


A

Aditya Ranjan (Excise Inspector) Selected Selection 48

Downloaded by Kunal Chauhan (kc6970929@gmail.com)


lOMoARcPSD|39315497

Average

SOLUTIONS
1. (b) Hence, The heights of student 13. (a)
will be: 49 cm, 56 cm and 63 cm
Total Tech Remaining A + B + C = 70 × 3 = 210 kg
7. (c)
Emp: x 5 x–5 A + B + C + D = 60 × 4 = 240 kg
Age of class teacher
Avg: 14000 18000 13200 D = 240 – 210 = 30 kg
= 17 + (2 × 10) = 37 years
ATQ, 5 × 18000 + (x – 5) × 13200 8. (c) E = 30 + 5 = 35 kg
= x × 14000 Number of people in the group B + C + D + E = 59 × 4 = 236 kg
On solving, x = 30 B + C = 236 – 30 – 35 = 171 kg
83kg – 63kg 20000g
Method-2 = = 500g = 40 A + B + C = 210 kg
500g
Tech Emp Non-Tech Emp A = 210 – 171 = 39 kg
18000 13200 9. (d)
14. (d)

r
Initially Left Join
A : B : C
No. of
x : 2x : 6x

si
45 6 8
14000 student
Average 52 40 43 x + 2x + 6 x

800
an by: 4000
New Average
45  52   8  43 – 6  40 
3
9x = 90
=30

n
1 5 =
45 – 6  8 x = 10
1 Unit = 5
Difference between A and C
6 units = 30 ja 2340 + 344 – 240
R s
2444 = 6x – x = 5x = 50
The total employees = 30 = = = 52
47 47 15. (b)
2. (d)
a th

10. (b) A + B + C = 70 × 3 = 210 kg


Total rings = 6 × 38 = 228
Average all student A + B + C + D = 60 × 4 = 240 kg
After removing the blue basket,
Total rings = 34 × 5= 170 95  30  85  10 D = 30 kg
  92.5
ty a

Rings in blue basket = 228 –170 40 E = 35 kg


= 58 11. (d) B + C + D + E = 59 × 4 = 236 kg
3. (c) Total First Last
di M

B + C = 236 – 30 – 35 = 171 kg
8th Result = 15 × 21 – (7×21+7×20) No. 12 4 5
A = 210 – 171 = 39 kg
= 315 – 287 = 28 Avg. 39 40 35
Deviation = 4 × 1 + 5 × ( – 4) = –16 D = 30 kg
4. (d)
E = 35 kg
 70 – 36  V : VI : VII
New Average = 36   Average of A , D and E
 17 
  x +5  : (x + 11) : x
39  30  35 104
= 36 + 2 = 38 Thus, = =
3 3
5. (a) x + 5 + x + 11 + x = 39 × 3 + 16
Average of all girls 3x + 16 = 39 × 3 + 16 = 34.66 = 35 kg
3x = 39 × 3 16. (c)
25  11.2  15  10 430
= = = 10.75 x = 39 Ratio = 3 : 4 : 7 = 14x
25  15 40
Average of V and VI number 14x = 14014 × 3
A

Method-2
x + 5 + x + 11 x = 3003
25  1.2 = = x + 8 = 47
Average = 10  2 Heigest price of chair (7)
40
= 3003 × 7 = Rs.21021
= 10 + 0.75 = 10.75 years 12. (c)
A + B + C = 65 × 3 = 195 kg 17. (d)
6. (d)
A + B = 63.5 × 2 = 127 kg M + W + F = 41 × 3 = 123°C
Given, ratio = 7 : 8 : 9
A + C = 67.5 × 2 = 135 kg W + Th + F = 42 × 3 = 126°C
Total = 24 units
Total height = 56 × 3 = 168cm A + B + A + C = 127 + 135 Th = 43°C
24 units = 168 cm 2A + B + C = 262 kg W + F = 126 – 43 = 83°C
1 units = 7 cm Weight of A = 262 – 195 = 67 kg M = 123 – 83 = 40°C

Aditya Ranjan (Excise Inspector) Selected Selection 49

Downloaded by Kunal Chauhan (kc6970929@gmail.com)


lOMoARcPSD|39315497

Average

18. (b) 24. (b) 29. (b)


Sum of 'N' number = 42x Boys Girls
Total First Last 6x
New sum of n number = 42x + 5x
No. 12 4 5
Avg. 39 40 35 
 3  
  1 

Deviation = 4 × 1 + 5 × ( – 4) = –16  x   4 –  x   8 

 4 
   4 
 69
V : VI : VII = 42x + 3x – 2x
 x + 5 :  x +11 : x = 43x
New average of 'x' number 20 22
x + 5 + x + 11 + x = 39 × 3 + 16
43 x ATQ, 20 × 6x + 22 × 5x = 42 × 69
3x + 16 = 39 × 3 + 16 =43
=  120x + 110x = 42 × 69
x = 39 x
Average of VI and VII Number 25. (d)  230x = 42 × 69
x + 11 + x Total First Next
42  69
= = x + 5.5 = 44.5 No. 10 4 3 x= = 12.6
2 230
Avg. 72 69 74
19. (a) Avg. score of boys = 6x = 6×12.6
Deviation = 4 × (-3) + 3 × ( +2)
A + B + C = 65 × 3 = 195 kg
= –6 = 75.6 Marks
B + C = 61.5 × 2 = 123 kg Let the 9th number is x. 30. (d)

r
A + C = 68.5 × 2 = 137 kg
8th 9th 10th Given, Batsman scored in 15Th
B + C + A + C = 123 +137
x+6 x x–6 Inning = 92

si
A + B + 2C = 260 kg So, 3x = 72 × 3 + 6
C = 260 – 195 = 65 kg Average score of 14 inning is
222
20. (c) an by
Total age of 40 students
= 40 × 16 = 640 years
x =
3
= 74
9th no.= 74, 8th no. = x + 6 = 80
= 92 – 15 × 4 = 32
New Average = 32 + 4 = 36
31. (c)

n
Total age after joing 10 students Avg of 8th and 9th number
Avg of 19 number = 22.8
= 50 × 15 = 750 years 74  80 154 Avg of next 10 number = 18.4
ja
Total age of 10 students =
2

2
= 77
R s
= 750 – 640 = 110 years Avg of last 10 number = 28.6
26. (d) Deviation = – 4.4 × 10 + 5.8 × 10
Total age of 5 students
a th

Boys Girls
= 5 × 11 = 55 years = 14
69.3 59.4
Age of remaining 5 students After Excluding the 10th number,
= 110 – 55 = 55 years the average of remaining number
Average age of remaining 5
ty a

14
55 63.8 will be = 22.8 – = 22(Appx.)
students = = 11 years 18
5
di M

21. (a) 32. (d)


Number of students Let the age of teacher = x year
4.4 5.5 ATQ, 24 × 15.5 + 1 × x = 25(x – 24)
93 – 63 30
=  = 60 4 : 5  372 + 1x = 25x – 600
0.5 0.5 Number of Boys in class
22. (c)  972 = 24x
44 4  x = 40.5
Required run rate per over = 100% = 44 %
99 9 33. (a)
282 – (2×4.5) 192
= = = 6.4 27. (b) Total First Next
30 30 ATQ, n × 40 + 1 × 80 = (n + 1) × 41 No. 12 5 4
23. (a)  40n + 80 = 41n + 41  n = 39
28. (c) Avg. 58 56 60
Let the number of students in
class is x Total First Next Deviation = 5 × (–2) + 4 × (+2)
No. 11 4 4 = –2
ATQ, 60.5x + 8 × 65 = (60.5 + 0.9) (x + 8)
A

Avg. 68 78 63
 60.5x + 520 = 61.4 (x + 8) 10th 11th 12th
Deviation = 4 × (+10) + 4 × (-5)
 60.5x + 520 = 61.4x + 491.2 = + 20 x+4 x x+1
 0.9x = 28.8  x = 32 9th 10th 11th So, 3x + 5 = 58 × 3 + 2
2x x–4 x  3x + 5 = 174 + 2
The number of student in the class
A.T.Q, 4x – 4 = 68 × 3 – 20  3x = 176 – 5
initially was = 32
4x = 184  x = 47
 x = 57
SMART APPROACH:- 9th  11th
Required Average = Avg. of 10th and 12th number
Number of students initially 2
(65 – 60.5) ×8 61  58 119
= = 32 94  57 = = = 59.5
0.9 = = 70.5 2 2
2

Aditya Ranjan (Excise Inspector) Selected Selection 50

Downloaded by Kunal Chauhan (kc6970929@gmail.com)


lOMoARcPSD|39315497

Time & Work

TIME AND WORK


09

1. To do a certain task X would take A 6. A! and B can complete a work in


3 times as long as Y and Z * +A B , 10 days and 15 days, respectively.
together; and Z would takes 4 They got a total of Rs. 1,250 for
times as long as Y and X
1 that work. What will be B's
2 ! B
together. Three of them together 2 share?
can complete the task in 10 days. , ! A B ’ 2 3
How much time is taken by X and " $ !
Z to complete the task?

r
SSC CPO 09/11/2022 (Shift-02) ’ 0 4 #$
X B 5 & "

si
Y Z 3 10 SSC CPO 09/11/2022 (Shift-03)
(a) days (b) days
10 3 (a) Rs. 400 (b) Rs. 500
an by Z Y X (c) 3 days (d) 4 days
7.
(c) Rs. 600 (d) Rs. 200
Aarush and Maahi together can

n
4. 12 men or 20 boys can finish a
do a piece of work in 10 days. If
work in 10 days. What part of the
Aarush alone can do the same
ja ! same work will be done by 4 men
work in 15 days, then how many
R s
and 4 boys in 4 days?
X Z days Maahi alone will take to do
a th

! "# # -. the same work?

SSC CPO 09/11/2022 (Shift-01) ! 46


-. ’
2 1 ’ $
(a) 18 days (b) 20 days
ty a

/ ! "
9 9

SSC CPO 09/11/2022 (Shift-02)
"
di M

1 2
(c) 20 days (d) 22 days 49 16 SSC CPO 10/11/2022 (Shift-01)
9 9 (a) (b)
75 75 (a) 25 (b) 15
2. If 25 persons can complete a work
(c) 30 (d) 20
in 140 days, then how many 26 59
persons will be required to (c) (d) 8. At a construction site, Raju can
75 75
complete the same work in 70 paint a wall in 36 hours, while
days? 5. A can do a piece of work in 10 Angad can do the same work in
days. B can do it in 15 days. With 18 hours. Sumit can paint the
#$ % & ’ same wall in 24 hours. In how
the assistance of C they completed
! ’ much time can they paint the wall
the work in 5 (days. Find in how
! if they all work together?
many days C &
% alone can do it?
) " ’7 5 *!
A

A ’ 8 9 : :
SSC CPO 09/11/2022 (Shift-01)
B , $ C ; 9 :
(a) 70 (b) 50
0 ’ $ ) # 9 :
(c) 40 (d) 80
! 1 C * ) /
3. A can do a piece of work in 10
days while A and B together can , " ) )
1 SSC CPO 09/11/2022 (Shift-03) : "
complete it in 2 days. How long SSC CPO 10/11/2022 (Shift-01)
2 (a) 20 (b) 25
will B alone take to complete the (a) 9 hours (b) 12 hours
(c) 30 (d) 35 (c) 6 hours (d) 8 hours
work?

Aditya Ranjan (Excise Inspector) Selected Selection 51

Downloaded by Kunal Chauhan (kc6970929@gmail.com)


lOMoARcPSD|39315497

Time & Work

9. 60 men can complete a work in 13. A group of men committed to 17. Reena, Veena and Vimla can do work
40 days. They start work together completing a task in 20 days. The in 4 h, 8 h and 12 h respectively.
but after every 10 day, 5 men work was completed in 40 days In how much time will they finish
leave the work. In how many days because 5 workers did not come the work working together?
will the work be completed? for work. Determine the number ) 7 )
of men who originally agreed to
8 2 3 9 : ; 9 :
do the work?
! ) ! )
46 ! ’ # !
4 + $
! ? +@ "
< -.
! &
SSC CPO 11/11/2022 (Shift-03)
! "
$ ’
SSC CPO 10/11/2022 (Shift-02) 2* 2
46 A B ’ (a) h * !
(b) 5 h
(a) 47.5 (b) 49.5 11 11
(c) 42.5 (d) 45.5
C
10. A can complete a work in 8 days 2 2
and B can complete the same (c) 3 h (d) 2 h
SSC CPO 11/11/2022 (Shift-01) 11 11
work in 11 days. How long will (a) 15 (b) 10 18. A is twice as good a workman as
they take, if both work together? B and together they finish a piece
(c) 12 (d) 18

r
A ; of work in 13 days. In how many
14.! 24 men can complete a work in days will B alone finish the work?
B ! How many men are

si
15 days. A, B
needed to complete the same
)
0 , work in
! 10 days?
an by
SSC CPO 10/11/2022 (Shift-03)
" # 46 $ ’ !
! B
!

n
SSC CPO 23/11/2020 (Shift-1)
12 12 46 (a) 18.5
) (b) 21
(a) 4 days (b) 5 days "
17 ja 19 (c) 39 (d) 42
R s
SSC CPO 11/11/2022 (Shift-02) 19. A and B can do a job in 10 days
11 12
(c) 4 days (d) 4 days (a) 16 (b) 20 and 5 days, respectively. They
a th

19 19
(c) 36 (d) 25 worked together for two days,
11. A and B can complete a work in after which B was replaced by C
15. 8 men and 12 women finish a job
36 days and 45 days respectively. and the work was finished in the
in 4 days. While 6 men and 14
They worked together for 2 days next three days. How long will C
women in 5 days. In how many
ty a

and then A left the work. In how alone take to finish 60% of the job?
days will 20 women finish the job?
many days will B complete the
; 46 # A B ’ 102 3
remaining work?
di M

A B 2 3 8 ! *5 + 8 46
$ ’
$ ! 0 + B 5 C ’
# =! #
’" !
A < -. B 6 !
C ’ 60% /
! " SSC CPO 11/11/2022 (Shift-02)
! "
SSC CPO 10/11/2022 (Shift-03) (a) 10 (b) 20
SSC CPO 23/11/2020 (Shift-1)
(a) 41.5 (b) 40 (c) 30 (d) 15
(a) 30 days (b) 25 days
(c) 41 (d) 40.5 16. Ankita can write a manuscript in
20 days. Ankita along her friend (c) 18 days (d) 24 days
12. P and Q can do a piece of work in 20. A and B can do a piece of work in
14 days. Q and R together can do Somya can write the manuscript
in 12 days. In how many days can 36 days. B and C can do the same
it in 21 days. If P is twice as good work in 60 days. A and C can do
Somya alone write the
A

a workman as R, then in how many the same work in 45 days. In how


days Q alone can do the work? manuscript? many days can A alone complete
P Q # the
- same work?A
A D B ’ 36
Q R , #
B C ’ 60
P, R # - A
A C ’
>< Q ’ D 45 A
- " A " ’ !
SSC CPO 11/11/2022 (Shift-01) SSC CPO 11/11/2022 (Shift-03) SSC CPO 23/11/2020 (Shift-2)
(a) 42 days (b) 40 days (a) 22 (b) 32 (a) 90 (b) 45
(c) 35 days (d) 38 days (c) 28 (d) 30 (c) 60 (d) 120

Aditya Ranjan (Excise Inspector) Selected Selection 52

Downloaded by Kunal Chauhan (kc6970929@gmail.com)


lOMoARcPSD|39315497

Time & Work

21. X can do a work in 3 days, Y does P Q ’12 28. P and Q together can do a work
three times the same work in 8 ! P in 12 days. P alone can do the
days, and Z does five times the same work in 36 days. In how many
’ 18 !
same work in 12 days. If they have days can Q alone complete two-
to work together for 6 hours in a Q ’ F , ’ third
/ part of the same work?
day, then in how much time can ! "
P Q
they complete the work? SSC CPO 24/11/2020 (Shift-2)
12 ! P
X, 3
’ (a) 30 (b) 21
36
Y ’ 8 (c) 24 (d) 36
Q
Z ’ 25. A can complete some work in 35
F ,’ / !
12 ) days and B can complete the same
work in 15 days. They worked SSC CPO 25/11/2020 (Shift-2)
? 6 9 : ’ E
together for 8 days, then B left
’ ! ) the work. In
1 how many days will (a) 12 (b) 18
A alone complete 60% of the (c) 21 (d) 15
SSC CPO 23/11/2020 (Shift-2) remaining work?
29. A can complete a certain work in
(a) 4 hours A, ’35 35 days and B can complete the

r
(b) 5 hours B ’15 ! same work in 15 days. They worked
(c) 5 hours 20 minutes 0 8 together for 7 days, then B left

si
(d) 4 hours 10 minutes ’ B + ’ < -the. . work. In how many days will
A alone complete 60% of the
22. A and B can do a piece of work in A 6 60%’ /
an by
36 days. B and C can do the same
work in 60 days. A and C can do SSC CPO 24/11/2020 (Shift-2)
!
remaining work?
"
A ’ 35 !

n
the same work in 45 days. In how B ’15
many days can B alone complete (a) 15 (b) 10
(c) 5 (d) 8 ! 7 0
the same work?
ja 26. A and B can do a job in 10 days ’ B = ’ < -
R s
A B 36
and 5 days, respectively. They A 6 60% ’/
B C 60
worked together for two days, after
a th

!
A C which B was replaced by C and
45 B the work was finished in the next SSC CPO 25/11/2020 (Shift-2)

! three days. How" long will C alone (a) 7 (b) 15


take to finish 40% of the job?
ty a

SSC CPO 24/11/2020 (Shift-1) (c) 10 (d) 8


A B 10
2 3
(a) 90 (b) 60 30. 4 boys from school A and 6 boys
5
di M

(c) 120 (d) 45 from school B together can set up


23. A and B can do a piece of work in +
an exhibition in 5 days, which 5
2 B 5 C boys from school A and 10 boys
25 days. B alone can do 66 % !C * school C together can do in
from
3
of the same work in 30 days. In 40% 5 4 days or 3 boys from school B and
how many days can A alone do ! " 4 boys from school C together can
4 do in 10 days. Then how many boys
part of the same work? SSC CPO 25/11/2020 (Shift-1)
15 from school A can set up the ex-
(a) 15 days (b) 18 days hibition in one day?
A B 25
(c) 10 days (d) 12 days
2 27. A is twice as good a workman as ) G A -. B ) G
B 66 %
3 B, and together they finish a piece 8 -. ? ’
/ 30 A of work in 13 days. In how many A ) $G
days will A alone finish the work?
A

4 -. )C G -.
15
/
A, B B
" * ! ) B -. C ) G
SSC CPO 24/11/2020 (Shift-1) 13 A! -.
(a) 15 (b) 20 ! ? ’ A)" G -.
(c) 18 (d) 12 SSC CPO 25/11/2020 (Shift-1) "
24. P and Q together can do a work
1
in 12 days. P alone can do the (a) 41 (b) 19 SSC CPO 09/12/2019 (Shift-01)
same work in 18 days. In how many 2
(a) 60 (b) 40
days can Q alone complete two- 1
third part of the same work? (c) 9 (d) 39 (c) 20 (d) 80
4

Aditya Ranjan (Excise Inspector) Selected Selection 53

Downloaded by Kunal Chauhan (kc6970929@gmail.com)


lOMoARcPSD|39315497

Time & Work

31. If 10 men can complete a piece of 4 2


work in 12 days by working 7 hours 34. A can do of a work in 20 days and A ’ 5 / #
5
a day, then in how many days can
14 men do the same work by work- 3 2
B can do of the same work in 15 *+ B ’66 % /
ing 6 hours a day? 4 3
% & ? ( 9 days.
: They work together for 10 8 !
days. C alone complete the re- ’
# Hmaining work in 1 days, B and C
, % & ? 8 9 : ’ B
3 "
H
together can complete of the
4
" SSC CPO 11/12/2019 (Shift-02)
same work in:
(a) 6 Days (b) 4 Days
SSC CPO 09/12/2019 (Shift-01)
4 (c) 8 Days (d) 9 Days
(a) 15 (b) 16 A / #
5 37. Working 7 hours a day, 18
(c) 10 (d) 12 persons can complete a certain
3
*+ B / $ work in 32 days. In how many
2 4 days would 14 persons complete
32. A can do of a work in 6 days

r
5 ! ) the same work, working 8 hours
C 6 a day?
2

si
and B can do of the same work ! B C ’ ; % & ?
3
3 # !
an by
in 12 days. A and B worked
together for 6 days. C alone
completed the remaining work in
, 4
!
/
"
’ % &
!
? ; 9

n
8 days. A and C working together SSC CPO 11/12/2019 (Shift-02)
will complete the same work in: SSC CPO 11/12/2019 (Shift-01)
(a) 42 (b) 30
ja (a) 8 days (b) 5 days (c) 35 (d) 36
R s
2
A / 8 !
(c) 4 days (d) 6 days
5 38. A works twice as fast as B and B
a th

2 35. A and B together can do a certain works twice as fast as C. All


B / work in x days. Working alone, A
3 three working together can finish
# A! B and B can do the same work in a task in 4 days with the help of
8 (x + 8) and (x + 18) days, D. If D alone can finish the same
ty a

6 C ; !respectively. A and B together will task in 16 days, then in how


A C 5 many days will A alone finish 75%
di M

! complete
" of the same work in:
6 of the same task?
SSC CPO 09/12/2019 (Shift-02)
A B x A, B *
(a) 12 Days (b) 9 Days
!A B F B, C *
(c) 10 Days (d) 8 Days (x + 8)2 3 D
33. A, B and C can do a piece of work (x + 18) ! A ! D
in 30 days, 45 days and 90 days
respectively. A started the work 5 , 8
B ’
and he is assisted by B and C 6 A , ($J /
together on every third day. In / ! " ! "
how many days will complete the
work? SSC CPO 11/12/2019 (Shift-01) SSC CPO 12/12/2019 (Shift-01)
(a) 9 days (b) 8 days (a) 9 (b) 8
A

A, B C 2 3
$ I (c)! 10 days (d) 12 days (c) 7 (d) 6
A C B C 39. A can finish one-third of a work
2
36. A can do of a work in 12 days
5 2
! * " in 5 days, B can finish of the
5
2
SSC CPO 09/12/2019 (Shift-02) while B can do 66 % of the sam same work in 10 days and C can
3 finish 75% of the same work in
(a) 23 (b) 24
work in 16 days. They work 15 days. They work together for 6
1 together for 10 days. B alone will days. The remaining work will be
(c) 22 (d) 30 complete the remaining work in: finished by B alone in:
2

Aditya Ranjan (Excise Inspector) Selected Selection 54

Downloaded by Kunal Chauhan (kc6970929@gmail.com)


lOMoARcPSD|39315497

Time & Work

A F ,’ / SSC
$ CPO 12/12/2019 (Shift-02) SSC CPO 13/12/2019 (Shift-01)

2 (a) 20 Days (b) 24 Days (a) 27 (b) 24


! B
5
(c) 18 Days (d) 22 Days (c) 30 (d) 32
/ !
C ($J $ 42. 3 men and! 5 women together can 44. If 45 persons can complete a
) 8 complete a work in 6 days, work in 18 days, working 8 hours
whereas 4 men and 9 women a day, then how many persons
B 6 together can do it in 4 days. How are required to complete two-
! " many women are required to do thirds of the same work in 20
the same work in 7 days? days, working 9 hours a day?
SSC CPO 12/12/2019 (Shift-01)
46 $ $ % & ? ; 9 :
1 8 ! *+
(a) 2 Days (b) 1 Days ;4 6 !
2 I F ,’ / ?
(c) 3 Days (d) 5 Days H , ( # !
H % & )
40. Vivek can do a certain work in 14
) "

r
days. Vishal is 75% more efficient
than Vivek. How many days will SSC CPO 13/12/2019 (Shift-02)
SSC CPO 13/12/2019 (Shift-01)

si
Vishal alone take to do the same
work? (a) 24 (b) 40
(a) 15 (b) 12
)) an by ))
(c) 14 !
43. A can) complete one-third of a
(d) 10 (c) 36 (d) 30

45. A is twice as efficient as B and C

n
($J B ) , 3 is thrice as efficient as B.
work in 10 days and B can do Working together, they can finish
! " 5
ja a certain work in 5 days. A and C
R s
SSC CPO 12/12/2019 (Shift-02) th of the same work in 24 days.
They worked together for 10 days. worked together for 5 days. B
(a) 8 Days (b) 9 Days alone would complete the
a th

The remaining work was


(c) 10 Days (d) 6 Days completed by C alone in 15 days. remaining work in.

41. Vaibhav can do a piece of work in In how many days can C alone do
A, B B ’ C,
60 days. He works there for 15 2 B B ’
of the same work?
ty a

days and then Sandeep alone 3


finishes the remaining work in 30
days. In how many days will A F ,’ / ! A C
di M

Vaibhav and Sandeep working ! B , *+ $


together finish the work? 3 B
)/ ) 8 5
/ ! # ! "
) $ 0 SSC CPO 13/12/2019 (Shift-02)
+ + + C $
! ) / ) ,2 (a) 8 Days (b) 4 Days
! ! ,3 / C
(c) 6 Days (d) 5 Days
" ! "
A

Aditya Ranjan (Excise Inspector) Selected Selection 55

Downloaded by Kunal Chauhan (kc6970929@gmail.com)


lOMoARcPSD|39315497

Time & Work

ANSWER KEY
1.(d) 2.(b) 3.(b) 4.(b) 5.(c) 6.(b) 7.(c) 8.(d) 9.(a) 10.(d)

11.(d) 12.(a) 13.(b) 14.(c) 15.(b) 16.(d) 17.(d) 18.(c) 19.(c) 20.(c)

21.(c) 22.(a) 23.(a) 24.(c) 25.(c) 26.(d) 27.(b) 28.(a) 29.(a) 30.(b)

31.(c) 32.(c) 33.(a) 34.(b) 35.(c) 36.(a) 37.(d) 38.(c) 39.(b) 40.(a)

41.(b) 42.(b) 43.(b) 44.(a) 45.(d)

r
si
an by
n
ja
R s
a th
ty a
di M
A

Aditya Ranjan (Excise Inspector) Selected Selection 56

Downloaded by Kunal Chauhan (kc6970929@gmail.com)


lOMoARcPSD|39315497

Time & Work

SOLUTIONS
1. (d) 5. (c) 9. (a)
Case- I Let the total work = LCM(10, 15) Total work = 60 × 40 = 2400 units
Time taken by x and (y + z) = 3 : 1 = 30 Work done by 60 men in 10 days
= 600
Eff. = 1 : 3 = 5 : 15 A = 10 days 3
Work done by 55 men in 10 days
Case- 2 30 = 550
Time taken by z and (y + x) = 4 : 1 B = 15 days 2 Work done by 50 men in 10 days
Eff. = 1 : 4 = 4 : 16 With the help of C the whole = 500
work done in 5 days. Work done by 45 men in 10 days
Total work = 20 × 10 = 200 units
Efficiciency of (A + B + C) = 450
Time taken by x and z to complete Left work = 2400 – 2100 = 300 units
Total Work 30 300 units work done by 40 men
200 2 = Time Taken  5 = 6
the work   22 days

r
9 9 340
C's Efficiency = 6 – (3 + 2) = 1 in = = 7.5 days
40

si
2. (b)
Hence, the C alone can finish the Total days = 10 + 10 + 10 + 10 +
Let the number of required
30 7.5 = 47.5 days
percent = xan by
M 1D 1H 1 M 2D 2H 2 6.
work in 

(b)
1
 30 days
10. (d)
Let the total work be 1 unit.

n
We know, 
W1 W2 Let the total work be 1. The work can be finished by A and
A  10 days
1 88 12

ja
25  140 x  70

B  15 days B in = = =4 days
R s
1 1  1 1  19 19
Concept: The amount is   
distributed in the ratio of  8 11 
a th

x = 50
efficiency. 11. (d)
3. (b)
Let the total work be 1  1  A = 35 5
A 's efficiency  10  3
Given,    180
B's efficiency  1  2
ty a

A  10 days   B = 45 4
 15 
1 5 Work done by A and B in 2 days
A + B  2  days
di M

2 = 2 × 9 = 18
2 2 B's share =  1 2 5 0 = Rs. 500
5 Remaining Work = 180 – 18 = 162
2 1 3 7. (c) Remaining Work done by B alone
B's Efficiency  – 
5 10 10 Let the total work be 1.
162
B will finish the work in A + M  10 days = = 40.5 days
4
A  15 days
1 10 12. (a)
= 3  days  Maahi's efficiency
3
1 1 1 P 2
10 Given P = 2R  
= –  R 1
4. (b) 10 15 30
Maahi can do the same work in ATQ,
12M × 10 = 20B × 10
(P + Q) × 14 = (Q + R) × 21
M 5 1
  =  30 days 2  Q 21
A

B 3 1  
Q  1 14
Total Work =12M × 10 30
8. (d)  2(2 + Q) = 3(Q + 1)
= 12 × 5 × 10 = 600  4 + 2Q = 3Q + 3
Raju  36 2
Work done by 4 Men and 4 Boys Q=1
in 4 days = (4 × 5 + 4 × 3) × 4 Angad  18 4 72 Thus, P = 2, Q = 1 and R = 1
= 32 × 4 = 128 Sumit  24 3 Total work = (P + Q) × 14
 Work done by Altogether in = (2 + 1)×14 = 42 units
Part of the work done by 4 men Hence, Q alone can do the work
128 16 72 72
= 2  4  3 = = 8 hours 42
and 4 boys in 4 days =  in =  42 days .
600 75 9 1

Aditya Ranjan (Excise Inspector) Selected Selection 57

Downloaded by Kunal Chauhan (kc6970929@gmail.com)


lOMoARcPSD|39315497

Time & Work


13. (b) 19. (c)
A+B 25 9
Let the number of committed men
A 10 1 225
to do the work be x. B 45
10 5
ATQ, B 5 2
 x × 20 = (x – 5) × 40 4
 20x = 40x – 200 2 Days work of (A + B) = (1 + 2) × 2 = 6 work done by A alone in
Nxet 3 days A'S work = 1 × 3 = 3 15
 – 20x = – 200
 x = 10 Remaing work = [10– (6 + 3)] = 1 4
225 
14. (c) 1 work, C complete in 3 days
= 15 = 225  4 = 15 Days
Let the total number of men 10 work, C complete in = 10 × 3 9–5 15  4
required to finish the work in 10 = 30 days
days be x. 24. (c) P + Q 12 3
60% of total work, C completed
Now, 36
24 × 15 = x × 10 60 P 18 2
in = 30 × = 18 days
360 100 Q alone can finish the two-third
 x  = 36
10 20. (c) 2
15. (b) 36 
A+B 36 5 of the work in = 3 = 24 Days
ATQ, 3–2
B+C 60 180 3

r
(8M + 12W) × 4 = (6M + 14W) × 5
4 25. (c)
 32M + 48W = 30M + 70W C+A 45

si
32M – 30M = 70W – 48W  2(A + B + C) = 5 + 3 + 4 = 12 A 35 3
 2M = 22W 105
A+B+C=6
 M = 11W an by B 15 7
 Efficiency of A = 3 (A + B)'s 8 Days work = (3 + 7) × 8
M 11 A alone completed the work in =
  = 80

n
W 1
180 Remaining work = 105 – 80 = 25
Total Work = (8M + 12W) × 4 = 60 days
= (88 + 12) × 4 = 400 units 3
ja 60% of Remaing work = 25 ×
60
= 15
R s
400 21. (c)
100
Work done 20 women only = x : y : z
20  1 Remaining work done by A in
a th

= 20 days 8 12
Time 3 : : 15
16. (d) 3 5 = = 5 Days
Ankita = 20 days 45 : 40 : 36 3
Ankita + Somya = 12 days 26. (d)
1 1 
ty a

Efficiency : :
1 1 45 40 36 A 10 1
Efficiency of Somya = –
12 20 8 : 9 : 10 10
di M

B 5 2
5–3 1  8 + 9 + 10 = 27
= = Work done by A & B together in
60 30 24 first 2 days = (1 + 2) × 2 = 6 unit
Somya can write manuscript in 1 hour work =
27 Work done by A in next 3 days
1 = 3 × 1 = 3 unit
= 1 = 30 days. 24 16
 In 6 hours = 27 × 6 = 3 Remaining work = [10 – (6 + 3)] = 1
30 Remaining work 1 unit is completed
17. (d) = 5 hours 20 min. by C in 3 days.
22. (a) A + B 36 5 Now,
Reena  4 6
B+C 60 180 3
3 40
Veena  8 24 C+A 45 4 40% of total work = 10 × =4
100
Vimla  12 2  2A + 2B + 2C = 12
A

C will complete this 4 unit work


Work done by altogether in A+B+C=6
in 3 × 4 = 12 days
B's Efficiency = 2
24 2 27. (b)
=  2 hours B alone can complete the work
11 11
A : B
18. (c) 180
in = = 90 days Efficiency 2 : 1
A : B 2
Efficiency 2 : 1 Total work done by (A + B) in 13
23. (a)
Total work = (2 × 1) × 13 = 39 days = (2 + 1) × 13 = 39
A + B  25 Days
39 39 unit work completed by A in
B completed work in = 3 39 1
1
B  30 × = 45 days = = 19 days
= 39 Days 2 2 2

Aditya Ranjan (Excise Inspector) Selected Selection 58

Downloaded by Kunal Chauhan (kc6970929@gmail.com)


lOMoARcPSD|39315497

Time & Work

28. (a) A and B together can do the work


A  15 6
3 in = 8 18 = 12
P+ Q 12 Days 90 (Total Work)
36
P 36 Days B  18 5 So, x = 12 days
1
Q can completed the whole work Work done by A and B in 6 days = 5
A and B together completed the
36 (6 + 5) × 6 = 66 6
= = 18 Days Remaining work = 90 – 66 = 24
3 –1 5
Two-third of work, completed by ATQ, 'C' complete the remaining of the work in = 12  = 10 days
work in 8 days. 6
2 36. (a)
Q in = 18 × = 12 Days 24
3 Efficiency of 'C' = =3 5
29. (a) 8 A  12  = 30 days
A & C will complete the same work 2
A 35 3
105 90 3
B 15 7 B  16  = 24 days
in = (6  3) = 10 days 2
(A + B)'s 7 days work = (3 + 7) × 7 A  30
= 70 33. (a) 4
120
Remaining work = 105 – 70 = 35 A  30 3 B  24 5
60% of remaining work

r
B  45 90 Work done by A & B in 10 days
2 = (4 + 5) × 10 = 90
60 1

si
= 35 × = 21 C  90
100 Remaining work = (120 – 90) = 30
Work done by A, B and C in 3 days 'B' alone will complete the
Remaining work completed by A

in =
21
3
an by
= 7 Days
= 3A + B + C
= 3 × 3 + 2 + 1 = 12 remaining work in =
30
5
= 6 days

n
Work done (A, B and C) in 21 days
30. (b) 37. (d)
= 12 × 7 = 84
ATQ, 18  32  7 14  D  8
ja Remaining work = (90 – 84) = 6
ATQ, 
R s
(4A + 6B) × 5 = (5A + 10C) × 4 A will complete 6 unit work in = 1 1
= (3B + 4C) × 10  D = 36
6
a th

= 2 days 38. (c)


Now, (4A + 6B) × 5 = (3B + 4C) × 10 3
A: B : C
 (4A + 6B) = (6B + 8C) Total no. of required days = 21 + 2 4: 2 : 3
 4A = 8C = 23 days
ATQ, (7 + D) × 4 = D × 16
ty a

34. (b)
A:C=2:1  28 + 4D = 16D
Again, 5  28 = 12D
di M

A  20 × = 25 days
(5A + 10C) × 4 = (3B + 4C) × 10 4
28 7
D= 
 20A + 40C = 30B + 40C 4 12 3
B  15 × = 20 days
 20A = 30B 3 7
A : B = 3 : 2 Total work = 16 ×
A  25 4 3
A:B:C=6:4:3 100 'A' alone will finish 75% of the
Total work = (4A + 6B) × 5 B  20 5
Work done by A and B in 10 days 7 3
= (24 + 24) × 5 = 240 16  
= (4 + 5) × 10 = 90 same task in = 3 4 = 7 days
Number of boy's from school A can 4
Remaining work = (100 – 90) = 10
set up the exihibition in one day 'C' complete the remaining work 39. (b)
240 in 1 days
= = 40 A  15
6 10 20
A

efficiency of 'C' = = 10 12
31. (c) 1 B  25 300

10  12  7 14  D  6 3 15
 B and C together can complete C  20
ATQ, 4
1 1 Work done by A, B and C in 6 days
of the same work in
 D = 10 = (20 + 12 + 15) × 6
32. (c) 3 = 47 × 6 = 282
100 
4  75
5 = = 5 days Remaining work = (300 – 282) = 18
(5  10) 15
A  6 = 15 days 'B' finished remaining work in =
2 35. (c)
3 A and B alone can do work in 18 1
B  12  = 18 days  1 days.
2 (x + 8) and (x + 18) 12 2

Aditya Ranjan (Excise Inspector) Selected Selection 59

Downloaded by Kunal Chauhan (kc6970929@gmail.com)


lOMoARcPSD|39315497

Time & Work


40. (a) 42. (b)
50 10
Vivek Vishal ATQ, efficiency of 'C' = 
15 3
4 7 (3M + 5W) × 6 = (4M + 9W) × 4 2
Vivek can do a work in 14 days 18M + 30W = 16M + 36W 'C' alone do of the same work
3
Total work = (14 × 4) = 56 2M = 6W
2
120 
Vivek can complete the work alone M 3 in = 3 = 80  3 = 24 days
  10 10
56 W 1
in = = 8 days. 3
7 Total Work = (3M + 5W) × 6 = (3 ×
44. (a)
41. (b) 3 + 5 × 1) 6 = 84
M1  D1  T1 M2  D2  T2
Let the total work = 60 Number of women are required to 
do the same work in 7 days W1 W2
Efficiency of Vaibhav = 1
W × 1 × 7 = 84 45 18  8 M2  20  9
Vaibhav's 15 day's work = 15  
 W = 12 1 2
Remaining work = 60 – 15 = 45 3
43. (b)
Sandeep Finishes the work in 30  M2 = 24
days. A  30 4
45. (d)

r
120
A B C
45

si
Efficiency of Sandeep = = 1.5 B  40 3 2 1 3
30
Work done by A and B in 10 days T.W = (6 × 5) = 30
an by
Both can do the work in =
60
2.5
= (4 + 3) × 10 = 70
Remaining work = (120 – 70) = 50
Work done by A and C in 5 days
= 5 × 5 = 25
Remaining work = 30 – 25 = 5

n
= 24 days 'C' alone completed the remaining 'B' alone complete the remaining
work in = 15 days
5
ja work in = = 5 days
R s
1
a th


ty a
di M
A

Aditya Ranjan (Excise Inspector) Selected Selection 60

Downloaded by Kunal Chauhan (kc6970929@gmail.com)


lOMoARcPSD|39315497

Pipe & Cristen

PIPE AND CRISTEN


10

1. There are 3 taps A, B and C in a . 1A. B ’ 16 6. Pipes A, B and C can fill an empty
tank. These can fill the tank in
24 C $%’ .301 .
10 h, 20 h and 25 h, respetively. tank in hours, if all the three
At first, all three taps are opened 1 -40 # 7
simultaneously. After 2 h, tap C . 1. " # pipes$ are opened simultaneously.

is closed and tap A and B keep 10 %A & . % 1. & ’& $
A and B are filling pipes and C is
running. After 4 h, tap B is also an emptying
+ ’ % &pipe. Pipe A2 can fill
closed. The remaining tank is
SSC CPO 23/11/2020 (Shift-1) the tank in 15 hours and pipe C
filled by tap A alone. Find the

r
can empty it in 12 hours. In how
percentage of done by tap A itself. 1 1
(a) 15 (b) 12 long (in hours) can pipe B alone

si
A, B C 2 2 fill the empty tank?
(c) 20 (d) 10
an by " # $
4. Two! pipes can fill a tank in 15
hours and 4 hours, respectively,
% &
while a third pipe can empty it in #
. A,
1 .B

.
C


30
#7
"

n
C % & ’& A$ 12 hours. How long (in hours) will
B ( ) # $ it* take to fill the empty tank if A B 6
% & B ja % & ’& all the
$ three pipes are opened
. 1. C # 6 . 1
R s
+ A , simultaneously?
&- . 1. ’ . 1A. 15
$ A , ’
a th

./’ 0 * $%. C ’1 1
. 12 #
SSC CPO 10/11/2022 (Shift-02) . 1. 1 # B. 1. 7 #
(a) 75% (b) 52% ’& . 1. ’ " 3
ty a

(c) 72% (d) 32% # $ ’


SSC CPO 24/11/2020 (Shift-1)
2. Two pipes A and B can fill a tank 3 4 2
di M

in 15 hours and 18 hours, SSC CPO 23/11/2020 (Shift-2) (a) 4 (b) 6


respectively. Both pipes are
20 15 (c) 3 (d) 5
opened simultaneously to fill the (a) (b)
tank. In how many hours will the 7 7
7. When operated separately, pipe
empty tank be filled? 50 30 A takes 5 hours less than pipe B
& A
. 1 - B. ’ (c) (d)
7 7 to fill a cistern, and when operated
15 18 5. When operated separately, pipe A together, the cistern gets filled
’ & . 1 .takes 5 hours
" #less than pipe B to in 6 hours. In how much time (in
$ # ’ fill a cistern, and when2 both pipes hours) will pipe A fill the cistern,
are operated together, the cistern if operated separately?
SSC CPO 23/11/2020 (Shift-1) gets filled in 6 hours. In how much
2 2 time (in hours) will pipe B fill the 5 . ( ) ’ $
(a) 7 (b) 9 cistern, if operated separately? ’ A, . 1B. . 1 .
11 11
A

2 2 5 .’ ( ’ . ’ 5
(c) 10 (d) 8 A . B. 1 .
. 1 " ( ) ’ $ 6 .
11 11
3. Pipes A and B can fill a tank in $% & . 1$. "A’& . 1. ( )
16 hours and 24 hours, .’ ( ’ 6 ’ $ 6 ’
respectively, whereas pipe C can $ ’B& . 1 . . ’ ( ’ 3 4 & 2
empty the full tank in 40 hours. 1 , 6
All three pipes are opened SSC CPO 24/11/2020 (Shift-1)
together, but pipe A is closed
3 4 0
after 10 hours. After how many SSC CPO 23/11/2020 (Shift-2) (a) 10 (b) 18
hours will the remaining part of (a) 18 (b) 15
the tank be filled? (c) 9 (d) 10 (c) 9 (d) 15

Aditya Ranjan (Excise Inspector) Selected Selection 61

Downloaded by Kunal Chauhan (kc6970929@gmail.com)


lOMoARcPSD|39315497

Pipe & Cristen


8. A pump can fill a tank with water . 1A. B 16 14. Pipes A and B can fill an empty
in 3 hours. Because of a leak, it 24 tank
$ %in’ 6 and
. 18 .hours respectively.
While pipe C can empty the full
1 C 7 1 40
- # tank in 10 hours. If all three pipes
took 3 hours to fill the tank. In
3 . 1 . "
are opened together, then the tank
how many hours can the leak # $ 10 ’ % & will
. 1get
. filled in.
alone drain all the water of the
C % & ’& $ . 1A. B +#
tank when it is full?
’ % & :2 ;
1- . . ’ . 3
6
’ % 8SSC CPO
1 25/11/2020 (Shift-1) C

1 1 ’& . 1.
3 (a) 2
$% (b) 2 ’& $ ’
3 2
’ 6 .) $ 2
1
. ’ ’ (c) 5 (d) 5 SSC CPO 09/12/2019 (Shift-01)
2
SSC CPO 24/11/2020 (Shift-2) 12. Two pipes A and B can fill a cistern 4 1
(a) 21 (b) 15 (a) 4 hrs (b) 6 hrs
1 23 5
(c) 30 (d) 10 in 12 hours and 25 hours,

r
2
9. Pipes A, B and C can fill a tank 5 1
respectively. The pipes are opened (c) 5 hrs (d) 7 hrs

si
in 15, 30 and 40 hours, 23 2
simultaneously and it is found
respectively. Pipes A, B and C are
that due to a leakage in the
opened at 6 a.m., 8 a.m. and 10
an by 15. Three pipes, A, B, C can fill an
bottom, it took 1 hour 40 minute
a.m., respectively, on the same empty cistern in 2, 3 and 6 hours
more to full the cistern. When the
day. When will the tank be full? respectively. They are opened to-
cistern is full in how much time

n
. 1A,. B C ’ gether. After what time should B
will the leak empty the cistern?
be closed, so that the cistern gets
15, 30 40 & A
. 1 .B ’ filled in exactly 1 hr 15 min?
’& ja
A,
. B1 . C 6,
R s
1 . A,1 B. C # $
8 10 %$ # $ 12 ’ 25
2 :
a th

%$ $ 2
& . 1. " # . 1. " #
SSC CPO 24/11/2020 (Shift-2)
. ’ . & ’ 6 ’ 8 %B & . %
1. & ’&
(a) 3:20 p.m. (b) 11:20 p.m.
1 40 ’ ’9 $ ( ’ ’ $ <
(c) 7:20 p.m. (d) 5:20 p.m.
ty a

$% ’ ’6 , $ 2
10. Two pipes A and B can fill a tank
in 12 hours and 18 hours, # ’ 2
SSC CPO 09/12/2019 (Shift-01)
di M

respectively. Both pipes are SSC CPO 25/11/2020 (Shift-2) (a) 20 min (b) 45 min
opened simultaneously. In how
(a) 48 hours (b) 45 hours (c) 30 min (d) 15 min
much time will the empty tank
be filled completely? (c) 42 hours (d) 50 hours 16. Pipes A, B and C together can fill
& .A 1 . B 13. A pump can fill a tank with water a cistern in 12 hours. All the
in 1 hour. Because of a leak, it three pipes are opened together
12 18
for 4 hours and then C is closed.
’ & . 1. 1 "
took 1 hours to fill the tank. A and B together take 10 hours to
# $ # ’ 3 & fill the remaining part of the
.) $ 2 In how many hours can the leak cistern C alone will fill two-thirds
alone drain all the water of the of the cistern in:
SSC CPO 25/11/2020 (Shift-1)
tank when it is full?
(a) 8 hours 1- . . ’ . . 1A,. B C ’
A

(b) 10 hours 24 minutes ’ 6 8 1


(c) 9 hours 30 minutes * ’ # C$
1
(d) 7 hours 12 minutes 1 % & ’& A $ B ’
3
11. Pipes A and B can fill a tank in 16 +
hours and 24 hours, respectively, $% .) 6 ’ 6
C & 5’ 1-
whereas pipe C can empty the full ’ .) . %
tank in 40 hours. All three pipes
& 2
$ 2
are opened together, but pipe C SSC CPO 09/12/2019 (Shift-02)
SSC CPO 25/11/2020 (Shift-2)
is closed after 10 house. After how
(a) 2 (b) 1 (a) 40 Hours (b) 48 Hours
many hours will the remaining part
of the tank be filled? (c) 4 (d) 5 (c) 60 Hours (d) 50 Hours

Aditya Ranjan (Excise Inspector) Selected Selection 62

Downloaded by Kunal Chauhan (kc6970929@gmail.com)


lOMoARcPSD|39315497

Pipe & Cristen

17. Pipes A, B and C can fill a tank . 1P. Q . ; 1A. B ’ :


in 10, 15 and 30 hours, ’ > ’ $%’ C $ % ’ .. ) 1 .
respectively. D is an emptying . 1R. . ) * ’ # * #
pipe which alone can empty the P. 1 .Q : ’ A B ’
full tank in x hours. A, B and C
’ " # $’ . = % & ’.= &
are opened together for 3 hours
’& R . #1 . ’& ’ . =C . 1 .# C $,
and then closed. Now D is
opened which alone empties the R , @ ’ # ’
tank in 30 hours. What is the "
SSC CPO 12/12/2019 (Shift-01)
value of x? SSC CPO 11/12/2019 (Shift-01)
(a) 18 hours (b) 12 hours
(a) 30 minutes (b) 40 minutes
. 1A,. B C (c) 10 hours (d) 15 hours
(c) 35 minutes (d) 45 minutes
D
20. Pipes A and B together can fill an 23. Pipes A and B are emptying pipes
# 6 . 1 x. $ and can empty a tank in 6 hours
2
.) # empty tank. in1 6. minutes. If A and 16 hours, respectively, C is
3 a filling pipe. All the three pipe
A, B C " ’
takes 3 minutes more than B to were opened together. They took

r
# $ ’.= % & the tank,
fill ’ & then$ the time (im
minutes) in which A alone would 5
% D. 1
#. $ $ 80 minutes to empty of the

si
fill &
one-third part of the tank is: 19
x#
. 1A. B ’ # tank. Pipe C alone can fill the
%an by 1 tank in:
2
SSC CPO 09/12/2019 (Shift-02) 6 ’ A ’ & . 1A. B #
3

n
(a) 40 (b) 50 B ’ ’9 . 1.
A 7 5’ 1: - # C
(c) 60 ja (d) 45
& 6 2 . 1. .
R s
’ 3’ 4
18. Three pipes, A, B and C, can fill a 5
SSC CPO 11/12/2019 (Shift-02)
a th

cistern in 12, 18 and 24 minutes, # 7 . 1 19


.
(a) 6 (b) 5
respectively. if all the pipes are
opened together for 7 minutes, (c) 5.5 (d) 4.5 # ; ’
what will be the volume of the 21. Pipes A and B can fill a tank in .C1 . ’
ty a

water that overflows as the 36 hours and 48 hours, 2


percentage of the total volume of respectively. Both pipes are
opened together for 9 hours and SSC CPO 12/12/2019 (Shift-01)
the cistern?
di M

then A is closed. Pipe B alone (a) 36 hours (b) 42 hours


. A,
1 .B $ C will fill the remaining part of the
tank now in: (c) 48 hours (d) 40 hours
; * ’
’& . 1. > ’ ’ & A
." 1 .B 24. Two
: pipes A and B can fill a tank
" *; in 18 minutes& and 24 .minutes
1.
# ’& $ $ respectively. If both the pipes are
’ ./’ $ ’ ./6 A ?. " # $ ’.=
opened simultaneously, then
% $ 2 . 1A. % & ’& B$ . 1 .
after how much time should pipe
+ B’ be closed so that the tank is
SSC CPO 11/12/2019 (Shift-01) & 2 full in 12 minutes?
7 1 SSC CPO 11/12/2019 (Shift-02) & A
. 1 .B
(a) 26 (b) 23
18 3 1 * ’
(a) 20 hours
A

(b) 25 hours
2 . 1. " # ’&
2 5 ’ .)
(c) 23 (d) 26 (c) 27 hours (d) 24 hours
3 18 ’ . B1 . ’ % & %
22. Pipes A and B together can fill a
19. Pipes P and Q can fill a tank in 18 tank in 16 hours, whereas pipe 2
and 27 minutes, respectively, C alone can empty the full tank SSC CPO 12/12/2019 (Shift-02)
whereas pipe R can empty the full in 24 hours. A and B were
(a) 6 minutes
tank in 54 minutes, P and Q were opened together for 10 hours and
opened together for 6 minutes and then closed. Pipe C was opened. (b) 9 minutes
then closed and R was opened. The The tank will now be emptied by (c) 8 minutes
tank was emptied by R alone in: C in: (d) 5 minutes

Aditya Ranjan (Excise Inspector) Selected Selection 63

Downloaded by Kunal Chauhan (kc6970929@gmail.com)


lOMoARcPSD|39315497

Pipe & Cristen

25. A tank is filled in 4 hours by 27. Pipes A and B can empty a full SSC CPO 13/12/2019 (Shift-02)
three pipes A, B and C. The pipe tank in 18 hours and 24 hours
C is twice as fast as B and pipe respectively. Pipe C alone can fill 1
B is thrice as fast as A. How the tank in 36 hours. If the tank (a) 7 minutes
5
much time will pipe A alone take
5
to fill the tank? is full and all the three pipes
6 2
. A,1 B. C * (b) 8 minutes
are opened together, then in how 5
B . 1. many hours will the tank be
. 1C. & $ A emptied?. 1 . (c) 6 minutes
B. 1. $ & A. 1 .B
A. 1 . ’ ; " * # (d) 5 minutes
& 2 . 1C. 7 : 29. Pipe A can fill a tank of capacity
SSC CPO 12/12/2019 (Shift-02) 5
’& 6 1
(a) 25 hours (b) 40 hours 350 litres in 3 minutes. Pipe B
2
(c) 30 hours (d) 32 hours
. 1. " # ’& $
can fill a tank of capacity 780
26. Taps A and B can fill a tank in 15
’ # $ 2

r
minutes and 10 minutes, 2
respectively while tap C can SSC CPO 13/12/2019 (Shift-01) litres in 8 minutes. How long
3

si
empty the full tank in x minutes. 1 1
If all the three taps are opened (a) 10 (b) 12 (in min) will it take to fill a tank
2 2
an by
together, the tank is filled
completely in 8 minutes. Tap C (c) 10
28. Pipes A and B can fill a tank in
(d) 12
of capacity 1615 litres if both
pipes are opened together?

n
3
alone will empty part of the . 1A,. 350 9 ’ 6
8 1
tank is: 18 minutes and 22 minutes, 1
ja 2 3 ’ B,
R s
A B ’respectively while pipe C can 2
’ C empty
$ % ’ the full tank in 12 780 9 ’ 6
a th

.) x ’ # minutes. A and B are opened


together for 6 minutes and then 2
’& " # ’ & Now
closed. $ C is opened. C alone 8 ’ ’
3
; ’ .) $ in.
will empty the tank
ty a

3 " # ’& $
C ’ . 1A. B ;
8 9 ’ 6
di M

1
# & 2 ’ 22 ’ 3’ 4 2
2
SSC CPO 13/12/2019 (Shift-01) SSC CPO 13/12/2019 (Shift-02)
(a) 10 minutes
$%’ .C 1..) ’ #
1A. . B : ’ 1 1
1 ’ " # $ (a) ’ 7
.= & (b) 8
(b) 10 minutes 2 2
2
% & ’& $ C #% . 1 .
(c) 9 minutes (c) 9 (d) 8
$ C 7 ’
1 # & 2
(d) 8 minutes
2
A

ANSWER KEY
1.(c) 2.(d) 3.(b) 4.(d) 5.(b) 6.(a) 7.(a) 8.(c) 9.(a) 10.(d)

11.(a) 12.(d) 13.(c) 14.(c) 15.(c) 16.(a) 17.(b) 18.(a) 19.(a) 20.(b)

21.(c) 22.(d) 23.(c) 24.(c) 25.(b) 26.(c) 27.(d) 28.(a) 29.(b)

Aditya Ranjan (Excise Inspector) Selected Selection 64

Downloaded by Kunal Chauhan (kc6970929@gmail.com)


lOMoARcPSD|39315497

Pipe & Cristen

SOLUTIONS
1. (c) A & B  6 hours 80 80 16
A 10
= = =
10 1 1 1 8  4  3 15 3
 
B 20 5 100
So, x – 5 x 6 = 5 hours 20 minutes
C 25 4 Thus, Tank will be filled at = 3 :
x = 15(satisfies the equation)
20 pm
Hence, B can fill the cistern in
Time 2 Hr. 2 Hr. 10. (d)
15 hours.
AB A 12 3
Tap AB A 6. (a) 36
C
B 18 2
Work
38 30 Remain  32 30
Done A+B +C= hours 14 36 36
7 Total time taken = =
32 5
Total work done by A in A = 15 hours 60 4

r
= 7 hours 12 minutes
C = 12 hours –5
10  4  32 11. (a)
 100

si
percentage =
100 B's efficiency = [14 – (4 –5)] = 15 A 16 15
= 72% B 24 240 10
2. (d)
A
an by
15 hours 6
B fill the tank =
60
15
= 4 hours C 40 6
(A + B + C)'s 10 hours work done

n
90 7. (a)
= (15 + 10 – 6) × 10
B 18 hours 5 Let, B be can fill the tank in = 190
ja
Total time when tank will be filled x hour. Remaining work = 240 – 190 = 50
R s
A  (x – 5) hours Remaining work (A + B) completed
90 2
by A and B =  8 hours A & B  6 hours 50
a th

11 11
in = = 2 hours
3. (b) 1 1 1 15  10
So,  x – 5   x  6 12. (d)
A 16 15
1
240 10 x = 15(satisfies the equation) 12 2
ty a

B 24 A
2
6 A can fill the tank in = x – 5 25
C 40
= 15 – 5 = 10 hours B 25 1
di M

Total work done by (A + B + C)


in 10 days = (15 + 10 – 6) × 10 8. (c) 25
(A + B) = hours
= 190 Total Time = 3 10 3
Remaining work = 240 – 190 = 50 30 25 2
With Leakage = + 1
Remaining work done by (B + C) Time with leakage = 10 9 3 3
3
50 50 1 25 5
= = = 12 hours Drain the whole water by leak =  = 10 hours
10 – 6 4 2 3 3
4. (d) 30
= = 30 hours 25
10 – 9 (A + B) 12
I 15 4 3
100
60 15 9. (a)
II 4 With Leakage 10 10
5 A 15 8
III 12 Time taken by the leak to empty
A

B 30 120 4 the cistern -


Total time taken by three pipes
40 3
C 100
60 = = 50 hours
to fill empty tank = A does work 6 am to 10 am 12 – 10
4  15 – 5
= 4 × 8 = 32 13. (c)
60 30 A 1 hour 4
= = hours B does work 8 am to 10 am
14 7 4
=2×4=8 4
With Leakage A hour 3
5. (b) 3
Total work done = 32 + 8 = 40
Let, B be can fill the tank in x Leakage pipe can empty the tank
hour. Remaining work = 120 – 40 = 80
4
A  (x – 5) hours (A + B + C) complete 80 work in = = 4 hours
4–3

Aditya Ranjan (Excise Inspector) Selected Selection 65

Downloaded by Kunal Chauhan (kc6970929@gmail.com)


lOMoARcPSD|39315497

Pipe & Cristen

14. (c) 18. (a) 23. (c)

A6 A  12 A6 8
40 6
30 B  18 4 72 48
B8 240 B  16 3
C  24 3
C  10 –24 All the pipes opened together for Let the efficiency of pipe 'C' be x.
All pipes are opened together, then 7 minutes = (6 + 4 + 3) × 7 = 91 80 5
the tank will get filled in So, the water that overflow Then, (8 + 3 – x) ×  48 
60 18
240 240 5 = (91 – 72) = 19
= (4030 – 24)  46  5 23 hours 19 7 4 40
% change = × 100% = 26 %  (11 – x) × 
15. (c) 72 18 3 3
19. (a) x=1
A  2
3 P  18 Hence, pipe 'C' alone can fill the
3
2
54 48
B  3 6 Q  27 2 tank in = = 48 hours
1
1 'R' can empty the full tank in 54
minutes 24. (c)
C  6

r
54 A  18 4
Pipe A & C can fill the tank in So, efficiency of 'R' = =1
54 72

si
1 5 P and Q opened together for 6 B  24 3
=1   hrs.
4 4 minutes = (3 + 2) × 6 = 30 Work done by pipe A in 12 min
an by 5
Pipe A & C filled the tank = × 4
4
R can empty the tank


30
 30 min.
= 4 × 12 = 48

n
1 Remaining work = 72 – 48 = 24
= 5 unit 20. (b)
Pipe B should be closed after Let, B can fill the tank in x 24
ja A  (x + 3)
Time taken by pipe B =
3
= 8 min
R s
1
= × 60 = 30 minutes 1 1 3 Thus, pipe B should be closed
2 So,  
a th

x  3 x 20 after 8 min.
16. (a) On solving, x = 12 25. (b)
(A, B and C) together can fill a B  12
cistern in 12 hours A  15 A B C
ty a

Let, the total work = 60 1 1 3 6


So, pipe A can fill of the tank Tank capacity = (1 + 3 + 6) × 4 = 40
60 3
di M

efficiency of (A, B and C) = =5


12 1 40
in = 15  = 5 minutes 'A' alone take to fill the tank =
Work done by all three pipes in 4 3 1
hours = 4 × 5 = 20 21. (c) = 40 hours
Remaining work = (60 – 20) = 40 26. (c)
A  36 4
A and B together take 10 hours to
144 A  15
fill the tank. 8
B  48 3 12
40 B  10 120
Both pipe opened together for 9
efficiency of (A and B) = =4 A+B–C8 15
10 hours = (4 + 3) × 9 = 63
efficiency of C = (5 – 4) = 1 Remaining = (144 – 63) = 81 Let tap 'C' will empty the tank in
'C' alone fill two-thirds of the 'B' alone fill the remaining part 'x' minutes.
2 81 ATQ,
60  = = 27 hours
cistern = 3 = 40 hours 3 8 + 12 – x = 15
A

1 22. (d) 20 – x = 15
17. (b) A + B  16 3 x = 15
A  10 3 48
Total Work = 120
B  15 2 C  24 –2
30
(A + B) opened together for 10 3
C  30 1 hours and then closed 'C' alone will empty part of the
(A, B, C) opened together for 3 8
Tank filled = 3 × 10 = 30
hours = (3 + 2 + 1) × 3 = 18 'C' will emptied the tank filled in 120  3
30 30 tank = 8 = 9 minutes
tank in = × 30 = 50 hours =  15 hours. 5
18 2

Aditya Ranjan (Excise Inspector) Selected Selection 66

Downloaded by Kunal Chauhan (kc6970929@gmail.com)


lOMoARcPSD|39315497

Pipe & Cristen


27. (d) T.W = 180 2
A  18 –4 Work done by pipe A and pipe B in Pipe B can fill a tank in 8 min
–3 3
B  24 72 6 min = (10 + 8) × 6 = 108
Pipe 'C' alone empty the tank = 780 litres
C  36 +2
Pipe B can fill a tank in 1 min
Total capacity of tank = 72 108 1
=  7 min 3
5 5 15 5 = 780 × = 90 litres
of the tank = 72  = 60 26
6 6 29. (b)
all three pipes are opened together Pipe A and pipe B together fill the
then, 1 tank in 1 min = 100 + 90 = 190
Pipe A can fill a tank in 3 min
Required time to emptied the tank 2 litres
60 60 = 350 litres pipe A & pipe B can fill the tank
=  = 12 hours Pipe A can fill a tank in 1 min of 1615 Litre in
–4 – 3  2 –5
28. (a) 2 1615 1
= 350 × =  8 min
A  18 7 190 2
10 = 100 litres
B  45 8 180
2
– 15
C  12

r
si
an by 

n
ja
R s
a th
ty a
di M
A

Aditya Ranjan (Excise Inspector) Selected Selection 67

Downloaded by Kunal Chauhan (kc6970929@gmail.com)


lOMoARcPSD|39315497

Time & Distance

TIME & DISTANCE


11

1. On a straight road, a bus is 30 4. Amit and Sumit start walking 7. A car travels with a speed of 21
km ahead of a car travelling in from same point in opposite m/sec in the first 10 minutes, 9
the same direction. After 3 hours, direction at the speed of 6 km/h km in the next 10 minutes and
the car is 60 km ahead of the bus. and 4 km/h respectively. How far 10 km in the last 10 minutes of
If the speed of the bus is 42 km/h, will they be from each other after its journey. What is the average
then find the speed of the car. speed of the car (in km/h) during
4 hours?
its entire journey?
% 2 % 6 2 3 # %& 5 10 #!

r
7 8 " )21 m/sec
! ’ %
! "

si
) ! % # = 2 9 # 9! km
% % = #
# $ % & ’(
’ ! 3 , 10 km %& #5 % #
) an by
! %
SSC CPO 09/11/2022 (Shift-01)
,
* %

SSC CPO 09/11/2022 (Shift-03)


0
:& 0
# (km/h%)

n
(a) 67 km/h (b) 72 km/h
(c) 65 km/h (d) 59 km/h (a) 500 m (b) 35 km SSC CPO 10/11/2022 (Shift-01)
2. ja
A car covers a particular distance (c) 40 km (d) 300 m (a) 60 km/h (b) 63.2 km/h
R s
in 3 hours with the speed of 54 5. A man goes from place A to B at (c) 62 km/h (d) 65.3 km/h
km/h. If the speed increased by 27
a speed of 12 km/h and returns 8. A car cover the first 100 km at a
a th

km/h, the time taken by the car


to cover the same distance will be: from B to A at a speed of 18 km/ speed of 50 km/h. It covered next
h. His average speed for the 140 km at a speed of 70 km/h.
+’ ) !
entire journey is: What is its average speed?
# % , ! %& %
ty a

& (- ) ! 1 & : %A ; <B %# = ( ) ! # 100 km ,50


. & $ % % / $, % B A % km/h % %& #
di M

%& # %# &
=> )0 ! 140
! % km ,,70 km/h %
SSC CPO 09/11/2022 (Shift-02) & 5 # %% & * % % % :&
(a) 2 h (b) 1 h SSC CPO 10/11/2022 (Shift-01)

1 1 SSC CPO 09/11/2022 (Shift-03) (a) 60 km/h (b) 62 km/h


(c) 2 h (d) 1 h
2 2 (c) 58 km/h (d) 64 km/h
2
3. Shivi drives her car on three (a) 14 km/h (b) 15 km/h 9. The speeds of two bodies are in
5
difference ways to reach the the ratio 2 : 3. If the difference
destination. Determine the (c) 6 km/h (d) 30 km/h in the time taken to cover 50 m
average speed during this journey. is 10 sec, then find the difference
6. If a car covers 75.5 km in 3.5
%1& % 2 # 3 of #petrol, how much
litres in their speeds.
% # 4 % %
distance (in km) will it cover in #2 %
A

& 5 # % % 28* litres


% of petrol?
$ & + , %& #
Speed Distance & ?+ ! !@ & - + ?%+ =
Way
(Km / h) (km)
, %& % % 3 (%> * % $
A 38 114
B 42 84
! !@ %# ,SSC CPO
A 10/11/2022
B (Shift-02)
C 46 230 %& 0 8 5
(a) m/sec (b) m/sec
SSC CPO 10/11/2022 (Shift-01)
9 6
SSC CPO 09/11/2022 (Shift-02)
(a) 42.8 km/h (b) 41.5 km/h (a) 603 (b) 602 6 7
(c) m/sec (d) m/sec
(c) 43.5 km/h (d) 41.8 km/h (c) 604 (d) 600 5 5

Aditya Ranjan (Excise Inspector) Selected Selection 68

Downloaded by Kunal Chauhan (kc6970929@gmail.com)


lOMoARcPSD|39315497

Time & Distance

10. A truck goes from Haryana to 13. A bus travels at 100 km/h for the 17. A theif is spotted by a policeman
Banglore with an average speed first 1/2 hour. Later it travels at from a distance of 250 m. When
of 60 km/h. The journey takes 30 80 km/h. Find the time taken by the policeman starts the chase,
hours. It returns from Banglore the bus to travel 290 km.
to Haryana on the same road with the theif also starts running.
an average speed of 40 km/h. =)( ! =
Assuming that the speed of the
What was the average speed of ) ! % thief is 10 km/h and that of the
the truck during the whole & > ) ! policeman
% is 12 km/h, how far
journey? (I , % & would
# the theif have run before
!@ & C / % " & & & * % he is $caught?
) ! % $ CPO
SSC % 10/11/2022 (Shift-03) 2 (+
& 5 ! % &
(a) 4 hours (b) 3.5 hours L % $ 2
& C ’
(c) 3 hours (d) 2.5 hours # F D % %
) ! % ! % of A and B are in the
14. The speeds F D % & #
, & 5 # ! @ ratio 3 : 5.%A takes 30 minutes
more than B to reach the
= ) ! 2
:& < 0
destination. In how much time = ( ? ) ! %
SSC CPO 10/11/2022 (Shift-02) does A reach the destination?

r
$ # %# ,
(a) 60 km/h (b) 40 km/h A B 8 + # 2SSC %
CPO 09/11/2022 (Shift-01)

si
(c) 50 km/h (d) 48 km/h % 1 & % A, B 2 J %#2 # (a) 4 km (b) 1.25 km
11. In a journey of three unequal #! &A % (c) 3 km
an by (d) 2 km
laps, a car covers a distance of %1& % %# & 18. A policeman
2J % 0
sees a theif at a
200 km in 4 h in the first lap,
SSC CPO 11/11/2022 (Shift-01) distance of 150 m. He starts

n
while another 162 km at the
(a) 1 hour 15 minutes chasing the theif who is running
speed of 15 m/s in the second at a speed of 5 m/sec, while the
lap. It covered the remaining (b) 1 hour 10 minutes
ja (c) 1 hour
policeman chases with a speed
R s
distance of the final lap in 4 h of 7m/sec. Find the distance
such that the average speed of (d) 1 hour 5 minutes covered by the theif when he is
a th

the car for entire journey was 50 15. During a journey of 120 km, Rahi caught by the policeman.
km/h. What was speed of the car drives first 60 km at the speed 2 =+ ,
in the third lap of the journey? of 60 km/h, next 30 km at 60 km/
L % 3 +
h and the remaining 30 km at the
% # # D & 5 speed of 30 km/h. Determine the D
ty a

D ( average
, ’ speed.
! %D $ 2
%& % $ , =D( = "& ( 5 # )
C K
di M

, =+ ) ? " %& , " ) ! 2 % / $ #


% # % D E , , " ) / ! %& , *
% E SSC CPO 09/11/2022
, (Shift-02)
’ ! F %&
) ! % %&(a) 285%m (b) 325 m
, & 5 %
% % * % $ (c) 375 m (d) 295 m
+ ) ! & 5 % D
SSC CPO 11/11/2022 (Shift-03) 19. A theif, seeing a policeman from
:& < 0 a distance of 120 metres, starts
(a) 48 km/h (b) 33.33 km/h
SSC CPO 10/11/2022 (Shift-02) running at a speed of 7 km/h. The
(c) 50 km/h (d) 40 km/h
policeman chases immdiately with
(a) 47 km/h (b) 52 km/h 16. A car starts at 3 p.m. at a speed a speed of 8 km/h and the theif
(c) 42 km/h (d) 45 km/h of 50 km/h. Another car follows is caught. What is the distance
it at 4 p.m. at a speed of 75 km/ run by the theif?
12. A man goes 24 m towards east
A

h. At what time will both the cars


and then 10 m towards north. meet? =( ! ,
How far is he away from his 2 L -
3 p.m. + ) !
initial position?
# F D % D, # F D %
(’ ! ,3 4 p.m. -+ ) ! %2 % > ) !
G = H $ % K 3# 29 % # %
# F D ;< % %# & , 0 0 D , %# 0
SSC CPO 10/11/2022 (Shift-02) SSC CPO 11/11/2022 (Shift-03) SSC CPO 11/11/2022 (Shift-02)
(a) 23 m (b) 25 m (a) 6 pm (b) 5 pm (a) 820 m (b) 780 m
(c) 26 m (d) 24 m (c) 7 pm (d) 8 pm (c) 840 m (d) 80 m

Aditya Ranjan (Excise Inspector) Selected Selection 69

Downloaded by Kunal Chauhan (kc6970929@gmail.com)


lOMoARcPSD|39315497

Time & Distance

20. A and B run a 12 km race on a 5 40 km , % &A #B


circular track of length 1200 m. 24. Geeta runs times as fast as 2 30! #! % A &
2
They complete one round in 300 # 2# %B %
Babita. In a race, if Geeta gives
seconds and 400 seconds
a lead of 40 m to Babita, then find 1 ! & %&
respectively. After how much time the distance from the starting
from start will the faster person A / 40 km , %& #
point where both of them will
meet the slower person for the meet (correct up to two decimal 3 & A ! B * %
last time? places). SSC CPO 23/11/2020 (Shift-1)
A B =( 3MH !@ (a) 7 (b) 5
5
=( % % 3 7 2%8 2 # % $ (c)
% 6 (d) 9
28. A person travels a distance of 300
’ :
, % F D %# & & % %km and then’ returns to the starting
point. The time taken by him for
% $ 1& :% 1& :% %. % % 0 % F the outward
D 2 is 5 hours more
journey
SSC CPO 10/11/2022 (Shift-03) 2 % , * % $ the
than $ time taken
3 for the return
(a) 2400 sec (b) 8400 sec # A 3 journey. If ;he< returns
# at a speed
of 10 km/h more than the speed
(c) 9600 sec (d) 10800 sec % ,C N %B of going, what is the average speed

r
21. In a race of 1200 m, Ram can beat SSC CPO 10/11/2022 (Shift-01) (in km/h) for the entire journey?
Shyam by 200 m or by 20 sec.
1300
& km
:% , %& %

si
What must be the speed of Ram? (a) 66.67 m (b) 65 m
1200 m 200& (c) 65.33 m (d) 66 m G D 6 2
m & (an by
:&
C
# 0
% Three persons P, Q, R run along
25.
a circular track at speeds of 3
/ $ #
& + !
&

n
km/h, 4 km/h, 6 km/h, $ # 10 km/h
SSC CPO 11/11/2022 (Shift-01)
(a) 14 m/sec (b) 12 m/sec respectively. If the length of the 3 !% %
ja track is 36 km, then after how # (km/h
% ) * %
(c) 10 m/sec (d) 16 m/sec
R s
much time will they meet again SSC CPO 23/11/2020 (Shift-2)
22. A gives B a head-start of 10
seconds in a 1500 m race and both at the starting point? (a) 24 (b) 30
a th

fininsh the race at the same % # 1P,& Q,: R


% 3MH (c)
! @20 (d) 15
time. What is the time taken by
A (in minutes) to finish the race 7 3 km/h,
8 4 km/h, 6 km/h 29. A bus covers a 50 km distance in 1
hour 15 minutes, whereas the same
if speed of B is 6 m/s? % & !@
ty a

distance is covered by a car in 45


A, =+ ! B = 36 km % 3 G 2 O What
minutes. % is the ratio
2 of the speed
29 % A 4 ; ! ! B % #% & 0 of the bus to the speed of the car?
di M

# & , % 50 km 1, 15 ! #!
SSC CPO 10/11/2022 (Shift-02)
B& " ) ?A % %& % $
, # A (a)# ! 38 hours
B %(b)# 36 hours 45 #! %& %
& 0 (c) 24 hours (d) 28 hours
#2 % *
SSC CPO 11/11/2022 (Shift-02) 26. How much percentage should a
SSC CPO 23/11/2020 (Shift-2)
(a) 3 (b) 4 racer increase speed to reduce
the time by 20% to cover a fixed (a) 1 : 3 (b) 3 : 1
(c) 8 (d) 5
distance? (c) 5 : 3 (d) 3 : 5
23. A can run 250 m in 25 sec and 30. A bus covers a 60 kilometre
B in 30 sec. How many metres # % , %& #distance%in2 1 (hour
P 30 minutes,
start can A give to B in a km race
& # 3 whereas
% # the same distance is
so that the race may end in a
covered by a car in 45 minutes.
dead-heat? F % % . #
A

What is the ratio of the speed of


(+ , %A& 25 # SSC CPO 10/11/2022 (Shift-03) the car to the speed to the bus?
sec & % $ %(a)
# 25% (b) 30% = ! #! "
, % & B # 30 sec & (c) 40% (d) 35% , %& % $
% A, B / ’+ #! %&
27. A takes 2 hours 30 minutes more
%# ! . % $ B to walk 40 km. If A doubles
than
! $ his speed, then he can make it #2 % * %
SSC CPO 11/11/2022 (Shift-03) in 1 hour less than B. How much SSC CPO 24/11/2020 (Shift-1)
(a) 169.53 m (b) 173.82 m time (in hours) does A require (a) 2 : 1 (b) 1 : 2
(c) 166.67 m (d) 186.34 m for walking a 40 km distance? (c) 3 : 5 (d) 5 : 3

Aditya Ranjan (Excise Inspector) Selected Selection 70

Downloaded by Kunal Chauhan (kc6970929@gmail.com)


lOMoARcPSD|39315497

Time & Distance

31. A person covers a distance of 300 34. A person has to travel a distance A B # & X 7 8
km and then returns to the of 30 km. He finds that he has Y Y X ;< # 4 ,
starting point. The time taken by
5 # 29 & 4 ,
him for the outward journey is 5 covered part of the distance in
hours more than the time taken 6 A B Y X % 2 #
for the return journey. If he 3 hours and 20 minutes. 7 8 (?’+ ! ’? +
returned at a speed of 10 km/h What is his speed (in km/h) ? &B >?’ ) !
more than the speed of going, what
was the speed (in km/h) for the
1& :% & 5A # A ) ! B *
outward journey? 5 SSC CPO 11/12/2019 (Shift-01)
& 5 6 D , & #
1 & 300
: %km , %& % (a) 10.8 (b) 9.9
G D 6 2 ! ! %( #! %& & 12.6
(c) % (d) 11.7
$ # / & & A
& 3 ) ! B : & 37. 0
A person takes 40 minutes more
# & + !
SSC CPO 09/12/2019 (Shift-01) than his usual time when he
& 3 $ # 10 % 2 (a) #6 covers a distance of 20 km at 5
(b) 54
km/h. If he covers the same
km/h ! %
(c) 6.3 $ % (d) 7.5 distance at 8 km/h, he take x
& (km/h ) : & < 35.0 A person travels a distance of 240 minutes less than the usual

r
SSC CPO 24/11/2020 (Shift-1) km partly by train and the rest by time.
(a) 30 (b) 15

si
What is the value of x?
1
(c) 25 (d) 20 bus. He takes 3 hours if he 1& :% ( , +
2
an by
32. A boy increases his speed to
9
5
travels 150 km by train and the
rest by bus. If the travels 140 km
’ #!
%& #
& %
#

n
times of his original speed. By & , > ) !
doing this, he reaches his school by bus and the rest by train, he
% x 6& #! & &
40 minutes before the usual time.
ja 2
takes 3 hours. What is the % x # :& 0
R s
How much time (in minutes) does
3
he take usually? SSC CPO 11/12/2019 (Shift-02)
a th

# , speed of the train? (a) 48 (b) 54


9 1& :% (’ , (c) 45 2K (d) 50
% 2 #5 2 # 3 M Q %
, % & 38. %
Ranjeet drives hisEcar at an
ty a

3 # ; , 6& & ’, #! / %& % &


average 3 = of
speed + 50 km/h and
2 % 6& ,# reaches
E his
, destination in 8
hours. Rahman covers the same
di M

%# & A #! B % 0
1 distance in 5 hours. If Ranjeet
SSC CPO 24/11/2020 (Shift-2) %& % 3
% ! & 5
2 increases his speed by 10 km/h
(a) 120 (b) 30
& % & 3 =’ and Rahman increases his speed
(c) 90 (d) 45 by 20 km/h, then what will be the
33. A takes 2 hours 30 minutes more , E , %&
difference in time taken by them
than B to walk 40 km. If A doubles 2 to reach their destination?
his speed, then he can make it % % 3 ! & %
3 $ % # + ) !
in 1 hour less than B. What is
the average time taken by A and %# 0 % > !
B to walks a 40 km distance?
SSC CPO 09/12/2019 (Shift-02) 2J $ % # %
40 km , % &A #B
%& % & $ %
2 30
! #! & % (a) 72 km/h (b) 75 km/h
) ! . %
A

& A # 2 # % $ (c) 80 km/h% (d) 70 km/h


% B 1 ! & % ( ) ! . % %
36. A and B started travelling towards
A B / 40 km , % &each other at the same time from J2 # # / $
# # 3 ; % places
& AX to Y! and Y to B X respec- %# % 0
* % tively. After crossing each other, SSC CPO 12/12/2019 (Shift-02)
SSC CPO 25/11/2020 (Shift-1) A and B took 2.45 hours and 4.05 (a) 2 hours 30 minutes
(a) 5 hours 45 minutes hours to reach Y and X respec-
(b) 3 hours 40 minutes
(b) 7 hours 15 minutes tively. If the speed of B was 8.4
km/h, then what was the speed (c) 3 hours 20 minutes
(c) 5 hours 15 minutes
(d) 6 hours (in km/h) of A? (d) 2 hours 40 minutes

Aditya Ranjan (Excise Inspector) Selected Selection 71

Downloaded by Kunal Chauhan (kc6970929@gmail.com)


lOMoARcPSD|39315497

Time & Distance

39. If abhi travels a certain distance SSC CPO 13/12/2019 (Shift-02) 1 & : % #A ; ! ; ! B #%# 40
at 6 km/h, he reaches his 3 km/h B A % 30 km/h
destination 12 minutes early, but (a) 4
7
if he travel at 4 km/h, he reaches , %& 6.3
, & !5
his destination 10 minutes late. 5 A B (km , )
The speed (in km/h) at which he (b) 4
7
should travel to reach his %# 0
destination on time is. 4
(c) 5 SSC CPO 13/12/2019 (Shift-02)
& D # 6 km/h
% , 4
%& % % 3% 1 1&# =( #! (a) 117
2J $ % # &(d) 5 38 ,
(b) 108
4 km/h %& % 40. A person
% 3travelled from station A to
# %1& = #! station B at 40
2 Jkm/h% and from B (c) 99
# %1& % & to A at2 30J # km/h. The entire
(d) 91
(km/h ) , % & journey took 6.3 hours. What is
the distance (in km) between A
# 0 and B?

r
si
an y ANSWER KEY

n
1.(b) 2.(a) 3.(a) 4.(c) 5.(a) 6.(c) 7.(b) 8.(a) 9.(b) 10.(d)

11.(a) ja
12.(c) 13.(b) 14.(a) 15.(a) 16.(a) 17.(b) 18.(c) 19.(c) 20.(a)
R s
21.(b) 22.(b) 23.(c) 24.(a) 25.(b) 26.(a) 27.(a) 28.(a) 29.(d) 30.(a)
a th

31.(d) 32.(c) 33.(a) 34.(d) 35.(b) 36.(a) 37.(d) 38.(d) 39.(b) 40.(b)
ty a
di M
A

Aditya Ranjan (Excise Inspector) Selected Selection 72

Downloaded by Kunal Chauhan (kc6970929@gmail.com)


lOMoARcPSD|39315497

Time & Distance

SOLUTIONS
1. (b) 6. (c) 11.(a)
Given that, the bus is 30 km Given, Lap Distance Time Speed

ahead of car and after 3 hour the Car covers 75.5 km in 3.5 litre of First 200 km 4h 50 km/h
D 162 5
car is ahead of 60km petrol  
Second 162 km S 15 18 15 m/s
Speed of bus = 42 km/h  Car will cover the distance in 3h
28 litre petrol Third — 4h —
Let the speed of car = x km/h
75.5 Total distance
Distance =  28 = 604 km = 50 × 11 = 550 km
3.5
Speed = Time Taken Remaining distance for third lap
7. (b)
= 550 – 362 = 188 km
90 Case I
 x – 42 Speed of car in third lap
3 Speed = 21 m/s

r
 x 30 4272 km /h Time = 10 m = 600 s 188
= = 47 km/h
Distance = Speed × Time = 21 × 4

si
2. (a)
600 = 12600
Distance covered by the car = 54
× 3 = 162 km = 12.6 km 12. (c)
an by
After the increment speed of car
by 27 km/hr.
Hence, Total Distance Covered
= 12.6 + 9 + 10 = 31.6 km
Distance from Initial Position

n
= 242  102
= (54 + 27) = 81 km/h Total Time Taken
1 = 576  100 = 676 = 26 m
162 km
Time taken  ja = 2 hour = 10 + 10 + 10 = 30 
60
h = 0.5h
R s
81 km / hr
3. (a)
31.6 km
a th

Time taken in way A Average Speed = = 13. (b)


0.5h
114 63.2km/h Speed Time Distance
 = 3 hour
38 8. (a) 1
100 km/h h 50 km
2
ty a

Time taken in way B


Total Distacne Remaining
Average speed =
84 Total Time 240 distance
 = 2 hour 80 km/h =3 h
di M

42 80 =290–50
Time taken in way C 100  140
= = 60 km/h =240 km
100 140
230  Total time = 0.5 + 3 = 3.5 hours
 = 5 hour 50 70
46 14. (a)
9. (b)
Total time taken = (3 + 2 + 5) = If distance is constant, time is
Given, inversely proportional to speed.
10 hour
Total Distance Covered S1 : S 2 = 2 : 3 Given sA : sB = 3 : 5
= (114 + 84 + 230) = 428 km If D is constant then T is Inverely tA : t B = 5 : 3
proportional to the S. Time difference, t A – t B = 30
428 minutes
Average speed  = 42.8 km/h T1 : T2 = 3 : 2
10  2 units = 30 minutes
1 unit = 10 s
4. (c)  1 units = 15 minutes
T1 = 30 s and T2 = 20 s
A

Relative Speed = (6 + 4) = 10 km/h Hence, Time taken by A, tA


Time = 4 h 50 50 = 5 × 15 minutes
S2 – S1  –
Distance = Speed × Time 20 30 = 75 minutes = 1 hour 15 minutes
= 10 × 4 = 40 km 15. (a)
5. (a)  3 – 2 5
 50   = m/s
 60  6 Total Distance
2  S1  S2 Average speed =
10. (d) Time Taken
Average Speed =  S  S 
1 2
2  60  40 120
2  12  18 2 Average Speed = = = 48 km/h
 60  40 60 30 30
12  18   14 5 km/h = 48 km/h
 
60 60 30

Aditya Ranjan (Excise Inspector) Selected Selection 73

Downloaded by Kunal Chauhan (kc6970929@gmail.com)


lOMoARcPSD|39315497

Time & Distance

16. (a) The ratio of distance covered by


Distance covered by first car in
SMART APPROACH:- Ram and Shyam
1 hour = 50 km. The ratio of speed of thief and police = 5 : 7
Then, the ratio of distance = 5 : 7 = 1200 : 1000 = 6 : 5
Now, relative speed
(7 – 5)x = 2x = 150 Here, Time is constant
= (75 – 50)km/h = 25 km/h x = 75 m
Time taken by second car to cover If time is constant, speed is
 5x = 5 × 75 = 375 m
50 km directly proportional to distance.
19. (c) The ratio of speed of Ram and
50 Relative Speed = (8 – 7) = 1 km/h
=  2 hours Shyam = 6 : 5
25 Distance Run by the theif
Hence, Speed of Ram
Hence, they will meet at 06:00 pm. 120
17. (b) = 7 km/h × = 840 m 10
1 =  6 = 12 m/s
5
Concept,
SMART APPROACH:-
Speed × Time = Distance SMART APPROACH:-
The ratio of speed of thief and police = 7 : 8
ratio of distance = 7 : 8 200
5 Speed of Shyam = = 10m/s
1km/hr = m/s (8 – 7)x = x = 120 m
20
18 Time is constant

r
7x = 840 m
Given that, Speed of police = 12 km/h ratio of Ram and Shyam = ratio of their

si
and speed of Theif = 10 km/h. 20. (a) speed distance
Total length of the race is 12 km ratio of speed = 6 : 5
Thief is spotted by police at a = 12000 m
an by
distance of 250 m.
Relative speed
Track Length = 1200 m
Number of round to complete the
5x = 10 m/s
6x = 12 m/s

n
= (12 – 10) km/h = 2 km/hr 22. (b)
12000
Time taken by the police to caught race =  10 Given
1200
ja Distance = 1500 m
R s
250 18 The race will be finished by A in
the theif =  = 450 seconds
2 5 = 300 × 10 = 3000 seconds Speed of B = 6 m/s
a th

Distance covered by the theif Now, When both of them start the Time taken by B to complete the
before he is caught running race
A and B will meet the first time
5 1500
= LCM (300, 400)
ty a

 10   450 =  250 sec


18 6
= 1200 seconds
 1250 m  1.25 km Time taken by A to complete the
A and B will meet the second time
di M

= 2400 seconds race


SMART APPROACH:-
For next meet, = 250 – 10 = 240 sec
The ratio of speed of police and theif is = 6 : 5
Then the ratio of distance = 6 : 5 Time would be 3600 seconds, We know, 60 sec = 1 min
1 unit = 250 m which is not possible as A  240 sec = 4 min
5 units = 1250 m =1.25 km
complete the race in 3000
18. (c) seconds. A will take 4 minutes to complete
Hence, Both of them will meet the race.
Speed of policeman = 7 m/s
Speed of thief = 5 m/s last time at 2400 seconds. 23. (c)
Relative speed = (7 – 5) m/s 21. (b) If d is constant,
= 2 m/s
SA tB 30 6
Distance = 150 m    
SB t A 25 5
A

Distance
Time  If T is constant
Speed
Given, Length of Race = 1200 m
150 d A SA 6
 Ram can beat Shyam by 200 m   
= 75 s dB SB 5
2 or 20 sec, it means Shyam is
The distance covered by the thief covering 1000 m in the race and
To end race at dead-heat in 1 km
when he is caught by the if he has given 20 sec he will
race A has to give start of B
policeman, finish the race.
= Speed of thief × time 200 1
Speed of Shyam =  10 m/s   1000m = 166.67m
= 5 m/s × 75 s = 375 m 20 6

Aditya Ranjan (Excise Inspector) Selected Selection 74

Downloaded by Kunal Chauhan (kc6970929@gmail.com)


lOMoARcPSD|39315497

Time & Distance

24. (a) 28. (a)


7
Given, 300 300 x=
– 5 2
Ratio of speed of Geeta and n n  10 A takes time to cover 40 km = 2x
Babita = 5 : 2
By Hit and Trial, = 7 hours
If time is constant then distance n = 20 km/h B takes time to cover 40 km = 4
is directly proportional to the
n + 10 = 30 km/h hours 30 minutes
speed
2  30  20 Total time taken by (A + B)
Hence, the ratio of distance Average speed =
covered by Geeta and Babita = 5 : 2 30  20  1
= 24 km/h =  7  4  hours
Given, lead distance = 5 – 2 = 3 units  2
29. (d)
3 units = 40 m distance constant 23
1 Average = = 5 hours 45 min.
40 2 2
5 units =  5 = 66.67m So, Speed 
3 Time
T  75 : 45 34. (d)
25. (b) S  3 : 5 Person has to travel a distance
Time taken by P to cover the track 30. (a) = 30 km

r
36 60
=  12 hour Speed of Bus= = 40 km/h 5
3 1 He has covered part of the
1

si
2 6
Time taken by Q to cover the distance in 3h 20 min
track 60

=
36
an by
 9 hour
Speed of Car =
3
4
= 80 km/h
= 30 ×
5
6
= 25 km

n
4 Ratio = 80 : 40
Time taken by R to cover the = 2:1 D 25  3 15
track Speed =   = 7.5
ja T 10 2
R s
36 SMART APPROACH:- km/h
=  6 hour
6 Here, distance is constant 35. (b)
a th

LCM of 12, 9 and 6 = 36 hour T = 90 : 45 = 2 : 1


7 150 90
26. (a)  
Speed = 1 : 2 2 x y = 7xy
–1 Hence, Required ratio = 2 : 1
ty a

20% = = 300y + 180x .......(i)


5
31. (d) 11 100 140
If distance is constant, speed is
di M

 
inversely proportional to time. 300 300 3 x y
– 5
t1 : t2 = 5 : 4 n n  10
= 11xy = 300y + 420x .......(ii)
s1 : s2 = 4 : 5 By hit and trial
1 n = 20 km/h 7 x y  300 y  180 x
Net increment =  100% = 25%
4 Speed for onward journey = 20 11x y  300 y  420 x
27. (a) km/h 4 x y  240 x
 Distance same 32. (c)
Distance same y = 60 (Speed of bus)
1
So, Speed  1 7 150 90
Time Speed  Speed of train =  
Time 2 x 60
(A) speed 1 : 2
Speed – 5  9 x = 75 (speed of train)
Time 2 : 1
A

Time – 9  5
1 SMART APPROACH:-
x=2+ +1 (4) = 40 minutes
2
Usual time (9) = 90 minutes
7 33. (a)
x= Distance same
2
Speed 1 : 2
240
7 Time 2 : 1 Speed of car = = 60 km/h
So 2x = × 2 = 7 hours 4
2
 1 150
A require 7 hours for walking x = 2   1 Speed of train = = 75 km/h
 2 2
40 km

Aditya Ranjan (Excise Inspector) Selected Selection 75

Downloaded by Kunal Chauhan (kc6970929@gmail.com)


lOMoARcPSD|39315497

Time & Distance

36. (a) Method:-2 3x – 36 = 2x + 20


We know that, x = 56 min
D 5  8 (40  x )
SA TB    = 20 6(56 – 12)
 S 8–5 60
SB TA Speed of the man = 56
60 
40 (40  x ) 60
  = 20
SA

4.05 3 60
6  44 33 5
8.4 2.45  80 + 2x = 180 = = = 4 km/h
 x = 50 min 56 7 7
SA 9 40. (b)
 Let the distance between station
8 .4 7 38. (d)
Distance = 50 × 8 = 400 km A and station B is = D km.
SA = 10.8 km/h.
New speed of Ranjeet = 60km/h d d 63
37. (d) A.T.Q,  
400 30 40 10
D 20 Speed of Rehman = = 80 km/h
5
T=  =4h 7d 63
S 5 New Speed of Rahman =100 km/h  
120 10
Usual time = 4 h – 30 min = 3h 20 400 400
Time Difference = –  d = 108 km
min 60 100

r
Speed of man = 8 km/h
20 4 8 SMART APPROACH:-
20 = –  = 2 h 40 min

si
3 1 3 S1  S2
T= = 2 h 30 min Distance =  total time
8 39. (b) S1  S2
an by
Time difference = (3h + 20 min) – ATQ, 30  40
  6.3  108 km
(2h + 30 min) = 50 min 6  (x – 12) (x  10) 70
 4
60 60

n
ja
R s

a th
ty a
di M
A

Aditya Ranjan (Excise Inspector) Selected Selection 76

Downloaded by Kunal Chauhan (kc6970929@gmail.com)


lOMoARcPSD|39315497

Train

TRAIN
12

1. Two trains A and B having the (a) 19 km (b) 262 km 9


length 195 m and 165 m (c) 243 km (d) 224 km 3 #
respectively, are running in the 4. Two stations are 120 km apart on
same direction on parallel lines. If " # E8 &’
a straight line. A train starts
the speed of A and B be 77 km/h from station A at 8 a.m. and moves # <9 &’ $
and 85 km/h respectively, then towards station B at 20 km/h and ! " + +
what will be the time (in seconds) another train starts from station / >
taken by them to cross each other? B at 9 a.m. and travels towards
station A at a speed of 30 km/ SSC CPO 10/11/2022 (Shift-03)
A B

r
h. At what time will they meet? (a) 11 sec (b) 12 sec
!

si
4 " ? @ 98
" # A $ ! (c) 13 sec (d) 10 sec
+ $ "A :4
B an by # %% &’ 7. Two trains with a speed of 80 km/
8 # :; 4 120 km/h. respectively, are
h and
( &’ )B * "9 8 &’ # km apart each other. Find the
500
+ ! , -

n
# + B : 4 distance between them 10
. ! "/ $ 9 # :; minutes
4 before crossing.
SSC CPO 09/11/2022 (Shift-01) A 30 &’ #
ja 80 &’ 120 &’
R s
(a) 162 (b) 164 # $ 3 ! / > # 3 500 /
-6! F
(c) 160 (d) 166 SSC CPO 10/11/2022 (Shift-01) + "
a th

2. Two trains start from Delhi and (a) 10:30 am (b) 10:00 am
Puna towards each other at 7 am 7 $ " 7 +
(c) 11:00 am (d) 11:30 am
with speeds of 85 km/h and 67 5. Two trains running between
8 ) #
km/h, respectively. If they cross Banglore and Chennai, start at the D "$
ty a

each other at 3.30 p.m., the same time from their respective SSC CPO 11/11/2022 (Shift-01)
distance between the stations is: locations and proceed towards
7:00 0 1 each
+ other at the speed of 80 km/ (a) 72.33 km (b) 33.33 km
di M

( &’ h%and 95 km/h.


& ’ When they meet, (c) 63.33 km (d) 52.74 km
it is found that one train has
# " + #
travelled 180 km more than the 8. A train crosses two persons
0 2 $ ! 3 other. The" distance
+ between travelling at 4 km/h and 6 km/h
Banglore and Chennai is _____. in the same direction in 12 sec
3:30
!
4 # + / 5: 5
55555 A5 $ # B #and# 14 sec, 3 respectively. The
7 4= speed of" the train is
!
SSC CPO 09/11/2022 (Shift-02)
# :; 80 " " E
(a) 1245 km (b) 1292 km
km/h 95 km/h # &’ #
(c) 1283 km (d) 1227 km
3. Two trains A and B start running
" 7 + C6 $ ) G! H ! 12 -
at 80 km/h and 82 km/h towards ! " +180 14 - $
each other from two different km ? + ! $D / :A "$
A

stations. They meet after 1 hour # B # + D "$


30 minutes. How far were they from SSC CPO 11/11/2022 (Shift-03)
SSC CPO 10/11/2022 (Shift-01)
each other when they started? (a) 1200 km (b) 2100 km (a) 18 km/h (b) 26 km/h
/A - 6 B
! /7 / (c) 345 km (d) 400 km (c) 20 km/h (d) 24 km/h
4 " + 6. Two( 8trains 121& ’ m and 99 m in 9. A train x running at 74 km/h
length respectively are running in
(9 &’ # # :; crosses another train y running
opposite directions, one at a
$ 3 ’ <8 speed of 40 km/h and the other at 52 km/h in the opposite
$ / -6! # at a speed
:; of! 32 km/h. In what direction in 12 seconds. If the
time=will they length of y is two-thirds that of
3 " 7 + + > be completely clear x, then what is the length of x
of each other from the moment
SSC CPO 09/11/2022 (Shift-03) they meet? (in m)?

Aditya Ranjan (Excise Inspector) Selected Selection 77

Downloaded by Kunal Chauhan (kc6970929@gmail.com)


lOMoARcPSD|39315497

Train

74 km/h # # 3 " / -400


6 km + # 15. A train
! of length 212 m is running
x, 3 52 km/h # $ ! 10 km/h# at 45 km/h. In what time (in sec-
onds) will it cross a platform of
# 3 y / 12
-6 I- C6 / ) ! length
M 188 m? +
$ ! y / -6 2! ’ $ ! 9 9B ! ; N " / -6 E
/ x- 6 7 ) ! M + # ! -6 $ !
/ x- 6 m , . D ,$’ . / $ J K L
SSC CPO 23/11/2020 (Shift-1) SSC CPO 25/11/2020 (Shift-2) . / >
(a) 168 (b) 252 (a) 12 (b) 15 SSC CPO 11/12/2019 (Shift - 01)
(c) 210 (d) 200 (c) 8 (d) 10 (a) 36 (b) 42
10. A train travelling at the speed of 13. The speed of two railway engines (c) 32 (d) 40
x km/h crossed a 300 m long is in the ratio 5 : 4. If they more 16. The speed of train A is 16 km/h
platform in 30 seconds, and on parallel tracks in the same di- less than the speed of train B. To
overtack a man walking in the rection and if the slower engine cover a distance of 384 km, B
same direction at 6 km/h in 20 is ahead of the faster engine by 8 takes 4 hours less time than A.
seconds. What is the value of x? km when the latter starts, then What is the speed (in km/h) of
how train B?
x km/h # # 3 / far
- 6will the faster engine have

r
to travel before it overtakes the / A- 6 # B / - 6#
300 m J 30
K L I- slower one? & ’ 384 $ + !

si
)6 km/h
3 # : E " B, $ / - /6 A- 6
# # 20
G! HI-
! 3 " : ! E ’ B
!
an by x
SSC CPO 24/11/2020 (Shift-2)
$ D $
#
# 3
2
(
# 3# (
SSC/ CPO
& ’)
= 12/12/2019 (Shift - 01)
>

n
(a) 60 (b) 96 # 3 # (a)
3 50 (b) 48
(c) 48 (d) 102 / " + (c) 45! (d) 32
ja
11. A train x running at 74 km/h 1
R s
/ $ 17. The speed of a train is 2 times
crosses another train y running 6
at 52 km/h in the opposite SSC CPO 09/12/2019 (Shift - 01)
the speed of a car. The car covers
a th

direction in 12 seconds. If the (a) 32 km (b) 48 km 486 km in 9 hours. How much


length of y is two-thirds that of distance will the train cover in 6
(c) 40 km (d) 36 km hours?
x, then what is the length of y
(in m)? 14. Two trains running in opposite " / -6 # "
ty a

74 km/h # # " direction on parallel tracks, at 1


speeds of 42 km/h and 57 km/h 2 /: $ ’
/ x,- 52
6 km/h # 3 6
di M

y+ 12 / take
- 6 18 seconds to cross each ! $ / -6 ’
I- $y ! other.
/ If- 6the length of one train
is 270 m, then the length of the ! / >
x / -6 7
other train is: SSC CPO 12/12/2019 (Shift - 02)
? y / - 6m , . (a) 612 km (b) 702 km
/ > / -6! F !
(c) 732 km
3 (d) 712 km
SSC CPO 25/11/2020 (Shift-1) E9 &’ 18. A train% of length5 342
& ’ m is running
(a) 252 (b) 200 # -6 at
$ 3 " 7 + 54 km/h. In how much time (in
seconds) will it cross a bridge of
(c) 168 (d) 210 ( I- ! length$ 438
! m?
12. A train covers 400 km at a uniform " / -6 9 % 8 342 5 " + 54 / - 6 & ’
speed. If the speed had been 10
/ -6 D "#$ -6 438 $ ! /: - 6
km/h more, it would have taken
2 hours less for the same journey. SSC CPO 09/12/2019 (Shift - 02)
! ,
A

What is the usual time taken (in SSC CPO 13/12/2019 (Shift - 01)
(a) 242 m (b) 225 m
hours) by it to complete the (a) 48 (b) 50
journey? (c) 250 m (d) 230 m
(c) 54 (d) 52

ANSWER KEY
1.(a) 2.(b) 3.(c) 4.(c) 5.(b) 6.(a) 7.(b) 8.(a) 9.(b) 10.(b)

11.(c) 12.(d) 13.(c) 14.(b) 15.(c) 16.(b) 17.(b) 18.(d)

Aditya Ranjan (Excise Inspector) Selected Selection 78

Downloaded by Kunal Chauhan (kc6970929@gmail.com)


lOMoARcPSD|39315497

Train

SOLUTIONS
1. (a) Time of meeting = 9 am. + 2 hour 8. (a)
Relative speed in same direction = 11 am. Let the speed of the train be S
= (85 – 77) km/h. = 8km/h. 5. (b) and its length be L.
Let the train starting from Now, it crosses the first person
Distance Banglore be A and from Channai
Time  in 12 sec
Speed is B. Relative speed as they move in
360 18 Given, same direction
  = 162 seconds Speed of A = 80 km/h
8 5 =S–4
Speed of B = 95 km/h
Time taken by them to cross each
Ratio of the speed of A and B = 12
other = 162 seconds.  L  S – 4  .........(1)
16 : 19 3600
2. (b) If time is constant then distance
Also, it crosses the second

r
Given is directly proportional to the
person in 14 sec
Speed of train started from Delhi speed.

si
Hence, The ratio of distance Relative Speed = S – 6
= 85 km/h
covered by A and B
14
= 67 km/h
an by
Speed to train started from Puna = 16 : 19
Total distance cover when A and
B meets = 16 + 19
 L  S – 6 

From (1) and (2)


3600
.......(2)

n
Travelling time of both the train
= 35 units
= Duration between 07 : 00 AM 12 14
to 03 : 30 PM B will cover more distance by   S – 4   S – 6 
ja = 19 – 16 = 3 units 3600 3600
R s
= 8.5 hour
3 units = 180 km  12S – 48 = 14S – 84
Distance covered by the train
a th

180  12S – 14S = – 84 + 48


started from Delhi = 85 × 8.5 = 35 units =  35
722.5 km 3  – 2S = –36
Distance covered by the train = 2100 km  S = 18
started from Puna = 67 × 8.5 = Hence, The speed of train
ty a

569.5 km SMART APPROACH:- = 18km/h


Hence, The distance between Here time is constant
di M

Delhi and Puna


A B SMART APPROACH:-
= 722.5 + 569.5 = 1292 km
Speed 80 95 Distance is same
3. (c)
Distance 80 95 Let speed of train = S
Relative Speed = (80 + 82) = 162
km/h 95x – 80x = 15x = 180 (S – 4) × 12 = (S – 6) × 14
Both trains meet After 1 hour 30 (80 + 95) = 175x =
180 S = 18 km/h
 175
minute, 15
2100 km 9. (b)
Time = 1.5 h
Speed of train x = 74 km/h
Distance between the Trains = 6. (a) If Length of train x = 3D
162 × 1.5 h = 243 km Relative Speed = 40 + 32 Speed of train y = 52 km/h
4. (c)
5 2
In such a type of question we will = 72× = 20 m/s
A

18 Length of train y = 3D × = 2D
make the time of starting the 3
Time taken in crossing each other
same for both the stations.  Both trains travelling in
121  99 220 opposite direction
Assume, The starting time of = = = 11s
station A is 9 am. 20 20 So the relative speed of train x and y
7. (b) = 72 km/h + 52 km/h
Now the distance
Distance between before 10 = 126 km/h
= 120 – 20 = 100 km minute
Relative speed 5
10 1 3D + 2D = 126 × m/s × 12
= 120  80   = 200  = 18
= 20 + 30 = 50 km/h 60 6 sec
100 33.33 km 5D = 84 × 5 = 84
Time  = 2 hour
50
Aditya Ranjan (Excise Inspector) Selected Selection 79

Downloaded by Kunal Chauhan (kc6970929@gmail.com)


lOMoARcPSD|39315497

Train

Length of train x(3D) 12. (d) 384 384


ATQ, – 4
= 84 × 3 = 252 m 400 400 x – 16 x
– 2
10. (b) n n  10 384x – 384x  384 16
 =4
Let length of train = D m By hit and trial x (x – 16)
D  300 n = 40 km/h  6144 = 4x2 – 64x
 x= 30
m/sec Usual time taken by train to
 4x2 – 64x – 6411 = 0
400  x2 – 16x – 1536 = 0
D complete the journey =
 20 40
5  x2 – 48x + 32x – 1536 = 0
 x – 6  = 10 hours
18  x(x – 48) + 32 (x – 48) = 0
13. (c)
 x – 48 = 0
D
 20 Speed  5 : 4  x = 48
D  300 5
– 6 Hence speed of train B = 48 km/h
30 18 Distance  5 : 4

 D  300 – 50 
SMART APPROACH:-

r
D = 20   1 unit  8 km (given) By Hit and Trial method,
 30 

si
Take speed of 'B'= 48 km/h
Faster engine have to travel to
 D  250  overtakes the slower one 384 384
D = 2   4

D = 500 m
3 an by

= (8 × 5) = 40 km
14. (b)
32 48
 4 = 4 (Satisfied)

n
Length of other train = x Hence,
500  300
x= m/sec The speed of train B = 48 km/h
30 270  x
ja   18
R s
5 17. (b)
800 18 (42  57) 
x=  = 96 km/h 18
a th

30 5 486
 270 + x = 495 Speed of car = = 54 km/h
11. (c) 9
 x = 225
Let length of x = 3D 13
15. (c) Speed of train = 54 ×
6
ty a

2 Length of train = 212 m


Lengh of y = 3D × = 2D = 117 km/h
3
5
di M

Relative speed of trains x and y Speed = 45 km/h = 45 × m/s Distance covered by the train in 6
18 hours
= 74 km/h + 52 km/h Platform length = 188 m = 117 × 6 = 702 km
= 126 km/h Time to cross a platform 18. (d)
5 (212  188) 400 18 Speed of the train = 54 km/h
3D + 2D = 126 × × 12 m =  = 32 sec.
18 5 45  5
45  5
5D = 84 × 5 18 = 54  = 15 m/s
18
D = 84 m 16. (b) Required time to cross the bridge
Length of y (2D) = 2 × 84 = 168 m Let, speed of train 'B' be x km/h
(342  438)
Speed of train A = (x – 16) km/hr = = 52 sec.
15
A



Aditya Ranjan (Excise Inspector) Selected Selection 80

Downloaded by Kunal Chauhan (kc6970929@gmail.com)


lOMoARcPSD|39315497

Boat and Stream

BOAT AND STREAM


13

1. A boat can travel 104 km # ! (a) 20 (b) 18


downstream in 8 hours. If the &. (c) 24 (d) 15
speed of the stream is 2 km/h,
/0 6. # The distance *between
1* two places
then find in what time will it be
2 ! A and B is 14 km. A boat travels
able to cover 13 km upstream? from A to B downstream and then
$ 3 . from
reurns 4 B to A upstream and
) ) takes5 a total of 3 hours 44
mintues for the entire journey. If
!" #
the speed of the current is 2 km/h,

r
" # SSC
$ CPO 11/11/2022 (Shift-02) then what is the speed of the
(a) 15.5 sec (b) 29 sec boat, (in km/h) in still water?

si
%
(c) 31 sec (d) 35 sec /A0 ) B ;
$ & ’
4. The speed of a boat in still water $: A

(a) 2
2
an by
SSC CPO 09/11/2022 (Shift-02)

hours (b) 1
4
hours
is 15 km/h, and the speed of the
current is 5 km/h. In how much
B
B A
# )
$ )
$< $:
6

n
3 9 time (in hours) will the boat travel
a distance of 60 km upstream and
3 4 . %
1 2 the same distance downstream? &
(c) 2 hours
2
ja (d) 1 hours
3 /0 #
R s
/0 $ 15 km/
2. A boat can go 40 km downstream h ! ) 5 km/h
- & ’
a th

and 25 km upstream in 7 hours SSC CPO 11/12/2019 (Shift-2)


, 60 km -
30 minutes. It can go 48 km (a) 8 (b) 7
downstream and 36 km upstream $ ) 6 7
1 1
in 10 hours. What is the speed & ’ (c) 7 (d) 6
2 2
(in km/h) of the boat in still
ty a

SSC CPO 25/11/2020 (Shift-2) 7. A boat takes 4 hours to travel


water? (a) 10 (b) 9 from a place X to Y downstream
( % (c) 20 (d) 12 and back from Y to X upstream.
di M

If the distance from X to Y is


) * 3
5. A person rows a distance of 3 km 10.5 km and the speed of the
$ # 4 current is 9 km/h, then the
1 speed of the boat in still water
upstream in 1 hours and a dis- (in km/h) is:
) %+ $ 2
tance of 13 km downstream in 2 $: X
# # hours. How much time (in hours)
will Y /0 # ) 7
, - & ’ the person take to row a dis-
tance of 90 km in still water? $ Y X /0 $ )
SSC CPO 09/11/2022 (Shift-03) 8 5 9 $: X &Y
(a) 6 (b) 12 3
3 10.5 km ! ) 9 km/
(c) 9 (d) 15
$ 4 h ! /0 # (km/h )
3. The speed of a boat in still water 1 & ’
1 )
is thrice the speed of the stream. 2
A

SSC CPO 13/12/2019 (Shift-1)


If the boat takes 15.5 sec to go $: 13
to a certain place downstream, 8 15
4(a) 10 (b) 15
then find the additional time /0 #90 2
required to cover the same , - & & ’ 1
distance travelling upstream. (c) 12 (d) 12
SSC CPO 12/12/2019 (Shift - 01) 2

ANSWER KEY
1.(b) 2.(c) 3.(a) 4.(b) 5.(a) 6.(a) 7.(c)

Aditya Ranjan (Excise Inspector) Selected Selection 81

Downloaded by Kunal Chauhan (kc6970929@gmail.com)


lOMoARcPSD|39315497

Boat and Stream

SOLUTIONS
1. (b) Distance covered = 4x × 15.5 Time to row a distance of 90 km in
Let the speed of boat in still = 62x m
water = x km/h Speed of boat in upstream 2
= 3x – x = 2x m/s still water= 90 × = 20 Hours.
Downstream speed of boat 9
Time taken to cover 62x m in
104 6. (a)
 x 2 62x
8 upstream =  31sec
2x 14 14 56
 x + 2 = 13 km/h  
 x = 11 km/h Additional time required to travel B  S B – S 15
upstream = 31 – 15.5 = 15.5 sec.
Speed of boat in upstream Speed of current = 2 km/h (given)
4. (b)
= (11 – 2) = 9 km/h By option
Speed of boat in still water (x)

r
Time take by the boat to cover the put speed of boat = 8 km/h
= 15 km/h
distance of 13 km in upstream

si
Speed of current still water (y)
13 4 14 14 56
 km/h =1 km/h = 5 km/h So,  
9 9 8  2 8 – 2 15
2. (c)
an by Total time =
60

60
15  5 15 – 5
14 14 56

n
Let the speed of the boat in still =  
=3+6
water = x km/h 10 6 15
= 9 hours
Speed of stream = y km/h
ja 5. (a)
R s
CASE-I: 56 56
Upstream Speed= (x – y) km/h =  (Satisfied)
Boat can go 40 km downstream Downstream Speed= (x + y) km/h 15 15
a th

and 25 km upstream in 7 hours


30 minutes. 15 Hence the speed of boat is 8 km/
3 h
40 25  4 
   7.5 ............(1) xy 2 7. (c)
ty a

xy x–y
Let speed of boat be x km/h.
CASE- II:
15 3
  Speed of current = 9 km/h
di M

Boat can go 48 km downstream 4(x – y) 2


Distance between X and Y = 10.5
and 36 km upstream in 10 h.  12 x – 12y = 30 ...(1) km
48 36 13 Upstream speed = (x – 9) km/h
   10 ...........(2)
xy x–y Again, (x  y) = 2 Downstream speed = (x + 9) km/h
ATQ,
On solving (1) and (2) we get,  2 x + 2y = 13 ...(2)
x = 9, y = 3 Now, Adding (1) & (2) × 6, 10.5 10.5
 4
Hence, Speed of boat = 9 km/h x 9 x 9
12x – 12y  30
3. (a)
12x  12y  78 Put x = 12 from the option.
Let the speed of stream be x m/s
and speed of boat be 3x m/s. 24x  108 (Satisfied the equations)
Speed of boat in downstream = So, Speed of boat = 12 km/h.
9
A

x + 3x = 4x m/s  x = 2 km/h



Aditya Ranjan (Excise Inspector) Selected Selection 82

Downloaded by Kunal Chauhan (kc6970929@gmail.com)


lOMoARcPSD|39315497

Number System

NUMBER SYSTEM
14

1. What is the least value of x so 6. The number of factors of 196 =8 8


that number 8x5215 becomes which are divisible by 4 is: * ! ! &
divisible by 9? ,- . / 01 2 ! &
% -6 7 )
x / 8 ,
x " 2 , &6
SSC CPO 09/11/2022 (Shift-01) SSC CPO 09/11/2022 (Shift-02)
(a) 3 (b) 1 (a) 228 (b) 4 SSC CPO 10/11/2022 (Shift-01)
(c) 5 (d) 6
(c) 57 (d) 3 (a) 3 (b) 5

r
2. Which of the following is a prime
(c) 11 (d) 2
number? 7. If the 9 digit number 72x8431y4

si
is divisible by 36, what is the 10. What are the values of R and M,
! respectively, if the given number
x y is perfectly divisible by 16 and 11?

(a) 54
an by
SSC CPO 09/11/2022 (Shift-01)
(b) 39
value of  –  for the smallest
y x
34R05030M6
" " */ % ! &

n
possible value of y, given that x
(c) 68 (d) 89 R ! M& + @ <
and y are natural numbers?
3. If A is greater than B by 7, B is
ja
greater than C by 16, and A + B " ! 72x8431y4 34R05030M6
R s
+ C is 255, then the value of 3A 8% x ! y&,
3 9 SSC CPO 10/11/2022 (Shift-01)
+ C – 4B is: y 4 : ;
a th

(a) 4 and 6 (b) 7 and 5


"A, B # ! $ B, C % (c) 5 and 5 (d) 5 and 7
x y
! $ A !+ B &
+C 255  –  11. If the number 636633P is divisible
y x by 132, then the value of P is:
3A + C – 4B ’ & (
ty a

SSC CPO 09/11/2022 (Shift-03) " 636633P, 132 5 &


SSC CPO 09/11/2022 (Shift-01)
P
(a) 5 (b) 10
di M

(c) 8 (d) 4 5 1 SSC CPO 10/11/2022 (Shift-02)


(a) 1 (b) 2
4. If the number 647592 is divisible 7 10 (a) 4 (b) 2
by 88 and if the digits are (c) 3 (d) 6
rearranged in increasing order, 2 9 12. If the number 8764x5 is divisible by
then the new number thus formed (c) 1 (d) 2 9, then find the least possible value
5 10
will be divisible by: of x, where x is a two-digit number.
%)# 8. If a 4 digit
( number
" x58y is exactly "8764x5 2
* ! ! & divisible
+ by 9, then the least value x
of (x + y) is:
,- . /. 01 2 * ,3 & 4 ’ 2 x "2 A!
*/ " ) ! x58y, 9 (
5 & , 6 / < (x + y) SSC CPO 10/11/2022 (Shift-02)
SSC CPO 09/11/2022 (Shift-02) (a) 15 (b) 06
(c) 14 (d) 18
A

(a) 22 (b) 66 SSC CPO 10/11/2022 (Shift-01)


(c) 44 (d) 3 13. If the number 476**0 is divisible
5. How many of the following (a) 4 (b) 5 both 3 and 11, then in the
numbers are divisible by 156? hundredth and tenth places, the
(c) 3 (d) 2
% non-zero digits are, respectively:
9. The number 150328 is divisible " 476**0, 3 ! & "
by 23. If the digits are rearranged 7B ! &
312, 620, 936, 1402, 1872, 3216, in descending order then five
7176, 8108 times of 13 is substracted from
,& &C@ ! + @ <
SSC CPO 09/11/2022 (Shift-02) the new number thus formed, SSC CPO 10/11/2022 (Shift-03)
(a) 5 (b) 3 then the resultant number will (a) 2 and 3 (b) 3 and 2
(c) 4 (d) 2 be divisible by: (c) 5 and 8 (d) 8 an 5

Aditya Ranjan (Excise Inspector) Selected Selection 83

Downloaded by Kunal Chauhan (kc6970929@gmail.com)


lOMoARcPSD|39315497

Number System

14. If 86y5 is exactly divisible by 3, 20. A six-digit number is divisible by ,& I ,3G J
then the least value of y is: 198. If the digits are rearranged,
(+5) ! " 2 ! & ,3
" 86y5, 3 , 6 / < even then the number will be
divisible by: JK & (–2) ! " 2
y
: ! 2 ,3@ (– JK
SSC CPO 10/11/2022 (Shift-03)
( " ! ,- /. 0 12)
1 ! , 32 L J
(a) 3 (b) 0 (
(c) 1 (d) 2
J JK &
15. Find the least value of (A + B) if
the number 27B58A4 is SSC CPO 11/11/2022 (Shift-02) SSC CPO 11/11/2022 (Shift-03)
completely divisible by 88. (a) 3 (b) 6 (a) 8 (b) 32
" 27B58A4, 88 5 & , 6 /(c) <2 (d) 66 (c) 16 (d) 20
(A + B) 21. A six-digit number 763254 is 25. Let x = 224 and y = 322. If the
’ 2 ( divisible by 18. If we subtract five highest common factor of 23x and
times of 41 from the number, the a × y is divisible by x and y, then
SSC CPO 10/11/2022 (Shift-03) what can be the possible value
new number which is formed the
(a) 5 (b) 2 number will divisible by: of a?
(c) 4 (d) 7 :< ! #%8 ) x2 M ) y! M& 8 N

r
16. If the 9 digit number 97x4562y8 ( " ) " x, 8! >a×y
& JO> J
is divisible by 88, what is the
- 6 *xQ / ! 7 y& a

si
value of (x2 + y2) for the smallest
- ? ; " 4
value of y, given that x and y are *

"
an by
natural number.
! 97x4562y8,
SSC CPO 11/11/2022 (Shift-02)
(a) 2 (b) 7
SSC CPO 11/11/2022 (Shift-03)
(a) 16 (b) 8

n
88 x ! y&, 3 9 (c) 5 (d) 3 (c) 12 (d) 4
y 4 : ; (x 2 22. If the number 55p1067q9 is 26. What are the values of '&' and
+ y2) ja exactly divisible by 99, then pq "#", respectively, if the following
R s
is equal to: number is perfectly divisible by
SSC CPO 11/11/2022 (Shift-01)
" 55p1067q9 9?
, &
(a) 64 (b) 68
a th

& pq 51&918#0
(c) 76 (d) 80
" , 6
17. Find the least value of p so that
246p48 is divisible by 8. SSC CPO 11/11/2022 (Shift-03) '&' ! "#",
& + @ <
ty a

p ’ 2 (a) 35 (b) 28
246p48, 8 ( (c) 36 (d) 42 51&918#0
di M

SSC CPO 11/11/2022 (Shift-01) 23. Two positive number differ by SSC CPO 11/11/2022 (Shift-03)
3951. When the larger number is (a) 9 and 8 (b) 2 and 2
(a) 2 (b) 1
divided by the smaller number,
(c) 4 (d) 0 (c) 4 and 5 (d) 8 and 4
the quotient is 12 and the
remainder is 13. The sum of the 27. If 1433 × 1433 × 1422 × 1425 is
18. Ramu had to select a list of
numbers between 1 and 1000 digits of the larger number is: divided by 12, then what is the
(including both) which are remainder?
" $ G ! 8 ! &
divisible by both 2 and 7. How ( 24 47B : ; "1433 × 1433 × 1422 × 1425
many such numbers are there? 12 2 2
2 2 ,H
& ! & === D" ! E ! &4 @
> F 8 4> @ F ,
( 47B H ’ & (
! > > !& SSC CPO 23/11/2020 (Shift-1)
1 2 ! & # " ( (a) 3 (b) 9
SSC CPO 11/11/2022 (Shift-03)
A
A

(a) 12 (b) 16 (c) 8 (d) 6


SSC CPO 11/11/2022 (Shift-01) 28. If a nine-digit number 785x3678y
(c) 18 (d) 14
(a) 142 (b) 71 is divisible by 72 then the value
24. In a test (+5) marks are given for
(c) 97 (d) 642 every correct answer and (–2) of (x – y) is:
19. The sum of the odd divisiors of marks are given for every incorrect " ! 785x3678y,
216 is: answer. Jay answered all the 72 (x – y) ’
% F 2 questions and scored (–12)
marks, though he got 4 correct & (
SSC CPO 11/11/2022 (Shift-01) SSC CPO 23/11/2020 (Shift-1)
answers.
(a) 16 (b) 14 How many of his answers were (a) 0 (b) –2
(c) 40 (d) 600 INCORRECT? (c) –1 (d) 2

Aditya Ranjan (Excise Inspector) Selected Selection 84

Downloaded by Kunal Chauhan (kc6970929@gmail.com)


lOMoARcPSD|39315497

Number System

29. The students of a class donated 34. How many numbers between 400 39. How many number between 300 and
a sum of Rs.2,209. If each and 700 are divisible by 5, 6 and 7? 700 are divisible by 5, 6 and 8?
student donated as many rupees 400 ! 700
& 4 > 300 ! 700 & 4 >
as the number of students in the 5,26 ! 7 & 5,
26 ! 8&
class, then the number of
SSC CPO 24/11/2020 (Shift-2) SSC CPO 25/11/2020 (Shift-2)
students in the class is:
(a) 20 (b) 10 (a) 5 (b) 20
I R 1 / S, =
(c) 2 (d) 5 (c) 3 (d) 2
& @ " & ( " ,
35. 3When
G a number R 1is successively
/
40. When a number is successively
J SL " 2 dividedI by 3, 4 and 7, the divided by 3, 4 and 7, the
R 1/ remainders
I obtained
R are1 / 2, 3 and remainder obtained is 2, 3 and 5
’ & ( 5, respectively. What will be the respectively. What will be the
remainder when 84 divided the remainder when 42 divides the
SSC CPO 23/11/2020 (Shift-2)
same number? same number?
(a) 49 (b) 53
24 3, 4 + ! @
7& <
(c) 51 (d) 47 24 + @ < 8
2
+ @ < 2 2,
30. The remainder when 75 × 73 × 78 2 2
× 76 is divided by 34 is: 3 ! 5& @ F ,HV ,3 L ( 24 J
+ @ < 8 ! & ,3 L

r
75 × 73 × 78 × 76 34 2 84 2 2
) 2

si
& ,& ,3 L @ F ,H ’ , 3 L& @( F ,H ’ & (
,3 L @ F ,H ’ & (
SSC CPO 23/11/2020 (Shift-2) SSC CPO 24/11/2020 (Shift-2)
an by SSC CPO 25/11/2020 (Shift-2)
(a) 15 (b) 22 (a) 71 (b) 30
(c) 18 (d) 12 (c) 48 (d) 53 (a) 41 (b) 31

n
31. The remainder when 72 × 73 × 78 36. If 1433 ×1433 × 1422 × 1425 is (c) 30 (d) 29
× 76 is divided by 35 is: divided by 10, then what is the 41. If the six-digit number 479xyz is
ja
72 × 73 × 78 × 76 35 2 remainder? exactly divisible by 7, 11 and 13,
R s
& ,& ,3 L @ F ,H ’ & × 1433
"1433 ( × 1422 × 1425 then {(y + z) ÷ x} is equal to:
SSC CPO 24/11/2020 (Shift-1) 10 xyz #
a th

2 2 " : ,C3! L )#
(a) 12 (b) 8 @ F ,H ’ & ( ! & 8 , 6 / [(y
< + z) ÷
(c) 22 (d) 15 x] 4& 4& (
SSC CPO 25/11/2020 (Shift-1)
(a) 9 (b) 0 SSC CPO 09/12/2019 (Shift-01)
ty a

32. If 6-digit number 5x2y6z is divisible


by 7, 11 and 13, then the value (c) 3 (d) 8 11
of (x – y + 3z) is: (a) 4 (b)
di M

37. What is the sum of the digit of 9


" % ! 5x2y6z; 7, 11 the least number when divided
by 15, 18 and 36 leaves the same 7 13
! 13
& (x – y + 3z) (c) (d)
remainder 9 in each case and is 13 7
’ & (
divisible by 11? 42. If the 7-digit number x468y05 is
SSC CPO 24/11/2020 (Shift-1)
(a) 0 (b) 4 J : ; : ; ! divisible by 11, then what is the
value of (x + y)?
(c) 7 (d) 9 15, 18
! 36&
2 2
33. The students of a class donated & , & , 3 G 9 @ 0 1F 4O "! # &! x468y05, 11
a sum of Rs.2,809 to the Fund. 11 (x + y)
Each student donated as many SSC CPO 09/12/2019 (Shift-02)
SSC CPO 25/11/2020 (Shift-1)
rupees as the number of students
(a) 16 (b) 17 (a) 8 (b) 10
in the class. The number of
A

students in the class is: (c) 15 (d) 18 (c) 14 (d) 12


I R 1 / S , 38. If=a nine-digit
& number
@ 785x3678y 43. How many natural number less
is divisible by 72, then than 1000 are divisible by 5 or 7
" & ( " ,3G "T 1 / J the value but NOT by 35?
of (x + y) is:
S, " 2 I R U1
785x3678y, 72 === ,3 9
I R "U 1 !
(x + y) ’ & #( 8
’ & (
SSC CPO 24/11/2020 (Shift-1) SSC CPO 25/11/2020 (Shift-1) SSC CPO 11/12/2019 (Shift-01)

(a) 47 (b) 51 (a) 10 (b) 5 (a) 285 (b) 313


(c) 49 (d) 53 (c) 12 (d) 20 (c) 341 (d) 243

Aditya Ranjan (Excise Inspector) Selected Selection 85

Downloaded by Kunal Chauhan (kc6970929@gmail.com)


lOMoARcPSD|39315497

Number System

44. When a certain number is divided SSC CPO 11/12/2019 (Shift-02) == ,3 9 A


by 65, the remainder is 56. When (a) 100 (b) 40
the same number is divided by 13,
(c) 103 (d) 33 SSC CPO 13/12/2019 (Shift-01)
the remainder is x. What is the
value of 46. If a 5-digit number 538xy is (a) 768 (b) 801
5x – 2 ?
divisible by 3, 7 and 11, then the (c) 934 (d) 1067
24 @> % value of (x²2+ y²) is: 49. If r is the remainder when each
2 @ F ,H % ,3 L ( of 6454, 7306 and 8797 is divided
" ! 538xy, 3, 7
J 24 8 2 by the greatest number d (d > 1),
! & , 6 / < (x² + y²) then (d – r) is equal to:
2 x ,3@ L F 5, xH– 2 ( ’ 2 ( r"
@F ,H 2 %) ) #8
SSC CPO 12/12/2019 (Shift-01) 47B d(d
4 >71)B 2
SSC CPO 11/12/2019 (Shift-02) & , & , 3 L (d – r)
(a) 25 (b) 10
(a) 2 7 (b) 13 ’ 2 (
(c) 17 (d) 13
SSC CPO 13/12/2019 (Shift-01)
(c) 2 2 (d) 3 2
47. Which of the following number is (a) 126 (b) 64
45. Let x be the least number exactly divisible by 6? (c) 137 (d) 149

r
divisible by 16, 24, 30, 36 and 45,
% number
50. The , 61563241234351
/ < is:
and x is also a perfect square.

si
What is the remainder when x is 2
divided by 123? 1563241234351:
an by SSC CPO 12/12/2019 (Shift-02)
SSC CPO 13/12/2019 (Shift-02)
2x : ; : ; 2
(a) 4325672 (b) 5643252 (a) Divisible by both 3 and 11
16, 24, 30, 36 ! 45
& 1

n
(c) 96543111 (d) 465466 (b) Divisible by 11 but not by 3
x , 6 / / x ( 28 4
48. How many natural numbers up to (c) Neither divisible by 3 nor by 11
2 2 @ F ,H
ja 2001 are divisible by 3 or 4 but (d) Divisible by 3 but not by 11
R s
4> NOT by 5?
a th

......-------......
ty a
di M

ANSWER KEY
1.(d) 2.(d) 3.(a) 4.(d) 5.(c) 6.(d) 7.(b) 8.(b) 9.(b) 10.(c)
A

11.(d) 12.(a) 13.(d) 14.(d) 15.(b) 16.(d) 17.(d) 18.(b) 19.(c) 20.(a)

21.(b) 22.(a) 23.(b) 24.(c) 25.(a) 26.(d) 27.(d) 28.(d) 29.(d) 30.(d)

31.(b) 32.(c) 33.(d) 34.(c) 35.(a) 36.(b) 37.(d) 38.(a) 39.(c) 40.(d)

41.(a) 42.(d) 43.(a) 44.(d) 45.(d) 46.(d) 47.(b) 48.(b) 49.(d) 50.(c)

Aditya Ranjan (Excise Inspector) Selected Selection 86

Downloaded by Kunal Chauhan (kc6970929@gmail.com)


lOMoARcPSD|39315497

Number System

SOLUTIONS
1. (d) 6. (d) 10. (c)
If the sum of digits is divisible The factor of 196 = 1, 2, 4, 7, 14, Given Number = 34R05030M6
by 9, Then the whole number will 28, 49, 98, 196 if the number's last 4 digits is
be divisible by 9. Number of factors which are divisible by 16, the entire no will
divisible by 4 = 3 also be divisible by 1
Sum of digits of the number
7. (b) Last 4 digits = 30M6
8x5215 = 21 + x
Given Number = 72x8431y4 With the help of basic division
Hence By Option(d), x = 6.
We know, if a number is divisible you will get M = 5
2. (d)
by 9 and 4, the number will be if the difference of the sum of
If any number N has only two divisible by 36 digit at odd position and sum of
factors 1 and N, Then the number if the number's last two digits are digits at even position in a
is the Prime Number. divisible by 4, the entire number number is 0 or 11, the number

r
Option(d) 89 has only two factors will also be divisible by 4. will be divisible by 11.
which are 1 and 89 only. Hence,

si
if the number's sum of digits is
Hence, Option(d) is correct. divisible by 9, the entire number (3 + R + 5 + 3 + M) – ( 4 + 0 + 0
will also be divisible by 9 + 0 + 6)
3. (a)
A B
an by C
Now,
Last two digit of 72x8431y4 is y4
= 11 + R + M – 10
= 11 + R + 5 – 10

n
  
We can assume y = 0, 2, 4, 6, 8 = 16 + R – 10
x  23 x  16 x (Let)
sum of digits = 29 + x + y =6+R
Given that, ja Here, assume such values of x Hence, R = 5
R s
A + B + C = 255 and y so that the sum will be Required Answer (c)
Then, (x + 23) + (x +16) + x = 255 divisible by 9.
a th

11. (d)
 3x + 39 = 255 if we assume y = 0 the option will Number = 636633P
 3x = 255 – 39 not satisfy hence we will assume ATQ, Number is divisible by 132
another value.
 3x = 216 We know, 132 = 2 × 2 × 3 × 11
ty a

Therefore, y = 2 then x = 5
216 Hence, We can say that number
x  = 72 is also divisible by 11.
3 x y 5 2
–  –
di M

Hence, C = 72, B = 88, A = 95 y x 2 5 We will use the concept of


divisibility of 11
Now the value of,
25 – 4 21 1  sum of odd places digit
3A + C – 4B = 3(95) + 72 – 4(88)   2
10 10 10 = sum of even places digit
= 285 + 72 – 352 8. (b) 6 + 3 + 6 + 3 = (3 + 6 + 3 + P)
= 357 – 352 = 5 Sum of the number is 18 = 12 + p
4. (d) = 13 + x + y 18 – 12 = P
Given Number = 647592 Assume the minimum value of (x 6 = P
Rearranged Number = 245679 + y) so that the sum will be 12. (a)
Here the sum of digits of divisible by 9. Sum of digit of the Number
rearranged number = 33 Hence, The value = 5 = 30 + x
Hence, the Number will be 9. (b)  30 + [15] = 45, which is
A

divisible by 3 Number = 150328 divisible by 9.


5. (c) Rearranged Number = 853210 Hence, x = 15
312, 936, 1872 and 7176 are Resultant Number 13. (d)
divisible by 156 = 853210 – 5 × 13 Let 476**0 = 476xy0
312 936 = 853210 – 65 Sum of Digits = 17 + x + y
 2;  6; By the divisibility rule of 11
156 156 = 853145
Last Digit of the resultant number 4+6+y =7+x+0
1872 7176 is 5  10 + y = 7 + x
 12;  46
156 156 From option (d)
Hence the resultant number will
Hence Required Answer = 4 be divisible by 5 y = 5 and x = 8

Aditya Ranjan (Excise Inspector) Selected Selection 87

Downloaded by Kunal Chauhan (kc6970929@gmail.com)


lOMoARcPSD|39315497

Number System
14. (d) 19. (c) On substracting Equation [1] &
[If sum of digits of a number is Prime factorization of 216 = 23 × 33 [2], we get–
divisible by 3, the entire number  x – y – x + 12y = 3951 – 13
Sum of odd divisors
will be divisible by 3]  11y = 3938
= (30 + 31 + 32 + 33)
Sum of digits = 19 + y  y = 358
= (1 + 3 + 9 + 27) = 40
Hence, y = 2 From Equation[1], x = 3951 + 358
20. (a)
15. (b) = 4309
Prime factor of 198 = 3 × 6 × 11
Given Thus, The sum of digits
Hence, Any number (Say N) is
Number = 27B58A4 = 4 + 3 + 0 + 9 = 16
divisible by 198 if the number is
[Divisibility by 8 = Check last 24. (c)
divisible by 3, 6 and 11.
three digit] Correct Answer = +5 Marks
If we rearrange the digit of the
 8A4  8  minimum value of number the divisibility rule of 6 Incorrect Answer = –2 Marks
A=2 and 11 will hamper; but the Marks Scored = –12 Marks
by the divisibility rule of 11– divisibility rule of 3 is based on Number of questions answered
2+B+8+4=7+5+A the sum of digits which will not correctly = 4
 14 + B = 12 + 2 change. Let the total number of question
Hence, The new Number is be x
 14 + B = 14
divisible by 3. ATQ,
B=0

r
21. (b)  4 × 5 + (x – 4) × (–2) = –12
Hence, A + B = 2
 20 – 2x + 8 = –12

si
16. (d) Number = 763254
New Number = 763254 – 5 × 41  –2x = – 12 – 28
Factor of 88 = 8 × 11
= 763254 – 205  –2x = – 40
an by
For the smallest possible natural
number of y– = 763049
 763049 is divisible by 7
 x = 20
Number of question answered

n
 2y8 will be divisible by 8 incorrectly = 20 – 4 = 16
 So, y will be 4 for 248, which 22. (a)
25. (a)
is divisible by 8 Given, 55p1067q9 is divisible by
ja  23 × 224 = a × 322
99
R s
As y = 4,
i.e., The number is also divisible 23  224
Now the number is – 97x456248  a
by 9 and 11. 322
a th

As y = 4, The possible value of x


Sum of digits = 33 + p + q .....(1)  a = 16
for which 97x456248 will be
divisible by 11. If the number of divisible by 9 i.e. 26. (d)
sum will be divisible by 9. Given Number = 51&918#0
Sum of odd places digit in
Again, By the divisibility rule of
ty a

number = 8 + 2 + 5 + x + 9 Sum of Digits = 24 + & + #


11– Assume such value of & and #
= 24 + x
 5 + p + 0 + 7 + 9 – (5 + 1 + 6 + from the options so that sum will
di M

Again, sum of even places digit q) be divisible by 9.


in number = 4 + 6 + 4 + 7 = 21  21 + p – (12 + q) By Option (d)–
Difference = 24 + x – 21  21 + p – 12 – q Sum of digits = 24 + 8 + 4 = 36
=3+x  9 + p – q ........(2) (divisible by 9)
Hence for x + 3 to be divisible by Assume such values of p and q Hence, Answer (d)
11, that satisfies the eqn. -1 & eqn.-2 27. (d)
x = 11 – 3 = 8 with respect to divisibility rule of [1433 × 1433 × 1422 × 1425] ÷ 12
Therefore, x2 + y2 = 82 + 42 9 and 11 Remaining = ?
= 64 + 16 So, p = 7 and q = 5
= 80  1433   1433   1422   1425 
Product = 35     
17. (d) 23. (b)  12   12   12   12 
We know, if the last three digits Assume Larger Number = x and Remainder 5569
of a number is divisible by 8, the Smaller Number = y
A

1350
number will be divisible by 8.  x – y = 3951 .........[1] 5 × 5 × 6 × 9 =
12
For the least possible value of p– ATQ, When the larger number is Remainder = 6
 p48 will be divisible by 8. divided by smaller number we
28. (d)
Hence, p = 0 get–
785x3678y divided by 72
18. (b) Quotient = 12 and Remainder =13
We know– 78y
LCM of 2 and 7 = 14 8×9 So y = 4
Dividend = Divisior × Quotient + 8
When we divide 1000 by 14 we get
71 as quotient and 6 as remainder. Remainder 785x36784 divided by 9
Therefore, There is 71 numbers  x = y × 12 + 13 x=6
which are divisible by both 2 and 7.  x – 12y = 13 ...............[2] x–y=6–4=2

Aditya Ranjan (Excise Inspector) Selected Selection 88

Downloaded by Kunal Chauhan (kc6970929@gmail.com)


lOMoARcPSD|39315497

Number System

29. (d)  7 × 1 + 5 = 12 41. (a)


x × x = 2209 12 × 4 + 3 = 51 6 digit number in which 3 digit
x2 = 2209 51 × 3 + 2 = 155 repeat xyzxyz is divisible by 1001
Comparing on 479479 to 479xyz
x = 47
155 x = 4, y = 7 and z = 9
30. (d) So, , Remainder = 71
84 (y  z) 7  9 16
75 × 73 × 78 × 76 Divisible by 34, Now,   =4
Remainder = ? 36. (b) x 4 4
1433 × 1433 × 1422 × 1425 42. (d)
75 73 78 76 divisible by 10, Remainder = ? x468y05 divisible by 11
  
34 34 34 34 Then,
 Last digit 5 and 2 multiple
Remainder = 7 × 5 × 10 × 8 Some of even place value
given last digit 0
 4 + 8 + 0 = 12
2800 If any numbers last digit is zero, Some of odd place value
 = Remainder = 12 and that number are divided by
34  x + 6 + y + 5 = x + y + 11
10, Remainder will be zero Put, x + y = 12
31. (b)
Remainder = 0 x  y  11 – 12 12  11 – 12 11
72 × 73 × 78 × 76 Divisible by   1
35, 37. (d) 11 11 11

Remainder = ? LCM of 15, 18, 36 So,

r
x + y = 12
72 73 78 76 180k + 9 4k + 9

si
    = ,k=6
35 35 35 35 11 11 43. (a)
Number less than 1000 are
an by
Remainder = 2 × 3 × 8 × 6 =
288
35
, 
180  6  9 1089
11

11
1000
divisible by 5 are = 

 5

– 1

n
Remainder = 8 Sum of digit = 1 + 0 + 8 + 9 = 18
= 199
32. (c) 38. (a) Number less than 1000 are
ja
5x2y6z, divisible by, 7, 11, 13 785x3678y,
R s
1000
Number 1001 divisible by 7, Divisible by 72, 8 × 9 divisible by 7 are == 142
7
11, 13
a th

78y Number less than 1000 are


So ___ 5x2y6z , y=4
x=6 8 1000
divisible by 35 are = = 28
y=5 35
785x 36784 Number less than 1000 are
ty a

z=2 , x=6
9 divisible by 7 or 5 but not 35 are
No – 562562 is divided by 7,
(x + y) = 6 + 4 = 10 = 199 + 142 – (2 × 28)
di M

11, 13 = 341 – 56 = 285


39. (c)
(x – y + 3z) = (6 – 5 + 3 × 2) = 7 44. (d)
LCM of 5, 6, 8 = 120
33. (d) 65  13 × 5
Number between = 300 – 700
x × x = 2809 = 13 × n + 56
x2 = 532 I – 120 × 3 = 360
put n = 10
x = 53 II – 120 × 4 = 480
= 13 × 10 + 56 = 186
34. (c) III – 120 × 5 = 600
divided 186 by 65 it gives
LCM of 5, 6, 7 = 210 Total = 3 remainder 56
I – 210 × 1 = 210 40. (d) So, After divideing 186 by 13 gives
II – 210 × 2 = 420 remainder = 4 x = 4
3 51
III – 210 × 3 = 630
2 = 5 x – 2  5  4 – 2  18  3 2
Between 400 and 700
A

420 and 630, 2 Numbers 4 12 45. (d)


35. (a) 3 LCM of (16, 24, 30, 36 and 45) is
7 1 LCM = 2 × 2 × 2 × 2 × 3 × 3 × 5
3 51
5 = 720
2 As we know,
4 12 7 × 1 + 5 = 12 Number is perfect square
3 12 × 4 + 3 = 51
LCM = 2 × 2 × 2 × 2 × 3 × 3 × 5 × 5
51 × 3 + 2 = 155 = 3600
7 1
155 When 3600 is divided by 123,
5 , Remainder = 29
42 Then the remainder is 33

Aditya Ranjan (Excise Inspector) Selected Selection 89

Downloaded by Kunal Chauhan (kc6970929@gmail.com)


lOMoARcPSD|39315497

Number System

46. (d) 50. (c)


2001
– = 667 + 500 – 166 = 10001 Divisibility rule of 3  A number
538xy is divisible by (3, 7 and 11) 123
is divisible by 3 if the sum of its
value of (x + y )
2 2
Number which are divisible by 3 digit is divisible by 3
= put, x = 2, y = 3 or 4 but not 5 = (Number divisible
by 3 or 4) – (Numbner divisible by Divisiblity rule of 11  If the
then, 53823 is exactly divisible by
LCM of 3 and 5) – Number divisible difference of the alternative sum
(3, 7, 11)
by LCM of 4 and 5) + (Number of digit of the number is a multiple
So, (x2 + y2) = ((2)2 + (3)2) = 13 of 11
divisible by LCM of 3, 4, and 5)
47. (b) Sum of the digit of the number
Number which is divisible by 3 or
As we know, 6 = 2 × 3 4 but not by 5 upto 2001 are (1563241234351)
Check the options using the = (1 + 5 + 6 + 3 + 2 + 4 + 1 + 2 + 3 +
2001 2001 2001
divisibility rule of 2 and 3. =  – = 1001 – 33 4 + 3 + 5 + 1) = 40
15 20 60
5643252 is exactly divisible by 6
– 100 + 33 = 801 40 is not divisible by 3
48. (b)
49. (d) Now, divisibility rule (11)
Number which are divisible by 'a'
Number 7306 – 6454 = 852

r
or 'b' = (Number divisible by 'a') + (5 + 3 + 4 + 2 + 4 + 5) – (1 + 6 + 2 +
(Number divisible by 'b') – Number 8797 – 7306 = 1491 1 + 3 + 3 + 1)

si
divisible by LCM of 'a' and 'b') 8797 – 6454 = 2343
= (23 – 17) = 6
Now, an by HCF of (852, 1491, 2343) = 213
6 is not multiple of (11) so the
Number which are divisible by 3 6454 divided by 213 number is not divisible by 11

n
2001 2001 Remainder 64
or 4 up to 2001 are =  Hence the given number is neither
3 4 d – r = (213 – 64) = 149 divisible by 3 nor by 11.
ja
R s
a th


ty a
di M
A

Aditya Ranjan (Excise Inspector) Selected Selection 90

Downloaded by Kunal Chauhan (kc6970929@gmail.com)


lOMoARcPSD|39315497

LCM & HCF

LCM AND HCF


15

1. A number when divided by 15 and (a) 16 (b) 18 11. Find the HCF of 238 and 832.
18 every time leaves 3 as a re- (c) 18 (d) 12 5! !5 1
mainder, the least possible num- 6. Find the HCF of two numbers 960
ber is ’(
and 1020 is
SSC CPO 10/11/2022 (Shift-01)
) :7; ;5; 1
2 3 4 (a) 16 (b) 8
! " # CPO
SSC $ %
09/11/2022 (Shift-03) (c) 2 (d) 14

r
& ’( (a) 40 (b) 120 12. The HCF and LCM of two
SSC CPO 09/11/2022 (Shift-01) numbers are 9 and 126,
(c) 60 (d) 80

si
respectively. If one of the
(a) 83 (b) 103 7. The LCM of x and y is 441 and their numbers is 18, then what is the
(c) 39 (d) 93 HCF is 7. If x = 49 then find y is: other number?
2.
an by
The ratio of two numbers is 4 :
5 and their HCF is 3. What is
x y
.*
0 +1
1 2
2 ), ,
9 ( )2
1

n
their LCM? C - : 57
x = 49 #y & ’( ’ # )@
) *+ , -
.* ja
/ / / ! ( .* 0 + 1 SSC CPO 09/11/2022
2 (Shift-03) 4
R s
3 4 (a) 56 (b) 36
SSC CPO 10/11/2022 (Shift-01)
SSC CPO 09/11/2022 (Shift-02) (c) 65 (d) 63
a th

(a) 63 (b) 36
(a) 48 (b) 80 8. Find the sum of the numbers
between 550 and 700 such that (c) 84 (d) 24
(c) 60 (d) 36
when they are divided by 12, 16 and 13. The least common multiple of 48
3. The least common multiple of a
24, leave remainder 5 in each case. and 64 is _______ times the
ty a

and b is 42. The LCM of 5a and highest common factor of 12 and


11b is:
; 9;; < .*
18.
6 & # = 5# 7# 5,
di M

a b 0 +1 5a 2 ,5 ( , 7, 0 +1
*
11b 0 +1 2 3 4" 1 _______2
6 (
SSC CPO 09/11/2022 (Shift-01) 6+* (
SSC CPO 09/11/2022 (Shift-03)
(a) 2310 (b) 4620
(a) 1980 (b) 1887 SSC CPO 10/11/2022 (Shift-01)
(c) 210 (d) 462
(c) 1860 (d) 1867 (a) 28 (b) 32
4. The sum of two numbers is 1224
and their HCF is 68. The number 9. The greatest four digit number (c) 24 (d) 16
of pairs of numbers satisfying the which is exactly divisible by 15,
14. The highest common factor of
above condition is: 24 and 40 is:
108, 72 and 5a is a. What can be
) 6 5 5 ,< .* >? & common multiple of
the least
/ / / 7 ( + # *5 , ,; @ A 2108,
- 72 andBa? (
+8 . 3 SSC
+ % CPO 09/11/2022
4 (Shift-03) ; # 9 5 5a .D< =
A

SSC CPO 09/11/2022 (Shift-02) (a) 9960 (b) 9940


6 + A a * 108,
> 72 a 0 +1
(a) 3 (b) 4 (c) 9990 (d) 9980
2 3 4
(c) 6 (d) 2 10. Find the largest number which
5. If three numbers are in the ratio completely divides 24, 56 and 96. SSC CPO 10/11/2022 (Shift-02)
of 1 : 3 : 7, and their LCM is 336, >? & (a)
’ 432 5,# (b) 324
then their HCF is:
7 :7 @A 2 - (c) 108 (d)( 216
) * ’ - ! - 9 *+
SSC CPO 09/11/2022 (Shift-03) 15. What is the greatest positive
.* 0 +1 2 !!7 integer that divides 554, 714 and
# .* 1 2 (a)34 4 (b) 1
213 leaving the remainder 43, 57
SSC CPO 09/11/2022 (Shift-02) (c) 8 (d) 2 and 67, respectively?

Aditya Ranjan (Excise Inspector) Selected Selection 91

Downloaded by Kunal Chauhan (kc6970929@gmail.com)


lOMoARcPSD|39315497

LCM & HCF

>? E* @21.
A The F ratio of*two numbers is 5 : 7 26. Find the LCM of 28 and 92.
,# 9 , 5 ! and their HCF is 3. Their LCM is: 5 :5 0 +1
* C - , ! #) 9 *+ - 9 ’(
79 " 4 .* 1 2 ! ( . * SSC CPO
0 + 11/11/2022
1 (Shift-01)
SSC CPO 10/11/2022 (Shift-02) 2 & ’ ( (a) 464 (b) 644
(a) 95 (b) 71 SSC CPO 10/11/2022 (Shift-03)
(c) 161 (d) 322
(c) 83 (d) 73 (a) 75 (b) 105 27. What is the LCM of a3b –ab3, a3b2
16. Find the LCM of 25, 30, 50 and (c) 125 (d) 35 – a2b3, ab(a – b)?
75. 3 7 13 a3b –ab3, a3b2 – a2b3, ab(a – b)
22. The HCF of , and is:
5 # !;# ; 9 0 +1 " 4 8 2 14 0 +1 2 & (
& ’( SSC CPO 11/11/2022 (Shift-02)
3 7 13
SSC CPO 10/11/2022 (Shift-02)
, 1 I (a)2 a2b2(a2 + b2) (b) a2b2(a2 – b2)
4 8 14
(a) 15.0 (b) 150 3 4 (c) a b (a + b )
2 3 2 2
(d) a3b2(a2 – b2)
(c) 1.50 (d) 75 SSC CPO 10/11/2022 (Shift-03)
28. The LCM of two numbers is 840
and their HCF is 7. If one of the

r
17. Find the HCF of 364 and 724. numbers is 56, find the other.
1 1
!7, 95, 1 " (a) 362 & (b)
) / / / ,;

si
56
’( / / / 9 ( ) ’ 7
an by
SSC CPO 10/11/2022 (Shift-02) 1 1 )@ & ’(
(c) (d)
70 60
(a) 4 (b) 2 SSC CPO 11/11/2022 (Shift-02)

n
(c) 6 (d) 364 23. If the LCM of two numbers a and (a) 120 (b) 112
b is 60 and their HCF is 15.
18. The greatest number of five digits (c) 105 (d) 84
Determine their mean proportion.
ja
which is divisible by 13, 15, 18
+29.
1 The LCM and ratio of three
R s
and 21 is: ) ) a b 0 numbers are 1386 and 3:7:11,
< >? > ? (LCM)
2 7; .* 1
respectively. The sum of the
a th

*G !# # 5 (HCF)
2 # . * greatest and least numbers is:
B 4 J *+ & ’( * 0 +1
SSC CPO 10/11/2022 (Shift-03) ! *+ C -
SSC CPO 10/11/2022 (Shift-02)
ty a

(a) 30 (b) 25 !-9- ( >?


(a) 99120 (b) 98280
(c) 60 (d) 4 6 & ’
(c) 96840 (d) 95830
di M

SSC CPO 11/11/2022 (Shift-03)


19. Find the greatest number that 11 9 16
24. Find the HCF of , ,
exactly divides 15, 30 and 40. 25 20 15 (a) 60 (b) 64

>? >? & 10 ’ # (c) 84 (d) 108


and . 30. The highest common factor of
# !; ,; @ A 2 H 33
120, 360 and 210 is:
( 11 9 16 10
, , / / / & 5;# !7; 5 ; 1
SSC CPO 11/11/2022 (Shift-02) 25 20 15 33
3 4
(a) 3 (b) 40 ’(
SSC CPO 11/11/2022 (Shift-03)
(c) 5 (d) 15 SSC CPO 10/11/2022 (Shift-03)
(a) 12 (b) 30
20. What is the greatest number of
1 1 (c) 3 (d) 15
six digits, which when divided by (a) (b)
3300 330
each of 16, 24, 72 and 84, leaves 31. Two numbers are in the ratio 7:11.
A

the remainder 15? If their HCF is 28, then the


1 1 difference between the two
(c) (d)
$ >? > ? 33 300 numbers is:
*G # 7 # 25.
5 , #Find
95 , 96 and 99.
the HCF of 69, ) 7:11 * + ( )
*7:# :7 :: 1 .* 2 1 (HCF)2 5 #
< 4 & ’( ) * <
SSC CPO 10/11/2022 (Shift-03) SSC CPO 11/11/2022 (Shift-01) SSC CPO 23/11/2020 (Shift-1)
(a) 999981 (b) 999951 (a) 3 (b) 9 (a) 28 (b) 196
(c) 999963 (d) 999915 (c) 1 (d) 6 (c) 112 (d) 308

Aditya Ranjan (Excise Inspector) Selected Selection 92

Downloaded by Kunal Chauhan (kc6970929@gmail.com)


lOMoARcPSD|39315497

LCM & HCF

32. What is the least number when ) M / /29N HCF #42. When 2388, 4309 and 8151 are
divided by 15, 18 and 36 leaves divided by a certain 3-digit number,
the same remainder 9 in each
. LCM
* M / / /N = the) remainder in each case is the
case and is divisible by 11? 6 + A 15
* >13 ( .* ) same. The remainder is:
$ % $ % & > ? # & ( 5! # ,!;:
15, 18 36 * SSC CPO 24/11/2020 (Shift-2) ’ * <
9 K " (a) 435
# (b) 377 # *
B ( (c) 406 (d) 464 ( & ’(
SSC CPO 23/11/2020 (Shift-1) 38. Two numbers are in the ratio 7 :
SSC CPO 09/12/2019 (Shift-02)
11. If their HCF is 28, then the
(a) 1269 (b) 1089 sum of the two numbers is: (a) 39 (b) 23
(c) 1080 (d) 1071 ) 7 : 11 * + ( ) (c) 19 (d) 15
33. The ratio of two numbers is 7:13 .* 1 (HCF)228 #43. Let x be the least 4-digit number
and their HCF is 8. Their LCM is: which divided by 2, 3, 4, 5, 6 and
) * 6 & (
) 7:13 * + 7 leaves a remainder of 1 each
SSC CPO 25/11/2020 (Shift-1) case. If x lies between 2800 and
.* / (HCF)
/ / 8 ( .* / / / 3000, then what is the sum of the
(a) 196 (b) 504

r
(LCM) & ( digits of x?
(c) 112 (d) 308
SSC CPO 23/11/2020 (Shift-2)

si
39. The HCF of two numbers is 29, * x ’ ,G $ %
(a) 628 (b) 728 and the other two factors of their # 5# !# ,# #
(c) 872 an by (d) 782 LCM are 15 and 13. The smaller *
of the two numbers is:
34. The least number which is exactly < x( ) * 5 ;; !;;;
) (HCF) /29 / / #

n
divisible by 5, 6, 8, 10 and 12 is: J x# 6
.* (LCM)
/ / / = )
$ % $ % 5, & # SSC CPO 09/12/2019 (Shift-02)
ja 6 + A 15
* >13 ( .* )
6, 8, 10 12 @A 2 - B $ %( ( (a) 15 (b) 16
R s
&
SSC CPO 23/11/2020 (Shift-2) SSC CPO 25/11/2020 (Shift-2)
(c) 12 (d) 13
a th

(a) 180 (b) 150 44. If r is the remainder when end of


(a) 435 (b) 377
4749, 5601 and 7092 is divided by
(c) 240 (d) 120 (c) 464 (d) 406 the greatest possible number d(>1),
35. The least number which is exactly 40. Five bells ring together at the then the value of (d + r) will be:
ty a

divisible by 4, 5, 8, 10 and 12 is: intervals of 3, 5, 8, 9 and 10


) ,9,:# 7; 9;:5
seconds. All the bells ring
$ % $ % 4, & simultaneously
# at the same time. >? >? d M
> N
di M

5, 8, 10 12 @A 2 - TheBwill again
( ring simultaneously #
SSC CPO 24/11/2020 (Shift-1)
after. r < d#+ r N M * &
O < 3,
0 5, 8,%
9 10 A> SSC CPO 11/12/2019 (Shift-01)
(a) 150 (b) 240
’ ( 0
% O (b) 271
(a) 276
(c) 180 (d) 120
’ ’ ( *
(c) 298 (d) 282
36. What is the least number of
) +*- ’ 6 4
45. Let x be the least number of 4
soldiers that can be drawn up in
troops of 10, 12, 15, 18 and 20 SSC CPO 25/11/2020 (Shift-2) digits number when divided by 2,
soldiers, and also in form of a (a) 8 minutes (b) 9 minutes 3, 4, 5, 6 and 7 leaves a remainder
solid square? of 1 in each case. If x lies between
(c) 4 minutes (d) 6 minutes 2000 and 2500, then what is the
.* * = @* & # 4 6 8 sum of the digits of x?
=10, 12, 15, 18 20 * 41. What is the HCF of 5 , 8 , 25 ?
A

* x ’< $ %
%+ >? L 62 H 5# !# ,# #
( 4 6 8
, , / (HCF)
/ & ’( *
5 8 25
SSC CPO 24/11/2020 (Shift-2) < x( ) * 5;;; 5 ;
SSC CPO 09/12/2019 (Shift-01)
(a) 400 (b) 625 < x # 6
(c) 900 (d) 180 1 1 ’(
(a) (b)
37. The HCF of two numbers is 29, 100 5 SSC CPO 11/12/2019 (Shift-01)
and the other two factors of their (a) 9 (b) 15
1 1
LCM are 15 and 13. The largest (c) (d)
50 200 (c) 10 (d) 4
of the two numbers is:

Aditya Ranjan (Excise Inspector) Selected Selection 93

Downloaded by Kunal Chauhan (kc6970929@gmail.com)


lOMoARcPSD|39315497

LCM & HCF


46. Let x be the least number !9! # 7 : : ; >7? # 9 7> #? ! (LCM)
,: /
divisible by 13, such that when x x k (# k (HCF) / /
is divisible by 4, 5, 6, 7, 8 and 12,
y < x (y P p #
the remainder in each case is 2.
The sum of the digits of x is: 6 & ’( " p (* Q’(
* x,
’ $ % $ % SSC CPO #12/12/2019 (Shift-01) SSC CPO 13/12/2019 (Shift-01)
! @A 2 - x B , ## (a) 3783
* ) (b) 3637 (a) 2520 (b) 5040
# 7# 9# 5 (c) 3673 (d) 3738 (c) 6720 (d) 3360
x 5 The
49. < HCF and
( LCM of two numbers 52. Let x be the least number when
6 & ’( are 8 and 48 respectively. If the divided by 8, 9, 12, 14 and 36
ratio of the two numbers is 2 : 3, leaves a remainder 4 in each case,
SSC CPO 12/12/2019 (Shift-01) but x is divisible by 11. The sum
then the largest of the two
(a) 10 numbers is: of the digits of x is.
(b) 11 * x ’ $ % $ %
) (HCF) / / / /
(c) 9
(LCM) C - , ( ) ) # :# 5# , !7
(d) 8 , x,
<11 #
47. In finding the HCF of two
*+ 5 - ! # )
Bx ( 6 3

r
numbers by division method, the >? *G 4
quotients are 1, 8 and 2 SSC CPO 13/12/2019 (Shift-02)
SSC CPO 12/12/2019 (Shift-02)

si
respectively, and the last divisor (a) 5 (b) 6
is 105. What is the sum of the (a) 24 (b) 18
an by (c) 9 (d) 4
number? (c) 48 (d) 16
53. The sum of two numbers is 1215
* E P ) 50. What is the sum
/ / of the greatest and their HCF is 81. If the

n
(HCF) & * 6 C three
- # digit number and the
numbers lies between 500 and
smallest four digit number such
5 " # ; 700, then the sum of the
that their HCF is 23?
( ) * ja 6 * 4 reciprocals of the numbers is?
R s
* . >? >? ) < 6 5
SSC CPO 11/12/2019 (Shift-02)
. $ % $ % (6 3 )
a th

(a) 3570 / (HCF)


/ 81 ’ ;
# * (HCF)/ 23
/ 4 9;; < #
(b) 3885
(c) 3780 SSC CPO 12/12/2019 (Shift-02) 6 6 4
(d) 3675 (a) 2001 (b) 2002 SSC CPO 13/12/2019 (Shift-02)
ty a

48. When 3738, 5659 and 9501 are (c) 1984 (d) 1998
5 5
divided by the greatest possible 51. The LCM of 165, 176, 385 and 495 (a) (b)
di M

number x, the remainder in each 1512 378


is k. When k is divided by the
case is y. What is the sum of x HCF of the numbers the quotient 5 5
and y? is p. What is the value of p? (c) (d)
702 1188

ANSWER KEY
1.(d) 2.(c) 3.(a) 4.(a) 5.(a) 6.(c) 7.(d) 8.(b) 9.(a) 10.(c)
A

11.(c) 12.(a) 13.(b) 14.(d) 15.(d) 16.(b) 17.(a) 18.(b) 19.(c) 20.(b)

21.(b) 22.(b) 23.(a) 24.(a) 25.(a) 26.(b) 27.(b) 28.(c) 29.(c) 30.(b)

31.(c) 32.(b) 33.(b) 34.(d) 35.(d) 36.(d) 37.(a) 38.(b) 39.(b) 40.(d)

41.(a) 42.(d) 43.(b) 44.(a) 45.(d) 46.(b) 47.(c) 48.(a) 49.(a) 50.(a)

51.(b) 52.(d) 53.(a)

Aditya Ranjan (Excise Inspector) Selected Selection 94

Downloaded by Kunal Chauhan (kc6970929@gmail.com)


lOMoARcPSD|39315497

LCM & HCF

SOLUTIONS
1. (d) 8. (b) 14. (d)
LCM of 15 and 18 = 90 Smallest number which is 108 = 72 × 1 + 36
Required Number = 90 + 3 = 93 divisible by 12, 16 and 24 72 = 36 × 2 + 0
2. (c) = LCM (12, 16 and 24) = 48 Hence, 5a = 5 × 36 = 180
When we divide 550 by 48 we get Now,
LCM of numbers
quotient = 11 remainder = 22
= HCF × [product of ratio] 108 = 2 × 2 × 3 × 3 × 3
Thus, The first number greater
= 3 × 4 × 5 = 60 than 550 which is divisible by 48 72 = 2 × 2 × 2 × 3 × 3
3. (a) = 550 + (48 – remainder) 36 = 2 × 2 × 3 × 3
Given that, LCM of a and b is = 42 = 550 + 26 = 576 LCM = 2 × 2 × 2 × 3 × 3 × 3
Hence LCM of 11a and 5b Hence, the number which are = 216
= 11 × 5 × (LCM of a and b) divisible by 48 and leave 15. (d)
remainder 5, 554 – 43 = 511

r
= 55 × 42 = 2310
1st number = 576 + 5 = 581
4. (a) 714 – 57 = 657
2nd number = 576 + 48 = 624 + 5

si
Let the two numbers be Ha and 213 – 67 = 146
= 629
Hb and their HCF is H. 3rd number = (576 + 48) + 48 = Now,
an by
Given, sum of Numbers = 1224
and HCF = 68
Therefore, Ha + Hb = 1224 9.
672 + 5 = 677
Sum = 581 + 629 + 677 = 1887
(a)
Prime Factorisation of
511 = 7 × 73
657 = 3 × 3 × 73

n
 H(a + b) = 1224 LCM of 15, 24 and 40 = 120 146 = 2 × 73
Largest 4 digit number = 9999 Hence, HCF = 73
1224
ja On dividing 9999 by 120, We get
 ab 
R s
68 Therefore, The greatest positive
quotient = 83 and remainder = 39
integer that divides 554, 714 and
 a + b = 18 Hence, Required number
a th

213 and leaves the remainder 43,


Now, Assume the such value of = 9999 – 39
= 9960 57, and 67 respectively is 73.
a and b whose sum is 18 and HCF
is 1. 16. (b)
SMART APPROACH:-
a = 1 and b = 17 Factor of 25 = 5 × 5
You can direct check which larger
ty a

a = 5 and b = 13 option is completely divisible by 120 Factor of 30 = 2 × 3 × 5


a = 7 and b = 11 10. (c) Factor of 50 = 2 × 5 × 5
Factor of 75 = 3 × 5 × 5
di M

There are 3 pairs. HCF of the 24, 56 and 96 = 8


5. (a) 11. (c) LCM = 5 × 5 × 3 × 2 = 150
LCM of numbers = HCF × [product Factor of 238 = 2 × 7 × 17 17. (a)
of ratio] Factor of 832 = 2 × 2 × 2 × 2 × 2 Factor of 364 = 2 × 2 × 7 × 13
× 2 × 13 Factor of 724 = 2 × 2 × 181
 336 = HCF × [1 × 3 × 7]
Therefore, HCF = 2 HCF = 4
336 12. (a) 18. (b)
 HCF  We know that,
21 LCM of 13, 15, 18 and 21 = 8190
First no. × second no = LCM ×
 HCF = 16 Greatest 5 digit number = 99999
HCF
6. (c)  18 × second no = 126 × 9 When we divide 99999 by 8190,
Prime Factorization of 960 we get,
126  9 quotient = 12 and remainder =
= 26 × 3 × 5  Second no =
Prime Factorization of 1020 18 1719
A

= 22 × 3 × 5 × 17  Second no = 63 Required number = 99999 – 1719


13. (b) = 98280
HCF = 22 × 3 × 5 = 60
LCM of 48 and 64 = 192
7. (d)
HCF of 12 and 18 = 6
SMART APPROACH:-
We know, We can direct result by
ATQ,
Product of numbers = LCM × HCF divisibilty 5 and 9 together
LCM of 48 and 64 = m × HCF of only option (b) is correct.
 x × y = 441 × 7 12 and 18
19. (c)
 49 × y = 3087  192 = m × 6
15 = 3 ×5
3087 192 30 = 2 ×3×5
y  m
49 6 40 = 2 ×2×2×5
 y = 63  m = 32 HCF = 5

Aditya Ranjan (Excise Inspector) Selected Selection 95

Downloaded by Kunal Chauhan (kc6970929@gmail.com)


lOMoARcPSD|39315497

LCM & HCF


20. (b) 27. (b) 36. (d)
LCM of 16, 24, 72 and 84 = 1008 a3b – ab3 = ab(a2 – b2) The least number of soldiers
When we divide largest 6 digit a3b2 – a2b3 = a2b2(a – b) = LCM (10, 12, 15, 18, 20) = 180
number – 999999 by 1008 we get– ab(a – b) = ab(a – b) Required number = 180 × 5 = 900
remainder = 63 LCM = a2b2(a2 – b2) Therefore, 900 solider can be
Largest 6 digit number which is 28. (c) drawan up in form of a sequare
divisible by 1008 First number × second number 37. (a)
= 999999 – 63 =LCM × HCF HCF = 29
= 56 × second number = 840 × 7 The largest Number
= 999936
= 15 × 29 = 435
Hence, Required Number 840  7
 Second number = 38. (b)
= 999936 + 15 56 The sum of two number = (7 + 11)
= 999951  Second number = 105 × 28 = 18 × 28 = 504
29. (c) SMART APPROACH:-
SMART APPROACH:-
Let the numbers be 3x , 7x and Divisibility Rule by 9
We can direct result by
divisibility 4, 8 and 9 11x .
Hence, HCF = x 39. (b)
 999951 – 15 = 999936
Only option (b) is correct.  LCM = HCF × product of ratios HCF = 29
The Smallest Number = 29 × 13
 1386 = x (3 × 7 × 11)

r
21. (b) = 377
LCM = HCF × product of ratios 1386
SMART APPROACH:-

si
 x  , x =6
=3×5×7 3  7  11
Divisibility Rule by 13
= 105 Least Number = 3x
22. (b)
an by
HCF of fraction
Greatest Number = 11 x
Sum = 14x
40. (d)
LCM [3, 5, 8, 9, 10]
360 seconds

n
= 14 × 6 = 84
HCF of Numerator 6 minutes
= LCM of Denominator After 6 minutes all the five bells
ja SMART APPROACH:- ring together.
R s
Sum of least and greatest ratio
HCF  3,7,13 1 = 3 + 11 = 14 units
41. (a)
= LCM  4, 8, 14  = 4 6 8
a th

56 Check which option is multiple


of 14 if there is only one multiple HCF of , ,
5 8 25
23. (a) this will be the best approach
Product = LCM × HCF HCF of Numerator
30. (b)
 ab = 60 × 15 LCMof Denomenator
ty a

120 = 2 × 2 × 2 × 3 × 5
Mean proportion = ab 360 = 2 × 2 × 2 × 3 × 3 × 5 HCF of (4,6,8) 2 1
210 = 2 × 3 × 5 × 7 = LCMof (5,8,25)  200  100
di M

= 60  15 = 30 HCF = 2 × 3 × 5 = 30
24. (a) 31. (c) 42. (d)
HCF of fraction Ratio = 7 : 11, HCF = 28 2388, 4309, 8151,
HCF of Numerator Difference = (11 – 7) × 28
= LCM of Denominator = 4 × 28 = 112
1921 3842
32. (b)
HCF 11, 9, 16, 10  1
= = (I) LCM of 15, 18, 36 = 180
LCM  25, 20, 15, 33 3300 Number 180 × k + 9 1921
25. (a) k  6
Factor of 69 = 3 × 23 180 × 6 + 9 17 × 113
Three digit number is = 113
Factor of 96  1089 When we divide 2388, 4309, 8151
=2×2×2×2×2×3 (II) By option by 113 we find remainder 15 in
A

Factor of 99 = 3 × 3 × 11 In option subtract 9 and divide each case


HCF = 3 by 15, 18, 36 Remainder = 15
26. (b) 33. (b) 43. (b)
Factor of 28 = 2 × 2 × 7 Ratio = 7 : 13, HCF = 8 LCM of (2, 3, 4, 5, 6, 7) = 420
Factor of 92 = 2 × 2 × 23 LCM = 7 × 13 × 8 = 420 n + 1
LCM = 2 × 2 × 7 × 23 = 56 × 13 = 728 Put n = 7(Because the x lies
= 644 34. (d) between 2800 and 3000)
5, 6, 8, 10, 12  LCM = 120 = 420 × 7 + 1
SMART APPROACH:-
35. (d)  2940 + 1 = 2941
Check which option are
divisible by 28 LCM of 4, 5, 8, 10, 12 The sum of digit = (2 + 9 + 4 + 1) =
 120 16

Aditya Ranjan (Excise Inspector) Selected Selection 96

Downloaded by Kunal Chauhan (kc6970929@gmail.com)


lOMoARcPSD|39315497

LCM & HCF

44. (a) 48. (a) 51. (b)


4749, 5601, 7092, LCM of (165, 176, 385 and 495)
3738 5659 and 9501
= 55440
852 1491 HCF of the number = 11
1921 3842
55440
639 So, P = = 5040
1921 11

213 × 3 H.C.F of 1921, 3842 and 5763 52. (d)


d = 213 = 1921 LCM of (8, 9, 12, 14, 36) = 1008
After dividing (4749, 5601, 7092) by When 3738, 5659 and 9501 are x be the least number and also
213 in each case get remainder = divided by 1921
63 divisible by 11
Remainder (y) = 1817
Value of (d + r) = (213 + 63) = 276 1008x  4
45. (d)  x + y = 1921  1817 = 3738 = put x = 1
49. (a) 11
LCM of (2, 3, 4, 5, 6, 7) = 420
= 420n + 1 Product = HCF × LCM = 1008 + 4 = 1012 (divisible by 11)
= 420 × 5 + 1 2x × 3x = 8 × 48 Least number = 1012

r
= 2100 + 1 = 2101 x2 = 4 × 16 Sum of digit = (1 + 0 + 1 + 2) = 4
Sum of digit = (2 + 1 + 0 + 1) = 4 x = 8 53. (a)

si
46. (b) Hence, Largest number = 8 × 3
LCM of (4, 5, 6, 7, 8, 12) = 840 Number be 81x and 81y
= 24
= 840n + 2
put n = 3
an by
= 840 × 3 + 2 = 2522
50. (a)
Greatest 3-digit number is 999
81x + 81y = 1215

x+y=
1215

n
2522 is least number which is When 999 is divided by 23, the 81
divisible by 13 remainder is 10 x + y = 15
ja
Sum of digit = (2 + 5 + 2 + 2) = 11 So, the greatest 3-digit number
Possible value of x and y for which
R s
47. (c) which is divisible by 23
1 8 2
81x and 81y lies between 500 and
= (999 – 10) = 989
a th

700.
Now, the smallest 4-digit number
Hence, sum of reciprocal of the
×

is 1000
85

5
0

10
21

number is
17

When 1000 is divided by 23, the


remainder is 11
ty a

1785 + 210 1680 + 105 210 + 0 1 1 1 1 1


So, the smallest 4-digit number  
81x 81y = 81  x y 
1995 1785 210 which is divisible by 23
di M

1st 2nd = 1000 + (23 – 11) = 1012


Some of these two number Hence, Sum of both the numbers 1  1 1
1995 + 1785 = 3780
=   
= 989 + 1012 = 2001 81  7 8 


A

Aditya Ranjan (Excise Inspector) Selected Selection 97

Downloaded by Kunal Chauhan (kc6970929@gmail.com)


lOMoARcPSD|39315497

Simplification

SIMPLIFICATION
16

1. Simplify the following 6. Solve the following equation. 12. Solve the following equation.

123 × (162 – 142 – 40) ÷ 2 – 94 = ? 3 76  19 


3
456 –  76  15 2   of  ?
814 + [(20 ÷ 5 of 3 × 6)+{(8 ÷ 24 4 18  72  24 
SSC CPO 09/11/2022 (Shift-03)
–2 SSC CPO 10/11/2022 (Shift-02)
 1 5 (a) 17280 (b) 6561
of 3)× 4} – 10 ÷ 5] –   (a) 443 (b) 256
 32  (c) 10719 (d) 986
(c) 356 (d) 401
SSC CPO 09/11/2022 (Shift-01) 7. Simplify the following expression.

r
13. The value of/
1 1
(a) 24 (b) 21

si
72 – 42  6 of 7 × 4 of 3  4 × 3 –5 5 3 4 6
4 9 – 1  3
2 7 5 7
SSC CPO 09/11/2022 (Shift-03)
4 4 3 2 1 1
(c) 27
5
an by (d) 29
9
(a) 32
(c) 58 (d) 45
(b) 16 2
 1  3 1
5 2 4
SSC CPO 10/11/2022 (Shift-03)

n
2. The value of/ 8. If {(1  x) × (x3 – 2x2) + (43 ÷ 24)×
x}  25 = 1, then the possible 3 2
3
–2744  3 –216 (a) 2 (b) 1
20 20
ja values of x is/are:
R s
64 3 7
3
{(1  x) × (x3 – 2x2) + (43 ÷ 24)× x} (c) 1 (d) 1
729 20 20
 25 = 1 x
a th

SSC CPO 09/11/2022 (Shift-01) 14. Solve/


(a) 164 (b) 512
2 
(c) 189 (d) 156
SSC CPO 09/11/2022 (Shift-03)
1  7  16  8  2   6  2
2
 6  .
(a) –5 (b) 5  36 
ty a

3. 72  8 × [{(57 – 49)  2} + {(85 –


(c) 0 (d) Both 5 and –5 SSC CPO 10/11/2022 (Shift-03)
60)  5} – {(28 – 19)  3}] is equal
to: 9. What is the simplified value of (a) 17 (b) 21
di M

(0.00243)(0.2)? (c) 19 (d) 12


72  8 × [{(57 – 49)  2} + {(85 –
60)  5} – {(28 – 19)  3}] (0.00243)(0.2) 15. The value of/
SSC CPO 10/11/2022 (Shift-01)  2  1  1 1   1 1
(a) 0.3 (b) 0.03 1  3   –      4  1 
SSC CPO 09/11/2022 (Shift-02)  7  2  2 7   5 2
(a) 56 (b) 46 (c) 0.003 (d) 0.0003
SSC CPO 11/11/2022 (Shift-01)
(c) 54 (d) 60 10. The simplified value of (0.2)3 × 400
 2000 of (0.2)2 is: 1 3
4. The value of 1801 ×1801 is: (a) (b)
3 4
(0.2)3 × 400  2000 of (0.2)2
1801 × 1801 (c) 2 (d) 1
SSC CPO 09/11/2022 (Shift-02) 16. Find the value of following expression.
SSC CPO 10/11/2022 (Shift-01)
(a) 3423601 (b) 3243601
1 3
(c) 2343601 (d) 3243106 (a) (b)
A

25 25  5  3  5 3  
5. What is the value of 'a' in the  –  –  –   of 8.8 – 1.2
below equation? 2  8  8  8 8  
(c) (d) 30 m
'a' 25 1 1 3 3
4  2.5  2  of 60   – 
11. Solve the following equation. 6 6 4 8

{(5 × 5 × 5 × 5 × 5 × 5)5 × (5 × 5 × SSC CPO 11/11/2022 (Shift-01)


(73 – 72 + 6) × (63 – 23 – 32 + 1) 
5 × 5 × 5) }  (5 × 5) = (625)
8 a
(212 + 202 – 41) = ? 22 23
(a) 5 (b) 3
SSC CPO 09/11/2022 (Shift-02) SSC CPO 10/11/2022 (Shift-02) 43 67
(a) 14 (b) 17 (a) 75 (b) 40 44 4
(c) 4 (d) 4
(c) 16 (d) 15 (c) 50 (d) 25 85 5

Aditya Ranjan (Excise Inspector) Selected Selection 98

Downloaded by Kunal Chauhan (kc6970929@gmail.com)


lOMoARcPSD|39315497

Simplification

17. The value of/ 22. Simplify the following expression. 28. The value of/
8 7  1 1 2 1  1 3 1
6  of 1 5  7 25  5 of 12 × 2 – 9  27 of 3 × 5  5 ÷ of  ÷
15 9  10 5 5 5  4 7 2
– 14  28 × 4
 1 7 9  11  1
SSC CPO 11/11/2022 (Shift-02) SSC CPO 11/11/2022 (Shift-03) 5 – 7 ÷ 9 × –  5 ÷ 2of 
 9 8 20  21  2
25 5 –41 –31 SSC CPO 23/11/2020 (Shift-2)
(a) (b) (a) (b)
16 14 91 9
35
–31 (a) 0 (b)
24
25 5 (c) 18 (d)
(c) (d) 18 15
18 18 (c) –2 (d)
23. Solve the following. 28
18. Find the value of a in the following
29. The value of/
equation. (given a < 10)
0.325 × 0.325 + 0.175 × 0.175 + 25 × 0.00455
 ! 16   28 of 7  22  
 2   5 × 0.0065 × 3.25 – 7 × 0.175 × 0.025
25  8  2 –   2
3
a" < 10 # 1   0.5
 – 18  12 of  
2
 –
   8   1.5
187  17  a – 3  3 1 SSC CPO 23/11/2020 (Shift-2)
8 2
– 9  7  a2 

r
SSC CPO 11/11/2022 (Shift-03)
(a) 3 (b) 0
SSC CPO 11/11/2022 (Shift-02) (a) 626 (b) 529 (c) –1 (d) –3

si
(c) 721 (d) 579 30. The value of/
(a) 2 (b) 1
1 1 3 3 1 3
(c) 4

1
x
an by

24
(d) 3 24. The value of/
(1018)2 – 1019 × 1017 + 1015 ×
3
3
÷ 2 of 1 +  + ×1 
2 5 8 7 4
SSC CPO 23/11/2020 (Shift-2)

n
19. If , then the value 1012 – 1016 × 1011
529 23 55 25
of x is: SSC CPO 09/11/2022 (Shift-01) (a) (b)
24 24
ja (a) 1 (b) 4
R s
x 24 35 5
1  x (c) 3 (d) 5 (c) (d)
529 23 24 24
a th

25. Arrange the given ratios in 31. The value of/


descending order 15 : 7, 5 : 11 1 1 3 +  3 + 1 ×1 3 
SSC CPO 11/11/2022 (Shift-02) and 21 : 77. 2  2 of 1  
3 2 5 8 7 4
(a) 15 (b) 27
’ () ’* + SSC CPO *24/11/2020
,- (Shift-1)
ty a

(c) 47 (d) 30
./ " 0 / " .. ’ 1 . "290 0 25
20. What is the value of 12 – 8  2 (a) (b)
24 24
di M

– {16 of (–2) + (3× 5 – 4)} SSC CPO 11/11/2022 (Shift-02)


12 – 8  2 – {16 of (–2) + (3× 5 – (a) 21 : 77 > 5 : 11 > 15 : 7 5 35
(c) (d)
4)} 24 24
(b) 15 : 7 > 5 : 11 > 21 : 77
32. The value of/
SSC CPO 11/11/2022 (Shift-02) (c) 15 : 7 > 21 : 77 > 5 : 11 0.325 × 0.325 + 0.175 × 0.175 + 25 × 0.00455
(a) 29 (b) 45 (d) 5 : 11 > 15 : 7 > 21 : 77 5 × 0.0065 × 3.25 – 7 × 0.175 × 0.025
(c) 1 (d) 0 0.5
26. The value of/ +
21. What is the positive value of the 1.5
following expression? 6.35 × 6.35 × 6.35 + 3.65 × 3.65 × 3.65
63.5 × 63.5 + 36.5 × 36.5 – 63.5 × 36.5 SSC CPO 24/11/2020 (Shift-1)
$ % &
is equal to: 11
(a) (b) 3
SSC CPO 23/11/2020 (Shift-1) 3
(a) 0.01 (b) 10 7
25  4  4 of  (c) 0 (d)
A

  (c) 1 (d) 0.1 3


36  15 of 2 of 29 –  8 – 11  33. The value of/
  27. The value of/
 9  5  5 of 3   3  1  1 3 1
40 –
of 32  5 ÷ 2of  +  5 ÷ of 
4  2  4 7 2
SSC CPO 11/11/2022 (Shift-02) 3
37 – of (34 – 6)  1 7 9  11
4 ÷ 5 – 7 ÷ 9 ×
5 1  9 8 20  21
(a) 1 (b) 1 SSC CPO 23/11/2020 (Shift-1)
6 5 SSC CPO 24/11/2020 (Shift-1)
(a) 1 (b) 0
35 15
4 3 (a) (b)
(c) 2 (d) 2 1 1 24 28
5 5 (c) – (d) (c) –2 (d) 8
2 2

Aditya Ranjan (Excise Inspector) Selected Selection 99

Downloaded by Kunal Chauhan (kc6970929@gmail.com)


lOMoARcPSD|39315497

Simplification

34. The value of/ 40. The value of/ 46. What is the simplified value of:
4.35  4.35  4.35  3.25  3.25  3.25 427  427  427  325  325  325
43.5  43.5  32.5  32.5 – 43.5  32.5 42.7  42.7  32.5  32.5 – 42.7  32.5
SSC CPO 25/11/2020 (Shift-2)  1  1  
2 2
SSC CPO 24/11/2020 (Shift-2) 1 
(a) 75200 (b) 75.2  x   – x –  
(a) 0.0076 (b) 0.76  8  y  y 

(c) 0.076 (d) 7.6 (c) 7520 (d) 752
35. The value of/ 41. The value of/ SSC CPO 09/12/2019 (Shift-01)

2 3 7 ÷ [5 + 1 ÷ 2 – {4 + (4 of 2 ÷ 4) + (5 x 2x
4 4 1 8 3 3 1
 of – 1  –   ÷ 5 of 2)}] (a) y (b)
3 10 9 5 9 15 4 4 2 y
SSC CPO 24/11/2020 (Shift-2) SSC CPO 25/11/2020 (Shift-2)
x 4x
25 14 7 (c) 2y (d)
(a) (b) (a) (b) –7 y
6 3 2

7 1
17 49 (c) – (d) 7
(c) (d) 2 47. The value of is closest to:
9 12 7–4 3

36. The value of/ 42. If (320 + 342 + 530 + 915) ÷ (20 + 22

r
– x + 18) = 43, then the value of x is: 1
– 7 ÷ [5 + 1 ÷ 2 – {4 + (4 of 2 ÷ 4) + 6
7–4 3

si
(4 ÷ 4 of 2)}] (320 + 342 + 530 + 915) ÷ (20 + 22
SSC CPO 24/11/2020 (Shift-2) – x + 18) = 43 x &
(a) –
7
2
an by (b) –7
SSC CPO 09/12/2019 (Shift-01)
(a) 15 (b) 11
SSC CPO 09/12/2019 (Shift-02)
(a) 4.1 (b) 4.2

n
7 (c) 26 (d) 23 (c) 1.2 (d) 3.7
(c) (d) 7 48. The value of
2 43. What is the simplified value of/
37. The value of/ ja 0.325 0.3250.175 0.175 – 25 0.00455
R s
5 0.0065 3.25 – 7 0.175 0.025
3  
40 + of 32  
4
a th

1 – 1   15  2 of 2 1 – 3  4
lies between:/ 8
3  2  16 3 4 33  43
37 + of (34 – 6) 4 – SSC CPO 09/12/2019 (Shift-02)
4  1 
 1  (a) 0.25 and 0.35
1
SSC CPO 25/11/2020 (Shift-1)  2
 3  (b) 0.05 and 0.15
ty a

 
3 9   (c) 0.15 and 0.25
(a) –1 (b) 1
29 29 SSC CPO 09/12/2019 (Shift-01) (d) 0.35 and 0.45
di M

3 3 5 4 49. The value of 5 ÷ [5 + 8 – {4 + (4 of 2


(c) 2 (d) 1 (a) (b) ÷ 4) – (2 ÷ 4 of 2)}] is:/
29 29 13 13
38. The value of/ 8 6 SSC CPO 09/12/2019 (Shift-02)
(c) (d)
5.35  5.35  5.35  3.65  3.65  3.65 13 13
53.5  53.5  36.5  36.5 – 53.5  36.5 5 5
44. What is the simplified value of : (a) (b)
SSC CPO 25/11/2020 (Shift-1) 8 7
(a) 0.9 (b) 90 20 20
(c) 0.09 (d) 9 1 1 3 3 1 3 5 (c) (d)
7  2 of 1 –    1  – 29 23
39. The value of/ 3 2 5 8 7 4  24
50. The value of is:/
SSC CPO 09/12/2019 (Shift-01)
 1
1 – 3 ÷ 6 of 2 +  4  4 of  ÷ 8 + (a) 1 (b) 2  1 3 1  1 7 9 
A

 4  5  of    5 – 7  9 ×
1 1  4 7 2  9 8 20 
(c) (d)
 1 1 24 12
4 8    11  1
 4 8   2  2 of 
45. What is (0.08% of 0.008% of 8)1/9 ? 21  2
SSC CPO 25/11/2020 (Shift-1) !2 234 2 2 . 25 3 4 3# SSC CPO 09/12/2019 (Shift-02)
69 7 9
(a) (b) – (a) (b) 5
4 4 SSC CPO 09/12/2019 (Shift-01) 4
69 7 (a) 0.2 (b) 0.08 7
(c) – (d) (c) 3 (d)
4 4 (c) 0.8 (d) 0.64 2

Aditya Ranjan (Excise Inspector) Selected Selection 100

Downloaded by Kunal Chauhan (kc6970929@gmail.com)


lOMoARcPSD|39315497

Simplification

51. What is the value of/ 55. The value of 60. The value of 8 ÷ [(9 – 5) ÷ {(4 ÷ 2 of 4)
– (8 ÷ 8 of 16) + (4 × 2 ÷ 8)}] is:/
0.74  1.23  0.13 4.669  4.669 – 9  (0.777)²
(0.37)3  (0.41)3 – 8(0.39)3 (4.669)²  (2.331)²  14(0.667)(2.331)
is (1 – k), where k = ? SSC CPO 12/12/2019 (Shift-01)
SSC CPO 11/12/2019 (Shift-01)
21 23
–1 4.669  4.669 – 9  (0.777)² (a) (b)
(a) (b) 1 8 8
(4.669)²  (2.331)²  14(0.667)(2.331)
3
(1 – k) k=? 12 32
1 (c) (d)
(c) –1 (d) 23 23
3 SSC CPO 11/12/2019 (Shift-02)
61. The value of/
52. The value of/ (a) 0.666 (b) 0.647
(c) 0.467 (d) 0.768  1 1 1 1 1 1 1
 3 – 2   of 1  of
0.6912  0.5292  3 2  4 4 4 3 3 9
56. The value of 3 1 1
of 
1 1 1
15
0.6912 – 0.5292    
10 6 3 9 3 6
SSC CPO 11/12/2019 (Shift-01) 1  3  1 1 
 4 –  3 – 2  SSC CPO 12/12/2019 (Shift-01)
(a) 1.5 (b) 0.9 3  4  6 3 

r
4 2
(c) 15 (d) 9  1 1 1   1 1 1  lies be- (a) (b)
 of        81 9

si
53. A student was asked to find the 5 5 5 5 5 5
9 27
4  3 3  9  tween:/ 8 (c) (d)
value an by
of  9   2  2   25  × 2 8
    SSC CPO 11/12/2019 (Shift-02)
2 4  3 4  62. The value of 6 – 17 – 2 72 is

n
(a) 10.2 and 10.8
 3 of 9   3  5 of 5   closest to:
  
(b) 4.2 and 4.4
ja
2
R s
His answer was . (c) 8.2 and 8.8 6 – 17 – 2 72
9
What is the difference between (d) 0.4 and 0.9 7
a th

his answer and the correct an- SSC CPO 12/12/2019 (Shift-01)
swer? 57. The value of 11  2 18 is clos-
est to: (a) 2.1 (b) 2.4
4  3 3  9 
 9:   2  2   25  ×
ty a

*9 -
    11  2 18 7 (c) 2.7 (d) 1.7
2 4  3 4 
di M

 3 of 9   3  5 of 5   SSC CPO 11/12/2019 (Shift-02) 63. If 0.00576  y  2.4 , then y is


  
equal to:
2 (a) 4.8 (b) 4.4
- ;
9
- ;< (c) 3.8 (d) 4.1 0.00576  y  2.4 y
; ;< ’ ;< = ’ &
58. The value of 9 × [(9 – 4) ÷ {(8 ÷ 8 of
SSC CPO 12/12/2019 (Shift-02)
4) + (4 ÷ 4 of 2)}] is:/
SSC CPO 11/12/2019 (Shift-01) (a) 3600 (b) 1200
SSC CPO 11/12/2019 (Shift-02) (c) 1000 (d) 2400
46 47
(a) (b) (a) 20 (b) 60 64. The value of/
243 243
(2.8)³  (2.2)³
53 59 15 15
(c) (d) (c) (d) (28)² – 28  22  484
243 243 4 2
A

SSC CPO 12/12/2019 (Shift-02)


54. The value of/ (0.13)²  (0.21)² (a) 0.02 (b) 0.05
59. The value of (0.39)²  81(0.07)²
5 – [2  3(2 – 2  2  5) – 5]  5 (c) 0.01 (d) 0.5
4  4  4 of (4  4  4 of 4) (2.4)4  3  (11.52)  9 65. The value of/

SSC CPO 11/12/2019 (Shift-01) (2.4)6  6(2.4)4  3  (17.28) lies be- 7
17.35  of 55 – 7
tween:/ = ,- 4
3 3 3 2 1
(42  6  8.35) – of  –   [291  (80  8)]
(a) 4 (b) 7 SSC CPO 12/12/2019 (Shift-01) 7 3 5
80 5
SSC CPO 12/12/2019 (Shift-02)
(a) 0.7 and 0.8 (b) 0.4 and 0.5
3 2 (a) 2 (b) 4
(c) 3 (d) (c) 0.6 and 0.7 (d) 0.5 and 0.6
16 5 (c) 1 (d) 3

Aditya Ranjan (Excise Inspector) Selected Selection 101

Downloaded by Kunal Chauhan (kc6970929@gmail.com)


lOMoARcPSD|39315497

Simplification

66. The value of/ 72. The value of


* > - :
2  1 1 21 1   4 7 3
56  of 27 – 8  1 1 1 1 2 2 1 1  1  –    2  1 of 
3  9 20 38 3   9 15 5 
 2  2 –   2  5  1 of 4
3 3 2 6 3 3 3 4 1 1 1 1 1 1
15 – of (29 – 14) of  –  of
1 1 1 1 3 2 5 5 125 25 5 5
8 3  4 of 5  5   2
5 2 3 3 4 3 lies between/ =
SSC CPO 12/12/2019 (Shift-02) ,- -
(a) 12 (b) 10
SSC CPO 13/12/2019 (Shift-02)
(c) 15 (d) 11 6
; ; <7 - ;< ’ ; 0.15 and 0.2
(a) (b) 0.2 and 0.25
67. The value of/
(c) 0.1 and 0.15 (d) 0.25 and 0.3
;< = ’
(0.321)³  (0.456)³ – (0.777)³ 73. Which one among the following is
SSC CPO 13/12/2019 (Shift-01)
0.9  (0.107)(0.76)(0.777) the smallest?
9 5
SSC CPO 13/12/2019 (Shift-01) (a) (b) ?
4 14
(a) 60 (b) – 6 SSC CPO 09/12/2019 (Shift-01)
(c) – 3 (d) 30 11 6
(c) (d) (a) 101 – 99

r
68. The value of/ = 6 49 49
(b) 201 – 199
5 – 2  4[5 – (3 – 4)]  5 4  2 of 4 70. The value of/

si
4  4  8 of 2(8 – 5) 2  3 – 8  2 of 8 (c) 301 – 299
4 – 3  2  (4 – 2) – 3  4  3  2  4

(a)
9
an by
SSC CPO 13/12/2019 (Shift-01)

(b)
9
4  3  4  (2 – 4)  4  3  4 of 3 (d) 401 – 399

n
8 4 SSC CPO 13/12/2019 (Shift-02) 74. The value of 9 – 2 11 – 6 2 is
15 89 32
(c) (d) (a) (b) – 32 closest to:/ 7
32 ja 4 7
SSC CPO 13/12/2019 (Shift-01)
R s
69. A student was asked to find the
value of –32 (a) 2.7 (b) 2.9
(c) (d) 32
a th

7 (c) 2.4 (d) 2.1


 1 1 1 1 2 2 1
 2  2 –   2  5  1 of 4 71. The value of/
3 2 6 3 3 3 4 1
1 1 1 1 3 2 75. The value of is clos-
3  4 of 5  5   2 (0.013)³  (0.007)(0.000049) 17  12 2
5 2 3 3 4 3
ty a

(0.007)²  0.013(0.013 – 0.007) est to/ 7


6
His answer was . What is the SSC CPO 13/12/2019 (Shift-02)
SSC CPO 13/12/2019 (Shift-02)
di M

7
(a) 0.07 (b) 0.02 (a) 1.2 (b) 0.14
difference between the correct
(c) 1.4 (d) 0.17
answer and his answer? (c) 0.06 (d) 0.04

ANSWER KEY
1.(d) 2.(c) 3.(c) 4.(b) 5.(b) 6.(c) 7.(c) 8.(d) 9.(a) 10.(a)

11.(a) 12.(a) 13.(c) 14.(c) 15.(c) 16.(c) 17.(c) 18.(b) 19.(c) 20.(a)

21.(b) 22.(d) 23.(d) 24.(d) 25.(b) 26.(d) 27.(a) 28.(c) 29.(a) 30.(c)
A

31.(a) 32.(a) 33.(d) 34.(c) 35.(d) 36.(d) 37.(d) 38.(c) 39.(a) 40.(a)

41.(b) 42.(b) 43.(b) 44.(a) 45.(a) 46.(c) 47.(d) 48.(a) 49.(c) 50.(b)

51.(a) 52.(c) 53.(a) 54.(a) 55.(a) 56.(b) 57.(b) 58.(b) 59.(c) 60.(b)

61.(a) 62.(b) 63.(c) 64.(b) 65.(c) 66.(d) 66.(b) 68.(a) 69.(d) 70.(c)

71.(b) 72.(a) 73.(d) 74.(c) 75.(d)

Aditya Ranjan (Excise Inspector) Selected Selection 102

Downloaded by Kunal Chauhan (kc6970929@gmail.com)


lOMoARcPSD|39315497

Simplification

SOLUTIONS
1. (d) 5. (d) 10. (a)
Using the simplification rule Rules of Surds and Indices (0.2)3 × 400  2000 of (0.2)2
BODMAS 1. am × an = a(m+n) = 0.008 × 400  2000 of 0.04
2. am  an = a(m–n) = 0.008 × 400  80
 20  5 of 3  6   –2 3. if am = an then m = n = 0.008 × 5
3
   1 5
 814   8  24 of 3   4 –   {(5 × 5 × 5 × 5 × 5 × 5)5 × (5 × 5
   32  1
 –10  5 
× 5 × 5 × 5)8}  (5 × 5) = (625)a = 0.040 =
25
 {(5)6×5 × (5)5×8}  (5)2 = (5 × 5 ×
5 ×5)a 11. (a)
–2
3  20  15  6    1  5 5  {(5)30 × (5)40}  (5)2 = (5)4a (73 – 72 + 6) × (63 – 23 – 32 + 1) 
 3
4
4  – 
(212 + 202 – 41)
 8  72  4 – 10  5   2   (5)30+40  (5)2 = (5)4a
 (5)70–2 = (5)4a = (343 – 49 + 6) × (216 – 8 – 9 +

r
1)  (441 + 400 – 41)
 20   8     (5)68 = (5)4a
  3   
3
 6      4 – 2 – 4 = (349 – 49) × (217 – 17)  (841 – 41)

si
  15   72     68 = 4a
 a = 17 300  200
 4  = = 75
 9 
an by
 27  8  – 2 – 4 6. (c)
Solve using the simplification
rule BODMAS
12. (a)
800

n
4 4 3 76  19 
= 27 + 8 – 2 – 4 +  29 123 × (162 – 142 – 40)  2 – 94 456 –  76  152   of  
9 9 4 18  72  24 
= 1728 × (256 – 196 – 40)  2
2. (c) ja – 6561
R s
19  19 
Given, = 1728 × 20  2 – 6561 = 456 –  76  225    
6  72  24 
= 17280 – 6561
a th

3
–2744  3 –216 = 10719 19  72  24 
 = 456 – 301  
64 7. (c) 
3 6  19 
729 Solve using the simplification
= 456 – 301 + 288
ty a

rule BODMAS
3 = 744 – 301
–14  –14  –14 72 – 42  6 of 7 × 4 of 3  4 × 3
–5 = 443
3 –6  –6  –6
di M

 13. (c)
= 72 – 42  42 × 12  4 × 3 – 5
444
3
= 72 – 1 × 3 × 3 – 5 5 3 4 6
999 – 1  3
–14  –6
= 72 – 9 – 5  2 7 5 7
 = 72 – 14 = 58 3 2 1 1
4  1  3 1
8. (d) 2 5 2 4
9
Given, 5 3 9 27 5 3 9 7
–14  –6  9 –   –  

4 1  x    x 3
– 2x 2
  4 3 5

 2   x  25  1
3 7 7 5
2 7 5 27
= 2 7 5 7 = 3 7 2 5
 189      
1  2 5 2 4 2 5 7 4
3. (c)   x  x – 2x   2  x 
2

x
72  8 × [{(57 – 49)  2} + {(85 –   1 5 1

60)  5} – {(28 – 19)  3}] 25 2 5 23 1 3
= =  =1
x2 – 2x + 2x = 25 3 1 10 2 20
A

= 9 × [{8  2} + {25  5} – {9  3}] 


 x2 = 25 2 2
= 9 × [4 + 5 – 3] 14. (c)
 x2 =  25
= 9 × [6] = 54  2 
 x 2 =  5 1  7  16  8  2   6  2
2
 6  
4. (b) 36 
9. (a)
1801 × 1801 = (1801)2  2
= 8  1   6  4  6   
1
 243  5 6
= (1800 + 1)2  0.00243  
0.2
 
= 18002 + 2 × 1800 × 1 + 12  100000 
 2
1 = 9  30  
= 3240000 + 3600 + 1  3 
5
5 3  6
   = 0.3
= 3243601  10  10 = 9 + 10 = 19

Aditya Ranjan (Excise Inspector) Selected Selection 103

Downloaded by Kunal Chauhan (kc6970929@gmail.com)


lOMoARcPSD|39315497

Simplification
15. (c)
98 7 63 2 36
=  of   25  4  4 of 
 2  1  1 1   1 1 15 9 10 5 5  
1  3   –      4  1  36  15 of 2 of  29  3   
 7  2  2 7   5 2 98 49 2 36 =  
 
=   
15 10 5 5
  45  15   
 9  7  1 1    21 3 
=  7   2   2 – 7     5  2 
       98 10 2 5 100  4 of 
=   
15 49 5 36 = 36  15 of 2 of 29  3  3
 9  7  7 – 2    63   
=  7   2   14     10  4 1
       = 
3 18 = 36  15 of 2 of 100  4 of30
 9  7 5   63 
=         24  1
 7  2 14   10  = = 36  15 of 2 of 100  120
18
 9  7 14   63 
=         25 5
 7  2 5   10  = = 36  15 of 2 of
18 6
126   63  18. (b)
=  =2 36 6 1
 10   10 

r
187  17  a – 3  3 = = =1
1 25 5 5
16. (c) 8 2
– 9  7  a2 

si
22. (d)
 5  3  5 3  
 –  –  –   of 8.8 – 1.2 
11a – 9
1
25  5 of 12 × 2 – 9  27 of 3 × 5

1
6
an by
 8  8  8 8  
1
6
3 3
4  2.5  2  of 60   – 
4 8
1  a2
 11a – 9 = 1 + a2
– 14  28 × 4
= 25  60 × 2 – 9  81 × 5 – 14 

n
 a2 – 11a + 10 = 0 28 × 4
 5  3 2   a2 – 10a – a + 10 = 0
 8 –  8 – 8  of 8.8 – 1.2 25 5 14 5 5
  ja  a(a – 10) – 1(a – 10) = 0 = 2 – – 4 = – –2
R s
60 9 28 6 9
= 25 6 – 3  (a – 1)(a – 10) = 0
 2.5  2  10   
6  8   a = 1 or 10 15 – 10 – 36 –31
a th

Hence, Answer (b) = =


18 18
5 1 
 8 – 8  of 8.8 – 1.2 SMART APPROACH:- 23. (d)
= 25 10 1 3 Put the value from the option
16   28 of 7  22  
ty a

 2   directly. 
6 25 10  8   2 3  
25  8  2 –   2 1  
19. (c) 
2
 – 18  12 of  
di M

1    8  
of 8.8 – 1.2
2 x 24
1   16   28 of 7  4  – 
= 1 3 529 23
   
625  8  8 – 

3 8 =  1  
Squaring both sides   324  144 of  
  8  
4.4 – 1.2 x 576
 1  = [625 + 1 – {16 + 49 – (324  18)}]
= 1   3  529 529
3 8 = [626 – {16 + 49 – 18}]
529  x 576
4.4 – 1.2   = 626 – 47 = 579
529 529
= 89  x = 576 – 529
24. (d)
24
 x = 47 = (1018)2 – 1019 × 1017 + 1015 ×
3.2  24 44 20. (a) 1012 – 1016 × 1011
= = 4
A

17 85 12 – 8  2 – {16 of (–2) + (3 × 5 – 4)} = 1036324 – 1036323 + 1027180


17. (c) = 12 – 4 – { – 32 + 11} – 1027176 = 1 + 4 = 5
8 7 = 12 – 4 + 21 'OR'
 1 1 2 1
6  of 1 5  7 = 33 – 4
15 9  10 5 5 5 Using the concept of unit Digit
= 29
98 7  11 26  2 36 21. (b) = (8)2 – 9 × 7 + 5 × 2 – 6 × 1
=  of    
15 9  10 5  5 5 =4–3+0–6
25  4  4 of 
  =4–9 =–5
98 7  11  52  2 36 36  15 of 2 of  29 –  8 – 11 
=  of    
  9  5  5 of 3  
15 9  10  5 5      Required Answer = 10 – 5 = 5

Aditya Ranjan (Excise Inspector) Selected Selection 104

Downloaded by Kunal Chauhan (kc6970929@gmail.com)


lOMoARcPSD|39315497

Simplification
25. (b) 29. (a)
a  b 
2
Given factions is: 0.5 a  b 0.5
0.325  0.325  0.175  0.175  25  0.00455 0.5
 2 2
  
15 5 21 3 - 1.5 =? a –b 1.5 a – b 1.5
, ,  5  0.0065  3.25 – 7  0.175  0.025
7 11 77 11 0.5 0.5 50 5
Descending order of the fractions: 0.325  0.325  0.175     
.15 1.5 15 15
15 5 3 0.175  0.175  0.65 0.5
   – 50 11
7 11 11 0.325  0.325 – 0.175  0.175 1.5  
15 3
26. (d)
0.325  0.325  0.175  33. (d)
6.35  6.35  6.35  3.65  3.65  3.65
? 0.175  0.175  0.325  2 0.5  1  1 3 1
63.5  63.5  36.5  36.5 – 63.5  36.5  –  5  2 of  +  5  of 
0.325  0.325 – 0.175  0.175 1.5  2  4 7 2
a 3 +b 3
 =  a+b   a = 0.325, b = 0.175  1 7 9  11
a +b 2 –ab
2
÷ 5 – 7  9  × =?
a = 6.35 0.325  0.175 0.5  9 8 20  21
 –
b = 3.65 0.325 – 0.175 1.5
 21 3   46 63 189  11
5   –  
1 1000
0.5 0.5  4 14   9 8 20  21
 635  365   0.1  –
.15 1.5

r
10000 10000
27. (a) 50 5  49   46 63 20  11
 – 5  –  

si
3 15 15  2   9 8 189  21
40 – of32
4 ? 45 49  46 5  11
37 –
3
4
an by
of  34 – 6

30. (c)
15
3 5  – 
2  9 6  21

n
49  184 – 30  11
3 1 1 3 3 1 3 5  
40 – ×32 3  2 of 1     1   ? 2  36  21
 4 3 2 5 8 7 4
3 ja 49  36 11
R s
37 –  28 5 
4 10 5 8  3  7  2  154 21
      
3 2 5 8 7 4
a th

40 – 24 5 + 3 = 8
 10 5 34. (c)
37 – 21  
34 8 4.35  4.35  4.35  3.25  3.25  3.25
?
16 43.5  43.5  32.5  32.5 – 43.5  32.5
 =1 5 5
ty a

16  
6 8 1  4353  3253 
28. (c)   
10000  435  3252 – 435  325 
2
35
di M


 1 3 1  1 7 9  24 1  760 
5  of    5 – 7  9  ×   0.076
4 7 2 9 8 20  31. (a) 10000  1 
1 1 3 3 1 3 35. (d)
11  1 2  2 of 1     1   ?
–  5  2of  = ? 3 2 5 8 7 4
21  2
2 3 4 4 1 8 3 3 1
 of –  1  –   ?
 21 3 1  7 5 8 3 1 7 3 10 9 5 9 15 4 4 2
          
÷ 3 2 5 8 7 4 2  90 4 10 15 3 3
 4 7 2  –   – 
3  12 5 9 8 4 2
 46 63 189  11 7 5
 –   × – (5 ÷ 1)    5 3 3
 9 8 20  21 34 8 5– – 
3 4 2
 21 14  14  15 29
   13 29 78 – 29 49
  11 24 24  –  
 4 3 
A

  –5 32. (a) 2 12 12 12
 46 63 20  21
 –   36. (d)
 9 8 189  0.325  0.325  0.175 
0.175  25  0.00455 0.5 –7  5  1  2 – 4   4 of 2  4   4  4 of 2
49  ?
11 5  0.0065  3.25 1.5
 2  –5 1
184 – 30 21 –7  0.175  0.025 –7 ÷ [5 + 1 ÷ 2 – {(4 + 2) + ( )}]
2
36 0.325  0.325  0.175 
 13 
49  36  11 0.175  2  0.325  0.175 0.5 –7 ÷ 5  1  2 –
 –5  
0.325  0.325 – 0.175  0.175 1.5  2 
2  154  21
–7 ÷ – 1
=3–5=–2  a = 0.325, b = 0.175
 7

Aditya Ranjan (Excise Inspector) Selected Selection 105

Downloaded by Kunal Chauhan (kc6970929@gmail.com)


lOMoARcPSD|39315497

Simplification

37. (d) 43. (b) 48. (a)


8 15 2 34 Given that,
3   –
40  of 32
4 13 16 3 27  64 0.325  0.325  0.175  0.175 –
?
3
37  of  34 – 6  8 5 7 25  0.00455
4   – 5  0.0065  3.25 – 7  0.175  0.025
13 8 91
40  24 64 Let,
  5 7 4
37  21 58 = – = A = 0.325, B = 0.175
13 91 13
Then,
32 3 44. (a)
 1
29 29 (A – B)² A–B
1 1 3 3 1 3 5
38. (c) = 7  2 of 1 –    1  – A² – B² = A  B
3 2 5 8 7 4  24
 a = 535, b = 365
22 5 8  3 1 7  5 0.325 – 0.175 0.15
1   –   –   0.3
= 0.325  0.175 0.5
 535  365 3 2 5  8 7 4  24
10000 This value lies between 0.25 to 0.35
22 40  3 1  5 49. (c)
900 =  –  –
  0.09 3 10  8 4  24 5 ÷ [5 + 8 – {4 + (4 of 2 ÷ 4) – (2 ÷ 4
10000

r
of 2)}]
39. (a) 11 5 5
 – –

si
6 8 24   1 
 1 = 5 ÷ 5  8 – 4  2 –  
1– 3 ÷ 6 of 2 +  4  4 of  ÷ 8 + 44 – 15 – 5 24   4 
 4
an by   1
24 24
 1 1  23 
4 8÷  × =? 45. (a) = 5 ÷ 5  8 –
 4 8  4 

n
1
3 1 (0.08% of 0.008% of 8)9 4 20
1–   4   8  128  = 5 
12 ja 8
8 8
29 29
R s
1 4 =  8 50. (b)
 1 –   16 10000 100000
4 8
a th

 1 3 1  1 7 9 
 5  of    5 – 7  9
1
4 1  2 9  9 
 17  –  4 7 2  9 8 20 
8 4 =   
 10   11  1
69    2  2of 
ty a

 2 21  2
4 = = 0.2
40. (a) 10  21 3   46 63 189  11
di M

46. (c) =     –    2
 a = 427, b = 325  4 14   9 8 20  21
100 (427 + 325)
 1  1  
2 2
1   21 14   46 63 20  11
75200  x   – x –   =     –   2
41. (b)  8  y  y    4 3   9 8 189  21
7 ÷ [5 + 1 ÷ 2 – {4 + (4 of 2 ÷ 4) + 49 18 11
(5 ÷ 5 of 2)}] = ?  1  1 2x 1 2x      2 = 5
   x2  2  – x2 – 2   2 77 21
8
  y y y y  
1 51. (a)
7 ÷ [5 + 1 ÷ 2 – {4 + 2 + }]
2 1 4x x (a3 + b3 + c3 = 3abc)
  
 1 13  8 y 2y 0.74 1.23  0.13
7 ÷ 5  –
 2 2  (0.37)³  (0.41)3 – 8(0.39)3
47. (d)
11 13  Given that,
A

7 ÷  – 2  (0.37)  3(0.41)  0.13


2 2  1 = (0.37)3  (0.41)3 – (2  0.39)3
–7 7–4 3
42. (b) 2  (0.37)  3(0.41)  0.13
(320 + 342 + 530 + 915) ÷ (20 + 22 1 = (0.37)3  (0.41)3 – (0.78)3
– x + 18) = 43 
7 – 2 12
2107 (0.37) (0.41) (0.78)
 = 43 = –3(0.37)(0.41)(0.78)
60 – x 1 2 3
 
 2107 = 2580 – 43x 2– 3 2 3
–1
 473 = 43x =
 2+ 3 = 3.7 3
 x = 11

Aditya Ranjan (Excise Inspector) Selected Selection 106

Downloaded by Kunal Chauhan (kc6970929@gmail.com)


lOMoARcPSD|39315497

Simplification

52. (c) 56. (b)   8  1  16 



=8÷ 

4 
0.6912  0.5292 1  3  1 1    16
  
 4 – 
3 –2 
0.6912 – 0.5292 3  4  6 3    16  23 23
= 8 ÷ 4  =8× 
(divided by 12 in both numerator 1 1 1  1 1 1  
 23  64 8

 of        61. (a)
and denominator) 5 5 5  5 5 5 
 1 1 1 1
3 – 2 
   of 1
0.6912 0.5292 1 47  3 2 4 4 4
  = of
12 12 3 1 1 15
= 3 12  4.25  
= 0.6912 0.5242 10 6 3
– 1
12 12 It lies between = (4.2 and 4.4) 1 1 1 10 5 1 5
 of  – 
  of
57. (b) 3 3 9  3 2 4 4
576  441 24  21 45 ÷ 1 1 1 = of
  3 1
=   
576 – 441 24 – 21 3 11  2 18 9 3 6 10 18
= 15 = 1 1 5 5
9 2  
53. (a) 4 6 16
= 3 + 1.4 = 4.4 3 27
÷ = 64 of
4  3 3  9  2 4  3 4 
58. (b) 15 1 1 6

r
 9   2  2   25    3 of 9   3  5 of 5    
    9 × [(9 – 4) ÷ {(8 ÷ 8 of 4) + (4 ÷ 4 of 9 3 1 180
2)}]

si
4 9  8 36  9
=   1    4 1 8 180 4 9 9
9
 25  
27 25    1 
 1   
15 2
=  of  
an by =9× 

5  
8  
 
4    3 64 15 1 2
4 9   8 25   
 32   8 
 9
=      
9 25  
 27 36  8 180 4 2 4

n
 1 1 
  =    =
3 64 15 81 81
4 50 8 =9× 

5    
    4 2 
 62. (b)
25 27  9 243 ja
R s
Required difference  4 6 – 17 – 2 72
= 9 × 5  = 60
2 8 
 3
a th

 – =
9 243 6 – 17 – 2  6 2
59. (c)
54 – 8 46 (0.13)2  (0.21)2 = 6 – 17 – 12 1.4
  = ÷
243 243 (0.39)2  81(0.07)2
ty a

54. (a) = 6 – 17 – 16.8


4
(2.4)  3  (11.52)  9
5 – [2  3(2 – 2  2  5) – 5]  5 = 6 – 0.2
di M

(2.4)6  6(2.4)4  3  (17.28)


4  4  4of (4  4  4of 4)
= 6 – 0.14  5.76 = 2.4
(0.13)2  (0.21)2
19 =  63. (c)
5  19  17  323  4 3 9 (0.13)2  9(0.07)2 
= 16 5 16 80 80 0.00576  y = 2.4
17 (2.4)4  3  (11.52)  9 Squaring both sides
55. (a) 2.42 (2.4)4  6(2.4)2  3  3
 
2

2
 0.00576  y = (2.4)²
4.669  4.669 – 9  (0.777) 2
(0.13)  (0.21) 2

=   0.00576×y = 5.76
(4.669)2  (2.331)2  14(0.667)(2.331) 9 (0.13)2  (0.21)2 
=1–K y = 1000
(2.4)4  3  (11.52)  9 64. (b)
(4.669)2 – (2331)2
2.42 (2.4)4  3  11.52  9 
A

(4.669)2  (2.331)2  2(4.669)(2.331) (2.8)3  (2.2)3


=1–K (28)2 – 28  22  484
1 1 5.76
=  = = 0.64
2 2 9 5.76 9 We know,
(4.669) – (2331)
= =1–K Value of the expression lies a³ + b³ = (a + b) (a² + b² – ab)
(4.669  2331)2 between 0.6 and 0.7. (28)3  (22)3 10 –3
(4669  2331)(4669 – 2331) 2338 60. (b) = (28)2 – 28  22  222
= (4669  2331)(4669  2331)  7000 8 ÷ [(9 – 5) ÷ {(4 ÷ 2 of 4) – (8 ÷ 8 of
16) + (4 × 2 ÷ 8)}]
= 0.334 (28  22)  2 2
(28) – 28  22  (22)  10
–3

 1 1 1
  =
= 0.334 = 1 – K (28)2 – 28  22  (22)2
=8÷ 

4 –  
= K = 1 – 0.334 = 0.666  
 2 16 1
 = 50 × 10–3 = 0.05

Aditya Ranjan (Excise Inspector) Selected Selection 107

Downloaded by Kunal Chauhan (kc6970929@gmail.com)


lOMoARcPSD|39315497

Simplification
65. (c) 69. (d)
10 21 38 1   22 22 3 
Solving using BODMAS rule- 
   – 
   
 1 1 1 1 2 2 1 9 20 21 3   9 15 5 
7  2  2    2  5  1 of 4 = 
17.35  of 55 – 7  3 2 6 3 3 3 4 1 125 1 25
=  – 
5 1 1 1 1 3 2 25 1 25 1
3 2 1 3  4 of5  5   2
(42  68.35) – of 
 –  [291(80  8)] 5 2 3 3 4 3
7 3 5 19 1   22 25 

 –    

 7 5 1  7 17 5 17 9 3   9 22 
7       of = 
17.35 
 55 – 7 3 2 6 3 3 3 4 5 –1
5 = 16 9 16 16 3 8
= 3 7  of   
 
(7  8.35) – of    29.1 5 2 3 3 4 3 57 – 9 9 48 9
7 15   
= 27 25  27 25
14 7 17 85 4 4
17.35  77 – 7   
3 3 3 12
= 58.45 – 1  29.1 = 16 16 3 3 16 4
5  24    =  = 0.16
5 3 4 8 25  4 25
87.35 0.16 lies between (0.15 and 0.2)
= (58.45 – 0.2  29.1) 17 12 8
2 
3 85 5 73. (d)

r
= =
87.35 16 1 3 2 3
1    201 – 199 = 0.10
= 5 24 2 15 2

si
87.35
66. (d)  8   30  48 101 – 99 = 0.07

2
56 
an by
Solving using BODMAS rule-

of 27 – 8
=    =
 5   49  49

48 6 6
301 – 299 = 0.057

n
3 Required difference = – 
3 49 7 49 401 – 399 = 0.050
15 – of (29 – 14)
5 70. (c)
ja The smallest value is
R s
2 4 – 3  2  (4 – 2) – 3  4  3  2  4 401 – 399 = 0.050
56   27 – 8 56  18 – 8 4  3  4  (2 – 4)  4  3  4of 3
a th

= 3 = 74. (c)
3 15 – 9
15 – 15
5 3 3
4– 2 – 3  4  4 9 – 2 11 – 6 2
2 2
66 =
3 1
ty a

= = 11 4   (–2)  4  3 
6 4 12
 2
2
67. (b) = 9 – 2 32  – 23 2
di M

We know, If a + b+ c = 0, then 4 – 3 – 36 4 8 –32


=  4 
1 –7 7
a3 + b3 + c3 = 3abc
Here, 0.321 + 0.456 – 0.777 = 0
4 –6
4 = 
9–2 3– 2 
71. (b)
(0.321)3  (0.456)3 – (0.777)3 = 9–62 2
 0.9  (0.107)(0.76)(0.777) (0.013)3  (0.007)(0.000049)
(0.007)2  0.013(0.013 – 0.007)
= 32 2
–3  0.321  0.456  0.777
= 0.9  (0.107)(0.76)(0.777) 3
(13)  (7) 3

= (72  132 – 7 13) 1000


 2
2
=–6 =  12  2 2
68. (a) (a3 + b3) = (a + b) (a2 + b2 – ab)
5 – 2  4  [5 – (3 – 4)]  5  4  2of 4 = 2 1
A

(7  13)(72  132 – 7 13)


4  4  8 of 2  (8 – 5)  2  3 – 8  2of 8 = (7 2  132 – 7 13) 1000 = 1.4 + 1 = 2.4
75. (d)
1 5
6
5– 20
2 2 = = 0.02 1 1 1
1000 = 
= 1 1 1 9 8
4  4 32 – 8 17  12 2 17  2 72
16 3 16 72. (a)
 1 1 21 1  4 7 3
5 3 5 9 1 1
  –   2  1 of 1 9 – 8 3–2 2
–   9 20 38 3  9 15 5 =  
1 1 2  2 9 9 8 9– 8 1
= 1 1 4 8 1 1 1 1 1 1
4 2– of  –  of
4 2 1 5 5 125 25 5 5 = 3 – 2 × 1.414 = 0.17

Aditya Ranjan (Excise Inspector) Selected Selection 108

Downloaded by Kunal Chauhan (kc6970929@gmail.com)


lOMoARcPSD|39315497

Algebra

ALGEBRA
17

1 9. If x + y + z = 8 and x2 + y2 + z2 =
x – y   y – z  z – x 
3 3 3
1. If a –  10, then the value 20 then the value of x3 + y3 + z3
a –5
x – y2    y2 – z2    z2 – x 2 
3 3
2 – 3xyz is.
1
of  a – 5  –
3
is: x+y+z=8 x 2 + y 2 + z2 =
a – 5
3
xyz
20 x3 + y3 + z3 – 3xyz
1 SSC CPO 09/11/2022 (Shift-02)
a–
a –5
 10 (a– 5)³
(a) 0
1 SSC CPO 09/11/2022 (Shift-03)
– 1

r
a – 53 (b) x  y  z (a) 16 (b) 10
SSC CPO 09/11/2022 (Shift-01) (c) 15 (d) –16

si
1
(a) 140 (b) 70 (c)  x  y  y  z  z  x  10. The simplified for of (7x + 4y)2 +
(c) 100 an by (d) 120 (7x – 4y)2 is:
(d) 1
1 6. If a + b + c = 10; a2 + b2 + c2 = (7x + 4y)2 + (7x – 4y)2
If x  4  1154, where x > 0,
4
2.

n
x 38, what is the value of (a – b)2
then what is the value of + (b – c)2 + (c – a)2? SSC CPO 10/11/2022 (Shift-01)
1 a + b + c = 10; a2 + b2 + c2 =
x3  3 ?
x
ja (a) 98x2 + 32y2
R s
38 (a – b)2 + (b – c)2 + (c – a)2
(b) 98x2 + 32y2 – 5xy
1
x 4  4  1154 x > 0 (c) 32x2 + 98y2
a th

x SSC CPO 09/11/2022 (Shift-02)


1 (d) 98x2 – 32y2
x3  3 (a) 15 (b) 12
x 11. What are the values of x and y,
(c) 14 (d) 13 respectively form the following
SSC CPO 09/11/2022 (Shift-01)
ty a

(a) 205 (b) 214 7. The value of equations?


! x " #
(c) 185 (d) 198
 m 2
 n2   m – n  –  m – n 
3

di M

3. If x4 + x –4 = 7, x > 0, then what y


is:
1 2
 m n – mn 
2 2
6x + 7y = 5xy
is the value of x  2 – 2?
x 10y – 4x = 4xy
x 4 + x –4 = 7, x > 0  m 2
 n2   m – n  –  m – n 
3
 SSC CPO 10/11/2022 (Shift-01)
1
x2  2 – 2  m n – mn 
2 2
(a) 3 and 4 (b) 4 and 5
x (c) 2 and 4 (d) 2 and 5
SSC CPO 09/11/2022 (Shift-01)
12. If x + y = 10, 2xy = 48 and x > y,
(a) 1 (b) 0 SSC CPO 09/11/2022 (Shift-03)
then find 2x – y.
(c) 5 (d) 3 (a) m + n (b) m – n
x + y = 10, 2xy = 48 x>y
4. If a + b + c = 10 and a2 + b2 + c2 (c) 2 (d) m/n
x–y %
= 48, then the value of ab + bc +  3  3
ca is _______. 8.
2
If 16y – k   4y    4y –  , SSC CPO 10/11/2022 (Shift-01)
A

 2  2 
a + b + c = 10 a2 + b2 + c2 = (a) 6 (b) 8
then the value of k is:
48 ab + bc + ca (c) 4 (d) 3
 3  3 13. If a + b + 1 = 2a, then the value
2 2
SSC CPO 09/11/2022 (Shift-02) 16y 2 – k   4y    4y – 
 2 2 of a4 + b7 is:
(a) 25 (b) 26
k
(c) 24 (d) 18 a2 + b2 + 1 = 2a a4 + b7
SSC CPO 09/11/2022 (Shift-03)
5. The value of
9 11
x – y  y – z  z – x  SSC CPO 10/11/2022 (Shift-02)
3 3 3
(a) (b)
4 4
x 2
– y2    y2 – z   z
2 3 2
–x 
2 3
6 7
(a) 1 (b) 0
(c) (d) (c) 2 (d) 4
where x  y  z , is: 4 4

Aditya Ranjan (Excise Inspector) Selected Selection 109

Downloaded by Kunal Chauhan (kc6970929@gmail.com)


lOMoARcPSD|39315497

Algebra

1 19. If a + b = 10 and ab = 9, then the 1


14. If x   2, then the value of value of a – b is: s  20
x s8
a + b = 10 ab = 9 a 1
s – 8
3
1 
x  4 .
4 –b s  8 
3

x SSC CPO 11/11/2022 (Shift-01)


1 1 (a) 7 (b) 5 SSC CPO 11/11/2022 (Shift-03)
x  2 x4
x x4 (c) 8 (d) 6 (a) 1324 (b) 1764
20. If a, b and c are positive integers (c) 1864 (d) 1944
such that a2 + b2 = 82 and b2 + 1
SSC CPO 10/11/2022 (Shift-02) 2
c2 = 65, then the value of 2a + 26. If x  2  9 8 , then the value of
(a) 0 (b) 2 x
7b – 3c is
(c) –1 (d) 1 1
a, b c ( ) ’! * + x,  is:
15. If a, b, c are all non-zero and a x
a2 + b2 = 82 b2 + c2 = 65
+ b + c = 0, find the value of 1 1
2a + 7b – 3c x 2  2  98 x
a 2 b2 c2 x x
  SSC CPO 11/11/2022 (Shift-01)
bc ca ab
(a) 2 (b) 5 SSC CPO 11/11/2022 (Shift-03)
a, b, c &# ’ a + b (c) 49 (d) 1 (a) 10 (b) 8

r
a 2 b2 c2 21. If x(x – 5) = –1, then the value (c) 7 (d) 9
 

si
+ c = 0 of x3(x3 – 110)? 27. If the equation k(21x2 + 24) +rx
bc ca ab
x(x – 5) = –1 x3(x3 – 110) + (14x2 – 9) = 0;
an by k(7x2 + 8) + px + (2x2 – 3) = 0; have
SSC CPO 10/11/2022 (Shift-02) both roots common, then the
SSC CPO 11/11/2022 (Shift-01)
(a) 3 (b) 4 p

n
(a) 0 (b) –1 value of is
1 r
(c) 1 (d) (c) 1 (d) 2
2 k(21x2 + 24) + rx + (14x2 – 9)
ja 22. If r = 55, then the value of r(r2 +
R s
x8 1 3r + 3) is = 0; k(7x2 + 8) + px + (2x2 – 3) =
16. If  14 , then the value of
x4 r = 55 r(r2 + 3r + 3) p
a th

0; ’ -.r /0
12
x 1
is SSC CPO 11/11/2022 (Shift-01)
x6
(a) 105000 (b) 176615 SSC CPO 11/11/2022 (Shift-03)
x8 1 x 12  1
ty a

 14 (c) 175616 (d) 175615 1 2


x4 x6 (a) (b)
1 1 3 5
23. If a   p2 , then find a 2  2 4 7
di M

a a (c) (d)
SSC CPO 10/11/2022 (Shift-03) 3 5
(a) 16 (b) 14 1 1 28. If a + b + c + 216 =
2 2 2
12 (a + b –
a  p2 a2 
(c) 52 (d) 64 a a2 2c), then ab – bc + ca is :
17. Simplify: (x + y)3 – (x – y)3 – 6y(x 2 a² + b² + c² + 216 = 12(a + b – 2c)
– y2) SSC CPO 11/11/2022 (Shift-02)
(a) p2 – 2 (b) P2 +2 ab – bc + ca
(x + y)3 – (x – y)3 – 6y(x2 – y2)
(c) P – 2
4
(d) P4 + 2 SSC CPO 23/11/2020 (Shift-1)
24. If a = 9.6, b = 4.44 and c =5.16, (a) 6 (b) 4
SSC CPO 10/11/2022 (Shift-03)
then the value of a3 – b3 – c3 – (c) 3 (d) 8
(a) 8y3 (b) x3
(c) 8x3
(d) y3
3abc is:
a = 9.6, b = 4.44 and c = 5.16

29. If 5 5x 3 – 3 3y 3 ÷   5x – 3y 
18. If x + 6x + 1 = 0, then the value
2
=  Ax 2 + By 2 + Cxy  , then the
A

a3 – b3 – c3 – 3abc
1
of (x + 6)3 +
x  6 3
?
SSC CPO 11/11/2022 (Shift-02)

value of 3A + B – 15C is: 
x2 + 6x + 1 = 0 (x + 6)3 (a) 0 (b) –1 5 5x 3 – 3 3y 3 ÷   5x – 3y 
1 (c) 2 (d) 1 = (Ax² + By² + Cxy)
+
 x  6 3A + B – 
3
25. If s  1  20 , then the value 15C
s8
SSC CPO 10/11/2022 (Shift-03) SSC CPO 23/11/2020 (Shift-1)
1
s – 8 (a) 3 (b) 12
3
(a) 245 (b) 216 of  is:
s  8
3
(c) 186 (d) 198 (c) 8 (d) 5

Aditya Ranjan (Excise Inspector) Selected Selection 110

Downloaded by Kunal Chauhan (kc6970929@gmail.com)


lOMoARcPSD|39315497

Algebra

30. If x 4 + x –4 =194, x > 0, then the x + y + z = 17, xyz = 171


(5 5 x³ – 3 3y³ )  ( 5 x – 3y ) =
xy + yz + zx = 111
1 (Ax² + By² + Cxy) (3A – B –
value of x + is :
x 3
x 3 + y 3 + z 3 + xyz  15C)
1 SSC CPO 25/11/2020 (Shift-1)
x4 + x–4 = 194, x > 0 x+
x SSC CPO 24/11/2020 (Shift-1) (a) 12 (b) 8
(a) – 64 (b) 0 (c) –3 (d) –5
SSC CPO 23/11/2020 (Shift-1) (c) 4 (d) – 4 42. If x4 + x–4 = 194, x > 0, then what
(a) 4 (b) 14 36. If x2 + 8y2 + 12y – 4xy + 9 = 0, then 1
the value of (7x + 8y) is: is the value of x + +2?
(c) 6 (d) 8 x
x² + 8y² + 12y – 4xy + 9 = 0
31. If x 2 + 8y 2 –12y – 4xy + 9 = 0 then x4 + x–4 = 194, x > 0
(7x + 8y)
the value of (7x – 8y) is: SSC CPO 24/11/2020 (Shift-1) 1
x+ +2
x
x² + 8y² – 12y – 4xy + 9 = 0 (a) – 33 (b) 9
(c) 33 (d) – 9 SSC CPO 25/11/2020 (Shift-1)
(7x – 8y)
37. If x + y + z = 13, x2 + y2 + z2 = 133 (a) 6 (b) 8

r
SSC CPO 23/11/2020 (Shift-2) and x3 + y3 + z3 = 847, then the
(c) 4 (d) 14
(a) 9 (b) 5

si
value of 3 xyz is: 43. If a² + b² = 82 and ab = 9, then a
(c) 12 (d) 21 possible value of a³ + b³ is:
x + y + z = 13, x² + y² + z² = 133
32. If x 2 – 5x + 1= 0 , then the value
an by a² + b² = 82 ab = 9 a³
x³ + y³ + z³ = 847 3 xyz
 4 1  + b³ . 1
of  x + 2  ÷  x +1 is :
2

n
 x  SSC CPO 25/11/2020 (Shift-2)
SSC CPO 24/11/2020 (Shift-2)
(a) 720 (b) 830
ja  4 1  (a) 8 (b) 7
x² – 5x + 1 = 0 x + 2  ÷ (c) 750 (d) 730
R s
 x  (c) – 9 (d) – 6
44. If x + y + z = 19, xyz = 216 and xy
(x² + 1) 38. If a3 + b3 = 217 and a + b = 7, then
+ yz + zx = 114, then the value of
a th

the value of ab is:


SSC CPO 23/11/2020 (Shift-2)
a³ + b³ = 217 a+b=7 x ³ + y³ + z³ + x yz is:
(a) 21 (b) 25
ab x + y + z = 19, xyz = 216
(c) 24 (d) 22
SSC CPO 24/11/2020 (Shift-2) xy + yz + zx = 114
ty a

33. If x + y + z = 19, xyz = 216 and xy


+ yz + zx = 114, then the value of (a) – 6 (b) – 1
x ³ + y³ + z³ + x yz
x3 + y 3 + z3 + xyz is: (c) 7 (d) 6
di M

39. If a2 + b2 + c2 + 84 = 4 (a – 2b + 4c), SSC CPO 25/11/2020 (Shift-2)


x + y + z = 19, xyz = 216 xy
then ab – bc + ca is equal to: (a) 32 (b) 28
+ yz + zx = 114 x³ + y³ + z³ +
a2 + b2 + c2 + 84 = 4 (a – 2b + (c) 30 (d) 35
xyz
45. If a + b + c = 0, then the value of
SSC CPO 23/11/2020 (Shift-2) 4c) ab – bc + ca
a 2 b2 c 2
(a) 1441 (b) 361 + + is:
bc ca ab
(c) 1225 (d) 577 SSC CPO 24/11/2020 (Shift-2)
34. If x2 – 3x + 1 = 0, then the value (a) 410 (b) 10 a 2 b2 c 2
a+b+c=0 + +
 4 1  (c) 510 (d) 210 bc ca ab
of  x + 2  ÷ x +1 is:
2

x 40. If x + y + z = 19, x2 + y2 + z2 = 133


and xz = y2, x > z > 0, what is the
value of (x – z)? SSC CPO 25/11/2020 (Shift-2)
 4 1 
A

x² – 3x + 1 = 0  x + 2  x + y + z = 19, x² + y² + z² = 133 (a) 0 (b) 3


x
xz = y², x > z > 0 (x – z) (c) 1 (d) –1
÷ (x² + 1)
46. (4x3y – 6x2y2 + 4xy3 – y4) can be ex-
SSC CPO 24/11/2020 (Shift-1)
SSC CPO 25/11/2020 (Shift-1) pressed as:
(a) 5 (b) 6 (4x3y – 6x2y2 + 4xy3 – y4)
(a) 0 (b) 5
(c) 7 (d) 9 1 $
(c) – 2 (d) – 5
35. If x + y + z = 17, xyz = 171 and xy +
SSC CPO 09/12/2019 (Shift-01)
yz + zx = 111, then the value of 41. If (5 5 x³ – 3 3y³ )  ( 5 x – 3y ) =
(a) (x + y)4 – x4 (b) (x + y)4 – y4
(Ax² + By² + Cxy), what is the value
3
x 3 + y 3 + z 3 + xyz  is: of (3A – B – 15C)? (c) (x – y)4 – x4 (d) x4 – (x – y)4

Aditya Ranjan (Excise Inspector) Selected Selection 111

Downloaded by Kunal Chauhan (kc6970929@gmail.com)


lOMoARcPSD|39315497

Algebra

47. If (2x + 3y + 4) (2x + 3y – 5) is a+b+c=6 a2 + b2 + c2 = 38 3 5


(a) (b)
equivalent to a(b2 + c2) + b(c2 + a2) – c(a2 + b2) 8 8
(ax2 + by2 + 2hxy + 2gx + 2fy + c), + 3abc
then what is the value of (g + f – 3 5
SSC CPO 11/12/2019 (Shift-01) (c) (d)
c)/(abh)? 4 16
(a) 3 (b) –3
(2x + 3y + 4) (2x + 3y – 5), (ax2 + (c) 6 (d) –6 8x
58. If = 1, x > 0, then
by + 2hxy + 2gx + 2fy + c)
2
2 53.
3 If (2x – 5y)3 – (2x + 5y)3 = y(Ax2 + 2 x ² 7 x – 2
By2), then what is the value of (2A 1
(g + f – c)/(abh) what is the value of x ³  ?
– B)? x³
(2x – 5y)3 – (2x + 5y)3 = y(Ax2 + 8x
SSC CPO 09/12/2019 (Shift-01) = 1, x > 0
By )
2
(2A – B) 2 x ² 7 x – 2
37 35 SSC CPO 11/12/2019 (Shift-01)
(a) (b) 1
216 432 x³ 
(a) 25 (b) 40 x³
19 19 (c) 15 (d) 10
(c) (d) SSC CPO 12/12/2019 (Shift-01)
108 216 1
54. If x  3, x  0 , then x² (x² – 3 3

r
48. If x4 + x2y2 + y4 = 21 and x2 + xy + y2 x (a) 17 (b) 17
8 4
= 3, then what is the value of 4xy? 47) = ?

si
x4 + x2y2 + y4 = 21 x2 + xy + y2 1 5 5
x  3, x  0 x² (x² – (c) 17 (d) 17
x 8 4
=3 an by
4xy
SSC CPO 09/12/2019 (Shift-02) 47) = ? 59. If a = 500, b = 502 and c = 504,
then the value of a³ + b³ + c³ – 3abc

n
(a) 12 (b) 4 SSC CPO 11/12/2019 (Shift-02)

(c) – 8 (d) – 4 (a) 0 (b) 2 a = 500, b = 502 c = 504


ja (c) – 2 (d) – 1 a³ + b³ + c² – 3abc
49. If x 2 – 7 x 1  0 , then (x3 + x–3) = ?
R s
55. If x and y are real numbers, then
the least possible value of 4(x –
2
x – 7 x 1  0 (x + x ) = ?
3 –3
a th

2)² + (y – 3)² – 2 (x – 3)² is: SSC CPO 12/12/2019 (Shift-01)


SSC CPO 09/12/2019 (Shift-02) (a) 15060 (b) 12048
x y 4 5 4 4(x –
(a) 10 7 (b) 4 7 2)² + (y – 3)² – 2 (x – 3)² . 4 (c) 18072 (d) 17040
ty a

(c) 7 7 (d) 3 7 60. If a – b = 5 and a² + b² = 45, then


SSC CPO 11/12/2019 (Shift-02) the value of ab is:
50. If x + y + z = 10, xy + yz + zx = 25
di M

(a) 3 (b) – 4
a– b=5 a² + b² = 45 ab
and xyz = 100, then what is the (c) 1 (d) – 8
value of (x3 + y3 + z3)?
56. If x = 5.51, y = 5.52 and z = 5.57,
x + y + z = 10, xy + yz + zx = 25 then what is the value of x³ + y³ SSC CPO 12/12/2019 (Shift-02)
+ z³ – 3xyz? (a) 25 (b) 10
xyz = 100 (x3 + y3 + z3)
x = 5.51, y = 5.52 z = 5.57 (c) 15 (d) 20
x³ + y³ + z³ – 3xyz 61. If x 4 + x –4 = 2599, then one of
SSC CPO 09/12/2019 (Shift-02) the values of x – x –1 , where x >
(a) 450 (b) 540 0, is equal to:
(c) 550 (d) 570 SSC CPO 11/12/2019 (Shift-02)
x 4 + x –4 = 2599 x – x –1
(a) 5.146 (b) 51.46
51. If x + y + z = 1, xy + yz + zx = – 26 x >0 6
and x3 + y3 + z3 = 151, then what (c) 0.05146 (d) 0.5146
SSC CPO 12/12/2019 (Shift-02)
will be the value of xyz?
A

21
57. If x + x² y² + y 
4 4
and x² + xy (a) 9 (b) 7
x + y + z = 1, xy + yz + zx = – 26 256
(c) 5 (d) 8
x3 + y3 + z3 = 151 xyz 3
+ y²  , then 2(x² + y²) =? 62. If a + b + c = 9 and ab + bc + ca =
16 18, then the value of a³ + b³ + c³
SSC CPO 11/12/2019 (Shift-01) – abc is:
21
(a) 24 (b) – 30 x4 + x² y² + y4  x² + xy a+b+c=9 ab + bc + ca = 18
256
(c) –18 (d) 32 a³ + b³ + c³ – abc
3
52. If a + b + c = 6 and a2 + b2 + c2 = 38, + y²  , 2(x² + y²) =? SSC CPO 12/12/2019 (Shift-02)
then what is the value of a(b2 + c2) 16
(a) 243 (b) 254
+ b(c2 + a2) – c(a2 + b2) + 3abc? SSC CPO 12/12/2019 (Shift-01) (c) 234 (d) 244

Aditya Ranjan (Excise Inspector) Selected Selection 112

Downloaded by Kunal Chauhan (kc6970929@gmail.com)


lOMoARcPSD|39315497

Algebra

63. If x² – 4x + 1 = 0, then what is the 65. If x4 + x²y² + y4 = 133 and x² – xy + y² 67. If x6 – 512y6 = (x² – Ay²) (x4 + Bx²y²
value of (x6 + x–6)? = 7, then what is the value of xy? + Cy4), then what is the value of
x² – 4x + 1 = 0 (x6 + x–6) (A + B – C) ?
x4 + x²y² + y4 = 133 x² – xy +
y² = 7 xy x6 – 512y6 = (x² – Ay²) (x4 + Bx²y²
SSC CPO 13/12/2019 (Shift-01) + Cy4) (A + B – C)
SSC CPO 13/12/2019 (Shift-01)
(a) 2786 (b) 2702
(c) 2716 (d) 2744 (a) 8
SSC CPO 13/12/2019 (Shift-02)
1
2
1
2 (b) 12 (a) – 72
   
64. If  x ³  – k    x  – p  0 ,
(c) 4
 x³   x  (b) 72
where k and p are real numbers and (c) – 80
(d) 6
k
x  0, then p is equal to: 66. If a + b + c = 19, ab + bc + ca = (d) 48
120, then what is the value of a³ 68. Solve the following:
2 2
 1   1  + b³ + c³ – 3abc? $
 x ³  – k    x  – p  0
  
x³ x a + b + c = 19, ab + bc + ca = 120 (a + b + c) (ab + bc + ca) – abc = ?
k p 4 5 4 a³ + b³ + c³ – 3abc
SSC CPO 13/12/2019 (Shift-02)

r
k
x0 p (a) (a + b) (b + c) (c – a)

si
SSC CPO 13/12/2019 (Shift-02) (b) (a – b) (b – c) (c – a)
SSC CPO 13/12/2019 (Shift-01)
(a) p² + 1 (b) p² + 3 (a) 31 (b) 23 (c) (a + b) (b – c) (c + a)
(c) p² – 1
an by (d) p² – 3 (c) 19 (d) 18 (d) (a + b) (b + c) (c + a)

n
ja
R s

ANSWER KEY
a th

1.(a) 2.(d) 3.(a) 4.(b) 5.(c) 6.(c) 7.(c) 8.(a) 9.(d) 10.(a)
ty a

11.(c) 12.(b) 13.(a) 14.(b) 15.(a) 16.(c) 17.(a) 18.(d) 19.(c) 20.(d)
di M

21.(b) 22.(d) 23.(c) 24.(a) 25.(b) 26.(a) 27.(a) 28.(a) 29.(a) 30.(a)

31.(a) 32.(d) 33.(c) 34.(b) 35.(d) 36.(a) 37.(d) 38.(d) 39.(d) 40.(b)

41.(c) 42.(a) 43.(d) 44.(d) 45.(b) 46.(d) 47.(b) 48.(c) 49.(b) 50.(c)

51.(a) 52.(d) 53.(d) 54.(d) 55.(b) 56.(c) 57.(b) 58.(c) 59.(c) 60.(b)

61.(b) 62.(a) 63.(b) 64.(d) 65.(d) 66.(c) 67.(c) 68.(d)


A

Aditya Ranjan (Excise Inspector) Selected Selection 113

Downloaded by Kunal Chauhan (kc6970929@gmail.com)


lOMoARcPSD|39315497

Algebra

SOLUTIONS
SOLUTIONS
1. (a) 4. (b) = 2(a2 + b2 + c2) – 2(ab + bc + ca)
Given, Given, = 2(38) – 2(31) = 76 – 62 = 14
a + b + c = 10
1 SMART APPROACH:-
a–  10 a2 + b2 + c2 = 48
a –5 By value putting,
We know that, a = 5, b = 3, c = 2
1
a – 5 – 5 (a + b + c) 2 = a 2 + b 2 + c 2 + 2(ab
 (a – b)² + (b – c)² + (c – a)²
a – 5  =2² + 1² + 3² = 14
+ bc + ca)
 (10)2 = 48 + 2(ab + bc + ca) 7. (c)
1
We know that, a –
a
 k then  100 – 48 = 2(ab + bc + ca)  m 2
 n2   m – n  –  m – n 
3

 52 = 2(ab + bc + ca)
1  m n – mn 
2 2

a3 –  k 3  3k.

r
 ab + bc + ca = 26
a3
 m – n  m2  n2  –  m – n  
2

SMART APPROACH:-

si
Hence, =
Let, c = 0 mn  m – n 
1 then, a + b = 10, a² + b² = 48
a – 5
3
an by

a – 5
3
3
 5  35 (a + b)² = a² + b² + 2ab
2ab = (10)² – 48
ab = 26
=
m2  n2 –  m2  n2 – 2mn 
mn

n
= 125 + 15
5. (c) 2mn
=140 = =2
mn
2. (d) ja We know that,
R s
If a + b + c = 0 then a3 + b3 + c3 8. (a)
Given,
= 3abc We know, (a + b)(a – b) = a2 – b2
a th

1
x  4  1154
4

x 
x – y 3  y – z 3 z – x 3  3  3
16y 2 – k   4y    4y – 
x 2
– y  y – z
2 2
  z
2 3 2
–x 
2 3
 2 2
1
x2  1 1 54  2
ty a

x2 2
3  x – y  y – z z – x  16y² – k   4y  –  
2 3

1 x 2
– y 2  y 2 – z 2  z 2 – x 2  2
di M

2
 x  2  1156
x On comparing
3  x – y  y – z  z – x 

1 3 x – y  y – z z – x x  y  y  z z  x  9
 x2 34 k
x2 4
1
Similarly,  9. (d)
x  y  y  z z  x 
Given,
1 6. (c)
x   3 4 2 x+y+z=8
x Given, x2 + y2 + z2 = 20
1 a + b + c = 10 We know that,
x 6
x a2 + b2 + c2 = 38 (x + y + z)2
We know that, = x2 + y2 + z2 + 2(xy + yz + zx)
1
A

 6  – 3  6 =198
3 3
Hence, x  (a + b + c)2 = a2 + b2 + c2 + 2(ab +  64 = 20 + 2(xy + yz + zx)
x3 bc + ca)
 64 – 20 = 2(xy + yz + zx)
3. (a)  (10)2 = 38 + 2(ab + bc + ca)
 44 = 2(xy + yz + zx)
Given,  100 – 38 = 2(ab + bc + ca)
 22 = xy + yz + zx
x4 + x–4 = 7  62 = 2(ab + bc + ca)
 x3 + y3 + z3 – 3xyz
x 2  x –2  7 2  31 = ab + bc + ca
= (x + y + z) {x2 + y2 + z2 – (xy + yz
x2 + x–2 = 3  a – b  b – c   c – a  + zx)}
2 2 2

1 = 8{20 –22}
2 = a2 + b2 – 2ab + b2 + c2 – 2bc +
The value of x  – 2 3 – 2 = 1 = – 16
x2 c2 + a2 – 2ca

Aditya Ranjan (Excise Inspector) Selected Selection 114

Downloaded by Kunal Chauhan (kc6970929@gmail.com)


lOMoARcPSD|39315497

Algebra

SMART APPROACH:- 16. (c)


SMART APPROACH:-
We can Assume value of x and y x 8
1
respectively 2 and 4 Given, 4
4
Now x
Put the value of x and y 2 and 4 then
satisfy the eq n. (i) 1
 x 
4
1 4
6x + 7y = 5 xy x4
12 + 28 = 5 × 2 × 4
40 = 40 Satisfy 1
 x 
2
eqn. (ii) 4
10 × 4 – 4 × 2 = 4 × 24 x2
32 = 32 Satisfy
Hence, 1
 4  – 3  4
3
The value of x and y is 2 and 4
 x6
x6
12. (b)
x 12  1
Given,   64 – 12 = 52
x6
x + y = 10 ....(1)
10. (a) 17. (a)
2xy = 48
We know that, (x + y)3 – (x – y)3 – 6y(x2 – y2)

r
 xy = 24 ....(2)
= x3 + y3 + 3xy(x + y) – {x3 – y3 –
(a + b)2 + (a – b)2 = 2(a2 + b2) Assume, such a value of x and y
3xy(x – y)} – 6yx2 + 6y3

si
 (7x + 4y)2 + (7x – 4y)2 which statisfies equation (1) and
(2). = x3 + y3 + 3x2y + 3xy2 – {x3 – y3 –
= 2{(7x)2 + (4y)2}
an by 3x2y + 3xy2} – 6yx2 + 6y3
x = 6 and y = 4
= 98x2 + 32y2 = x3 + y3 + 3x2y + 3xy2 – x3 + y3 +
 2x – y = 2(6) – 4
3x2y – 3xy2 – 6yx2 + 6y3

n
11. (c)
=8
6x + 7y = 5xy ......(1) = 8y3
SMART APPROACH:- SMART APPROACH:-
10y – 4x = 4xyja ......(2)
R s
We can Assume value of x and y Put x = 0, y = 1
On dividing equation (1) and (2) respectively 6 and 4 (x + y)³ – (x – y)³ – 6y (x² – y²)
by xy, we get
a th

n
Now, putting the value of x and y in eq . 1+1+6=8
x + y = 10 Putting the value of x and y in option (a)
6 7 6 + 4 = Satisfy
 5 ......(3) then satisfy the eqn.
y x 2xy = 48
2xy × 4 = 48 Satisfy
ty a

18. (d)
10 4 Now,
x2 + 6x + 1 = 0
and x – y  4 ......(4) 2x – y = 2 × 6 – 4 = 8
x(x + 6) + 1 = 0
di M

13. (a) Dividing both side by (x + 6)


Again multiplying equation (3) and
Put b = 0, a2 + 0 + 1 = 2a 1
equation (4) by 10 and 7 x 0
respectively  a2 – 2a + 1 = 0 x 6
 (a – 1)2 = 0 Adding 6 both sides
60 70
  50  a = 1 1
y x x 6 6
Therefore, a + b = 1 + 0
4 7 4 7
=1  x  6
70 28 14. (b) 1
–  28 x 63   6 3 – 3 6
x y Special Case: x 63
On substracting, we get: 1 = 216 – 18 = 198
if x   2 then x = 1 19. (c)
x
A

60 70  70 28  Given,
 – – = 50 – 28
y x  x y  1 a + b = 10 and ab = 9
Put x = 1 in x 
4
2
x4 We know,
88 (a – b)2 = (a + b)2 – 4ab
 y  22 15. (a) = 100 – 36 = 64
Put a = 2, b = – 1 and c = –1 a–b=8
y = 4 SMART APPROACH:-
a 2 b2 c2 4 1 1
On putting y = 4 in equation (3),   =  
bc ca ab 1 –2 –2 By value putting,
we get x = 2 a = 9, b = 1
Hence, x = 2 and y = 4 =4–1=3  a–b =9–1=8

Aditya Ranjan (Excise Inspector) Selected Selection 115

Downloaded by Kunal Chauhan (kc6970929@gmail.com)


lOMoARcPSD|39315497

Algebra
20. (d) 25. (b) 29. (a)
Given,
a2 + b2 = 82
Given
5 5x 3 – 3 3y 3    5x – 3y 
b2 + c2 = 65 1 = Ax2 + By2 + Cxy
S–  20
Assume, a = 9, b = 1 and c = 8 S–8
 2a + 7b – 3c = 2(9) + 7(1) – 3(8)  3A  B – 
15C  ?
= 18 + 7 – 24 = 1 1  a – b = (a – b) (a2 + b2 + ab)
3 3
 S – 8  –  12
21. (b) S – 8
Given, x (x – 5) = –1  
5 x – 3y 5 x 2  3y 2  15xy 
–1 1 5x – 3y
  S – 8 –
3
 x – 5
x  S – 8
3
= Ax2 + By2 + Cxy
On comparing
1
 x 5 = 123 + 3 × 12 = 1728 + 36 = 1764
x A = 5, B = 3, y =  15
26. (a)
1 Given 3A + B – 15 C
 x 
3
5 3 – 35
x3 =3×5+3– 15  15
1
1 x 2 9 8 = 15 + 3 – 15 = 3
x 3  3 1 1 0 x2

r

x 30. (a)
 x6 + 1 = 110x3 1

si
x  982 1 1
 x6 – 110x3 = –1  100 = 10 x4  , x ?
x x 4  194 x
 x3 (x3 – 110) = –1
22. (d)
Given,
an by 27. (a)
k(21x2 +24) + rx + (14x2 – 9) = 0 x4 
1
x4
 2  196

n
r = 55  21kx2 + 24k + rx + 14x2 – 9 = 0
 r(r2 + 3r + 3) 1
 (21k + 14)x2 + rx + 24k – 9 = 0 x2   14
= {r3 + 3r2 + 3r +1} – 1 x2
ja
R s
= {r3 + 1 + 3r (r +1)} – 1 .....(1)
= (r + 1)3 – 1 = 563 – 1 1
k(7x2 + 8) + px + (2x2 – 3) = 0 x 4
x
a th

= 175616 – 1 = 175615
 7kx2 + 8k + px + 2x2 – 3 = 0
31. (a)
SMART APPROACH:-  (7k + 2)x2 + px + 8k – 3 = 0 ....(2) x2 + 8y2 – 12y – 4xy + 9 = 0, (7x –
We can get the direct result
On dividing (1) by (2), we get– 8y) = ?
by divisibility rule 11 and 5
ty a

together x2 – 4xy + 4y2 + 4y2 – 12y + 9 = 0


On only option (d) is correct 21k  14 r 24k – 9 (x – 2y)2 + (2y – 3)2 = 0
  
7k  2 p 8k – 3 x – 2y = 0
di M

23. (c)
Given x = 2y
r 3  8k – 3  2y – 3 = 0
1  
a   P2 p 8k – 3
a 3
y=
2
2 1 r
 a   P4 – 2  3
a2 p 3 3
24. (a) x=2× x = 3, y =
2 2
Given a = 9.6, b = 4.44 and c = p 1
  3
5.16 r 3 7x – 8y = 7 × 3 – 8 ×
We know that 2
If a + b + c = 0 then, a3 – b3 – c3 28. (a)
= 21 – 12 = 9
– 3abc = 0 a2 + b2 + c2 + 216 = 2 (6a + 6b – 32. (d)
A

Here, 12c)
a – b – c = 9.6 – 4.44 – 5.16 = 0 (a – 6)2 + (b – 6)² + (c + 12)² = 0  4 1 
x2 – 5x + 1 = 0,  x  2  ÷ (x2 + 1) = ?
Therefore, a3 – b3 – c3 – 3abc = 0  x 
then
SMART APPROACH:- a = 6, b = 6, c = – 12 1
We know that,
x 5
Now, x
If a – b – c = 0 then a³ – b³ – c³ – 3abc = 0
Here,
a = 9.6, b = 4.44 and c = 5.16 = ab – bc + ca  4 1  1  3 1 
x  2   x  3 
a – b – c = 9.6 – 4.44 – 5.16 = 0 x x x 
Now, = 6  6 – (6)  (12)  (–12)  (6)   
1 1
a³ + b³ – c³ – 3abc = 0  x  1  x
2 
x  
x
= 36  72 – 72 = 6 

Aditya Ranjan (Excise Inspector) Selected Selection 116

Downloaded by Kunal Chauhan (kc6970929@gmail.com)


lOMoARcPSD|39315497

Algebra

1 = –748 + 684 = – 748 + 684 38. (d)


x  5
x x + y + z + xyz = – 64
3 3 3
a3 + b3 = (a + b) [(a+b)2 – 3ab]
217 = 7[49 – 3ab]
1 3
x 2  y 3  z 3  xyz  3 –64 = – 4
x3   110 31 = [49 – 3ab]
x3
SMART APPROACH:- 3ab = 18
 3 1  a + b + c – 3abc = (a + b + c)
3 3 3
ab = 6
 x  3  110
 x  [(a + b + c)2 – 3 (ab + bc + ca)]
 = 22

x 
1 5  x3 + y3 + z3 + xyz = (17) [(17)2 – SMART APPROACH:-
 
 x 3 × 111] + 171 + 513 = – 64 Value putting,
33. (c) a = 6, b = 1
3
x 3  y 3  z 3  xyz  – 4  ab = 6
(x + y + z)2 = x 2 + y2 + z2 + 2(xy +
yz + zx) 36. (a) 39. (d)
361 = x + y + z + 2 × 114
2 2 2 x2 + 8y2 + 12y – 4xy + 9 = 0, (7x +
a2 + b2 + c2 + 84 = 2(2a – 4b + 8c)
8y) = ?
x2 + y2 + z2 = 133 (a – 2)² + (b + 4)² + (c – 8)² = 0
x2 – 4xy + 4y2 + 4y2 + 12y + 9 = 0
x3 + y3 + z3 – 3xyz = (x + y + z)
(x – 2y)2 + (2y + 3)2 = 0 Then,
[x2 + y2 + z2 – (xy + yz + zx)]

r
–3 a = 2, b = – 4, c = 8
x3 + y3 + z3 + xyz x = 2y, y =
2

si
= 19 × [133 – 114] + 216 + 648 = ab – bc+ca
x + y + z + xyz = 1225
3 3 3
–3
an by
Short method:
a3 + b3 + c3 – 3abc = (a + b + c)
x = –3, y =
2

–3
=

=
2  –4  4  8  8  2

40 = 2 10

n
[(a + b + c)2 – 3 (ab + bc + ca)] 7x + 8y = 7 × –3 + 8 × = –33
2 40. (b)
 x3 + y3 + z3 + xyz = (19) [(19)2 – 3 ×
37. (d) (xy + yz + zx) =
ja
114] + 216 + 648 = 1225 x + y + z = 19, x2 + y2 + z2 = 133,
R s
x  y  z  – x  y  z  xz = y2, (x – z) = ?
2 2 2 2
SMART APPROACH:-
a th

Value putting, 2 (xy + yz + zx) =


a = 6, b = 4, c = 9
 x³ + y³ + z³ + xyz = 1225 169 – 133 x  y  z  – x 2  y2  z 2 
2

=  18
34. (b) 2 2
ty a

x3 + y3 + z3 – 3xyz = (x + y + z)
 4 1 
x  2  [x2 + y2 + z2 – (xy + yz + zx)] 361 – 133
x 
x2 – 3x + 1 = 0 ,  2 =
di M

? xyz =
 1
x  2

1 xy + yz + zx = 114
x 3 x 3  y3  z 3 –  x  y  z  x 2  y2  z 2 –  xy  yz  zx  
x 3
 zx  y 2
1
 x3   27 – 9 = 18 847 – 13 133 – 18 xy + yz + y2 = 114
x3 =
3 y(x + y + z) = 114
 4 1  1 1 847 – 1495
x  2  x3  3 = 114
 x  x
 x = 18  6 3 y= =6
1 1 3 19
 x  1  x
2
x
x
xyz = – 216
x + 6 + z = 19
35. (d)
3 xyz  3
–216 = – 6
x + z = 13
A

x2 + y2 + z2 = (x + y + z)2 – 2(xy +
yz + zx)
SMART APPROACH:- xz = 36

= 289 – 2 × 111 1 (x – z)2 = (x + z)2 – 4xz


a³ + b³ + c ³ – 3abc = (a + b + c)
= 289 – 222 = 67 2  169 – 4 × 36
x3 + y3 + z3 – 3xyz = (x + y + z) [ x2 [3(a² + b² + c²) – (a + b + c)²]  169 – 144
+ y2 + z2 – (xy + yz + zx)] 1 (x – z) = 5
x3 + y3 + z3 + xyz = (x + y + z) [x2 + 847 – 3xyz = ×13 [3 × 133 – 169]
2
y2 + z2 – (xy + yz + zx)] + 4xyz SMART APPROACH:-
= 17[67 – 111] + 4 × 171 3
xyz  3 216  – 6 Value putting, x = 9, y = 6, z = 4
x –z=9 –4 =5
= – 17 × 44 + 684

Aditya Ranjan (Excise Inspector) Selected Selection 117

Downloaded by Kunal Chauhan (kc6970929@gmail.com)


lOMoARcPSD|39315497

Algebra
41. (c) 46. (d) x3 + y3 + z3 – 3(100) = 10[(10)2 – 3 × 25]
(4x³y – 6x²y² + 4xy³ – y ) x3 + y3 + z3 – 300 = 10 [100 – 75]
  
4
5x – 3y 5x 2  3y 2  15xy
We know, x3 + y3 + z3 = 550
 5x – 3y  (x – y) = x – 4x y + 6x y – 4xy + 4y
4 4 3 2 2 3
51. (a)
= (Ax2 + By2 + Cxy) with option = x4 – (x – y)4 (satisfied) As we know,
On comparing 47. (b) x3 + y3 + z3 – 3xyz = (x + y + z) [(x + y
A = 5, B = 3, C = 15 (2x + 3y + 4) (2x + 3y – 5) + z)2 – 3 (xy + yz + zx)]
Now, 4x² + 6xy – 10x + 6xy + 9y² – 15y + 151 – 3xyz = 1[1 – 3 × – 26]
8x + 12y – 20 – 3xyz = 79 – 151
3A – B – 15 C
4x² + 9y² + 12xy – 2x – 3y – 20 3xyz = 79 – 151
3 × 5 – 3 – 15 × 15 Compare with
3xyz = 72
 15 – 3 – 15 (ax² + by² + 2hxy + 2gx + 2fy + c)
xyz = 24
–3
42. (a) –3 52. (d)
a = 4, b = 9, h = 6, g = – 1, f = ,
2 Given that,
1 1 c = – 20
x4   194 , x   2  ? a + b + c = 6, a² + b² + c² = 38

r
x4 x
Then, Let, c = 0
1

si
2
 x  2  14 3 Then,
x –1 –  20
gf –c 2 17.5 a + b = 6, a² + b² = 38, ab = – 1
an by   
1 abh 4 9 6 216 then, the value of
x 4
x ab² + ba² or ab (a + b)
35

n

1 432 – 1 (6) = – 6
x  2 6
x 53. (d)
ja 48. (c)
R s
43. (d) Given, that a3 – b3 = (a – b) [(a – b)2 + 3ab]
 (a + b)2 = a2 + b2 + 2ab (2x – 5y)3 – (2x + 5y)3 = y (Ax2 + By2)
x4 + y4 + x² y² = 21, x² + y² + xy = 3
a th

= 82 + 2 × 9 = 100
Then, (2x – 5y – 2x – 5y) [(– 10y)2 + 3 × (2x
a + b = 10 – 5y) × (2x + 5y)]
a3 + b3 = (a + b) (a2 + b2 – ab) x² + y² – xy = 7
xy = – 2 then 4xy = – 8 = – 10y [100 y2 + 3 × ((2x)2 – (5y)2)
= 10(82 – 9) = 730
ty a

= – 10y [100y2 + 3 × (4x2 – 25y2)]


SMART APPROACH:- SMART APPROACH:-
= – 10y [100 y2 + 12x2 – 75y2]
We know that,
di M

Value putting, a = 9, b = 1
 a³ + b³ = 730
4 4
x + x²y² + y = (x² + xy + y²) (x² – xy + y²) = – 10y [25y2 + 12x2] = y (Ax2 + By2)
Given, On compring
44. (d) x + x²y² + y = 21
4 4

x² + xy + y² = 3 ........ (i) = A = – 120, B = – 250


x2 + y2 + z2 = (x + y + z)2 – 2(xy +
x² – x y + y² = 7 .........(ii)
yz + zx)  2A – B = – 240 + 250 = 10
(i) – (ii)
= 361 – 2 × 114 = 133 xy = 2 54. (d)
x3 + y3 + z3 + xyz = (x + y + z) Now,

[x2 + y2 + z2 – (xy + yz + zx) + 4xyz


4 xy = 4 × (–2) = – 8 1
x =3
= 19[133 – 114] + 4 × 216 49. (b) x
= 19 × 19 + 864
x2 – 7x + 1 = 0
x3 + y3 + z3 + xyz = 1225 1
x+ =9–2=7
1 x
x 3 + y 3 + z 3  xyz  1225 = 35 =x+  7x
A

x
SMART APPROACH:- 1
1 x2 + = (7)2 – 2 = 47
Value putting, x = 4, y = 6, z = 9
 7 x2
3
x3 + 3  – 3 7
 x 3 + y 3 + z3 + xyz = 1225  35 x

45. (b) = 7 7 –3 7  4 7 x4 1
= 47
If a + b + c = 0 then, 50. (c) x2
 a3 + b3 + c3 = 3abc We know, x4 + 1 = 47x2
a 3  b3  c 3 3abc x3 + y3 + z3 – 3xyz = (x + y + z) x4 – 47x2 = – 1
  =3
abc abc [(x + y + z) – 3 (xy + yz + zx)]
2
= x2 (x2 – 47) = – 1

Aditya Ranjan (Excise Inspector) Selected Selection 118

Downloaded by Kunal Chauhan (kc6970929@gmail.com)


lOMoARcPSD|39315497

Algebra
55. (b) 2
4(x – 2)2 + (y – 3)2 – 2(x – 3)2 1  1 2 17  3 1  1
So,  x  3 – k  = 0  x  3 = k
3
x   
  4 =
For least possible value put x 2  2  x  x
(y – 3)2 = 0 We know, 2
 1  1
4(x – 2)² – 2 (x – 3)² &  x  – p = 0  x  =p
1 1  x  x
= 4(x2 + 4 – 4x) – 2 (x² + 9 – 6x) If x   N ,then x ³  3 =N3 – 3N
= 4x2 + 16 – 16x – 2x2 – 18 + 12x x x We know,
= 2x2 – 2 – 4x 1 17 17 3 17 5 17 3
 x3 +  – =  1 1  1
= 2(x2 – 1 – 2x) 3
 x   = x  3  3 x  
x3 8 2 8  x x  x
= 2[(x – 1)2 – 2]
59. (c) p³ = k + 3p
We know,
a = 500, b = 502, c = 504 k = p³ – 3p
For least possible value put (x –
1)2 = 0 We know,
a3 + b3 + c3 – 3abc = (a + b + c) × 3d2 k
=2[(x – 1)² – 2] = 2 × (– 2) = – 4  p = p2 – 3
56. (c) = (500 + 502 + 504) × 3 × (2)2
x = 5.51, y = 5.52, z = 5.57 = 1506 × 12 = 18072
60. (b) SMART APPROACH:-
value of x3 + y3 + z3 – xyz
We know, Put x = 1
we know that

r
(2 – k)² + (2 – p)² = 0
(a – b)2 = a2 + b2 – 2ab  k = 2 and p = 2

xyz  25 = 45 – 2ab
x3 + y3 + z3 – 3xyz = k 2

si
2  – 20 = – 2ab   1
p 2
[(x – y)² + (y – z)² + (z – x)²] =  ab = 10 Option (P² – 3) (satisfied)

2
an by
(5.51  5.52  5.57) 61. (b)
1
65. (d)

n
x + 4 = 2599
4 x4 + x2y2 + y4 = 133
[(5.51 – 5.52)2 + (5.52 – 5.57)2 + (5.57 x
– 5.51)2] x2 – xy + y2 = 7 ...(1)
1 We know,
16.60 ja x2 +  2599  2 = 2601
R s
= [(0.01)2 + (0.05)2 + (0.06)2] x2 (x2 –xy + y2)(x2 + xy + y2)=x4 + x2y2 + y4
2 7(x2 + xy + y2) = 133
1
a th

16.6 x2 + = 51 x2 + xy + y2 = 19 ...(2)


= [0.0001 + 0.0025 + 0.0036] x2 Subtract eqn (1) from eqn (2)
2
1 2xy = 12
= 8.3 [0.0062] = 0.05146 x – =
x 51 – 2  49  7 xy = 6
57. (b)
ty a

62. (a) 66. (c)


Given,
We Know, a³ + b³ + c³ =(a + b + c) {(a We know that, a3 + b3 + c3 – 3abc
3
di M

+ b+ c)2 – 3 (ab + bc + ca)} = (a + b + c) [(a + b + c)2 – 3 (ab + bc


x 2  xy  y 2 = ----(1)
16 = 9{81 – 3 × 18} + ca)]
= 9(27) = 19(361 – 360) = 19
As we know, x 4  y 4  x 2y 2
= 243 67. (c)
=  x  y  xy  x  y  xy  x3 – y3 = (x – y) (x2 + y2 + xy)
2 2 2 2

SMART APPROACH:- x6 – 512 y6 = (x2 – Ay2) (x4 + Bx2y2 + Cy4)


By value putting, a = 3, b = 6
 21   3  2
 
 8y  [x + 8x y + 64y ]
   =   x  xy  y
2  a³ + b³ = 243  2 
= x –
2
 256   16  4 2 2 4
63. (b) 
7 x2 – 4x + 1 = 0 = (x2 + Ay2) (x4 – Bx2y2 + Cy4)
2
 2
 x  xy  y =
16

----(2)
1 On Comparing
x+ =4
On adding (1) and (2), We get- x A = – 8, B – 8 and C = 64
1 Now,
5
 2 x ²  y ²  = x3 +
A

= (4)3 – 3×4 = 52 A + B – C = (– 8 – 8 – 64) = – 80


8 x3
58. (c) 1 68. (d)
x6 + = (52)2 – 2 = 2702
x6 (a + b + c) (ab + bc + ca) – abc
8x
 1 64. (d) put a = b = c = 1
2x 2  7x – 2 As we know,
If, (a – b)2 + (c – d)² = 0 (1 + 1 + 1) (1 + 1 + 1) – 1
 8x = 2x + 7x – 2
2

 2x2 – x – 2 = 0 a = b and c = d =3×3–1=8


 3 1  
2
1 
2 Hence, Option 4
2 1 1
 2x – 1  x –   x  3 – k    x  – p = 0 (a + b) (b + c) (c + a) (satisfied)
x x 2  x   x 

Aditya Ranjan (Excise Inspector) Selected Selection 119

Downloaded by Kunal Chauhan (kc6970929@gmail.com)


lOMoARcPSD|39315497

Trigonometry

TRIGONOMETRY
18

2 sin tanx = 5 A ! B" # $ %


1. If x  1 co s sin , then the x
  1
sinx sin A = ! sin
" B=
1 – cos   sin 
3
SSC CPO 09/11/2022 (Shift-02)
value of 1  sin   is: 1
1 5 cos (A – B)
(a) (b) 5
2 s in 3 26
x SSC CPO 09/11/2022 (Shift-03)
1  c o s s in

r
2 5
(c) (d) 4 3 1 8 3 –1
1 – cos   sin  28

si
5 (a) (b)
1  sin   15 15
6. Simplify/

x
an by
SSC CPO 09/11/2022 (Shift-01) sin 8 cos  – sin 6 cos 3
cos 2 cos  – sin 3 sin 4
(c)
8 3 1
15
(d)
4 3 –1
15

n
(a) (b) x
1 x  SSC CPO 09/11/2022 (Shift-02)
11. What is the value of/
(a) cot (b) cot2
1 ja 1 x  sin  A  B 
R s
(c) tan (d) tan2
(c) (d) ?
x x 7. The value of/ ? sin A cos B
a th

n (sin 45° + cos 60°) + (tan445° +


4 4
SSC CPO 09/11/2022 (Shift-03)
 sec  – 1  
2. If     cosec  – cot  cot445°) is: (a) 1 + cot A tan B
 sec   1  
SSC CPO 09/11/2022 (Shift-02) (b) 1 + tan A cot B
then n =?:
ty a

37 33 (c) 1 – sin A cos B


(a) (b)
 sec  – 1  
n
16 16 (d) 1 – cot A tan B
    cosec  – cot 
di M

 sec   1   35 39 12. Which of the following is the value


(c) (d)
n =? 16 16 of/ (
SSC CPO 09/11/2022 (Shift-01)
8. The value of/
(1 + cot A – cosec A)(1 + tanA + 1 – sin 45
(a) 1 (b) 0.5 ?
sec A)–1 is: 1  sin 45
(c) –1 (d) – 0.5
SSC CPO 09/11/2022 (Shift-02) SSC CPO 09/11/2022 (Shift-03)
3. 16sec A –16tan A = _______.
2 2
(a) 1 (b) 2 (a) cos 45° – tan 45°
SSC CPO 09/11/2022 (Shift-01)
(c) 3 (d) 0 (b) tan 45° – sec 45°
(a) 9 (b) 0
9. The value of/ (c) tan 45°
(c) 16 (d) 1
23cosec2A – 23cot2A is: (d) sec 45° – tan 45°
4. The value of/
SSC CPO 09/11/2022 (Shift-03)
13. The value of the expression
A

1  tan  1 – tan   cos245° + cos2135° + cos2225° +


(2cos² –1)    is: (a) 23 (b) 1
1 – tan  1  tan   cos2315° is:
(c) 27 (d) 0 ) cos 2 45° + cos 2 135° +
SSC CPO 09/11/2022 (Shift-01)
10. Given that A and B are second
(a) 2 (b) 0 cos2225° + cos2315° *
1 SSC CPO 10/11/2022 (Shift-01)
quadrant angles, sin A = and
3 3
(c) (d) 1 1
2 (a) 2 (b)
1 2
5. In a right triangle for an acute sin B = , then find the value
5 3
angle x, find sinx. It is given that of cos (A – B). (c) (d) 1
tanx = 5. 2

Aditya Ranjan (Excise Inspector) Selected Selection 120

Downloaded by Kunal Chauhan (kc6970929@gmail.com)


lOMoARcPSD|39315497

Trigonometry

14. The value of/ 19. If the value of cosecA + cotA = m, cos A = sin2A ! asin " 12A +
then the value of cosecA – cotA is: bsin10A + c sin8A + sin6A = 1 a
2cos 3  – cos 
is: cosecA + cotA = m +b+c=?
sin  – 2sin3 
cosecA – cotA SSC CPO 11/11/2022 (Shift-01)
SSC CPO 10/11/2022 (Shift-01)
SSC CPO 10/11/2022 (Shift-02) (a) 7 (b) 8
(a) sec  (b) sin 
1 (c) 9 (d) 6
(c) cot  (d) tan  (a) (b) m
m 26. Simplify/
m cos(36° – A) cos(36° + A) + cos(54°
15. If cos 48° = (c) m (d) m2
n – A)cos(54° + A)
Then, sec 48° – cot 42° is equal to: 20. If cos   sin   2 cos  , find the SSC CPO 11/11/2022 (Shift-01)
m value of  cos  – sin   (a) cos A (b) sin 2A
cos 48° = sec 48° – cot
n (c) cos 2A (d) sin A
42° +" +" cos   sin   2 cos  27. Evaluate/
SSC CPO 10/11/2022 (Shift-01)  cos  – sin   sin54 sec 46

SSC CPO 10/11/2022 (Shift-02) cos 36 cosec 44

r
m – n2 – m2
(a)
m (a) 2 sin  (b) 2 cos  SSC CPO 11/11/2022 (Shift-01)

si
(a) 0 (b) –1
m – n2 – m2 1 1 (c) 2 (d) 1
(b)
n
an by (c)
2
sin  (d)
2
cos 
28. If 4 cos  + 3 sin  = x and
4 sin  – 3 cos  = y, find the value

n
21. Simplify the following:/
n – n2 – m2 of x2 + y2.
(c)
n 4cos  + 3 sin  = x ! "
ja (1 + cot2)(1 – cos)(1 + cos)
4 sin  – 3 cos  = y
R s
SSC CPO 10/11/2022 (Shift-03)
n – n2 – m2
(d) (a) 1 (b) –5
a th

m SSC CPO 11/11/2022 (Shift-01)


(c) 3 (d) –3
16. Solve the following equation. (a) 16 (b) 9
22. If cos + sec = 2, then the value
( " of sin6 + cos6 is: (c) 25 (d) 1
ty a

cos + sec = 2 sin  +


6 29. Simplify/
sec 2  1 – sin2  
cos6 cos 4  – sin4 
di M

SSC CPO 10/11/2022 (Shift-01)


SSC CPO 10/11/2022 (Shift-03) sin2 
(a) tan  (b) cosec
1 SSC CPO 11/11/2022 (Shift-02)
(a) (b) 0
(c) sec  (d) 1 3 (a) 1 – tan2  (b) tan2  – 1
17. If cosA + cosB + cosC = 3. Then 1
what is the value of sinA + sinB (c) 1 (d) (c) cot 2  – 1 (d) 1 – cot 2 
2
+ sinC? 30. The value of/
 7 
cosA + cosB + cosC = 3 sinA 23. Find/ cos  – . sin A sin A
 2  – –1
+ sinB + sinC cot A  cosec A cot A – cosec A
SSC CPO 10/11/2022 (Shift-02) SSC CPO 10/11/2022 (Shift-03)
SSC CPO 11/11/2022 (Shift-02)
(a) 1 (b) 2 1
(a) (b) 1 1
(c) 0 (d) –1 2 (a) (b) 3
2
A

18. What is the value of following in (c) –1 (d) 0 (c) 0 (d) 2


the term of trignometric ratios?
24. The value of/ 31. If sin2   cos 3  , then the value
! , - (
(cosec A + cot A) (1 – cosA) is:
of cot 2   cot 6  is:
SSC CPO 10/11/2022 (Shift-03)
sin A 1  cos A sin2   cos 3 
 (a) cos A (b) tan A
1  cos A sin A (c) cot A (d) sin A cot 2   cot 6 
SSC CPO 10/11/2022 (Shift-02) 25. If cos A = sin A, and asin12 A +
2 SSC CPO 11/11/2022 (Shift-02)
(a) 2 cosec A (b) 2 cos A bsin 10 A + c sin 8 A + sin 6 A = 1, (a) –1 (b) 0
(c) 2 sec A (d) 2 sin A then a + b + c = ? (c) 2 (d) 1

Aditya Ranjan (Excise Inspector) Selected Selection 121

Downloaded by Kunal Chauhan (kc6970929@gmail.com)


lOMoARcPSD|39315497

Trigonometry

37. If 4 – 2 sin²– 5 cos = 0,0º <  43. If 4 is an acute angle, and cot
1
32. If sin A  , then the value of < 90º, then the value of cos + 4= tan (– 5º), then what is the
2 value of ?
tan is:
(tanA + cosA) is: 4 cot 4 != tan
"
4 – 2 sin²– 5 cos = 0,0º <
1 ( –5º)  "
sin A  (tanA + cosA) < 90º cos + tan
2 SSC CPO 24/11/2020 (Shift-2)
" (a) 19º (b) 45º
SSC CPO 23/11/2020 (Shift-1) (c) 21º (d) 24º
SSC CPO 11/11/2022 (Shift-02)
2+ 3 2– 3 1
2 3 (a) (b) 44. Solve for  : cos² – sin² = 0
(a) (b) 2 2 2
3 3 2 3 <  < 90º.
5 5 1 2 3 1– 2 3  cos²"  –* sin² 
(c) (d)
(c)
2 3
(d)
3 3
2 2 1
38. If sec 3x = cosec (3x – 45º), where = 0 <  < 90º
33. If 1 + sin  – 3sincos =0, then
2 2
3x is an acute angle, then x is
the value of cot is: SSC CPO 25/11/2020 (Shift-2)
equal to:
1 + sin2 – 3sincos = 0, (a) 45º (b) 60º
sec3x = cosec(3x – 45º) .

r
cot (c) 40º (d) 30º
3x x "
SSC CPO 11/11/2022 (Shift-03) 1

si
SSC CPO 23/11/2020 (Shift-2)
(a) 0 (b) 2 45. If cot = 0º <  < 90º, then
(a) 27.5º (b) 45º 3
1 an by 1 (c) 35º (d) 22.5º 2 – sin2 
(c) (d) the value of + (cosec²
2 3 39. The value of/ " 1 – cos2 

n
4 4 5 sin2 30º +cos 2 60º –sec35º.sin55º  – sec) is:
34. If cos  – sin   , then the sec60º +cosec30º
6 1
cot = 0º <  < 90º
ja
value of 2 cos  – 1 _____.
2 SSC CPO 23/11/2020 (Shift-2) 3
R s
5 1 1 2 – sin2 
cos 4  – sin4   (a) – (b) + (cosec² – sec)
a th

6 4 4 1 – cos2 
2 cos 2  – 1 1 1 "
(c) – (d)
SSC CPO 11/11/2022 (Shift-03)
8 8 SSC CPO 25/11/2020 (Shift-2)
40. If sin 3x = cos (3x – 45º), 0º < 3x < (a) 5 (b) 0
ty a

11 5 90º, then x is equal to:


(a) (b) (c) 1 (d) 2
6 6 sin 3x = cos (3x – 45º), 0º < 3x
di M

< 90º x " 46. If 0°    90°, sec107 + cos107 


6 6
(c) (d) SSC CPO 24/11/2020 (Shift-1)
= 2, then, (sec + cos ) is equal
11 5 to:
35. If tan 45° + sec60° = x, find the (a) 35º (b) 45º
value of x. (c) 22.5º (d) 27.5º
0°    90°, sec107  + cos107
tan 45° + sec60° = x x 41. The value of/ "
= 2, (sec + cos )
" sin2 30º +cos 2 60º +sec 45º.sin 45º
SSC CPO 11/11/2022 (Shift-03) sec 60º +cosec 30º
(a) 1 (b) 4 SSC CPO 09/12/2019 (Shift-01)
SSC CPO 24/11/2020 (Shift-1)
(c) 3 (d) 2 (a) 2 (b) 1
1 3
36. If 4(cosec² 57º – tan² 33º) – cos (a) – (b)
4 8 1
y (c) (d) 2–107
90º – y tan² 66º tan² 24º = then 3 1 2
2
A

(c) – (d)
value of y is: 8 4 47. If 0° <  < 90°, sin (2 + 50°) = cos
4(cosec² 57º – tan² 33º) – cos 42. The value of/ " (4 + 16°), then what is the value
y of  (in degrees)?
sin2 52º  2  sin2 38º
90º – y tan² 66º tan² 24º = is:
2 4 cos2 43º –5  4 cos2 47º 0° <  < 90°, sin (2 + 50°) = cos (4
y " SSC CPO 24/11/2020 (Shift-2) + 16°),  /! ’ 0
SSC CPO 23/11/2020 (Shift-1) 1 SSC CPO 09/12/2019 (Shift-01)
(a) –4 (b) 8 (a) 3 (b)
3 (a) 10° (b) 8°
8 1
(c) 4 (d)
3 (c) – (d) –3 (c) 4° (d) 12°
3

Aditya Ranjan (Excise Inspector) Selected Selection 122

Downloaded by Kunal Chauhan (kc6970929@gmail.com)


lOMoARcPSD|39315497

Trigonometry

1 1
52. If 7 sin²  + 3 cos²  = 4, 0° <  <
57. If 2 sin (60° – ) = 1, 0° <  < 90°,
48. If c o se c 1  c o se c  – 1 = 2 sec 90°, then the value of (tan² 2 +
cosec² 2) is: then  is equal to:
, 0° <  < 90°, then the value of
7 sin²  + 3 cos²  = 4, 0° <  < 90° 2 sin (60° – ) = 1, 0° <  < 90°
tan   2sec 
is: (tan² 2 + cosec² 2)
cosec  

1 1 SSC CPO 11/12/2019 (Shift-02) SSC CPO 12/12/2019 (Shift-02)



c o se c 1 c o se c  – 1 = 2 sec
15 (a) 45° (b) 30°
tan   2sec  (a) 7 (b)
4 (c) 15° (d) 60°
, 0° <  < 90°
cosec 
13 13 58. If 3(cot² – cos²) = 1 – sin²
* (c) (d) 0° <  < 90°, then  is equal to:
3 4
SSC CPO 09/12/2019 (Shift-02)
53. If 21 tan = 20, then, 3(cot² – cos²) = 1 – sin²
2 3 4 2 (1 + sin + cos): (1 – sin + cos) = ? 0° <  < 90°  + 1
(a) (b)
2 2 21 tan = 20
SSC CPO 13/12/2019 (Shift-01)
(1 + sin + cos): (1 – sin + cos) = ?
2 2 4 3

r
(c) (d) SSC CPO 11/12/2019 (Shift-02) (a) 30° (b) 60°
2 2
(a) 5 : 2 (b) 3 : 1 (c) 45° (d) 15°

si
49. If (cos  + sin ) : (cos  – sin ) = (c) 7 : 3 (d) 2 : 1
( 3  1) : ( 3 – 1) , 0° <  < 90°, then 54. If 2sin  + 15 cos²  = 7, 0° <  < 90° 59. The value of/
an by
what is the value of sec  ?
(cos  + sin ) : (cos  – sin ) =
then what is the vlaue of
3 – tan 
2  tan 
?
tan²48° – cosec²42° + cosec (67° +
) – sec (23° – ) is:

n
( 3  1) : ( 3 – 1) , 0° <  < 90° 2sin  + 15 cos²  = 7, 0° <  < 90° SSC CPO 13/12/2019 (Shift-01)

sec  ja 3 – tan  (a) – 1 (b) 0


R s
SSC CPO 09/12/2019 (Shift-02) 2  tan  (c) 1 (d) – 2
(a) 1 (b) 2 SSC CPO 12/12/2019 (Shift-01)
a th

cos  cos 
2 3 1 3 60. If  = 4, 0° <  < 90°,
(c) (d) (a) (b) 1 – sin  1  sin 
2 3 4 4 then what is the value of (sec +
50. If 3 + cos2 = 3(cot2 + sin²), 00 < cosec + cot)?
1 5
ty a

 < 900, then what is the value of (c) (d)


2 8 cos  cos 
(cos + 2sin)? 
55. If cosec  = 1.25, then = 4, 0° <  < 90°
di M

3 + cos2 = 3(cot2 + sin²), 00 < 1 – sin  1  sin 


4 tan  – 5 cos   1 (sec + cosec + cot)
 < 900 (cos + 2sin) ?
sec   4 cot  – 1

SSC CPO 11/12/2019 (Shift-01)


cosec  = 1.25 SSC CPO 13/12/2019 (Shift-02)
4 tan  – 5 cos   1
32 ?
(a) 3 2 (b) sec   4 cot  – 1 1 2 3
2 (a) 2  3 (b)
SSC CPO 12/12/2019 (Shift-01) 3
2 3 1 3 3 1 9
(c) (d) (a) (b) 2 2 3
2 2 10 (c) 1  2 3 (d)
3
51. If tan (11) = cot (7), then what 10 1
is the value of sin² (6) + sec² (9) (c) (d) 61. If cos² – sin² = tan² , then which
11 2
of the following is true?
A

+ cosec² (12)? 56. If sin – cos = 0, 0° <  < 90°,


tan (11) = cot (7) sin² (6) then the value of sin4 + cos4 is: cos² – sin² = tan²
+ sec² (9) + cosec² (12) sin – cos = 0, 0° <  < 90° ( 2
sin4 + cos4
SSC CPO 13/12/2019 (Shift-02)
SSC CPO 11/12/2019 (Shift-01) SSC CPO 12/12/2019 (Shift-02)
(a) cos cos = 1
35 23 1
(a) (b) (a) (b) 1 (b) cos cos =2
12 6 3
1 1 (c) cos² – sin² = cot²
31 43
(c) (d) (c) (d)
12 12 2 4 (d) cos² – sin² = tan²

Aditya Ranjan (Excise Inspector) Selected Selection 123

Downloaded by Kunal Chauhan (kc6970929@gmail.com)


lOMoARcPSD|39315497

Trigonometry

ANSWER KEY
1.(b) 2.(b) 3.(d) 4.(a) 5.(b) 6.(d) 7.(a) 8.(a) 9.(a) 10.(c)

11.(a) 12.(d) 13.(a) 14.(c) 15.(d) 16.(c) 17.(c) 18.(a) 19.(a) 20.(a)

21.(a) 22.(c) 23.(d) 24.(d) 25.(a) 26.(c) 27.(c) 28.(c) 29.(c) 30.(b)

31.(a) 32.(c) 33.(b) 34.(b) 35.(c) 36.(d) 37.(c) 38.(d) 39.(c) 40.(c)

41.(b) 42.(d) 43.(a) 44.(d) 45.(c) 46.(a) 47.(c) 48.(b) 49.(d) 50.(c)

51.(d) 52.(c) 53.(c) 54.(c) 55.(c) 56.(c) 57.(c) 58.(b) 59.(a) 60.(a)

r
si
61.(d)

an by
n
ja
R s
a th
ty a
di M
A

Aditya Ranjan (Excise Inspector) Selected Selection 124

Downloaded by Kunal Chauhan (kc6970929@gmail.com)


lOMoARcPSD|39315497

Trigonometry

SOLUTIONS
1. (b) 5. (b)
sin2 
Given,  We know that,
sin  1  cos  
2sin Perpendicular
x   tan  
sin  Base
1  cos sin  ............... 2
1  cos 
by the rationalization 5
Multiplying [1] and [2], we get– tanx =
1
2sin  1 – cos   sin 
  1 – cos  Hence, Perpendicular = 5 and
  cosec  – cot  
2
1  cos   sin  1 – cos   sin  Base = 1
1  cos 
By Pythagoras Theorm,
2sin  1 – cos   sin  
 sec  – 1
  cosec  – cot  
2
1  sin  
2
–  cos  
2 H  P 2  B2

r
sec   1
= 52  12

si
2sin  1 – cos   sin    sec  – 1 

   cosec  – cot  = 26
1  2sin   sin2  – cos2   sec   1 


an by
2sin  1 – cos   sin  
Hence, n =
1
2
= 0.5
Sinx 
Pe rp en dicu lar
H yp ote nu se

n
sin2   cos2   2sin   sin2  – cos2 
3. (d) 5
2sin  1 – cos   sin   =
26
 ja
2sin   2sin2 
We know that, sec A – tan A =1 2 2
R s
16sec 2A – 16tan 2 A= 16(sec 2 A – 6. (d)
tan2 A)
2sin  1 – cos   sin  
a th

sin 8 cos  – sin 6 cos 3


 = 16 × 1 = 16
2sin  1  sin   cos 2cos  – sin 3sin 4
4. (a)
Multiplying and dividing by 2
1 – cos   sin 
 2cos  – 1 11 – tan  1 – tan  
ty a

 =x
2

1  sin  tan  1  tan   sin 8 cos  – sin 6 cos 3 2
 
cos 2 cos  – sin 3 sin 4 2
di M

SMART APPROACH:-  1  tan2  1– tan 2 


We can solve this problem by putting   2cos2 –1   2sin 8 cos  – 2sin 6 cos 3
 1– tan1  tan  
the value of  = 90° 2cos 2 cos  – 2sin 3 sin 4

2. (b)  2  2 tan2   We know,


  2 cos 2  – 1  
Given 2
 1 – tan   2sinAcosB = sin(A + B) + sin(A –B)
2cosAcosB = cos(A + B) + cos(A –B)
 sec  – 1 n 
     cosec  – cot     2sinAsinB = cos(A – B) + cos(A –B)
 sec   1    2 1  tan 2   
  2 cos  – 1 
2
2  sin 9 sin 7 –  sin 9 cos 3 
R.H.S = cosec – cot  1 – sin   
 cos 2   cos 3  cos  –  cos  – cos 7 
1 cos  sin 7 – sin 3
 – 
A

sin  sin    cos 3  cos 7


2sec2 
  2cos 2  – 1  
1 – cos  
2

 sec  cos2  – 1 – cos2    

We know,
 ................ 1
sin  C  D C – D
  sinC – sinD  2cos  2  sin  2 
2
  2 cos  – 1 
   
2
=2
1 – cos  1  cos    2 cos  – 1 
2
   C  D C – D
sin  1  cos  cosC – cosD = 2cos   sin  
 2   2 
SMART APPROACH:-
1 – cos 2  2cos 5 sin 2
 We can easily solve this problem by putting

sin  1  cos   the value of  = 30°
=
2cos 5 cos 2 tan 2

Aditya Ranjan (Excise Inspector) Selected Selection 125

Downloaded by Kunal Chauhan (kc6970929@gmail.com)


lOMoARcPSD|39315497

Trigonometry

7. (a)
 cos A  1 – sin2 A
 2 – 1
2
We know,

 2 – 1
2 2
1 1 2 2 2
sin 45   1–   
2 3 3

1  2 –1
1 sin B 
cos 60  5
2 We know, 2 = sec 45° and 1 =
tan45° = 1  cos B  1 – sin B 2 tan 45°
cot45° = 1 Hence Answer (d)
 (sin445° + cos460°) + (tan445° 1
2
13. (a)
 1–   cos245° + cos2135° + cos2225° +
+ cot445°) 5 cos2315°
 1 4  1 4  = cos245° + {cos(180° – 45°)}2 +
2 6
        1  1 = {cos(180° + 45°)}2 + {cos(360° – 45°)}2
 2   2   5
= cos245° + {– cos45°}2 + (–cos45°}2
Since the angles given in the
1 1  + {cos45°}2
  2 second quadrant.
 4 16  So,
2 2 2 2
 1   1   1   1 
      

r
 4  1 
 2 –2 2  2  2  2  2
 16  cosA 

si
3 1 1 1 1
5 37     =2
 2  2 2 2 2
16 16 –2 6
8. (a)
an by
(1 + cotA – cosecA) (1 + tanA +
cosB =
5
We know that–
14. (c)
2cos3  – cos 

n
secA) –1 cos(A – B) = cosAcosB + sinAsinB sin  – 2sin3 
 cos A 1  sin A 1  2 2 2 6 1 1 cos   2cos 2  – 1
 1 
 ja
–  1 
sin A sin A  
  –1
cos A cos A      =
sin  1 – 2sin2  
R s
3 5 3 5
  sin A  cos A 2 – 12  2 2 2 2 3 1 1 cos cos 2
a th

=   –1      = cot 
 sin A.cos A  3 5 3 5 sin cos 2
15. (d)
8 3 1
 sin2 A  cos 2 A  2sin A cos A – 1  m
=   –1 15 cos 48 
ty a

 sin A.cos A  n
11. (a)
put sin2A + cos2A = 1 We know that– m
cos  90 – 42  
di M

sin(A + B) = sinAcosB + cosAsinB n


1  2sin A cos A – 1 
=  –1 sin  A  B  m
 sin A.cos A   sin 42 
sin A cos B n
 2sin A cos A 
=   –1 sin A cos B  cos A sin B cos 42  1 – sin2 42
 sin A.cos A  
sin A cos B
=2–1=1 m2
sin A cos B cos A sin B  1–
  n2
SMART APPROACH:- sin A cos B sin A cos B
= 1 + cotA tanB n2 – m 2 n2 – m 2
We can easily solve this problem by putting  
the value of  = 45° 12. (d) n2 n
cos 42
9. (a) 1 cot 42 
1– sin 42
We know that, 1 – sin 45 2

A

cosec2A – cot2A = 1 1  sin 45 1 n2 – m2


1
2 n n2 – m 2
23 cosec2A – 23cot2A  
m m
= 23(cosec2A – cot2A) n
2 –1
= 23(1)
2 n
= 23 = sec 48 
2 1 m
10. (c)
2  sec 48 – cot 42
Given,
1 2 –1 2 –1 n n2 – m 2 n – n2 – m2
sin A    = – 
3 2 1 2 –1 m m m

Aditya Ranjan (Excise Inspector) Selected Selection 126

Downloaded by Kunal Chauhan (kc6970929@gmail.com)


lOMoARcPSD|39315497

Trigonometry
16. (c) 21. (a) 26. (c)
 sec 2   1 – sin2   (1 + cot2)(1 – cos)(1 + cos)
= (1 + cot2)(1 – cos2)
cos(36° – A)cos(36° + A) + cos(54°
– A)cos(54° + A)
= sec 2   cos  = sec  = cosec2× sin2 = cos(36° + A)cos(36° – A) + cos{90°
=1 – (54° – A)} cos{90° – (54° + A)}
22. (c) = cos (36° + A)cos(36° – A) + sin(36°
17. (c)
Given, cos + sec= 2 + A) sin(36° – A)
Given,
We know, We know
cosA + cosB + cosC = 3
cos0° = 1 and sec0° = 1 [cos(A – B) = cosAcosB +
A = B = C = 0° sinAsinB]
put = 0°
sinA + sinB + sinC = 0 = cos{36° + A – (36° – A)}
sin6+ cos6= (sin0°)6 + (cos0°)6
18. (a) = cos2A
=0+1
sin A 1  cos A =1 27. (c)

1  cos A sin A 23. (d) sin54 sec 46

 –7    cos 36 cos ec44
sin2 A  1  cos A 
2
cos    cos  4 –  cos  90 – 54  cosec  90 – 46 
=  2   2
sin A 1  cos A  = cos 36

cosec 44

r
= cos cos 36 cosec 44
sin2 A  1  2cos A  cos2 A 2
= cos 36  cosec 44

si
= sin A 1  cos A  =0
24. (d) =1+1
2 1  cos A 
an by  cosec A  cot A 1 – cos A  =2
= sin A 1  cos A  = 2cosecA 28. (c)
 1 cos A  4cos + 3sin = x
 1 – cos A 

n
=  
 sin A sin A  squaring both sides–
SMART APPROACH:- 16cos2+ 9sin2+ 24sincos
ja
We can easily solve this problem by putting 1  cos A   = x2 ......(1)
= 1 – cos A 
R s
the value of  = 30° sin A Again,
4sin– 3cos= y
19. (a) 1 – cos2 A
a th

= squaring both sides–


We know that– sin A 16sin 2   + 9cos 2  – 24sincos
cosec2A – cot2A = 1 sin2 A = y2 ......(2)
 (cosecA + cotA)(cosecA – cotA) = Adding equation (1) and (2), we get–
sin A
x2 + y2 = 25sin2+ 25cos2
ty a

=1 = sinA = 25(sin2+ 25cos2)


 cosecA – cotA
= 25(1)
SMART APPROACH:-
di M

1 = 25
= cosec A  cot A We can easily solve this problem by putting
the value of  = 30° SMART APPROACH:-
1 Special Case:
 cosecA – cotA = 25. (a) If Acos + Bsin = X and Asin – Bcos = Y
m Given, then –
20. (a) asin12A + bsin10A + csin8A + sin6A X² + Y² = A² + B²
=1 ....(1) Given,
cos   sin   2 cos  4cos + 3sin  = X
cosA = sin2A
squaring both side, we get– 4sin – 3sin = Y
squaring both sides:
 sin2 + cos2+ 2sincos  cos2A = sin4A
 X² + Y² = 4² + 3²
= 16 + 9 = 25
= 2cos2  1– sin2A = sin4A
 2sincos = 2cos 2  – sin 2  –  1= sin4A + sin2A 29. (c)
cos2  cubing both sides:
A

cos 4  – sin4 
 2sincos= cos2– sin2  1 = (sin4A + sin2A)3 sin2 
 2sincos = (cos – sin)(cos  1 = sin 12 A + sin 6 A + 3sin 4 A
– sin) sin2A (sin4A + sin2A)
=
 cos 2
 – sin2   cos2   sin2 
 1 = sin12A + sin6A + 3sin10A + sin2 
2sin  cos  3sin 8 A
 cos  – sin   cos2  – sin2 
cos   sin   sin 12 A + 3sin 10 A + 3sin 8 A + =
sin6A = 1 .....(2) sin2 
2sin  cos 
 cos  – sin   On comparing (1) and (2), we get cos2  sin2 
2 cos  = –
a =1, b = 3 and c = 3 sin2  sin2 
 cos  – sin   2 sin  Hence, a + b + c = 7 = cot 2  – 1

Aditya Ranjan (Excise Inspector) Selected Selection 127

Downloaded by Kunal Chauhan (kc6970929@gmail.com)


lOMoARcPSD|39315497

Trigonometry
30. (b) 33. (b)  A + B = 90
1 + sin2 – 3sincos = 0 secA = cosec B
sin A sin A
– –1 Dividing the expression by sin2 3x + 3x – 45° = 90°
cot A  cosec A cot A – cosec A
cosec2 + 1 – 3 cot = 0 6x = 135°
 1 1   (cosec2 – 1) + 1 – 3 cot + 1 = 0
= sin A  cot A  cosec A – cot A – cosec A  – 1 135
  cot2 –3cot + 2 = 0 x=
cot2 –2cot – cot + 2 = 0 6
 cot A – cosec A – cot A – cosec A 
= sin A   cot A  cosec A  cot A – cosec A   – 1 cot(cot – 2) –1 (cot –2) = 0 x = 22.5°
 
(cot – 2)(cot – 1) 39. (c)
 –2 cosec A  Hence, cot = 2 or 1
= sin A   –1 sin2 30  cos2 60 – sec 35.sin55
cot 2
A – cosec 2
A Answer (B) ?
  sec 60  cosec 30
34. (b)
–2sin A cosec A Given
1 1
= –  cosec 2 A – cot2 A  – 1 5
 – cosec 55 sin55
4
cos  – sin  4
 4 4
6 22
=2–1
5
=1   cos 2  – sin2   cos 2   sin2    1
6 –1

r
–1
SMART APPROACH:-  2 
5 4 8
  cos 2  – sin2   1 

si
We can easily solve this problem by putting
6 40. (c) sin3x = cos(3x – 45°), x = ?
the value of  = 45°

31. (a)
Given
an by  cos 2  – 1 – cos 2   
5
6
 A + B = 90°
sinA = cosB
3x + 3x – 45° = 90°

n
sin2 = cos3 5
 2cos 2  – 1  6x = 135°
Squaring both sides 6
x = 22.5
ja 35. (c)
R s
sin4 = cos6...........[1] 41. (b)
tan45° + sec60° = 1 + 2 = 3
cos 6  sin2 30  cos2 60  sec 45.sin 45
a th

cot2 – cot6= cot2– x = 3 ?


sin6  sec 60  cosec 30
36. (d)
From equation [1], 1 1 1
4(cosec257° + tan233°) – cos90° –   2
2 sin4  4 4 2
= cot  – y 
ty a

sin6  y tan266° × tan224° = 22


2
2 1 1
= cot  – 4(cosec257° – cot257°) – cos90° – 1
di M

sin2  = 2
y 4
= cot 2  – cosec 2 y tan266° cot266° =
2
3
= –  cosec  – cot   =
2 2
y 8
4–0–y=
2
=–1 sin2 52  2  sin2 38
y 42. (d) ?
32. (c) 4= +y 4 cos2 43 – 5  4cos2 47
2
Given,
3y sin2 52  cos 2 52  2
1 =4 
sin A  2 4(cos 2 43  sin2 43) – 5
2 8
 A = 30° y= 3
3   –3
Now, 37. (c) –1
A

tan A + cos A = tan 30° + cos 30° 4 – 2sin2 – 5cos = 0, 0° <  < 43. (a) If A + B = 90°
90°, cos + tan = ? tan A = cot B
1 3 Put = 60°
=  4 + – 5 = 90°
3 2
1  2 3  5 = 95°
1
23 cos + tan =  3 =   = 19°
= 2  2 
2 3 1
38. (d) 44. (d) cos2 – sin2 =
2
5 sec3x = cosec (3x – 45°), 3x is cos2 = cos60°
= angle, x = ?
2 3  = 30°

Aditya Ranjan (Excise Inspector) Selected Selection 128

Downloaded by Kunal Chauhan (kc6970929@gmail.com)


lOMoARcPSD|39315497

Trigonometry
So,
1
45. (c) cot = 3 1 (tan22 + cosec22)
3 
2 2  3 1 = (tan2 2 × 30 + cosec22 × 30)

= 60° 3 1 3 –1 = (tan2 60° + cosec2 60°)

2 2 4
2 – sin2 θ =3+
 + (cosec2 – sec) 3
1 – cos 2 θ 3 1 3 1
=  (satisfied) 13
3 –1 3 –1 =
3 3
2–
 4   4 – 2 2 2 3 53. (c)
3   sec 30° = 
3  3 3 21 tan = 20
4 find value of (1 + sin + cos) : (1 –
50. (c)
sin + cos)
3 + cos2 = 3(cot2 + sin2)
5 put  = 60° 20 P
–2 = tan =  , h = 29
 4 1 3  21 B
3 3 1
=3+ = 3
   = (1 + sin + cos) = (1 – sin +
4 4 3 4  cos)
 20 21   20 21 

r
5 2 13 13
 – 1 =  3 = 
1   
: 
1 –  

3 3 4 12  29 29   29 29 

si
46. (a) 29  20  21 29 – 20  21
13 13 :
sec107  + cos107  = 2 =  (satisfied) =
an by 4 4 29 29
put  = 0°
So, (cos + 2sin) = 70 : 30
= 1107 + 1107 = 2
= (cos60° + 2sin 60°) =7:3

n
= 1 + 1 = 2 (satisfied)
54. (c)
Now,
1 3 Given, 2sin + 15 cos² = 7
sec + cos
ja =  2
2 2 Put P = 4, B = 3, h = 5
R s
 sec0° + cos 0°
1+1 4 9
1 3 1 2 3 2 × + 15 × =7
a th

=2 =   5 25
47. (c) 2 1 2 8 27
2 + 50° + 4 + 16° = 90 51. (d)   7
5 5
= 6 + 66 = 90 tan (11) = cot (7)
35
ty a

= 6 = 24 tan(11) = tan(90° – 7)  =7


= 11 = 90° – 7 5
=  = 4°
= 18 = 90° 7 = 7 (satisfied)
48. (b)
di M

 = 5° Thus,
1 1 Now, 4 5

co se c   1 co se c  – 1 = 2 sec sin2 (6) + sec2 (9) + cosec2 (12) 3 – tan
3–
3  3  5  1

Put = 45° = sin2 (6 × 5) + sec2 (9 × 5) + cosec2 2  tan  2  4 10 10 2
(12 × 5) 3 3
1 1
+ = 2 2 = sin2 30° + sec2 45° + cosec2 60° 55. (c)
2 1 2 –1 Given,
1 2 4
2 2 = 2 2 =   125 5 H
4 1 3 cosec = 1.25 = = =
(both conditions are satisfied) 100 4 P
Then, 3  24  16 43 Here, H = 5, P = 4 & B = 3
=  4 tan– 5 cos 1
tan   2sec  12 12 
cosec  52. (c) sec 4cot– 1
A

7sin2  + 3 cos2  = 4 4 3 16
1 2 2 24 4 2 4 – 5 1 – 3 1
We have to find value of (tan22 + 3 5 3 10
  = = =
2 2 2 cosec22) 5 3 5
 4 – 1  3 – 1 11
49. (d) Let, put  = 30° 3 4 3
 7sin2 30° + cos2 30° = 4 56. (c)
cos sin 3 1 Given, sin-cos=0
 1 3
cos – sin 3 –1  7 ×  3 = 4 sin = cos
4 4
put  = 30° Putting  = 45° satisfied.
16 Hence, sin4 + cos4
cos 30 sin 30 3 1  =4
 4 1 1 2 1
cos 30 – sin 30 3 –1  4 = 4 (Satisfied) =   
4 4 4 2

Aditya Ranjan (Excise Inspector) Selected Selection 129

Downloaded by Kunal Chauhan (kc6970929@gmail.com)


lOMoARcPSD|39315497

Trigonometry
57. (c) 59. (a) 61. (d)
 tan248° – cosec242° + cosec (67° cos²– sin² = tan²
2 sin (60º – ) = 1
+ ) – sec (23° – )
(cos2– sin2)
1 = tan2 48° – cosec2 (90° – 48°) +  = sin²/cos²
1
sin (60° – ) = cosec (67° + ) – sec [90°– (67° + )]
2
= tan248° – sec248° + cosec (67° + (cos 2– sin2) sin2
 (cos 2 sin2) = cos 2
sin (60 – ) = sin 45° ) – cosec (67° + )
 60° –  = 45° = tan248° – sec248° By Componendo and Dividendo,
= – (sec248° - tan248°)
 a = 15° cos 2 (sin2 cos 2)
We know, sec  - tan  = –1
2 2  (– sin2) = (sin2 – cos 2)
58. (b)
60. (a)
3(cot2 – cos2) = 1 – sin2
cos cos (– sin2) (sin2 – cos 2)
 =4  =
3(cot  – cos ) = cos 
2 2 2
1 – sin 1  sin cos2 (sin2 cos 2)
put  = 60°
Put  = 60°
1 1 sin2 (sin2– cos2)

r
 2 =
1 1 1 2 2 cos  1
 3 –     =4

si
3 4 4 3 3 tan² = (cos² – sin²)
1– 1
2 2
3 ×
1
12
an by
=
1
4 
2 3 – 2 3
4
SMART APPROACH:-
If , then,

n
1
cos² – sin² = tan²
1 1 4 = 4 (Satisfied)
  (Satisfied) cos² -sin² = tan²
4 4 ja sec + cosec + cot
R s
2 2 1
    2 3
1
a th

3 3
ty a


di M
A

Aditya Ranjan (Excise Inspector) Selected Selection 130

Downloaded by Kunal Chauhan (kc6970929@gmail.com)


lOMoARcPSD|39315497

Height and Distance

HEIGHT & DISTANCE


19

1. A ladder leaning against a wall 3. The length of the shadow of a 6. Asha and Suman’s mud forts have
makes an angle  with the vertical pole on the ground is 18 heights 9 cm and 16 cm. If the
horizontal ground such that m. If the angle of elevation of the fort tops are at 25 cm apart from
5 sun at that time is , such that each other, then the distance (in
cosθ = . If the height of the cm) between two forts is:
13 12
cosθ = then what is the height
top of the ladder from the wall is 13 " ,=, >
18 m, then what is the distance (in m) of the pole? % &9’ cm $ 16 cm ! >

r
(in cm) of the foot of the ladder 8 2 ) 2+ 4 +" # $ 925 cm
$ +(
from the wall?
2 +18 m $ ! 5 ! ! $> + cm ’ +- . *

si
12 SSC CPO 24/11/2020 (Shift-1)
5: !  cosθ =
an by
cosθ =

5 ( 4 + m +%
13
- &. ’ $ *(c)
+ 7
(a) 24 (b) 16
(d) 25

n
13
! SSC CPO 23/11/2020 (Shift-2) 7. A ladder is resting against a wall.

ja " #18 cm
$ ( & ’ (a)
% $ 7.5 (b) 9 The angle between the foot of the
(c) 18 (d) 12 ladder and wall is 60º, and the foot
R s
(foot) ) of the ladder is 3.6 m away from
4. Asha and Suman’s mud forts have
* + , , +- . + the wall. The length of the ladder
a th

heights 9 cm and 16 cm. They are


(in m) is:
SSC CPO 23/11/2020 (Shift-1) 24 cm apart. How far (in cm) are
the fort tops from each other? ? $
(a) 19.5 (b) 13
" ; ! < ,=,
60º )
ty a

(c) 7.5 (d) 18 2 + 9 cm


% & ’ 16 $cm + 3.6 m m 2
+ +-
2. A person was standing on a road 8 24 cm + 5. +
di M

near a mall. He was 1425 m away " # +$ cm ’+ - . * +SSC CPO 24/11/2020 (Shift-2)
from the mall and able to see the
SSC CPO 23/11/2020 (Shift-2)
top of the mall from the road in (a) 5.4 (b) 3.6
such a way that the top of a tree. (a) 16 (b) 7
(c) 14.4 (d) 7.2
which is in between him and the (c) 25 (d) 24
8. Let A and B be two towers with
mall. was exactly in line of sight 5. The length of the shadow of a
same base. From the midpoint of
with the top of the mall. The tree vertical pole on the ground is 36
m. If the angle of elevation of the the line joining their feet, the
height is 10 m and it is 30 m away
sun at that time is , such that angles of elevation of the tops of
from him. How tall (in m) is the
A and B are 30º and 60º,
mall? 13
sec = then what is the height respectively. The ratio of the
/! 0 12 , 3 4 123 heights of B and A is:
(in m) of the pole?
1425
12 m A B ) 2
A

3 12 " # $ ) 2 +4 6 + 3
+ + +
+ 5 1 22 9 ’ $ 36 m 2 + !$ 5
3 2 4 A @!
6 3 " # $ 12 ! " !#$ $ 5 : !
B " # $ 5 : ! 30º
7 #, 4 + 10m 3 % & ’ 13$ 60º
sec =
12
( 6 + %&’ $B A %&’ $
! 5 / !300m . +
6 1 2m %-& .’
+ $ * *+ , +- . + SSC CPO 24/11/2020 (Shift-2)
SSC CPO 23/11/2020 (Shift-1) SSC CPO 24/11/2020 (Shift-1)
(a) 1 : 3 (b) 3 : 1
(a) 475 (b) 300 (a) 12 (b) 9
(c) 425 (d) 525 (c) 18 (d) 15 (c) 1 : 2 (d) 1 : 3

Aditya Ranjan (Excise Inspector) Selected Selection 131

Downloaded by Kunal Chauhan (kc6970929@gmail.com)


lOMoARcPSD|39315497

Height and Distance

9. A ladder leaning against a wall 12. A ladder is resting against a wall. 02 1 , 1 3


makes an angle  with the The angle between the foot of the
horizontal ground such that
6 + 5E 8
ladder and the wall is 45º and the
foot of the ladder is 6.6 m away " + P, Q, R S +
12
tan  =  If the height of the from the wall. The length of the 3 A " B 5E ( $(
5
ladder (in m) is: "’ " + + 6
top of the ladder from the wall is
24 m, then what is the distance 4 $ > +, 8> " # $ , 5:
(in m) of the foot of the ladder 45º
@! ( 2
from the wall?  PQ 
6.6 m    +  .
 RS 
2 + m $ +* - . +

SSC CPO 25/11/2020 (Shift-2)
12 SSC CPO 09/12/2019 (Shift - 01)
tan  = !
5 (a) 6.6 2 (b) 2.2 2 tan2   tan2 
" # $ 24 m% & (’ $ (a)
tan2   tan2 
(c) 3.3 2 (d) 3.6 2
) m + - .* +
SSC CPO 25/11/2020 (Shift-1) 13. A pole stands vertically on a road, cot 2   cot 2 

r
(b)
(a) 18 (b) 19.5 which goes in the north-south cot 2   cot 2 
direction P, Q are two points

si
(c) 10 (d) 7.5
towards the north of the pole, such cot 2   cot 2 
10. A person was standing on a road (c)
near a mall. He was 1215 m away that PQ = b, and the angles of cot2   cot2 
an by
from the mall and able to see the
top of the mall from the road in
elevation of the top of the pole at
P, Q are , respectively. Then the
(d)
tan2   tan2 

n
such a way that the top of a tree, height of the pole is: cot2   cot2 
which is in between him and the
D+ 3 2 + 15. DFrom
3 the (top of a house A in a
ja
mall, exactly in line of sight with street, the angles of elevation and
R s
the top of the mall. The tree height 5E 8 "P, Q ) + depression of the top and foot of
is 20 m and it is 60 m away from 5E PQ+= b, + ( another house B on the opposite
a th

him. How tall (in m) is the mall? P, Q @ " # $ side


5 :of !the street are 60° and 45°,
/ !0 12 , 3 4A 3 " ,B respectively. If the height of house
+ D+ % A+is’ 36$m, then
B what is the height
1 21215 m of house B? (Your answer should
SSC CPO 09/12/2019 (Shift - 01)
ty a

3 12 " # $ 4 be nearest to an integer)


+ ( 5 12 b’ 6 <2 + A 6 " # $ (
" (a) #tan$  tan 
di M

3 " # $ 12 7 #, 6 B :!
4 + 20 m3 % & !’ $ " # $ 5: !
5 / ! 0 60 m 6 b A " 60° B 45° + ! A
(b)
12 m&’ +$
% - .* + tan  – tan 
5 ? +36’ $ , ( B 5?+’ $
SSC CPO 25/11/2020 (Shift-1) F * 5E
b
(a) 375 (b) 300 (c) , ’ -
(c) 405 (d) 250 cot  – cot 
SSC CPO 09/12/2019 (Shift - 02)
11. Let A and B be two towers with
the same base. From the mid point b tan  (a) 91 m (b) 93 m
of the line joining their feet, the (d) (c) 94 m (d) 98 m
tan 
angles of elevation of the tops of 16. The angles of elevation of the top
A and B are 30º and 45º, 14. A clock tower stands at the of a tower from two points on the
crossing of two roads which point ground at distance 32 m and 18
respectively. The ratio of the
A

m from its base and in the same


heights of A and B is: in the north-south and the east-
straight line with it are
A B ) 2 west directions. P, Q, R and S are complementary. The height (in m)
+ + + point on +the roads) due north, of the tower is _______.
’ C @A ! B( " # $ east, south and west respectively, , 1 " # $ (@
where the angles of elevation of HI J KL J
5: ! 30º
A " 45º
B A
the top of the tower are 6 + +
B %&’ $ . respectively,
+ ,   and  Then
SSC CPO 25/11/2020 (Shift-2)
+ , 1 5?&’ $ *
2 SSC CPO 09/12/2019 (Shift - 02)
(a) (b) 1 : 3  PQ 
3 :1   is equal to: (a) 24 (b) 20
 RS 
(c) 3 : 1 (d) 1 : 3 (c) 28 (d) 16

Aditya Ranjan (Excise Inspector) Selected Selection 132

Downloaded by Kunal Chauhan (kc6970929@gmail.com)


lOMoARcPSD|39315497

Height and Distance

17. A kite is flying at a height of 123 II J 2P * + 3 "


m. The thread attached to it is
- 60° 2< $ 3 H° Q Q ,
assumed to be stretchead
straight and makes an angle of < $ 5 + 7 3
5?&’ $ * , ( + -5 " D
60° with the level ground. The . + 5 6: °" $ R<Q! 3
length of the string is (nearest to SSC CPO 12/12/2019 (Shift - 02)
a whole number): (a) 342 m (b) 364 m
SSC CPO 11/12/2019 (Shift - 02)
+< KIH J 5?&’ $ 53 (c) 384 m (d) 346 m
3 ) < ) 22 2 24. The angle of elevation of a ladder
(a) (b) 11 2
260° 3 leaning against a wall is 60° and
) < 2+ $ * $ the foot of the ladder is 6.5 m
(c) 11 (d) 11 3 away from the wall. The length of
+4! ,- .
the ladder is:
SSC CPO 11/12/2019 (Shift - 01) 21. An observer who is 1.62 m tall is
45 m away from a pole. The angle 9 4 3
(a) 140 m (b) 139 m
of elevation of the top of the pole 5: ! 60°
(c) 142 m (d) 138 m from his eyes is 30°. The height 6.5 ,
18. From a point 12m above the (in m) of the pole is closed to:

r
water level, the angle of elevation
of the top of a hill is 600 and the 1.62 2+ 2 $
! $ SSC CPO 412/12/2019
+ (Shift - 02)

si
angle of depression of the base of 45 4 3 5 (a) &4 + (b)
6.5 3 m 13 3 m
the hill is 300. What is the height 4 + " # 30° $ 5: !
an by
(in m) of the hill? (c) 12 m (d) 13 m
4 + ( 5) ?+2&<’ $
< F
2 6 KI J 5? 6 + ( (Shift - 01) 25. A ladder leaning against a window
SSC CPO 12/12/2019 of a house makes an angle of 60°

n
3 ’ , 60°
5: ! with the ground. If the distance
(a) 26.8 (b) 25.8
3 ) 30° of the foot of the ladder from the
(c) 26.2 (d) 27.6
ja
3 5?+’ $ * , +- F wall is 4.2 m, then the height of
R s
SSC CPO 11/12/2019 (Shift - 01) 22. From the top of a lamp post of the point, where the ladder
height x metres, two objects on touches the window from the
a th

(a) 36 (b) 48 3 ground is closed to:


the ground on the same side of
(c) 48 (d) 36 3 it (and in line with the foot of > 4 3 4 3
the lamp post) are observed at 60°
19. As observed from the top of a angles of depression of 30° and
ty a

4.2m. (
lighthouse 45 m high above the 60°, respectively. The distance
sealevel, the angle of depression
5 + 5?&’ $ 0!
between the objects is 32 3 m.
di M

of a ship, sailing directly towards 4 3 6 " $


it, changes from 30° to 45°. The The value of x is: SSC CPO 13/12/2019 (Shift - 01)
distance travelled by the ship (a) 7.3 m (b) 6.8 m
during the period of observation x , 5&?’ " 6 + (c) 7.8 m" # $ (d) 7( m
is: (Your answer should be correct
to one decimal place.) ? 6 A pole of length
26. 6 7 m in + fixed vertically
on the top of a tower. The angle of
M 2 NO J 5 ? & ’ *$ 2" 6 +" 6 + ) 4 +-
elevation of the top of the pole
" # $ 2 ( 2 " 6 +! < A " from
observed B a point on the ground
) 30° 60° + 6 + ’is 60° and the angle of depression
30° 45° 2 of the same point on the ground
32 3 m. x . from the top of the tower is 45°.
) ; ! < $
. * 5E " 2SSC CPO 12/12/2019 (Shift - 01) , 1 7
" m #2 +$ 93
5@ $) S 6 <
A

6 ’ - (a) 54 (b) 48
SSC CPO 11/12/2019 (Shift - 02) + 93 " # $
(c) 45 (d) 36 ! < ! 5 : 60°
! , 1
(a) 32.9 m (b) 33.4 m
23. The angle of elevation of the top " # $ 5 +
(c) 36.9 m (d) 24.8 m of a tree from a point on the 45° , 1 (m 5
) ?+& ’ $ F
ground which is 300 m away from SSC CPO 13/12/2019 (Shift - 01)
20. A 22 m long ladder (whose foot is
on the ground) leans against a the tree is 30°. When the tree
grew up, its angle of elevation of 7
wall making an angle of 60° with (a) 7(2 3 – 1) (b) ( 3  2)
the top became 60° from the same 2
the wall. What is the height (in m)
of the point where the ladder point. How much did the tree 7
(c) 7 3 (d) ( 3  1)
touches the wall from the ground? grow? 2

Aditya Ranjan (Excise Inspector) Selected Selection 133

Downloaded by Kunal Chauhan (kc6970929@gmail.com)


lOMoARcPSD|39315497

Height and Distance

27. Two points A and B are on the SSC CPO 13/12/2019 (Shift - 02) 4 + 75 m ) 48 m
ground and on opposite sides of a (a) 98.6 m 6 + +
tower. A is closer to the foot of A " B +  ! 
tower by 42 m than B. If the angles (b) 99.4 m
of elevation of the top of the tower,
+( 4 + 5?&’ $
(c) 88.2 m
as observed from A and B are 60° SSC CPO 13/12/2019 (Shift - 02)
and 45°, respectively, then the (d) 87.6 m
height of the tower is closed to: (a) 54.5 m
28. The angles of elevation of a pole
A B +6 + ( from two points which are 75 m (b) 60 m
, 1 + 8 and 48 m away from its base
(c) 61.5 m
" A, B
+ + 2 + , 1 a n d makes angle  and 
respectively. If  and  are
42 m @ , A B! (d) 50 m
complementary, then the height
2 < , 1 " #of the
$ tower
5 : is:!
A60°" B 45° +( , 1
5?&’ $ 2< < F

ANSWER KEY

r
si
1.(c) 2.(a) 3.(a) 4.(c) 5.(d) 6.(a) 7.(d) 8.(b) 9.(c) 10.(c)

11.(b)
an by
12.(a) 13.(c) 14.(b) 15.(d) 16.(a) 17.(c) 18.(c) 19.(a) 20.(d)

n
21.(d) 22.(b) 23.(d) 24.(d) 25.(a) 26.(d) 27.(b) 28.(b)
ja
R s
a th

SOLUTIONS
ty a

1. (c) 2. (a)
B AB = 132 – 122
di M

A
= (5unit)
(90–  )  12unit = 18 m
13 E Height of a pole AB (5unit)
12  ) 18
0– 10m = × 5 = 7.5 m
(9
 12
C 30m D A 4. (c)
A
1425 m
C 5 B AB = ?
16 – 9 = 7cm
5 Base 10 AB
cos θ    
13 Hypotenuse 30 1425 C E
24cm
AB = 475 m
A

3. (a) 9cm 9cm


AB = 152 – 52 = 12 A

D B
 12 = 18 m 24cm
(13)
(5)  AE = 7 cm
18 CE = 24 cm
5= 5
12  AC = ?
C (12) B
AC = 242  72 = 576  49
= 7.5 m 12 B
 cos θ  
13 H = 625 = 25 cm

Aditya Ranjan (Excise Inspector) Selected Selection 134

Downloaded by Kunal Chauhan (kc6970929@gmail.com)


lOMoARcPSD|39315497

Height and Distance

5. (d) 12. (a)


A
(2) 30°
 3 The length of the ladder AC = ?
 In a right angle triangle
(13) 60°
(5) (1)
2 45°
1
(12) 45°
C B
2 3 1
13 H 2 3
 sec θ   (1) A
12 B
30° 60°
AB = 132 – 122
A 3 C 3 B
= (5unit)
Ratio, height of B : A = 3 : 1
 (12unit) = 36 m
9. (c)
(5unit) = 15 m A
45°
Height of the pole = 15 m
6. (a) C 6.6 m B
(12)

r
A ATQ
cm 1unit = 6.6m

si
C (5) B
25 16 – 9 = 7
then,

D
an by E  tan θ 
12 P
5

B 2unit AC = 6.6 2 m

n
 (12unit) = 24 m 13. (c)
9cm 9cm
A
(5unit) = 10 m
ja 10. (c)
R s
C B
AD = 25 cm
A
a th

AE = 7 cm
DE = BC = ?
(90–  )
BC = 252 – 72  
E
ty a

= 625 – 49 B P b Q
)
= 0– 20m Let hight of the pole is x cm
576 (9
di M


= 24 cm InABQ
C 60m D B
7. (d)
1215 m
A x
AB = Height of a mall = ? = tan 
BQ
20 AB
  BQ = x cot
60 1215

1215 Similarly
AB =
3 In ABP
60° = 405 m
C 3.6 m B 11. (b) x
tan 
BP
A

Length of Ladder AC = ?
B
 cos θ  BP = x cot
H
3.6 2
2 3
3 We know,
cos60° =
AC (1)
BQ = BP + PQ
1 3.6
 A
30° 45° B x cot = x cot + b
2 AC 3
3
AC = 7.2 m
8. (b) b
x 
Ratio, Height of A : B = 1 : 3 cot– cot
 In a right angle triangle

Aditya Ranjan (Excise Inspector) Selected Selection 135

Downloaded by Kunal Chauhan (kc6970929@gmail.com)


lOMoARcPSD|39315497

Height and Distance

14. (b) Given that, 19. (a)


P
1 unit = 36 A
K Then,

3  1 unit = 36 ( 3  1 ) = 98
16. (a)
1 E
 A
O Q
O K

K O K 45° 30°
B B D 3–1 C
C D B 45


Note : When complementry are

1
  
3 – 1 = 32.9m
S O R
given in the question then, height 20. (d)
Let K is the point on the top of of tower A
the tower and the height of the = BD  BC
clock tower OK be h cm OK is Height of the tower (AB) 30°

r
perpendicular to PR and SQ.
= 32 18 = 24m
In POK, 3

si
17. (c) 2
OK
tan 

OP 
OP an by
h
A

30
°
60°

n
tan B 1 C
2 Ladder (AC)2unit = 22m
OP = h cot then,
Similarly, ja
R s
3unit(AB) = 11 3 m
In QOK
a th

OK 60° 22
tan AB =  3  11 3
OQ 2
B 1 C
21. (d)
h height (AB) = 123m (given) E
OQ 
ty a

tan
123
OQ = h cot The length of string (AC) = ×2 1
3
di M

In POQ, OP is perpendicular to
30°
OQ, then = 82 3 = 82 × 1.73 = 142 m A D
3
PQ² = OP² + OQ² 18. (c) 1.62
PQ² = h²cot² + h² cot²
E
B C
PQ² = h² (cot²+ cot²) 45
Similarly, 45
DE= 1  15 3 = 25.98
RS² = h² (cot²y + cot²) 3 × 3 =3 3
Now, 60° Height of the pole = CD + DE
A D
30° 3 = 1.62 + 25.98 = 27.6 m
 PQ 2 [h²(cot ² cot ²)]

 
  60° 22. (b)
 RS  [h²(cot ² cot ²)] 1 1 A
A

2
 PQ  (cot ² cot²)

 
  90° 30°
 RS  (cot ² cot ²)
B C
15. (d) 3
B AB = 12m (given)
2 height of the hill = (ED + CD) = (3
3 60° 30°
60° + 1) = 4unit
A 1uniti = 12
1 B 1 D 2 C

1 1 Then, Height of the lamp-post,

45°
12 32 3 96
4unit =  4 = 48m AB=  3 = 48m
1 1 2 2

Aditya Ranjan (Excise Inspector) Selected Selection 136

Downloaded by Kunal Chauhan (kc6970929@gmail.com)


lOMoARcPSD|39315497

Height and Distance


23. (d) 25. (a) 27. (b)

A A P
75°
2

3 3
3
1

60°
60° 45°
A
3
B A 1 B
C 1 B 3
3 unit = 300 1 unit(AB) = 4.2 cm Given that
Length of the tree increased by then, 3  1unit : 42
300 4.2 then,
= × 2 = 100 × 2 3 3unit AB =  3
3 1 3unit
= 200 × 1.732 = 346.4m = 4.2 × 1.73 = 7.3 m 42 3 42 3 3 1

r
26. (d) = = 
24. (d) 3 1 3 1 3 1

si
A = 63  21 3  99.4m
A

3–1
an by 28. (b)
A
2 D 3

n
3
1
ja
R s
60° C B
1
a th

C 1 B Let the pole be AD and Tower be


BD.
Given, BC = 6.5 m Given, AD = 7 m
In ABC  
 
3  1 unit = 7 m
ty a

BC C D B
cos 60° =
AC
7 When  =  = 90º then, AB
So, The tower BD= ×1
di M

 
1 6.5
2 AC
 3 –1  = BD  BC
Given, BD = 48 and BC = 75
AC = 2 × 6.5 = 13m

7
×
3 1 7
=
 3 1 m Height of the pole =
3 1 3 1 75  48
2
= 60 m


A

Aditya Ranjan (Excise Inspector) Selected Selection 137

Downloaded by Kunal Chauhan (kc6970929@gmail.com)


lOMoARcPSD|39315497

Geometry

GEOMETRY
20

1. The angles of a triangle are in A 7. The areas of two similar triangles


the ratio 1 : 1 : 2. What ABC and PQR are 64 cm 2 and
percentage of the total internal 144cm 2 , resectively. If the
angle is the greatest angle? greatest side of the smaller ABC
D is 24 cm, then what is the
greatest side of the PQR?
! %< ABC PQR
= > ?% @ 11 1 % ,%

r
B C ABC
" ! , ,
SSC CPO 09/11/2022 (Shift-01)
SSC CPO 09/11/2022 (Shift-02)

si
% , PQR , ,
(a) 50% (b) 65%
(a) 4.68 cm (b) 5.36 cm % : ,
(c) 45% (d) 40%
2.
an by
ABC and DEF are two triangles
such that ABC  FDE. If AB =
5.
(c) 3.76 cm
PQ and RS are two parallel chords
(d) 8.5 cm
SSC CPO 09/11/2022 (Shift-02)
(a) 32 cm (b) 24 cm

n
of a circle such that PQ is 48 cm
5 cm, B = 40° and A = 80°, and RS is 40 cm. If the chords (c) 42 cm (d) 36 cm
then which of the following option are on the opposite sides of the
ja 8. If two tangents to a circle of
is true? centre and the distance between
R s
radius 3 cm are inclined to each
them is 22 cm, what is the radius
ABC DEF ! other at angle of 60°, then the
(in cm) of the circle?
a th

length of each tangent is:


ABC  FDE "AB! = 5 cm, PQ RS &/0 , ! %" ! A %, 7" &
B = 40° A = 80° # $, & " ) PQ, 1 2 B - $ C
!6
% & ’ (" % , RS, 1 3 %, " ! ,& D , (" B
ty a

SSC CPO 09/11/2022 (Shift-01) ! , & , ! % )- 4,


(a) DE = 5 cm, E = 60° ,5 , !6 , %, &/0 ,
di M

SSC CPO 09/11/2022 (Shift-02)


7" 8 %, % 9 :
(b) DE = 5 cm, F = 60°
SSC CPO 09/11/2022 (Shift-02) 3 3
(c) DE = 5 cm, D = 60° (a) cm (b) 3 3 cm
(a) 25 (b) 24 4
(d) DF = 5 cm, E = 60°
(c) 35 (d) 22
3. In ABC, DE || BC in such a way 6. In the given circle with diameter (c) 3 cm (d) 6 cm
that A-D-B and A-E-C. If ACB AB, find the value of x. 9. In the following figure, if PT = 12
= 40°, then  DAE +  ADE = ! + &/0 % AB x ;" cm and PB = 24cm, then find AB
_________. % : , # $ . PT =%12 %
" ,!
ABC %DE  BC ) % C PB = 24 % , AB : ,
A-D-B A-E-C "ACB
! = 50° B A
A

40° DAE + ADE * " + P

SSC CPO 09/11/2022 (Shift-01) A


xº B
(a) 240° (b) 120°
T
(c) 140° (d) 230°
4. In the given figure, AB = 8 cm,
AC = 17 cm. What is the length
of AD? D SSC CPO 09/11/2022 (Shift-03)
SSC CPO 10/11/2022 (Shift-02)
!, +)- AB
. = 8%cm, AC =17 (a) 24 cm (b) 18 cm
(a) 40° (b) 20°
cm. AD , )- , (c) 16 cm (d) 12 cm
(c) 30° (d) 50°

Aditya Ranjan (Excise Inspector) Selected Selection 138

Downloaded by Kunal Chauhan (kc6970929@gmail.com)


lOMoARcPSD|39315497

Geometry
10. In the given figure, the circle with 13. A tangent is drawn from an ! %< %
centre O has radius 10 cm. The external point 'A' to a circle of %, 1 %, 4
radius of the circle with centre radius 12 cm. If the length of the
P is x cm. STR is a common tangent is 5 cm, then the : ,
tangent to the two circles at distance from the centre of the SSC CPO 10/11/2022 (Shift-03)
points R and S as shown in the circle to point 'A' is: (a) 6 : 13 (b) 2 : 3
figures. RT = 16 cm and TS = 24
cm. What is the value of x (in %, 7" & & / 0 (c) 9 : 4 H (d)
! 35 : 24
cm)? 'A' B - $ $ , 17.5 , If in ABC,
, ! and ACD =
AB" =AC
!, +)- .O % ! & & / 0B ,- $ , )- I 125°
% , then BAC is:
&/0
7" 3 %P ,& & / 0! , ! 'A' , !! 6 , , " !ABC %, AB =AC ACD
= 125° BAC % *" +
7x " cm STR, 5 % ! $SSC CPO 10/11/2022 (Shift-02)
! & / 0R S ! (a) 17 cm (b) 9 cm
4 " E F B RT- = 16 $ (c) 7 cm (d) 13 cm
cm TS = 24 cm x %
14. Two circles of radii 8 cm and 3
(in cm) * " c m re sp ec tive ly , ar e 13 c m
apart. AB is a direct common

r
tangent touch to both the circles SSC CPO 11/11/2022 (Shift-01)
at A and B respectively, then the (a) 75° (b) 55°

si
length of AB is: (c) 60° (d) 70°
2 %, A %, 18.
7 " Two& circles! touch & / each
0 other
an by A
?%
%,
A B
, ! 6AB, !
B -
&/0 externally. The radius of
circle with centre A is 18 cm. The
&radius
, of the second circle with
the first

n
SSC CPO 09/11/2022 (Shift-03) 4 " EF % J ," BAB - K
centre B is 8 cm. Find the length
(a) 15 (b) 16 , )- of their common tangent CD.
ja
R s
(c) 12 (d) 18 SSC CPO 10/11/2022 (Shift-03) ! &/0 !6 H
11. Two angles of a triangle are equal (a) 10 cm (b) 12 cm A & J ! & / 0 ,
a th

and the third angle measures 18 cm B J &! ! 6 & / 0


70°.What is the measure of each (c) 8 cm (d) 6 cm
78 "cm 4 , 4 " EF B
of the unknown angles? 15. In the given figure, MNP, SQP,
NQR and MSR are straight lines. CD , ) - : ,
!
ty a

NPQ = 54° and QRS = 68°, SSC CPO 11/11/2022 (Shift-01)


, 70°, % (" : what is the degree measure of
, % *" (a) 23 cm (b) 26 cm
di M

SMN?
SSC CPO 09/11/2022 (Shift-03) (c) 24 cm (d) 25 cm
!, +)- . MNP,%SQP, NQR
(a) 75° (b) 45° 19. In the given figure, DE||BC. If
MSR ,L, $ NPQ =
(c) 65° (d) 55° AD = 5 cm, DB = 10 cm, and AE
54° QRS = 68° SMN
12. AB is a common tangent to both = 8 cm, then AC is
the circles in the given figure.
+ , % :
R !, +)- .DE||BC
% "AD!
Find the distance (correct to two
decimal places) between the = 5 cm, DB = 10 cm AE = 8
centres of the two circles. cm AC % * " +
68º
!, +)- . AB %
! &/0 ,
4 " EF B - $ ) S ! &/0
! ,5 , !6 , : 180º–  Q % &
8 !
A

! ! BG , + 180º– 9

  54º
B M N P
12
X
E SSC CPO 10/11/2022 (Shift-03)
C D
5 8 (a) 29° (b) 38°
A (c) 54° (d) 68°
SSC CPO 10/11/2022 (Shift-01) 16. The perimeters of two similar
SSC CPO 11/11/2022 (Shift-02)
triangles are 36 cm and 24 cm,
(a) 18.98 units (b) 23.58 units (a) 24 cm (b) 32 cm
respectively. Find the ratio of
(c) 26.58 units (d) 21.62 units (c) 8 cm (d) 16 cm
their areas.

Aditya Ranjan (Excise Inspector) Selected Selection 139

Downloaded by Kunal Chauhan (kc6970929@gmail.com)


lOMoARcPSD|39315497

Geometry

20. AB is the common tangent to both ABC %2A = 3B = 6C ) PA PB, O J! & &/0
circles as shown in the given , % JP! $ , 5, +)- ! B
figure. What is the distance
% *" " !A B J! &/0 )
between the centre of the circles.
SSC CPO 11/11/2022 (Shift-03) BG APB = 128º OAB
AB ! &/0 , 4 " EF $
! + 5 % !$ " (a) 170°
+" (b) 90° % :
SSC CPO 23/11/2020 (Shift-1)
&/0 ! , 5(c) 80°
, !6 , (d) ,150°
24. AB is chord in a circle of radius (a) 64º (b) 72º
13 cm. From centre O, a (c) 52º (d) 38
perpendicular is drawn through 28. In ABC, BD  AC at D. E is a
AB, intersecting AB at point C. point on BC such that
The length of OC is 5 cm. What
is the length of AB? BEA = xº. If EAC = 46º and
AB, 13 cm 7" & EBD ,= 60º then the value of x
&/0
,& O AB
! AB is:
SSC CPO 11/11/2022 (Shift-02) ABC %D BD  AC BC
C ! OP !
(a) 20 cm (b) 15 cm JE ! ) BEA = xº
$ , 5 OC , 5) -
(c) 10 cm (d) 30 cm "EAC
! = 46º EBD =

r
cm AB , )- ,
21. Study the given figure and answer 60º x % :
SSC CPO 11/11/2022 (Shift-03)

si
the question that follows.
SSC CPO 23/11/2020 (Shift-1)
(a) 24 cm (b) 12 cm
!, +)- . M"" , (a) 76º (b) 68º
,5 ! an by
+ 40 ! (c)
, 20 cm (d) 15 cm
(c) 78º (d) 72º
2020 29. In a ABC the bisectors of B and

n
25. A circle is inscribed in a triangle C meet at O. If BOC = 142º then
ABC. It touches sides AB, BC and the measure of A is:
AC at points R, P and Q, ABC %B C %R!Q &
ja respectively. If AQ = 3.5 cm, PC
R s
J O! % BOC "= !
= 4.5 cm and BR=7 cm, then the
perimeter (in cm) of the triangle 142º A % :
a th

ABC is: SSC CPO 23/11/2022 (Shift-1)

ABC ! &/0 $ , (a)


5 52º (b) 68º
(c) 116º (d) 104º
Find the length of AB in the given " AB, BC AC ? %30. In ABC, A = 66º. AB and AC are
ty a

triangle, if it is given that the JR,! P Q B - " !


produced to points D and E,
length of BD is 4 unit.
AQ = 3.5 cm, PC = 4.5 cm BR respectively. If the bisectors of
AB
di M

! + ,% )- : = 7 cm ABC cm
% 8 angle CBD and angle BCE meet
, " ! " !" +BD
" % 9 : at the point O, then BOC is
, )- 1 ) )- equal to:
SSC CPO 23/11/2020 (Shift-1)
ABC %A = 66º AB AC
SSC CPO 11/11/2022 (Shift-02)
(a) 45 (b) 28 ?% D J E! S "
(a) 3 (b) 3.5
(c) 15 (d) 30 +" " ! CBD BCE
(c) 2.5 (d) 4 %R!Q & O J%!
26. One side of a rhombus is 13 cm
22. In a circle of radius 5 m, AB and and one of its diagonals is 10 cm. BOC % :
CD are two equal and parallel What is the area of the rhombus SSC CPO 23/11/2020 (Shift-2)
chords of length 8 m each. What (in cm2)? (a) 114º (b) 93º
is the distance between the
chords? , %5 - 13, cm (c) 57º (d) 66º
) 10&cm - % 5 31. ABCD
- is a cyclic quadrilateral such
A

5 m 7" & AB & /0 % that AB is a diameter of the circle


CD, ( "8 m )- , ! % = cm²> ’% 8 9 :
and ADC =148º. What is the
% ,& N ,SSC
& CPO 23/11/2020
, 5 (Shift-1)
measure of the BAC ?
, !6 , *" (a) 30 (b) 60
SSC CPO 11/11/2022 (Shift-03)
) - 5 ? , " ABCD
5 / ) -
(c) 120 (d) 90
(a) 5 (b) 6 AB & / 0 ; " ADC =
27. PA and PB are two tangents from
(c) 3 (d) 8 148º BAC % :
a point P outside the circle with
23. In ABC, 2A = 3B = 6C. What centre O. If A and B are points SSC CPO 23/11/2020 (Shift-2)
is the value of the largest angle on the circle such that APB = (a) 60º (b) 45º
among these three angles? 128º then OAB is equal to: (c) 58º (d) 32º

Aditya Ranjan (Excise Inspector) Selected Selection 140

Downloaded by Kunal Chauhan (kc6970929@gmail.com)


lOMoARcPSD|39315497

Geometry

ar   A B C  36. ABCD is a cyclic quadrilateral such 40. In ABC, A = 68º. If I is the


32. Let ABC  RPQ and . that AB is a diameter of the circle incentre of the triangle, then the
ar   R P Q 
and ADC = 118º. What is the measure of BIC is:
4
= If AB = 3 cm, BC = 4 cm and measure of BAC ?
9 ABC %A = 68º "I !
AC = 5 cm, then PQ (in cm) is ABCD 5?," 5 - J ! BIC , % :
equal to: AB &/0 ; " ADC = 118º
ar   A B C 
% ABC  RPQ . BAC % :
ar   R P Q  SSC CPO 24/11/2020 (Shift-2)
SSC CPO 24/11/2020 (Shift-1)
4
= AB
" != 3 cm, BC = 4 cm (a) 32º (b) 28º (a) 124º (b) 68º
9
AC = 5 cm PQ % (c) 45º (d) 30º (c) 148º (d) 54º
8cm % 9 : 37. Let  ABC ~  RPQ and
41. In ABC, D is the median from A
SSC CPO 23/11/2020 (Shift-2)
ar  ABC  4 to BC. AB = 6 cm, AC = 8 cm, and
(a) 12 (b) 6 = If AB = 3 cm BC = BC = 10 cm. The length of median
ar  RPQ 9
(c) 4.5 (d) 5 AD (in cm) is:
33. PA and PB are two tangents from 4 cm and AC = 5 cm, then RP (in
a point P outside the circle with cm) is equal to: ABC %D, A BC % M"

r
centre O. If A and B are points %  ABC ~  RPQ AB = 6 cm, AC = 8 cm, BC =

si
on the circle such that APB = 10 cm % ADM " , cm)- 8
100º then OAB is equal to: ar  ABC  4
AB
" != 3 cm, = % 9 :
PA an by
PB, O
J! & & / 0 ar  RPQH 9
JP! $ , 5, +)- ! B BC- = 4 cm
$ SSC CPO 24/11/2020 (Shift-2)
AC = 5 cm RP

n
" ! &/0 A B
J!) 8cm % 9 % : (a) 4.5 (b) 5
B G APB = 100º OAB SSC CPO 24/11/2020 (Shift-1)
% : ja (c) 4 (d) 3
(a) 12 (b) 6
R s
SSC CPO 23/11/2020 (Shift-2) 42. PA and PB are two tangents from
(c) 5 (d) 4.5
(a) 50º (b) 35º a point P outside the circle with
a th

38. Chord AB of a circle is produced centre O. If A and B are points


(c) 45º (d) 70º to a point P, and C is a point on on the circle such that APB =
34. In ABC, AB and AC are produced the circle such that PC is a tangent 142º, then OAB is equal to:
to points D and E, respectively. to the circle. IF PC = 12 cm, and
ty a

If the bisectors of CBD and BCE BP = 10 cm, then the length of AB PB, O J! & &/0 PA
meet at the point O. and BOC = (in cm) is:
57º, then A is equal to:
JP! $ , 5, +)- ! B
di M

, &/0 AB
, ,& PJ! " !A B J! &/0 )
ABC %AB AC ?% J!
S " C &/0 J! ) BG APB = 142º, OAB
D E S " CBD " !
B GPC & / 0 , B - $% :
BCE %!Q & O J!
PC
" != 12 cm, BP = 10
% BOC = 57º A SSC CPO 25/11/2020 (Shift-1)
cm AB , cm
)- %8 9 :
% :
(a) 58º (b) 31º
SSC CPO 24/11/2020 (Shift-1)
SSC CPO 24/11/2020 (Shift-2) (c) 71º (d) 64º
(a) 93º (b) 57º
(a) 5.4 (b) 6
(c) 66º (d) 114º 43. In a ABC, the bisector of B and
35. PA and PB are two tangents from (c) 5 (d) 4.4 C meet at O in the triangle. If
a point P outside the circle with 39. PQRS is a cycle quadrilateral. If BOC = 134º, then the measure
centre O at the points A and B P is 4 times R, and S is 3 times of A is:
A

on it. If APB = 130º, then OAB Q, then the average of Q and
is equal to: ABC %B C
R is:
O J! & , & / 0P ,
J! % T O % % J!
&/0 AB G B J! PQRS 5 ? , " 5 -P, R " !
"BOC
! = 134º A
$ , 5 , + ) - ! PA B PB - $ N4 + S, Q 3 +
% :
" !APB = 130º OAB Q R :
% : SSC CPO 25/11/2020 (Shift-1)
SSC CPO 24/11/2020 (Shift-2)
SSC CPO 24/11/2020 (Shift-1)
(a) 40.5º (b) 45.7º (a) 116º (b) 88º
(a) 50º (b) 35º
(c) 65º (d) 45 (c) 90º (d) 81º (c) 52º (d) 104º

Aditya Ranjan (Excise Inspector) Selected Selection 141

Downloaded by Kunal Chauhan (kc6970929@gmail.com)


lOMoARcPSD|39315497

Geometry
44. A circle is inscribed in a triangle 48. In a circle with centre O, AD is a 52. D is a point on the side BC of a
ABC. It touches side AB, BC and diameter and AC is chord. Point ABC such that ADC = BAC.
AC at points R, P and Q, B is on AC such that OB = 7 cm If CA = 10 cm and BC = 16 cm,
respectively. If AQ = 2.6 cm, PC and OBA = 60º. If DOC = 60º, then the length of CD is:
= 2.7 cm and BR = 3 cm, then the then what is the length of BC?
perimeter (in cm) of the triangle ABC BC
, D !)
ABC is: O J! & AD &; /"0 %
AC ,& B, ACJ ! ) B GADC = BAC
ABC ! &/0 $ , 5 " CA
! = 10 % , BC = 16 %,
BG OB = 7 cm OBA =
" AB, BC AC ?% CD , )- , +,
60º DOC
" ! = 60º BC
JR,! P Q B - " !
, )- : SSC CPO 09/12/2019 (Shift-02)
AQ = 2.6 cm, PC = 2.7 cm BR
SSC CPO 25/11/2020 (Shift-2)
= 3 cm ABC %cm 8 (a) 6.25 cm (b) 6.5 cm
% 9 : (a) 3 7 (b) 5 7
(c) 7 cm (d) 6 cm
SSC CPO 25/11/2020 (Shift-1) (c) 7 cm (d) 3.5 cm
53. P is a point outside a circle with
(a) 28 (b) 30 49. The sides PQ and PR of PQR are centre O, and it is 14 cm away
produced to points S and T, from the centre. A secant PAB
(c) 16.6 (d) 33.2 respectively. The bisectors of

r
drawn from P intersects the circle
SQR and TRQ meet at U. If at the point A and B such that PA
45. In ABC, BD  AC at D. E is a
QUR = 59º, then the measure

si
point on BC such that BEA = xº. = 10 cm and PB = 16 cm. The
of P is: diameter of the circle is:
If EAC = 62º and EBD = 60º,

ABC
an by
then the value of x is:

%D BD  AC
PQR
JS!
, PQ
T J !
N PR, ? %
S )- ,
OV &
BG
&/0
" V
P
1

n
SQR TRQ %!Q &
BC EJ ! ) P PAB OP ! $
JU! % QUR =" 59º
!
BEA = xº EAC
" ! = 62º , & /A0 B ) !
ja P , % :
O P !PA = 10 , % ,
R s
EBD = 60º, x % : SSC CPO 25/11/2020 (Shift-2)
PB = 16 %, &/0 ;
SSC CPO 25/11/2020 (Shift-1) (a) 41º (b) 31º
a th

,
(a) 78º (b) 68º (c) 49º (d) 62º
SSC CPO 09/12/2019 (Shift-02)
(c) 76º (d) 92º 50. In a ABC, angle BAC = 90°. If BC
= 25 cm. then what is the length (a) 10 cm (b) 13 cm
ty a

46. Two circles of radii 15 cm and 10


of the median AD?
cm intersect each other and the (c) 12 cm (d) 11 cm
length of their common chord is ABC % BAC = 90° BC
" != 54. I is the incentre of ABC. If BIC
di M

16 cm. What is the distance (in


25 %, AD %, M " ) - : = 108°, then A = ?
cm) between their centres?
, ABC J I V "BIC
! =
15 cm 10 cm , & 7" !
SSC CPO 09/12/2019 (Shift-01) 108° A = ?
&/0 R!6 U 4 ,
4 " EF , & 16 ,cm ) - (a) 10 cm (b) 12.5 cm SSC CPO 09/12/2019 (Shift-02)
4 J! cm , 5
% , 9 ! 6: ,(c)814.5 cm (d) 24 cm (a) 72° (b) 65°
51. A circle touches the side BC of a
(c) 81° (d) 36°
SSC CPO 25/11/2020 (Shift-2) ABC at P and also touches AB
and AC produce at Q and R, 55. In ABC, C = 90° and CD is per-
(a) 15+2161 (b) 12 + 37
respectively. If the perimeter of pendicular to AB at D. If AD : BD
(c) 6 + 161 (d) 10 + 161 = k , then AC : BC = ?
ABC = 26.4 cm, then the length
A

47. In ABC, A = 54º. If I is the of AQ is: ABC %C = 90° CD AB


incentre of the triangle, then the
measure of BIC is:
& ABC
/0 , BC P ! D, ABN ! " !
B - AB AC
G , " AD : BD =k
ABC %A = 54º "I !4 AC : BC = ?
S ?Q% R !
 BIC
J! , SSC CPO 11/12/2019 (Shift-01)
B - ABC " ! 26.4
%
% :
cm AQ , )- (a) k (b) k
SSC CPO 25/11/2020 (Shift-2)
SSC CPO 09/12/2019 (Shift-02)
(a) 54º (b) 148º 1
(a) 8.8 cm (b) 13.2 cm (c) (d) 4
k
k
(c) 68º (d) 117º (c) 15.4 cm (d) 17.6 cm

Aditya Ranjan (Excise Inspector) Selected Selection 142

Downloaded by Kunal Chauhan (kc6970929@gmail.com)


lOMoARcPSD|39315497

Geometry

56. ABC is an equilateral triangle in 60. The sides AB, BC and AC of a 64. ABCD is cyclic quadrilateral.
which D, E and F are the points ABC are 12 cm, 8 cm and 10 cm Sides AB and DC, when produced,
on sides BC, AC and AB, respec- respectively. A circle is inscribed meet at E, and sides BC and AD,
tively, such that AD  BC, in the traingle touching AB, BC when produced, meet at F. If
and AC at D, E and F, respectively. BFA = 60° and AED = 30°, then
BE  AC and CF  AB. Which of the measure of ABC is:
The difference between the length
the following is true?
of AD and CE is: 5?," 5 - AB
ABCD
ABC % D, E %
ABC , AB, BC) AC + S DC "
F ? % BC, AC AB
?% %, 2 %, & E 3 ! %% , , )
! AD  BC, BE  AC
, &/0 "BC AD + S "
CF  AB # $ % F ! % , BFA = "60°
!
AB, BC AC D, E
?% F
& ’ ,
B - AD CE , ) - AED = 30° ABC % *"
SSC CPO 11/12/2019 (Shift-01)
,5 +
(a) 7AB2 = 9AD2 (b) 2AB2 = 3AD2
(c) 4AC2 = 5BE2 (d) 3AC2 = 4BE2 SSC CPO 11/12/2019 (Shift-02) SSC CPO 12/12/2019 (Shift-01)
57. A line touches a circle of radius 6 (a) 65° (b) 75°
(a) 4 cm (b) 5 cm
cm. Another line is drawn which (c) 70° (d) 80°
(c) 3 cm (d) 2 cm

r
is tangent to the circle. If the two 65. In ABC, C = 900 and D is a
lines are parallel, then the dis- 61. In ABC, A = 90°, AB = 16 cm and
point on CB such that AD is the
AC = 12 cm. D is the midpoint of

si
tance between them is: bisector of A. If AC = 5 cm and
$ @7" %, & AC and&DE / 0 CB at E.
B What- is the BC = 12 cm, then what is the
area (in cm²) of CDE?
,
, B
an by
- $ " ! !
J"
$ , 5, $ ,
$ABC, %A
AC = 12 %
&/0
% = 90°, %AB = 16
AC
, % M D
"
%,
!
ABC %C = 90° D, CB
length of AD?

n
4 ,5 , !6 , , +, ! AD, A
SSC CPO 11/12/2019 (Shift-01) E DE  CB CDE %!Q & AC = 5 cm" ! BC
(a) 6 cm ja
(b) 12 cm = (cm²
>%) : , = 12 cm AD , )- ,
R s
(c) 8 cm (d) 10 cm SSC CPO 11/12/2019 (Shift-02) SSC CPO 12/12/2019 (Shift-01)
58. PT is a tangent at the point R on (a) 8.64 (b) 7.68
a th

a circle with centre O. SQ is a 10 5 13


(c) 5.76 (d) 6.25 (a) cm (b) cm
diameter, which when produced 3 6
meets the tangent PT at P. If 62. The sides of ABC are 10 cm, 10.5
SPT = 32°, then what will be the cm and 14.5 cm. What is the 5 13 20
ty a

measure of QRP? radius of its circum circle? (c) cm (d) cm


3 3
JOV & &/0 R ,B G ABC ! , 3 %, 3WI %,
66. A circle is inscribed in a
di M

B -PT $ SQ ; " +1 W I %, ) & / 0equilateral


, 7" ,
triangle of side 24 cm.
S " PTB - P $! )- : , What is the area (in cm²) of a
% , SPT
" =! 32° QRP SSC CPO 11/12/2019 (Shift-02) square inscribed in the circle?
% *" + (a) 5 cm (b) 7.5 cm 24 cm , & %
SSC CPO 11/12/2019 (Shift-01) (c) 5.25 cm (d) 7.25 cm &/0 " +" 4 , &/0
(a) 32° (b) 58° 63. P is a point outside a circle and = (cm²>%
) : ,
(c) 30° (d) 29° is 26 cm away from its centre. A
59. Two circles of radii 5 cm and 3 cm secant PAB drawn from P SSC CPO 12/12/2019 (Shift-01)
interest each other at A and B, intersects the circle at points A
and the distance between their (a) 48 (b) 72
and B such that PB = 32 cm and
centres is 6 cm. The length (in PA = 18 cm. The radius of the (c) 96 (d) 54
cm) of the common chord AB is: circle (in cm) is: 67. In ABC, AB = c cm, AC = b cm and
A

I %, A
%, & / 0& , 7"
! P &B/ 0G ! CB = a cm. If A = 2 B, then which
R!6 A B OP
! &/0 26 cmV , ! 6 , P of the following is true?
4 V ,5 ,
! 6 , @$ , % 5 ,, + ) - 4 " PABE FO) P &! / 0 $
ABC %AB = c cm, AC = b cm
, &AB , )- 8 %,
% 9 : ,A ! B U PB, CB = a cm "A! = 2 B
SSC CPO 11/12/2019 (Shift-02)
= 32 cm PA = 18 cm &/0 # $ % &
4 13 2 14 , (cm
7" %) : ,
(a) (b) SSC CPO 12/12/2019 (Shift-01)
3 3
SSC CPO 12/12/2019 (Shift-01)
(a) a² = b² – bc (b) a² = b² – ac
2 13 4 14 (a) 12 (b) 10
(c) (d) (c) a² = b² + bc (d) a² = b² + ac
3 3 (c) 13 (d) 8

Aditya Ranjan (Excise Inspector) Selected Selection 143

Downloaded by Kunal Chauhan (kc6970929@gmail.com)


lOMoARcPSD|39315497

Geometry

68. The tangent at a point A on a P 75. AB is a chord in a circle with


circle with centre O intersects the centre O. AB is produced to C
diameter PQ of the circle, when such that BC is equal to the
extended, at point B. If BAQ = radius of the circle. C is joined to
105°, then APQ is equal to: O and produced to meet the circle
O JV & &/0A B GQ ! at D. If ACD = 32°, then the
T S R measure of AOD is.
$ , 5 , + ) - BPQ - $B ! ; " SSC CPO 13/12/2019 (Shift-01)
O P ! BAQ, = 105°
" ! OV & &AB/ 0 %, &
(a) 42° (b) 39°
APQ % *" + AB C ) S "
(c) 30° (d) 40° BC, & / 0 , 7" C
A
72. ABCD is a cyclic quadrilateral in O % " D &/0
which AB = 15 cm, BC = 12 cm and % ) + S "
Q CD = 10 cm. If AC bisects BD,
B P O ACD = 32° AOD %
then what is the measure of AD?
SSC CPO 13/12/2019 (Shift-02)
ABCD 5?," 5 -AB = %
SSC CPO 12/12/2019 (Shift-02) 15 cm, BC = 12 cm CD = 10 cm (a) 48° (b) 108°

r
(a) 55° (b) 60° "AC,! BD %!Q & ,
(c) 80° (d) 96°
(c) 65° (d) 75° AD % : ,

si
69. Two equilateral triangles of 76. ABCD is a cyclic quadrilateral.
SSC CPO 13/12/2019 (Shift-01) The tangents to the circle at the
side 10 3 cm are joined to form a
an by points A and C on it, intersect at
(a) 15 cm (b) 13.5 cm
quadrilateral. P. If ABC = 98°, then what is the
What is the altitude of the (c) 18 cm (d) 20 cm measure of APC?

n
quadrilateral? 73. In ABC, C = 90°. M and N are
ABCD 5?," 5 - &/
! % ,the midpoints, of %
sides AB and AC,
ja respectively. CM and BN intersect A C , B - $ N
R s
10 3 %, each5 other at- D and BDC = 90°. P O P ! ABC
, = 98° " !
" ) 5 - , 4 > N= 58 cm,
If BC ) - then the
, length of APC % *" +
a th

BN is:
SSC CPO 13/12/2019 (Shift-02)
SSC CPO 12/12/2019 (Shift-02) ABC %C = 90° M ?% N
(a) 12 cm (b) 14 cm AB AC BG %M" (a) 14°
! (b) 22°
ty a

(c) 16 cm (d) 15 cm CM BN !6 D ! (c) 16° (d) 26°


70. From a point P which is at distance OP ! BDC = 90°
77. In ABC, AB = AC and D is a point
di M

of 10 cm from the centre O of a " !BC = 8 cm BN , ) - * " on side AC such that BD = BC. If
circle of radius 6 cm, a pair of
tangents PQ and PR to the circle
+, AB = 12.5 cm and BC = 5 cm, then
at point Q and R respectively, are what is the measure of DC?
SSC CPO 13/12/2019 (Shift-01)
drawn. Then the area of the ABC %AB = AC D, AC
quadrilateral PQOR is equal to (a) 6 3 cm (b) 6 6 cm
) BD!= BC
@ %, 7" & O 3& / 0 V " AB
! = 12.5 cm BC = 5 cm
(c) 4 6 cm (d) 8 3 cm
%, , !6 , PB G & / 0 ! DC , % : ,
! Q! R ! B - 74. $ Two ? chords
% AB and CD of a circle
intersect each other at P SSC CPO 13/12/2019 (Shift-02)
PQ PR $ , 5 , , 5 -
internally. If AP = 3.5 cm. PC = 5
PQOR = > +
cm and DP = 7 cm, then what is (a) 2.5 cm (b) 3 cm
SSC CPO 12/12/2019 (Shift-02) the measure of PB?
(c) 1.8 cm (d) 2 cm
A

(a) 30 sq.cm (b) 40 sq.cm


&/0 % AB ! CD , & R ! 6 78. I is the incentre of ABC of A =
(c) 24 sq.cm (d) 48 sq.cm
< P ! OP ! then BIC = ?
46°,
71. In the figure, in PQR, PT  QR
, AP =
" 3.5
! cm, PC = 5 cm
at T and PS is the bisector of " A
! = 46° & ABC %
I J JV
QPR. If PQR = 78° and TPS = DP = 7 cm PB , % :
BIC = ?
24°, then the measure of PRQ is: ,
. PQR
% %T PT  QR SSC CPO 13/12/2019 (Shift-02)
SSC CPO 13/12/2019 (Shift-01)
PS QPR %!Q & " ! (a) 113° (b) 124°
(a) 8 cm (b) 12 cm
PQR = 78° TPS = 24°
(c) 10 cm (d) 10.5 cm (c) 134° (d) 93°
PRQ % : ,

Aditya Ranjan (Excise Inspector) Selected Selection 144

Downloaded by Kunal Chauhan (kc6970929@gmail.com)


lOMoARcPSD|39315497

Geometry

ANSWER KEY
1.(a) 2.(a) 3.(c) 4.(c) 5.(a) 6.(a) 7.(d) 8.(b) 9.(b) 10.(a)

11.(d) 12.(b) 13.(d) 14.(b) 15.(a) 16.(c) 17.(d) 18.(c) 19.(a) 20.(d)

21.(d) 22.(b) 23.(b) 24.(a) 25.(d) 26.(c) 27.(a) 28.(a) 29.(d) 30.(c)

31.(c) 32.(b) 33.(a) 34.(c) 35.(c) 36.(b) 37.(d) 38.(d) 39.(a) 40.(a)

41.(b) 42.(c) 43.(b) 44.(c) 45.(d) 46.(c) 47.(d) 48.(c) 49.(d) 50.(b)

51.(b) 52.(a) 53.(c) 54.(d) 55.(d) 56.(d) 57.(b) 58.(d) 59.(d) 60.(a)

61.(a) 62.(d) 63.(b) 64.(b) 65.(c) 66.(c) 67.(c) 68.(d) 69.(d) 70.(d)

r
71.(c) 72.(c) 73.(c) 74.(c) 75.(d) 76.(c) 77.(d) 78. (a)

si
an by
SOLUTIONS

n
ja
R s
1. (a) 3. (c)
AB2
a th

Sum of all 3 angles of a triangle AD 


AC
= 180°
x + x + 2x = 180° 88
 AD 
4x = 180° 17
ty a

 x = 45°  AD  3.76 cm
Largest angle = 90° 5. (a)
di M

90 From figure,
 100%  50%
180 R N S
SMART APPROACH:- ACB = AED = 40
Angles’s ratio = 1 : 1 : 2
Largest angles = 2 units In ADE,
Total Angles = 4 units DAE + ADE + AED = 180º O
Largest angle : Total Angles = 2 : 4
Hence, The largest angle is DAE + ADE = 180º – AED
50% of the total angles
DAE + ADE = 180º – 40º
2. (a) DAE + ADE = 140º
Given, P M Q
4. (c)
A  80º, B  40º By the property of R – A – T, OP = OS = radius
A

and ABC  FDE Given,


So that, PQ = 48 cm
RS = 40 cm
A  F, B  D and C  E
Distance between centre, MN =
C  180 –   A B  22 cm
 C  180  – 120  Let OM = x cm
ON = MN – OM = 22 – x
C  60
We know that, perpendicular from
 C  E  60  the center of the circle bisects
and AB = FD = 5 cm the chord
Hence, Option (a) is correct.

Aditya Ranjan (Excise Inspector) Selected Selection 145

Downloaded by Kunal Chauhan (kc6970929@gmail.com)


lOMoARcPSD|39315497

Geometry

PQ 48 8. (b)  TOR  TPS


PM = QM =   24cm Given, radius = OA = 3 cm
2 2 by the AAA similarity,
ORT  PST
RS 40
RN = NS =   20 cm
2 2 OR RT
 
Now, PS ST

 PM2  OM2  NS2  ON2 10 16


 
 24 + x = 20 + (22 – x)
2 2 2 2 x 24
 576 + x2 = 400 + 484 – 44x + x2 By the property of circle,  x = 15 cm
 576 – 884 = – 44x
 –308 = – 44x APB 11. (d)
OPA 
 –308 = – 44x 2 Let the all three angles of
 x = 7cm triangles a, b and c.
60º
Hence, radius (OP) OPA   30º According to questions, one angle
2
is 70 degree and other two angle
 PM2  OM2  242  x 2 In OAP, is equal.
OA Hence, a = b = x and c = 70º

r
 242  72  576  49  625 tan P 
AP We know that, sum of all three
= 25 cm
angles of a triangle is 180º

si
3
6. (a)  tan30º 
AP a + b + c = 180º
C
an by 1 3  x + x + 70º = 180º
 
50° 3 AP  2x = 180º – 70º
 2x = 110º

n
 AP  3 3 cm
 x = 55º
A B Unknown angle = 55°
xº ja 9. (b)
R s
12. (b)
a th

In figure,
B
D 12
ATQ, X

AB is a diameter of the circle E


ty a

C D
then, DCB = 40°
5 8
ATF, BD is a chord of the circle
We know that, PT2 = PA × PB
di M

A
then, DAB = DCB = 40º
 122 = PA × 24 CAE = DBE
7. (d)  144 = PA × 24
Let the greatest side of PQR is AEC = BED
 PA = 6
x cm ECA = EDB
 AB = PB – PA
Given, = 24 – 6 Hence,  CAE  DBE
ABC  PQR = 18 cm CA AE

By the property of similarity, 10. (a) DB BE
Ratio of sides of ABC Given,
 5 8
Ratio of sides of PQR  
x 12
arABC  x = 7.5

arPQR In  CAE
A

24 64 CE  AC2  AB2
 
x 144 = 52  82  89 = 9.43
RT = 16 cm
24 8 Again in  DBE
  TS = 24 cm
x 12 OR = 10 cm DE  BD2  BE2 = 7.52  122
12  24 PS = x cm
= 56.25  144 = 200.25 = 14.15
 x  = 36 cm In figure,
8 Distance between Centre,
ORT  PST  90 CD = CE + DE
Length of the greatest side of the
PQR = 36 cm. OTR  PTS = 9.43 + 14.15 = 23.58 units

Aditya Ranjan (Excise Inspector) Selected Selection 146

Downloaded by Kunal Chauhan (kc6970929@gmail.com)


lOMoARcPSD|39315497

Geometry
13. (d) 2 = 112° – 54° 19. (a)

58 Since, DE  BC in ABC


 =
2 AD AE

 = 29° DB EC
Therefore, RMN = SMN = 29°
5 8
 
SMART APPROACH:- 10 EC
Just remember this direct result
B  EC = 16 cm
x = 90 – ( + ) A x
2
P

AC = AE + EC
C  D
= 8 + 16 = 24 cm
OA  OB2  AB2 Q
20. (d)
16. (c)
= 122  52 Ratio of areas of triangles
= 144  25 = 169 = 13 cm 2
 36  9
=  
 24  4
14. (b)
Thus, Ratio = 9 : 4

r
17. (d)
In ACE
Given, AB = AC

si
CE  42  32 = 25 = 5 cm

AB 
an by
 DBC  –  R – r 
2 2


CAE  DBE
CE AE

n
DE EB
132 –  8 – 3 
2
=
Hence, ABC  ACB 5 3
ja  
R s
= 169 – 25 = 144 = 12 cm By the linear pair DE 15
15. (a) ACB  ACD  180  DE  25 cm
a th

R
ACB  180 – ACD Distacne between Center, CD
= 180° – 125° = 55° = CE + ED = 5 + 25 = 30 cm
68º 21. (d)
By the exterior angle.
ty a

ABC  BAC  125 Given,


S

180º–  Q  55  BAC  125


di M

180º– 

 BAC  125 – 55 = 70º
 54º
M N P 18. (c)
AC = R = 18 cm
Let RMN =  BD = r = 8 cm
and MNR =  AB = 18 + 8 = 26 cm
We know, sum of opposite angles
in cyclic quadrilateral is 180°
So, PSM = 180° –  BD = 4 unit
Let AB = x unit
In RMN,
AD = AB + BD
 RMN + MNR + MRN = = x + 4 unit
180° ABC  ADE
A

 + + 68° = 180°


AB BC
 + = 112° ..........(1)  
AD DE
DCT, CD  AB  2
– R – r 
2

Again, in PSM, x 3
 
AB2 –  AC – BD
2
PSM +SMP + MPS = 180° = x 4 6
 180° –  +  + 54° = 180°  6x = 3x +12
262 – 18 – 8 
2
=  6x – 3x =12
  + 54° = 
=  3x = 12
From equation(1) 676 – 100
x = 4 unit
+  + 54° = 112° = 576 = 24 cm Hence, Length of AB = 4 unit.

Aditya Ranjan (Excise Inspector) Selected Selection 147

Downloaded by Kunal Chauhan (kc6970929@gmail.com)


lOMoARcPSD|39315497

Geometry
22. (b) 26. (c)
k 180
B    60
3 3
D 13cm C
k 180
C    30
6 6
5
The value of largest angle = 90° o
13cm 13cm
OR 10
Using Ratio– 5
Given, Radius = 5 m 2A  3B  6C A 13cm B
AB = CD = 8 m
We know that, 2A 3B 6C In a rhombus two diagonals
  
Perpendicular from the center of 36 36 36 intersect each other at 90° in
the circle bisects the chord. equal parts.
A B 3
  
AB
Hence, AM 
18 12 6 O
2
Largest Angle,
8m

r
= =4m A  180 
18 5
2 18  12  6

si
AB 8m = 90°
And, CN   =4m
2
an by 2 24. (a) A 13 B
In AMO, OM2 = OA2 – AM2 In ACO,
= 52 – 4 2 = 9 In a triangle AOB

n
 OM  3 m AO = 132 – 52
ja
In CNO, ON = OC – CN
2 2 2
R s
= 52 – 4 2 = 9 = 12 cm
 ON = 3 m  AO = 12 cm So, OC = 12 cm
a th

Thus, The distance between


center = OM + ON AC = 24 cm
= 3 + 3 = 6m
OR 1
Area of a rhombus = ×BD× AC
ty a

Distance between center 2 2 2


AC  AO – OC
di M

2
2 8  132 – 52 1
= 2  5 –   = 2 5 2 – 42 =
2
× 10 × 24
2
 169 – 25
= 2 25 – 16 = 2 9 =6m = 120 cm2
 144 = 12 cm
23. (b) 27. (a)
AB = 2 × AC = 24 cm
Let 2A  3B  6C  k 25. (d)
k k k A
A  B  C  A
2 3 6
We know that, 3.5cm O 128°
A  B  C  180 P
R Q
k k k
    180
B
A

2 3 6
7cm
3k  2k  k  AOB + APB = 180°
  180
6 B C
P 9.5cm AOB = 180° – 128° = 52°
6k
  180 Perimeter of ABC = 2(AQ +PC +
6 180 – 52
BR) OAB = = 64º
 k = 180° 2
Thus, = 2(3.5 + 4.5 + 7)
k 180 = 2 × 15
A    90
2 2 = 30 cm

Aditya Ranjan (Excise Inspector) Selected Selection 148

Downloaded by Kunal Chauhan (kc6970929@gmail.com)


lOMoARcPSD|39315497

Geometry

28. (a) 34. (c)


B 66 A
= 90° –
60° 2
= 90° – 33°
= 57° B C
x° E
31. (c) 57°
P
D o E
46° o
A B BOC = 57°
A D C 32°
P A = ?
148° A
D C BOC = 90° –
2
46° A
A D  ADC = 148° = 90° – BOC
2
B ABC = 180° – 148° = 32°
A
BAC = 90° – 32° = 58° = 90° – 57°
60° 2
32. (b) If ABC ~ RPQ
x° A = 66°
44°

r
P E
AB BC AC 35. (c)
Then,  
 APD = BPE RP PQ RQ

si
A
In Triangle APD
APD = 44° ar  ABC 
BPE = 44°
BEP = x°
an by =
ar  RPQ 
O 130° P

n
B
x + 44° + 60° = 180° ar  ABC  BC 2
  AOB = 180° – 130° = 50°
x = 76° ar  RPQ  PQ 2
ja In OAB
R s
29. (d) A 4 16 AOB = 50°
 [ BC = 4cm]
9 PQ 2 
a th

130
OAB = = 65°
PQ2 = 36 2
o PQ2 = 62  OA = OB = radius of circle
PQ = 6 cm 36. (b)
ty a

142°
33. (a)
B C
di M

A
o
A B
A 100° 62°
BOC = 90° + O P
2 118°
A D C
142° = 90° + B
2
A ABC = 180° – 118° = 62°
= 52°  APB = 100° BAC = 90° – 62°= 28°
2
APB + AOB = 180° 37. (d)
A = 104°
AOB = 180° – 100 = 80°  ABC ~ RPQ
30. (c)
A In OAB -
AB BC AC ar  ABC 
66° A   =
ar  RPQ 
A

RP PQ RQ
B C
4 AB

O 80° 9 RP
D o E
2 3
 [ AB = 3cm]
A = 66° 3 RP 
BOC = ? 9
B RP =
A 2
BOC = 90° – OA = OB = radius of circle
2 OAB = 50° RP = 4.5 cm

Aditya Ranjan (Excise Inspector) Selected Selection 149

Downloaded by Kunal Chauhan (kc6970929@gmail.com)


lOMoARcPSD|39315497

Geometry

38. (d) 44. (c)


41. (b)
C A
12 cm
A

A 6 cm 2. 6cm
B 10cm P 8 cm

R Q
 PC = 12 cm
PB = 10 cm B D C
AB = ? 10 cm 3cm
AP × BP = PC2
 BC2 = AB2 + AC2 B C
(AB + 10)× 10 = 122 P 2. 7cm
This is a right angle triangle Perimeter of ABC = ?
144
AB + 10 = A = 90° Perimeter of ABC = 2(AQ + PC
10
+ BR)
AB = 14.4 – 10 we know, In a right angle triangle
= 2(2.6 + 2.7 + 3)
AB = 4.4 cm the length of the median is = 2 × 8.3
39. (a) always half of the hypotaneus
= 16.6 cm
then,

r
R 45. (d)
S  BD = DC = AD
3 B

si
AD = 5cm 60°

Q 42. (c)
an by
4

P
A
x° E

n
In a cyclic qudrilateral sum of O 142° P P
opposite angles is 180° 28°
62°
 + 4 = 180º ja B A D C
R s
then,
x° = ?
 = 36º  APB = 142°
a th

APD = 90° – 62°


Similarly AOB = 180° – 142°
 + 3 = 180º = 28°
= 38°
then,  APD = BPE
In OAB –
 = 45º In triangle BPE
ty a

 OA = OB = radius of circle 60° + x° + 28° = 180°


36  45
Average of and  = 180 – 38 142 x° = 180° – 88°
2 OAB =  = 71°
di M

2 2 x° = 92°
81 43. (b)
= = 40.5° 46. (c)
2 A A
15cm 10cm
40. (a) A
o’
o
M
68° O
B
I AB = 16 cm
B C AB
AM = = 8 cm
BOC = 134° 2
B C
In an incetric triangle – In OMA –
A

A = 68° OM =
A 152 – 82 = 161 cm
BIC = ? BOC = 90° +
2 In O'MA –
In an incertric triangle–
O'M = 102 – 82
A A
BIC = 90° + 134° = 90 + = 6 cm
2 2
Distance between their centre
68 A (OO') = O'M + OM
= 90° + = 44°
2 2
= 90° + 34° = 124°

= 6  161 cm 
A = 88°

Aditya Ranjan (Excise Inspector) Selected Selection 150

Downloaded by Kunal Chauhan (kc6970929@gmail.com)


lOMoARcPSD|39315497

Geometry
47. (d) A 52. (a)
49. (d) P
A
54°

I Q R
10
59°

S T
B C U
In an incentric triangle B D C
A QUR = 59° ADC = BAC (given)
BIC = 90° +
2 P = ? DCA ~ ACB
54
BIC = 90° + P CD AC CD 10
2  QUR = 90° –   
2 AC BC 10 16
= 90° + 27°
= 117° = CD = 6.25 cm
P
 90 – QUR
48. (c) 2 53. (c)
C

r
30° = 90° – 59°
B O Q

si
60°
r = 31° R P
60°
7 P = 31° × 2
A 10
A
r
an by
30°
o
60°
r
60°
D

50. (b)
P = 62°
Given,
B

n
B PA = 10
ja PB = 16
R s
25
In a right angle triangle ACD D OP = 14
a th

AD2 = DC2 + AC2 OR = OQ,


AC2 = (2r)2 – r2 90° PQ = (14 – OQ)
A C
AC = r 3 PR = (14 + OQ)
BAC is right-angle triangle, BC is
ty a

In AOB hypotenuse We know,


B We know,
PQ × PR = PA × PB
di M

in right triangle
(14 – OQ) (14 + OQ) = 10 × 16
60° hypotenuse 25
7 Median =
2

2
= 12.5 cm (142 – OQ2) = 160
30° 51. (b) OQ2 = 36
A r O A OQ = 6cm
7 Diameter = 2 × 6 = 12 cm
tan30° =
r 54. (d)
B P C
r = 7 3 cm
Q R A
7
sin30° =
AB x°
AQ = AR
A

1 7

BP = BQ
I
2 AB
AB = 14 cm CP = CR 108°
Perimeter of ABC = AB + BP + PC B C
 AC = r 3 + AC
AC = 7 3  3 = 21 cm 26.4 = AB + BQ + CR + AC x
BIC = 90 +
26.4 = AQ + AR 2
AB = 14 cm
BC = AC – AB 26.4 = 2AQ x
= 90 + = 108°
= 21 – 14 26.4 2
AQ = = 13.2 cm
= 7 cm 2 = x = 36°

Aditya Ranjan (Excise Inspector) Selected Selection 151

Downloaded by Kunal Chauhan (kc6970929@gmail.com)


lOMoARcPSD|39315497

Geometry
55. (d) 59. (d) 61. (a)
A A
B
5 3
m
P Q
D
16 E
B
AP = 5 cm, AQ = 3cm and PQ =
C B 6cm
Given that, Let PM = x cm
In, AMP A D 6 C
AD k (PA)2 = (AM)2 + (PM)2
 We know triplet = (12, 16, 20)
DB 1 25 = (AM)2 + x2
(AM)2 = 5 – x2 So, BC = 20
We know,
AC² : BC² = AD : BD In AQM 1
(AQ)2 = (AM)2 + (6 – x)2 Area of ABC = × 12 × 16
 AC : BC = 4
k :1 2
9 = (AM)2 + 36 + x2 – 12x
56. (d) = 96cm2

r
(AM)2 = 9 – 36 – x2 + 12x
A 25 – x2 = 9 – 36 – x2 + 12x ABC ~ EDC

si
52 13 Area of EDC  DC 
2
x=   
12 3  
an by Area of ABC  BC 
F E 169 56
(AM)2 = 25 – x2 = 25 –  96  36
9 9

n
Area of EDC = = 8.64 cm2
400
56 2 14 62. (d)
AM =  cm
B D ja
C 9 3 Radius of circumcircle
R s
3AC² = 4BE² (This is a theorem) So,
57. (b) a bc
=
a th

2  2 14 4 14 4 ABC
AB = 2 × AM =  cm
3 3 Given triangle is a right angled
6
60. (a) triangle area
ty a

6 A 1
= × 10 × 10.5
2
x x
di M

Required distance = 52.5 cm2


(6 + 6) = 12cm D F Radius of circumcircle
58. (d)
(10 10.5 14.5)
T y z = = 7.25 cm
R 29 4  52.5
63. (b)
B y E z C
29 58 32 B
S P
O Q We know, A
AD = AF, FC = EC and BE = BD
Q P
Given that SPT = 32° AB = 12 cm, BC = 8cm, BE = BD O T
Then, AB = 12 cm, BC = 8 cm, AC = 10cm
(given)
A

in ROP
 R = 90°, P = 32° 2(x + y + z) = (12 + 8 + 10)
Given, PA = 18cm, PB = 32cm &
Then, = 2(x + y + z) = 30 cm OP = 26cm
O = 180° – (90 + 32) = 58° = (x + y + z) = 15 cm
Thus, PQ = (26 + r) & PT = (26 – r)
Then, In ABC (x + y) = 12 cm
We know, PT × PQ = PA × PB
58 y + z = 8 cm
RSQ =  29  (26 + r) × (26 – r) = 18 × 32
2 z = 3 cm and x = 7 cm
We know,  676 – r2 = 576
Difference = AD – CE
RSQ = QRP = 29° = (7 – 3) = 4 cm  r = 10 cm

Aditya Ranjan (Excise Inspector) Selected Selection 152

Downloaded by Kunal Chauhan (kc6970929@gmail.com)


lOMoARcPSD|39315497

Geometry
64. (b) 66. (c) 69. (d)

F 2
D 1
60°
180°– 
A  180°–  C As we know, height of triangle and
 height of quadrilateral will be same
 Height of the triangle = Height
30° 24
of the quadrilateral
B
E Side of triangle = 24 cm
In-radius of triangle 3 3
= a 10 3 = 15 cm
2 2
In  ABF,  ++ 60  = 180 a 24
=  4 3 70. (d)
  += 120 ----(1) 2 3 2 3
In  ADE,  + 180 -+ 30 = 180 Diagonal of inscribed square = Q
Diameter of the circle = 2 × 4 3 6
  -+ 30  = 0  -----(2)

r
On solving (1) and (2), we get- = 8 3 cm O P
  = 45  and = 75 
(diagonal)2 6

si
  ABC = 75 Area of square=
2
65. (c) R

A
an by =
8 3 8 3
2
= 96 cm² in OQP
(OP)2 = (OQ)2 + (QP)2

n
67. (c)
102 = 62 + (QP)2
Given, In right angle  ABC
(QP)2 = 64
ja A = 2B
R s
QP = 8 cm
1
a th

45° Area of OPQ = × Base × height


2
C B
D a 1
= × 6 × 8 = 24 cm
c 2
ty a

In  ABC, AB²  AC²  BC²


Area of quadrilateral PQOR
 AB²  5²  12² =169
= 2 × Area of OPQ
di M

 AB  13 45°
= 2 × 24
Here, AD is angle bisector of  A.
b = 48 cm²
AC CD Let A = 90
So,  71. (c)
AB DB B = 45
As we know, Given, PQR = 78° & TPS = 24°
5 CD
  BC2 = AC2 + AB2 Q – R
13 DB a2 = b2 + c2 ...(1) We know, TPS =
Let CD=5x and DB=13x 2
If B = C
BC = CD + DB So, AC = AB  78° – R = 24° × 2
 12 = 5x + 13x B = C R = 30°
12 2 from equation (1) 72. (c)
x=  a2 = b2 + c.c
18 3
a2 = b2 + bc D C
2 10
A

68. (d)
CD = 5x = 5  
3 3 We know,
In  ACD, AD²  CD²  AC² PAQ = 90° [PQ is a diameter]
BAQ = 105° (Given)
2 A B
 10  2 00 BAP + PAQ = 105°
 AD²     5  9  25 BAP = 15°
 3  AB = 15cm, BC = 12cm, CD = 10cm
As we know,
00  225 325 BAP = AQP = 15°[By Alt-Seg-T] AD AB
 AD²   By the property, 
9 9 in APQ, BC CD
APQ + PAQ + AQP = 180°
5 13 APQ + 90° + 15° = 180° AD 15 12  15
 AD = cm   AD = =18 cm
3 APQ = 75° 12 10 10

Aditya Ranjan (Excise Inspector) Selected Selection 153

Downloaded by Kunal Chauhan (kc6970929@gmail.com)


lOMoARcPSD|39315497

Geometry
73. (c) As we know, When two chords Given ABC = 98°
intersect each other internally- B + D = 180°
D = 180° – 98° = 82°
A AP × PB = PC × PD
PAC = PCA = ADC = 82°
3.5 × PB = 7 × 5 [Alternate segment Theorem]
PB = 10 cm In PAC
APC + PAC + PCA = 180°
75. (d) APC + 82° + 82° = 180°
N M
D APC = 16°
A
B
SMART APPROACH:-
APC = 180° – 2 ADC
32° C
C 8 B D = 180° – 2 × 82° = 16°
O
77. (d)
As BN and CM are medians of A
ABC, D is centroid of the traingle
So, D will divide BN in the ratio In OBC

r
2:1 If OB = BC, then D

si
BD : DN = 2 : 1 BOC = BCO = 32º
BN = 3x As we know,
an by
Also, In right angled CNB, CD is
perpendicular to BN
OBA = BOC+ BCO = 32° + 32° B C

n
= 64° We know,
BC2 = BD × BN ABC ~ BDC
If OA = OB, then
ja
(8)2 = 2x × 3x
DC BC

R s
OBA = OAB = 64° 
BC AC
6x2 = 64 In AOB
a th

DC 5
AOB + OAB + OBA = 180°  
4 6 5 12.5
x =
3 AOB + 64° 64° = 180° DC = 2 cm
78. (a)
ty a

AOB = 180° – 128° = 52°


4 6 A
 BN = 3x = 3 × = 4 6 cm AOD + AOB + BOC = 180°
3
di M

AOD + 52° + 32° = 180°


74. (c)
AOD = 180° – 84° = 96°
A D 76. (c) I
3.5 90º+A/2
7 A
B C
P P A = 46°
B D
5
C 46
B C BIC = 90 + =113º
2
A



Aditya Ranjan (Excise Inspector) Selected Selection 154

Downloaded by Kunal Chauhan (kc6970929@gmail.com)


lOMoARcPSD|39315497

Mensuration- 2D

MENSURATION -2D
21
2D
1. In the given figure, the length of ) # ) ( ! #
arc AB is equal to twice the (! 01 $2 $ 3 24 C
length of radius r of the circle.
Find the area of sector OAB in
# ) *5 E *4 F #
@ !" D ! $2 C
the terms of the radius r. $2 6 ) 78$
G ( ) %
AB # ! ! ,8,
> ?
r # 9 : " ! ; ! &

r
r ! " # $ # < = Blue
)

si
OAB % & ’ ()
SSC CPO 09/11/2022 (Shift-01)
Red Red Red
an by (a) 27 cm
(c) 28 cm (d) 24 cm
(b) 25 cm
SSC CPO 10/11/2022 (Shift-01)
(a) 4 : 3 (b) 1 : 2

n
4. The perimeter of a rectangle is (c) 2 : 1 (d) 3 : 4
equal to the perimeter of a
square. If the length and the 7. What is the length of the longest
ja breadth of the rectangle are 10 pole that can fit itself in a hall
R s
cm and 8 cm respectively, then 60 feet long, 30 feet broad and
what will be the area of the 20 feet high?
a th

square? 61 & , H1 & ,


SSC CPO 09/11/2022 (Shift-01)
) ) & , I J &,
(a) 3r (b) 2r ! ! *1 # "
(c) r2 (d) r2
ty a

! $2 5 ! SSC CPO 10/11/2022 (Shift-01)


2. Find the perimeter of major (a) 50 feet (b) 70 feet
% & > ?
sector of a circle of radius 12 (c) 30 feet (d) 20 feet
di M

metres, whose minor sector SSC CPO 09/11/2022 (Shift-03)


8. The length of a chord of a circle
subtends an angle of 75° at the (a) 72 cm2 (b) 98 cm2 is equal to the radius of the circle
centre. (c) 81 cm2 (d) 64 cm2 Find the angle subtended by it in
*+ , ) 5. The sides of- a triangular field are major segment.
62 m, 186 m and 279 m. Find the ) (
# $ ’ (greatest
) ( !length of tape that would
be able to exactly measure each
! ! K -
- # 75°$ . /
of them without any fractions. / ’ ()
) " ( "(SSC ) CPO6 +10/11/2022 (Shift-02)
SSC CPO 09/11/2022 (Shift-01) (a)! 90° (b) 60°
*56 +30 @ ,
(a) 24 + 5metres (c) 30° (d) 120°
A ’ () ( !
(b) 24 + 19metres 9. The area of a circle is 1386 cm2.
(c) 24 – 5metres
.B , $ 2 What )is the
! , radius of the circle?
A

! !%
(d) 24 – 19metres 22
SSC CPO 10/11/2022 (Shift-01) (Use  = )
3. A field in the shape of a rectangle 7
(a) 62 m (b) 93 m
length 90 m and breadth 75 m. ) % &
+
*H56
(c) 31 m (d) 30 m
In one corner of the field a pit > ?
6. As shown in the given figure,
which is 18 m long 15 m broad
inside large semicircle, three 22
and 6 m deep has been dug out. E = 7 F
semicircle (with equal radii) are
The earth taken out of it is evenly drawn so that their diameters all
spread over the remaining part of SSC CPO 10/11/2022 (Shift-02)
sit on the large semicircle's
the field. Find the rise in the level diameter. What is the ratio (a) 7 cm (b) 14 cm
of the field? between the red and blue areas? (c) 18 cm (d) 21 cm

Aditya Ranjan (Excise Inspector) Selected Selection 155

Downloaded by Kunal Chauhan (kc6970929@gmail.com)


lOMoARcPSD|39315497

Mensuration- 2D
10. The area of the sector of a circle 10 m $ 2 6 m I S 19. The perimeter of a square is the
5m
is 128 cm2. If the length of the J , ( same asE the perimeter of a
arc of that sector is 64 cm, then rectangle.
) &F , ( ! O / The perimeter
:G of the
find the radius of the circle. square is 40 m. If its breadth is
% & ’ () E # $2 (
) # $ % & *+5 two-thirds of its length, then the
% & ( ( areaF(inEm²)&of9 the rectangle is:
! +
@! # $
% & , ( F
6L ! ’ ! !
SSC CPO 11/11/2022 (Shift-01)
() 40
(a) 300 m2 (b) 280 m2
SSC CPO 10/11/2022 (Shift-03) m ! $2 !
(c) 330 m2 (d) 230 m2
(a) 4 cm (b) 8 cm U m² %
15. Find the area of triangle whose
(c) 2 cm (d) 16 cm sides are 10 cm, 12 cm, and 18 cm. F ’
11. If the arc of a circle of radius 30 10 cm, 12 cm 18 cm " ( SSC CPO 23/11/2020 (Shift-1)
cm has a length of 19 cm, then
the angle (in degrees, rounded off " ( % & ’ ( )96
(a) (b) 84
to two decimal places) subtended SSC CPO 11/11/2022 (Shift-02)
(c) 100 (d) 121
at the centre of the circle is:
(a) 22 2 cm2 (b) 30 2 cm2 20. The sides of a triangle are 24 cm,

r
22 26 cm and 10 cm, At each of its
(Take  = )
7 (c) 28 2 cm2 (d) 40 2 cm2 vertices, circles of radius 4.2 cm

si
30 cm ) 16. One side of the triangle is 15 cm are drawn. What is the area (in
19 cm andMthe. corresponding height is cm2) of the triangle, excluding the

<N
an by / E $ .
O/ P F ’
6 cm, then9area of the triangle is:
" ( )( ) " ( *4 !
portion covered by the sectors of

 22 

n
! I 6 ! " (  π = 7 
the circles?
22
E = 7 ()F % & ?
ja SSC CPO 11/11/2022 (Shift-02)
! " ( 24 cm,
" 26( cm
)
R s
SSC CPO 11/11/2022 (Shift-01) 10 cm ! . B 4.2 9 :
(a) 46 sq.cm (b) 45 sq.cm cm # (
(a) 51.25 (b) 16.27
a th

(c) 47 sq.cm (d) 48sq.cm # $ K ) )


(c) 36.27 (d) 46.25
17. If the hypotenuse of an isosceles Q $2 " (cm² % F
12. If the sum of the diagonals of a right angled triangle is10 cm,
rhombus is L and the perimeter  22 
then the other two sides (in cm) ’ π = 7 
is 4P, then find the area of the
ty a

are_________.  
rhombus?
) ! K ! / "SSC( CPO 23/11/2020 (Shift-1)
) " ( L/ P
di M

/ *1 ! M (a) 27.72 " ( (b) 120


@! 4P @! " (
E! F T T T T T(c)T 105.86
TTTTT (d) 92.28
% & > ?
SSC CPO 11/11/2022 (Shift-03) 21. A rectangular lawn whose length
SSC CPO 11/11/2022 (Shift-01)
(a) 10 2 and 10 2 is twice of its breadth is extended
1 2 1 2
(a)
4
 L – P2  (b)  L – 4P 
4
2 by having four semi-circular
(b) 8 2 and 8 2 portions on its sides. What is the
1 2 1 2
(c)
2
 L – 4P 2  (d)  L  3P 
4
2

(c) 6 2 and 6 2
total cost (in Rs.) of levelling the
entire lawn at the rate of Rs.100
13. What is the length of the longest per square metre, if the smaller
rod that can be placed in a room (d) 5 2 and 5 2 side of the rectangular lawn is 12
of dimensions 12 m × 9 m × 8 m? m? (Take  = 3.14)
18. The difference between the semi-
12 m × 9 m × 8 m perimeter and the sides of PQR J (!
# ( ! !are! 18 cm, 17 cm and 25,
A

$2 (!
Q$2 R > ? respectively. Find the area of the
triangle. A S
SSC CPO 11/11/2022 (Shift-01)
(a) 15 m (b) 17 m PQR A U " ( ( O J X *
(c) 16 m (d) 14 m V 189cm,
W 17 cm, ! ! )
14. A hall of 10 m length, 5 m width 25 cm " ( % & X’ Y #( )
and 6 m height is to be painted (one SSC CPO 11/11/2022 (Shift-03) " ( 12 m ( = 3.14 )
side only). Determine the total
SSC CPO 23/11/2020 (Shift-2)
surface area to be painted. (neglect (a) 330 510cm2 (b) 230 510cm2
the area of windows and doors) (a) 86,540 (b) 97,625
(floor area is not to be painted) (c) 30 510cm2 (d) 130 510cm2 (c) 85,320 (d) 78,650

Aditya Ranjan (Excise Inspector) Selected Selection 156

Downloaded by Kunal Chauhan (kc6970929@gmail.com)


lOMoARcPSD|39315497

Mensuration- 2D

22. If a wheel has diameter 42 cm, 26. A rectangular lawn whose length 10 , D !
then how far does the wheel go is twice of its breadth is extended
64
Wcm $2 <
(in metres) in 12 revolutions? by having four semi-circular
portions on its sides. What is the < %m² &F E"
 22  total area (in m2) of the lawn if SSC CPO 24/11/2020 (Shift-2)
 Take  = 
 7  the smaller side of the rectangle (a) 21 (b) 11
is 12 m?
! 42 cm
D ! (c) 15 (d) 9
(Take  = 3.14)
12 > O E , F 30. The perimeter of a square is half
( ) ? ) J (! @!
the perimeter of a rectangle. The
$2 perimeter" of the
( square is 40 m.
 22  A " If !itsNbreadth
72 is two-thirds of its
  
 7  ( 12Qm length,
, " ( then what is the area (in
m2) of the rectangle?
SSC CPO 23/11/2020 (Shift-2) J m² % F ’ & E
(a) 15.84 (b) 23.27 ) )
SSC CPO 24/11/2020 (Shift-1) A 40
(c) 21.45 (d) 17.64
m ! $2 !

r
23. The two parallel sides of a (a) 548.32 (b) 444
trapezium are 27 cm and 13 cm, U m² %
(c) 853.2 (d) 308.64

si
respectively. If the height of the F ’
trapezium is 7 cm, then what is SSC CPO 25/11/2020 (Shift-1)

! !
an by
its area (in m2)?
(trapezium)
" (
27. The base of a triangle is equal to
the perimeter of a square whose
(a) 400 (b) 196

n
diagonal is 62 cm, and its height (c) 384 (d) 321
! " ( 27
) cmV 9 13Wcm is equal to the side of a square 31. The sides of a triangle are 24 cm,
ja ! "7 cm( I S whose area is 144 cm2. The area 26 cm and 10 cm. At each of its
R s
(m² ) ! % & ’ of the triangle (in cm2) is: vertex, circles of radius 4.2 cm
SSC CPO 23/11/2020 (Shift-2) are drawn. What is the area (in
a th

! " ( A @ !
cm2) of the portion covered by the
(a) 140 (b) 0.014 (! + ! / 6 three sectors of the circle?
(c) 1.4 (d) 0.14 ! IS @! " (
 22 
24. The two parallel sides of a trapezium ( ! 144% cm² &  Take  = 
ty a

 7 
are 17 cm and 15 cm, respectively.
" ( %cm² &
F ’E
If the height of the trapezium is 6 ) " ( 24 "cm,(26) cm
di M

cm, then its area (in m2) is: SSC CPO 24/11/2020 (Shift-2)
10 cm ! .B 9
! ! " ( ! (a) 288
" ( ) (b) 216
S ) # ( 4.2 (!
cm # $
V 9 17Wcm 15 cm @ (c)
! 144 (d) 72
% cm²
& EF ’
! " ( 6 cmI S 28. The perimeter of a right angle
 22 
! % m² & FE ’ triangle is 60 cm and its   
hypotenuse is 26 cm. What is the  7 
SSC CPO 24/11/2020 (Shift-1)
area (in cm2) of the triangle? SSC CPO 25/11/2020 (Shift-1)
(a) 9.6 (b) 960
! ! / " 60
( cm (a) 92.28 (b) 105.86
(c) 0.96 (d) 0.0096
25. The area of a field in the shape
! 26 cm
/ " ( (c) 120 (d) 27.72

of a hexagon is 19443m². What % m²& EF ’ 32. One side of a rhombus is 13 cm


and one of its diagonals is 24 cm.
A

will be the cost (in Rs.) of fencing SSC CPO 24/11/2020 (Shift-2)
it at the rate of Rs.11.50 per metre? What is the area (in cm²) of
(a) 60 (b) 96 rhombus?
) : ,8" ( #19443m² % &Z
(c) 90 (d) 120 ! ! " ( 13 )cm " (
@! 11.50 . X ,
29. A 64 cm wide path is made around ! ) 24 cm/ ! "
! $2 a circular garden having a diameter % cm²& Z FE ’
EX F ) ? of 10 metres. The area (in m2) of
the path is closest to: SSC CPO 25/11/2020 (Shift-1)
SSC CPO 24/11/2020 (Shift-1)
22  (a) 30 (b) 60
(a) 2,256 (b) 3,200 
 Take  =  (c) 120 (d) 90
 7 
(c) 2,785 (d) 2,484

Aditya Ranjan (Excise Inspector) Selected Selection 157

Downloaded by Kunal Chauhan (kc6970929@gmail.com)


lOMoARcPSD|39315497

Mensuration- 2D

33. The base of a triangle is equal to 37. The two parallel sides of a trape- 41. A circular park whose diameter is
the perimeter of a square whose 210 m has a 5 m wide path run-
zium are 27 cm and 13 cm respec-
diagonal is 92 cm, and its height ning around it (on the outside).
is equal to the side of a square tively. If the height of the trape-
What is area (in m2) of the path?
whose area is 144 cm². The area zium is 8 cm, then what is its area
of the triangle (in cm²) is: in m2? +*1 !, D
)
E &F 4
! " ( A @! ) ! R " ( ! " ( ) %+ F& E
( ! 92 cm / V 9 W +3 ! *H ! ! R
SSC CPO 11/12/2019 (Shift-01)
! IS @! " (
" ( IS 5 ! !
(a) 1100 % (b) 1050
& 
( ! 144 % cm² & +
’ () (c) 1075 (d) 1020 
" ( %cm² &F Z ’ E
SSC CPO 09/12/2019 (Shift-01) 42. The sides of a triangular park are
SSC CPO 25/11/2020 (Shift-2)
(a) 0.032 (b) 0.056 35 m, 53 m and 66 m. The cost of
(a) 288 (b) 216
leveling the park at the rate of
(c) 72 (d) 144 (c) 0.016 (d) 0.32 Rs. 9.25 m² is:
34. If each side of an equilateral tri- 38. The sides of a triangular park are ) " ( 35 m, 53" ( )
angle is 12 cm, then its altitude 60 m, 112 m and 164 m. The cost
m 66 m 9.25/m²
XT !

r
is equal to: of leveling the park at the rate of
) ! " ( . B Rs. 8.50
" m ( 2 is:* + !

si
! ! IS ?
) "? ( 60 m, 112
" ( )
SSC CPO 09/12/2019 (Shift-01) SSC CPO 11/12/2019 (Shift-02)

(a) 6 3 cm
an by (b) 3 6 cm
m 164 m
!
8.50/m
XT 2 !
’ (a) Rs.
( )8,584 (b) Rs. 8,547

n
SSC CPO 09/12/2019 (Shift-02) (c) Rs. 8,621 (d) Rs. 8,510
(c) 6 2 cm (d) 3 2 cm
(a) Rs. 18,316 (b) Rs. 17,136 43. A field is in the from of a circle.
35. A chord of the larger among two
ja
concentric circles is of length 10 (c) Rs. 18,164 (d) Rs. 17,085 The cost of fencing around it at
R s
cm and its is tangent to the Rs. 12 per metre is Rs. 2,640.
39. The circumference of a circle What is the area (in m2) of the
smaller circle. What is the area
a th

exceeds its diameter by 60 cm.


(in cm2) of the annulus portion 22
The area of the circle is.
between the two circles? field? (Take   )
7
! . ! $2 22
(Take   ) ) *+ XT
ty a

( *1 ! Q7 ,
< 9 # ! ! $2
) A @! D ! ! 61X T! + 6 L 1 (m² ) %
di M

+
" % &
F E!
A % & ’ ()
> ? 22
22
? E   7 (F)
SSC CPO 09/12/2019 (Shift-01)  ( )
E 7
F
(a) 10 (b) 25
SSC CPO 11/12/2019 (Shift-02)
5 SSC CPO 09/12/2019 (Shift-02)
(c) 15 (d) (a) 1925 m² (b) 3850 m²
2 (a) 536 cm2 (b) 1078 cm2
36. PQRS is a rectangle T is a point (c) 2772 m² (d) 5544 m²
(c) 346.5 cm2 (d) 616 cm2
on PQ such that RTQ is an isos- 44. The lengths of the parallel sides
celes triangle and PT = 5 QT. If 40. A field is in the shape of a trape-
of a trapezium are 51 cm and 21
the area of triangle RTQ is 12 3 zium whose parallel sides are 200
cm, and that of each of the other
sq. cm, then the area of the rect- m and 400 m long, whereas each
two sides is 39 cm. What is the
angle PQRS is: of other two sides is 260 m long.
area (in cm²) of the trapezium?
A

What is the area (in m2) of the field?


PQRS ) T, PQ ) ) ! ) ! R " ( !
RTQ ) ! K " () ! R " ( ( !
51 cm 21 cm M
PT = 5 QT RTQ " (% & (! ! " ( +11
" ( 39
! cm. B
12 3 ! +
PQRS L11 ( M " (
! R " ( (cm² %) ’ &
% & ? ! .B +61
+ ()
% & EF ?
SSC CPO 09/12/2019 (Shift-01)
SSC CPO 11/12/2019 (Shift-01) SSC CPO 12/12/2019 (Shift-01)
(a) 144 3 sq.cm (b) 142 sq.cm (a) 48000 (b) 52000 (a) 1206 (b) 1296

(c) 134 3 sq.cm (d) 142 3 sq.cm (c) 72000 (d) 60000 (c) 1152 (d) 1260

Aditya Ranjan (Excise Inspector) Selected Selection 158

Downloaded by Kunal Chauhan (kc6970929@gmail.com)


lOMoARcPSD|39315497

Mensuration- 2D

33 ! D ! ) 50. When the radius of a sphere is


45. If the perimeter of an isosceles
increased by 5 cm, its surface
right triangle is (16 2  16) cm, $2 ) ( (!
area increases by 704 cm². The
then the area of the triangle is:
! $2 * L +diameter
\ of the original sphere, is
% & ? 22
) ! K ! / " ( (Take  = )
22 7
(16 2  16) ! " E(  ( ) F
7 ( ) 5 cm =
% & ? SSC CPO 12/12/2019 (Shift - 02) ( ! 704
: Gcm² %
SSC CPO 12/12/2019 (Shift - 02) (a) 3520 sq.cm (b) 3450 sq.cm = ( <
(a) 76 sq.cm (b) 64 sq.cm (c) 3550 sq.cm (d) 3620 sq.cm
22
(c) 58 sq.cm (d) 66 sq.cm 48. The sides of a triangle field are ? E  7 (F)
46. The diameter of a cycle wheel is 120m, 170m. and 250m. The cost
of leveling the field at the rate of SSC CPO 13/12/2019 (Shift - 02)
1 Rs. 7.40/m² is: (a) 8.2 cm (b) 5.2 cm
126 cm. A cyclist takes 16
2 ) " ( 120m, " ( (c)) 6.8 cm (d) 6.2 cm
minutes to reach the destination 170m. 250m. 7.40 X T m²
] 51. The sides of a triangular park are
at a speed of 72 km/hr. How
in the
Z ratio of 12 : 17 : 25 and its

r
! !
many revolution will the wheel perimeter is 1080 m. The area (in
make during the journey? (Take / ()

si
hectares) of the park is.
SSC CPO 13/12/2019 (Shift - 01)
22 ) " ( " (
 ) (a) Rs. 65,120 (b) Rs. 63,640

)
7

!
an by D ! 49.
(c) Rs. 59,200
* + The
(d) Rs. 66,600
6 ! internal length of a room is
*+ W *3 W +4
m %
1080
& E
!
>

n
) ! 3+ [ - two, times its breadth and three SSC CPO 13/12/2019 (Shift - 02)
times its height. The total cost of
1 (a) 3.6 (b) 4.5
! ja D 16 S , painting its four walls at the rate
R s
2 of Rs. 25/m² is Rs. 3,600. What (c) 4.2 (d) 4.8
O is the cost of laying a carpet on 52. The area (in m²) of a circular path
a th

> ) ? E its floor


() at the rate of Rs. of uniform width x meters
900.50m²? surrounding a circular region of
22
 F ) ! diameter
$ 2d metres is.
7
@! @&S d , D ! ) %
ty a

SSC CPO 12/12/2019 (Shift - 02) ! 25/m² XT x , ) ! $2


(a) 5000 (b) 5200 , 3,600 XT < (m²
% ) &
di M

(c) 4000 (d) 4500 900.50 X T m²


] ! ! & 9SSC CPO 13/12/2019 (Shift - 02)
47. A circular wire of diameter 77 cm Q /
(a)  x (x + d) (b)  x (x + 2d)
is bent in the form of a rectangle ()
whose length is 142% of its  d
SSC CPO 13/12/2019 (Shift - 01)
breadth. What is the area of the (c)  x  x   (d)  x (2x + d)
(a) Rs. 6,516 (b) Rs. 7,240 2
22 (c) Rs. 5,430 (d) Rs. 5,970
rectangle? (Take   )
7

......-------......
A

Aditya Ranjan (Excise Inspector) Selected Selection 159

Downloaded by Kunal Chauhan (kc6970929@gmail.com)


lOMoARcPSD|39315497

Mensuration- 2D

ANSWER KEY
1.(d) 2.(b) 3.(b) 4.(c) 5.(c) 6.(b) 7.(d) 8.(c) 9.(d) 10.(a)

11.(c) 12.(b) 13. (b) 14.(d) 15.(d) 16.(b) 17.(d) 18.(c) 19.(a) 20.(d)

21.(c) 22.(a) 23.(b) 24.(d) 25.(d) 26.(c) 27.(c) 28.(d) 29.(a) 30.(c)

31.(d) 32.(c) 33.(b) 34.(a) 35.(b) 36.(a) 37.(c) 38.(b) 39.(d) 40.(c)

41.(c) 42.(b) 43.(b) 44.(b) 45.(b) 46.(a) 47.(c) 48.(d) 49.(a) 50.(d)

51.(a) 52.(a)

r
SOLUTIONS
SOLUTIONS

si
an by
n
1. (d) 3. (b) 6. (b)
Formula used, volume of cuboid Let the diameter of blue semi-

ja
Length of arc   2r = length × breadth × height circle = 6 units
R s
360
Area of rectangular field = length
 Radius of blue semi-circle = 3
× breadth
a th

 units
 2r   2r = 90 × 75 = 6750 sq. m
360
Area of pit = 18 × 15 = 270 sq. m Diameter of red semi-circle
360
 Remaining area of the field where

ty a

the earth has to spreaded over 6


= =2 units
Now, Area of sector OAB = 6750 – 270 = 6480 m2 3
di M

 360 1 Volume of the earth = Volume of


  r    r 2 = r2  Radius of red semi-circle = 1
360  360 pit unit
Remaining Area × h = 18 × 15
SMART APPROACH:- ×6 Ratio between red and blue area is
Given that
Radius = r, length of arc = 2r 6480 × h = 1620
1  1 2
 1 :   3 – 3   1 
2 2
We know, 1 h = 0.25 m  3
Area of arc = × length of arc
2
2 2 2 
× radius Rise in the level of the earth =
Then,
1
25 cm
Area of arc = × 2r × r = r²
2 4. (c)  3 : 6 = 1 : 2
Perimeter of square = Perimeter
2. (b) 7. (b)
of rectangle
Perimeter of sector  4 × side = 2(Length + Breadth) Length of longest pole
A

  4 × side = 2(10 + 8)
=  r  2r  4 × side = 36
360  l 2  b 2  h2
 side = 9 cm
Angle of Major Sector = 360º – 75º  Area of square = side2 = 92 =
 60   30    20 
2 2 2
= 285º 81 cm2 
Perimeter of Major Sector 5. (c)
Factor of 62 = 2 × 31  3600  900  400
285
=  2  12  2  12 Factor of 186 = 2 × 3 × 31
360
Factor of 279 = 3 × 3 × 31  4900  70 feet
= 19  24 meter HCF = 31 m

Aditya Ranjan (Excise Inspector) Selected Selection 160

Downloaded by Kunal Chauhan (kc6970929@gmail.com)


lOMoARcPSD|39315497

Mensuration- 2D

8. (c) 12. (b) SMART APPROACH:-


Given, Given that,
d + d = L, perimeter of rhombus 4P
1 2

than,
side of rhombus = P
than,
d + d = 4P²
2 2
60 1 1

we know,
(d1 + d2)² = d + d + 2d1 d2
2 2
1 2

d1 d2 = 1/2 (L² – 4P²)


If chord length is equal to radius Then,
Area of rhombus = 1/4 (L² – 4P²)
of circle, the formed  will be
equilateral. Perimeter = 4P 13. (b)
Angle subtended by the chord in Sum of diagonals = L Length of the longest rod
60 In Rhombus ABCD
major segment   30 = l 2  b 2  h 2 = 122  92  82
2
4P
9. (d) AB  BC  CD  DA  P
4 = 144  81  64 = 289 =17m
Area of circle = r2

r
22 2 We know that, diagonals of 14. (d)
1386 = r rhombus bisect each other at 90°.
7 Given,

si
1386  7 C L = 10 m
 r2 = OA  OC 
22 2
r2 = 441
r = 21cm
an by OB  OD 
BD
B=5m
H=6m

n
2
10. (a) Area of hall to be painted
In AOB, excluding surface
ja 
R s
Area of sector =  r 2 OA2 + OB2 =AB2
360 = 2(l + b)h + l × b
a th

 2 2
= 2(10 + 5)6 + 10 × 5 = 180 + 50
 128   r 2 ........(1)  AC   BD  2
360       AB = 230m2
 2   2 

Arc length =  2r 15. (d)
360  AC   BD 
2
ty a

2
     P Let the sides of triangle be
 64   2r .......(2)  2   2 
360 a = 10 cm
di M

AC2 + BD2 = 4P2 ... (1)


On dividing (1) by (2), we get:
Again, b = 12 cm
r
2 AC + BD = L
2 c = 18 cm
 r = 4 cm squaring both sides abc
(AC + BD)2 = L2 Semi-Perimeter, S=
2
SMART APPROACH:-
Given that AC2 + BD2 + 2AC. BD = L2 10  12  18
Area of arc = 128 cm², length of From eqn(1) = = 20 cm
2
arc = 64 cm
We know, 4P2 + 2AC. BD = L2
1 Area of = S  S – a  S – b  S – c 
128 = × 64 × r
2 2AC.BD = L2 – 4P2
r = 4 cm
1 2 = 20  20 – 10  20 – 12 20 – 18 
11. (c)  AC.BD   L – 4P 2 
A

2
= 20  10  8  2
 Area of rhombus ABCD
Length of arc =  2r
360 = 40 2 cm2
1
 22 =  AC  BD 16. (b)
 19  2  30 2
360 7
1
1 1 2 Area of  =  Base  Height
19  360  7 =   L – 4P 2  2
  2 2
2  22  30
1
47880 1 2 =  15  6 = 45 cm2
=  L – 4P 
2
= = 36.27° 2
1320 4

Aditya Ranjan (Excise Inspector) Selected Selection 161

Downloaded by Kunal Chauhan (kc6970929@gmail.com)


lOMoARcPSD|39315497

Mensuration- 2D
17. (d) = Perimeter of a square Area of rectangle = 12 × 24 = 288 m2
Let the two equal sides of ABC 2 (3x + 2x) = 40 Area of circular portion of lawn
AB and BC be x cm. 5x = 20 = × 122 + × 62
x=4 = 3.14 × (144 + 36)
= 3.14 × 180
Length = 3x = 12 m
= 565.2 m2
Breadth = 2x = 8 m
Total area of the lawn
Area of a rectangle = 288 + 565.2 = 853.2 m2
= Length × Breadth Total cost = 853.2 × Rs.100
= 12 × 8 = 96 m2 = Rs. 85320
20. (d) 22. (a)
A Wheel covered distance in one
A revolution = 2r
4.2
A weel covered total in 12
revolution = 12 × 2r
By the pythagorean theorem, 24 cm 10 cm = 2r = 42 cm
AB2 + BC2 = AC2
r = 21 cm
 x2 + x2 = 102
 2x2 = 100 22

r
C = 12 × 2 × × 21
 x2 = 50 B
4.2
7
4.2
= 24 × 66

si
26 cm
x 5 2 = 1584 cm
 BAC is a right angle triangle–
SMART APPROACH:- = 15.84 m
an by
Equal sides of isosceles right
angle triangle
262 = 242 + 102
redius of each circle = 4.2 cm
23. (b)
A 13 cm B

n
hypotenuse Area of seetorian circles –

2 180
= r2 7 cm

10

ja
2
 5 2 cm 360
R s
2 2
[  a triangle is always 180°]
18. (c) D C
a th

Given, 1 22 27 cm
s – a = 18 cm =   4.2  4.2 Area of the trapezium –
2 7
s – b = 17 cm = 27.72 cm2 1
= × (13 + 27) × 7
s – c = 25 cm 2
ty a

On adding 1 = 140 cm2


Area of ABC = × BA × AC
 3s – (a + b + c) = 60 2
140
di M

 3s – 60 = a + b + c = m2 = 0.014 m2
1 10000
We know that, = × 24 × 10 = 120 cm2 24. (d)
2
abc Area of the trapezium –
s Area of remaining region
2 = 120 – 27.72 1
= × (17 + 15) × 6
= 92.28 cm2 2
3s – 60
s = 96 cm2
2 SMART APPROACH:- = 0.0096 m2
 2s = 3s – 60 Sides of triangle is 10cm, 25. (d)
 s = 60 24cm and 26cm
(10, 24, 26) is triplate  Area of a hexazon
Area of   Area of remaining region
= area of triangle – area of 3
=6× × (side)2
= s  s – a  s – b  s – c  sectarian circles 4
A

1 1 22
= 60  18  17  25  ×10 ×24 - × × 4.2× 4.2 3
2 2 7 =6× × a2 = 1944 3 (a = side)
= 92.28 cm² 4
= 30 510 cm2
21. (c) 1944  4
19. (a) a2 =
6
If length of rectangle = 3x
a2 = 324 × 4
Breadth of a rectangle 12 m a = 36 m
6 m
2 Perimeter of the hexazon
= 3x × = 2x = 6 × 36 = 216 m
3
24 m Total cost = 216 × 11.5 = 108 × 23
Perimeter of a rectangle = Rs.2484

Aditya Ranjan (Excise Inspector) Selected Selection 162

Downloaded by Kunal Chauhan (kc6970929@gmail.com)


lOMoARcPSD|39315497

Mensuration- 2D
26. (c) 29. (a) In a rhombus, two diagonals
intersect each other on 90° in
equal parts
D C
12 m In APB –
6 m
12 m 5m
P
.64 m
A 24 m B 12

r=5m
Area of rectangle = 12 × 24 = 288 m2 A 13 B
= (5.642 – 52)
Ar. of circular portion of the lawn
= × 122 +62 ×  22
= × 10.64 × .64 PB = 132 – 122 = 5 cm
= 3.14 × (144 + 36) 7
= 3.14 × 180 = 22 × 1.52 × .64 Area of a rhombous
= 565.2 m2 21.4016
Total area of the lawn = 21 m2 1
= × AC × BD
30. (c) 2

r
= 288 + 565.2 = 853.2 m2
Length of the rectangle = 3x

si
27. (c) Breadth of the rectangle = 3x × 1
= × 24 × 10 = 120 cm2
Side of square = a 2
2
an by
diagonal = a 2 3
= 2x

 (3x + 2x) = 40
33. (b)

n
a 2 = 6 2 Side of square = a
x = 8m
a = 6 cm Area of the rectangle 3x × 2x =
ja diagonal = a 2
Perimeter of square = 4 × 6 = 24 cm 6x2
R s
Base of triangle = 24 cm = 6 × 64 = 384 m2
Height of triangle = ? a 2 = 9 2
a th

31. (d)
 Height of triangle is equal to a = 9 cm
side of that square whose's area C
is 144 cm2 Perimeter of square = 4 × 9 = 36 cm
ty a

Side = 12 cm 24 10 Base of triangle = 36 cm


Triangle's height = 12 cm
 Height of triangle is equal to
di M

1
Ar of triangle = × Base × height side of that square whose's area
2 A B
is 144 cm2
26
1 Side = 12 cm
= × 24 × 12 = 144 cm2 Radias of each circle = 4.2 cm
2
 ABC is a right angle triangle Height of the triangle = 12 cm
262 = 242 + 102
28. (d) Area of the triangle
Total area of sectoriam circles –
A
180 2 1
= r = × Base × height
360 2
26 [  A triangle is always 180°]
10
1
1 22 = × 36 × 12 = 216 cm2
A

=  × 4.2 × 4.2 2
 2 7
C 24 B = 27.72 cm2 34. (a)
Perimeter = 60 cm 32. (c)
AB + BC = 60 – 26 = 34 cm each side of an equilateral 
By hit and trial – D 13 C
= 12cm
One side = 24 cm
5 12
Second side = 10 cm P Altitutde
13 13
12
1 5
Area of triangle = × 24 × 10 = 120 cm2 3a 3  12
2 13 =   6 3 cm
A B 2 2

Aditya Ranjan (Excise Inspector) Selected Selection 163

Downloaded by Kunal Chauhan (kc6970929@gmail.com)


lOMoARcPSD|39315497

Mensuration- 2D
35. (b) r = 35
= 168(168–60)(168–112)(168–164)
22
= 168 108  56  4 Area of circle = r2 = × 35 × 35
7
= 2016 m2 = 3850
Cost of levelling the park = Rs. 44. (b)
O 8.5/m2
R r Cost of levelling of 2016 m2 of the A 21 B
A B park
5 x 5 = 2016 × 8.5 = 17136
36 39
39. (d)
AB = 10cm (given) 2r – 2r = 60
= 2r ( – 1) = 60 D 15 E 21
10 F 15 C
AX = = 5cm  22 1 
2
= 2r  –  = 60 1
R = r + 25
2 2  7 1 Area of trapezium = (a + b) × h
2
Area of the annulus portion = r = 14
between two circle 1
22 = (21 + 51) × 36
= (R2 – r2) Area of circle = r2 = × 14 × 14 2
= (25) = 25 7

r
36. (a) = 616 cm2 1
40. (c) = × 72 × 36 = 1296 cm²
2

si
P 5x T x Q
A 200 B 45. (b)
an by 2x x 260 240 260
Given, Perimeter = 16 2 +16 cm

n
a 2
D C a
100 X 200 Y 100
ja
R s
S R A.T.F
a
A.T.Q, AX = 120 × 2 = 240
a th

Given that, Then, We know,


PT : QT = 5 : 1
1
Area of trapazium = × (600) × Perimeter of Iscoceles 
1 2
Area of  RTQ = × x × x  12 3
2 240 = 72000m² = a 2 ( 2 +1)
ty a

41. (c)
 x² = 24 3
Then, 210 a 2 ( 2 +1) = 16 2 +16
di M

Radius of park = = 105m


Area of PQRS = 6x × x = 6x² 2
Radius of park and path combine a = 8 2
= 6 × 24 3 = 144 3 cm²
= (105 + 5) = 110
37. (c) Area of path = (R2 – r2) 1
Two parallel sides of a trapezium Area of triangle= ×a²
= ((110)2 – (105)2) 2
are 27cm and 13cm height = 8cm = ((110 + 105) (110 – 105))
1 =  × 215 × 5 1
Area = (a  b)  h = 1075 m2 = × 8 2  8 2 = 64 cm²
2 2
42. (b)
1 46. (a)
= (27  13)  8 (35  53  66)
2 S= = 77 m Distance Covered = S × T
2
1
=  40  8 160 cm2 = 0.016 m2 Area of  = S(S – a)(S – b)(S – c) 100000 33
A

2 = 72 × ×
60 2
38. (b) = 77  42  24 11
We know, = 11 × 7 × 6 × 2 = 924 m2 = 1980000 cm
a bc Costing of leveling of 924 m2 = 924 Distance covered by wheel in 1
S (Semi perimeter) = × 9.25 = Rs. 8547
2 22
43. (b) revolution= d = ×126 = 396 cm
Area of D = S(S – a)(S – b)(S – c) 7
2640
Total length of fencing = =
60  112  164 336 12  Number of revolutions
S=  = 168 220m
2 2 1980000
Area of  perimeter = 220 = = 5000 rev.
2r = 220 396

Aditya Ranjan (Excise Inspector) Selected Selection 164

Downloaded by Kunal Chauhan (kc6970929@gmail.com)


lOMoARcPSD|39315497

Mensuration- 2D
47. (c) 49. (a) 54x = 1080
Diameter of circle = 77 cm
L B H x = 20
Circumference of circle = d
6x 3x 2x Sides of the triangle are
12 × 20 = 240
22 Cost of painting its four wall at
= ×77 = 242 cm 17 × 20 = 340
7 the rate of Rs 25/m2 is Rs 3600

= 2(l + b) × h × 25 = 3600 25 × 20 = 500


71
We know, 142% =
50 = 2 × 9x × 2x × 25 = 3600
(240  340  500) 1080
S= 
Let Rectangle's Breadth = 50x =x=2 2 2
Rectangle's Length = 71x l = 6 × 2 = 12 m = 540
ATQ, Perimeter = 2(l + b) b=3×2=6m Area of triangle
 2(l + b) = 242 Cost of laying a carpet on its floor at
the rate of Rs. 90.50/m2 = S(S – a)(S – b)(S – c)
 2(71x + 50x) = 242

r
= (Area of floor) × 90.50
121x = 121 = 540  300  200  40
= 72 × 90 .50 = Rs. 6516

si
x = 1 = 36000 m2
50. (d)
an by
Area of rectangle = l × b

= 71x × 50x = 71 × 1 × 50 × 1
Surface area = 4r2
10000 m2 = 1 Hectares

36000 m2 = 3.6 hectares

n
New Radius = r + 5
= 3550 cm2 52. (a)
ATQ, 4(r + 5)2 – 4r2 = 704
48. (d) ja 4 [(r + 5)2 – r2] = 704
R s
d
Given, a = 120cm, b = 170cm, c = Radius of circular region =
22 2 2
a th

250cm. 4 × [r + 25 + 10r – r2] = 704


7 Width of path = x m
We know,
External radius of path (R)
7 1
a  b  c 120  1740  250 25 + 10r = 704 × 
ty a

S= = 22 4 d
2 2 = +x
25 + 10r = 56 2
di M

= 270 m
10r = 56 – 25 = 31 Area of path =  (R2 - r2)
ar = S(S – a)(S – b)(S – c) r = 3.1
 d 2
 d 2 
  x 
 –   
Diameter = 2 × 3.1 = 6.2 cm =    
= 270  150  100  20  2   2  
51. (a)
= 9000 m²
Ratio of side of  = 12x : 17x : 25x d2 d2 
Costing of levelling the field at the =   x 2  dx – 
4  =  [x + dx]
2

rate of Rs 7.40/m2 Perimeter of the triangle = 1080  4


= 9000 × 7.40 12x + 17x + 25x = 1080 = x (x + d) m2
= Rs. 66600
A



Aditya Ranjan (Excise Inspector) Selected Selection 165

Downloaded by Kunal Chauhan (kc6970929@gmail.com)


lOMoARcPSD|39315497

Mensuration-3D

MENSURATION -3D
21
3D
1. Determine the total surface area 5. A frustum of a right circular cone 8. The radius of the hollow sphere is
of a hemisphere closed at bottom. whose height is 18 cm, large base 21 cm. It is cut into two equal
radius is 25 cm and small base halves by a plane passing through
25 radius is 20 cm, is melted to form its centre. What is 75% of the
Radius of the hemisphere is
 a small sphere of diameter 2 cm. curved surface area (in cm2, rounded
How many spheres will be off to the nearest tens) of the
unit.
formed? 22
hemispheres? (take  = )
; < = ’ >?# 2 7

r
$ % +- 1 !"2 @A G& G
25 8* 1 "& > 7 & H 8B ’ &# ;
!"

si
 1 !"2 5 & 8 1 C & & : 1 7
SSC CPO 09/11/2022 (Shift-01) > 7 # !" ; 9# & 8
(*
an by
(a) 75 unit2
(c) 60 unit2
(b) 70 unit2
(d) 50 unit2
#
SSC CPO 09/11/2022 (Shift-02)
# 6 1 2 # 7 1
22
!

n
2. The ratio between the height and !  6= 3 4
(a) 6287 (b) 6862 7
radius of the base of a cylinder
SSC CPO 09/11/2022 (Shift-03)
ja
is 7 : 5. If its volume is 14836.5 (c) 4575 (d) 5857
R s
(a) 4160 (b) 3470
cm 3 , then find its total surface 6. Water is flowing through a
area. (take  = 3.14) (c) 2080 (d) 2270
cylindrical canal with an internal
a th

diameter of 7 m at he speed of 9. The total surface area of a solid


# $ % "& & metallic hemisphere is 462 cm2.
18 km/h. Find the volume of water
#’ ( ) * !" # This is melted and moulded into
flowing through the canal in 30
+ , - . / 0 *. ! " 2
1 a right circular cone. If the radius
22 of the base of the cone is the
ty a

 = 3.14 13 # 4 mintues. (take  = ) same as that of the hemisphere,


7
SSC CPO 09/11/2022 (Shift-01) 22
( 1 & C ; then#its height
& is: (use  =
di M

(a) 3391.2 cm2 (b) 5391.2 cm2 )


#!& 1 # ! ; +- 1 D5 7 7
(c) 4391.2 cm2
(d) 5591.2 cm2
3. What is the volume of a cube
!" 1 E 1 .B 1#7 1 ! #M ;
; # #  3 , 8/ 8! " 1 5 &
whose surface area is 54 cm2?
; < = ’ N 1
5 # # & 22 = ’ &
= F 4
54 cm 2
!"6 7
& & !"2
SSC CPO 09/11/2022 (Shift-01) SSC CPO 09/11/2022 (Shift-02) 22
(a) 28 cm3
(b) 3 cm 3
(a) 3,76,500 m3 (b) 3,56,500 m3 & = 7 3 H & 4
(c) 27 cm3 (d) 9 cm3 (c) 3,66,500 m3 (d) 3,46,500 m3 SSC CPO 09/11/2022 (Shift-03)
4. A hemispherical bowl whose 7. A hemisphere has 42 cm (a) 14 cm (b) 7 cm
radius is 21 cm is full of ice diameter. Find its curved surface
cream. Find the volume of the ice (c) 21 cm (d) 28 cm
area and the total surface area.
A

10. How many metallic spheres, each


22 C ,8 1 of !diameter
" 6 cm can be melted to
cream. (take  = )
7 mould into a solid cylinder of
;9 "&
& 7 & 2 diameter 8 cm and height of 18 cm?
&
8+ 1 !"2 9 1 : & !" 9 1 / 1 C ; #
SSC CPO 09/11/2022 (Shift-02) 5 & - 1 C
22
# = 3 4 (a) 2772 cm2, 4158 cm2 $ % ; # 1
7
(b) 3772 cm , 5158 cm
2 2 !"6
SSC CPO 09/11/2022 (Shift-02)
(c) 4770 cm , 4238 cm
2 2 SSC CPO 10/11/2022 (Shift-01)
(a) 20000 cm3 (b) 21000 cm3
(a) 6 (b) 12
(c) 22000 cm3 (d) 19404 cm3 (d) 3072 cm2, 4058 cm2
(c) 8 (d) 10

Aditya Ranjan (Excise Inspector) Selected Selection 166

Downloaded by Kunal Chauhan (kc6970929@gmail.com)


lOMoARcPSD|39315497

Mensuration-3D
11. The volume of a wall six times 15. A cylinder of height 8 cm and 19. A hemispherical bowl of internal
as high as its breadth and half radius 6 cm is melted and diameter 18 cm contains water.
as long as its height, is 23,958 converted into 3 cones of the This water is to be filled in
cm3. What is the breadth of the same radius and height that of cylindrical bottles of diameter 6
wall? cylinder. Determine the total cm and height 3 cm. The number
; & # 8. . 2! "P2* - curved
1 surface area of cones. of bottels required to empty the
$ % F % "@A - bowl is:
1 > !$ T ’%# "& / 1 ;
"& F $ % & C +- 1
# !" 5 !" "& #
; & % "@A # !"6 7 & 1 # !" #
"& $T% 1 #
SSC CPO 10/11/2022 (Shift-01) 1 "& $ % . 1
; . = ’ 1 &;
(a) 21 cm (b) 15 cm 1 : & # !"
!" = ’ ’ ;9
(c) 11 cm (d) 18 cm ;=
12. What is the total surface area of ! 6
a cuboid whose length, breadth SSC CPO 10/11/2022 (Shift-03)
SSC CPO 11/11/2022 (Shift-02)
and height are 7 cm, 5 cm and 9 (a) 180  cm2 (b) 60  cm2
cm, respectively? (a) 3 (b) 9
(c) 144  cm2 (d) 120  cm2
5 # : ’ Q
16. The heights of two right circular (c) 6 (d) 18

r
!" 2 % "@A cones are in%
" & $ 9 11 =: 5 )and
the ratio 20. The area of the base of a cone
is 616 cm2. If its slant height is

si
( 1 2 * 1 "& P 1the !perimeter
"6 of their bases are
SSC CPO 10/11/2022 (Shift-01) in the ratio 5 : 3. Find the ratio 20 cm, then what is the total
of their volumes. surface area of the cone?
(a) 190 cm2
(c) 286 cm2
an by (b) 403 cm2
(d) 386 cm2 ; < = ’ $ % 22
& (Use  = &7 )
#’

n
13. The length of a cold storage is 5 + ) * !" "& F#
times its breadth and its height #’ * ) . !" F# #=
is 8 m. The area of its four walls
’ &
ja
(including the doors) is 720 m2.
#’ & 1
8
!" $ %
R s
Find the total surface area of cold SSC CPO 10/11/2022 (Shift-03) 1 !"2 = ’
storage.
a th

(a) 8 :11 (b) 7 : 6 Q ! 6


= : @ & 2 (c) 5 : 9 % " @ (d)
A 3:4
22
* ’# "& $ % - 1 ! "
17. A cone-shaped storage tank's 3 = 7 F 4
% & ; & 3 &; height
! 4 and radius are 9 feet and
ty a

( 8 B 8 1! " = : @ & 7 feet, respectively. Determine SSC CPO 11/11/2022 (Shift-02)


how much liquid the tank can (a) 1352 cm2
di M

contain. (Take  = 3.14)


SSC CPO 10/11/2022 (Shift-02) (b) 1296 cm2
=Q ; & R 7 & 7" $T% "&
(a) 1290.6 m2 (b) 1275.3 m2
" & (c)
( 14967cm! "
2
91= ) P 7
(c) 1265.25 m2 (d) 1282.5 m2 (d) 1524 cm
7" 1 # 1 1 &2 S
14. Three metallic spheres of radii 10 &G !" 21. A hemispherical bowl of internal
cm, 8 cm and 6 cm, respectively, radius 6 cm contains a liquid.
are melted to form a single solid SSC CPO 11/11/2022 (Shift-01)
This liquid is to be filled into
cone of radius 12 cm. Find the (a) 471 cubic feet cylinderical shaped small bottels
curved surface area of the cone. of diameter 2 cm and height 4 cm.
(b) 450 cubic feet
(Correct to two places of decimal) How many bottles will be need
(Take  = 3.14) (c) 435.32 cubic feet to empty the bowl?
91= ) +B 1 2 - 1 2 (d) 461.58
" & / cubic1feet & / 1 ;
18. A cylindrical5tank of&capacity 64
A

; ’ # 7 & 1 H; !" H
litres has equal height and
+8 1 =radius. What
# would be the radius C "& , 1 $ % ;
!" = ;9 of the cylinder? (1 litre = 1000 cm3) & > 7 1
3 = 1 ; R64
S #7 & & 7
# & & G &#
! !" 4 7" $T% "& ; =
& & !" ! # 6
3 = 3.14 4 Q ! 6 SSC CPO 11/11/2022 (Shift-02)
SSC CPO 10/11/2022 (Shift-02) SSC CPO 11/11/2022 (Shift-01)
(a) 32 (b) 37
(a) 1664.50 cm2 (b) 1669.86 cm2 (a) 5 cm (b) 40 cm
(c) 1876.79 cm2 (d) 1864.41 cm2 (c) 38 (d) 36
(c) 50 cm (d) 4 cm

Aditya Ranjan (Excise Inspector) Selected Selection 167

Downloaded by Kunal Chauhan (kc6970929@gmail.com)


lOMoARcPSD|39315497

Mensuration-3D

22. The area of a trapezium shaped 26. A sphere is of radius 5 cm. What 31. The radius of the base of a cylinder
field as 1785 square feet. The is the surface area of the sphere? is 14 cm and its curved surface
distance between the two parallel area is 880 cm2, Its volume (in
* 1 !"
sides is 35 feet and one of the
parallel sides is 42 feet long. Q ! 6 cm3) is:  22 
 Taken   
What is the length of the other SSC CPO 10/11/2022 (Shift-01) 7 
parallel side? (a) 100cm2 (b) 150cm2
# 14
& cm ! "
1 1 " (c)# 200cm
& (d) 120cm2 2

"& ;9 880 cm² ! "


1785 feet2 ! " 27.
1 #The curved
& surface area and
circumference at the base of a cm
# 33 1 4 &
: ’ 35 feet ! " &
%
solid right circular cylinder are
"& 1 # & : ’ 12200 cm2 and 110 cm respectively.  22 
42 feet ! " & 1 # & :the’ height of the cylinder. = 
Find 7
# !"6 ; < # ; 9 SSC CPO 23/11/2020 (Shift-1)
SSC CPO 11/11/2022 (Shift-03) "& & & (a)91078
1 = ) 8 8 B(b)
B 3080
(a) 60 feet (b) 35 feet 1
8
"& ++B 1 !" #
(c) 9240
$ % (d) 6160
(c) 40 feet (d) 65 feet 32. A solid metallic cube side of 9 cm

r
SSC CPO 10/11/2022 (Shift-01) and a solid metallic cuboid having
23. What is the difference between
the total surface area and the (a) 24 cm (b) 22 cm dimensions 5 cm, 13 cm, 31 cm

si
curved surface area of a cone (c) 20 cm (d) 18 cm melted and recast into a single
whose radius is 35 cm? 28. What is the radius of a normal cube. What is the total surface area

(take  =
an by
22
7
)
cylinder whose height is 21 cm
and curved surface area is 1386
(in cm2) of the new cube?
9 cm : ’ R M;

n
5 cm, 13 cm, 31 cm ;1
35 cm ; = ’ 22
cm2? (Take  = ) (dimensions) ; M; 5
ja "& ;9 1 E7 5 & 5 # #
R s
# & ! 6 1 ? # Q ! cm² 1
5 # ’ 4
$ % 8+ 1 "& ;9
a th

SSC CPO 11/11/2022 (Shift-03) &


22
(a) 3850 cm2 (b) 3704 cm2  =3
+ 8. -! /" 6 17 4 SSC CPO 23/11/2020 (Shift-2)
(c) 3750 cm 2
(d) 3675 cm 2
SSC CPO 10/11/2022 (Shift-02) (a) 865 (b) 1362
ty a

24. If the area of the base of a cone (a) 10.5 cm (b) 3.5 cm (c) 1176 (d) 2744
is 154 cm 2 and the area of its (c) 7 cm (d) 10 cm 33. If a wheel has diameter 42 cm,
curved surface is 550 cm2, then then how far does the wheel go
di M

29. The height and curved surface


its volume is: area of a right circular cylinder are (in metres) in 12 revolutions?
= ’ 154
& 7 cm and 70  cm 2 . Its total
 22 
cm2 ! " "& ;9 ! surface area is:  Taken   
7 
550 cm2 !"2 # &U ; < # $T% "&
42 cm
SSC CPO 11/11/2022 (Shift-03) ;9 91= ) ( 1 " &! C !"2
70  8
1 !" ’ ! 12 % Q & 1 #
(a) 1232 cm3 (b) 1122 cm3
Q ! 6  22 
(c) 1434 cm3 (d) 1535 cm3 =
6 
SSC CPO 10/11/2022 (Shift-03) 7
25. What is the total surface area of
a solid right circular cylinder of (a) 140  cm2 (b) 150  cm2 SSC CPO 23/11/2020 (Shift-2)
radius 7 cm and height 8 cm? ( (c) 180  cm 2
(d) 120  cm2 (a) 15.84 (b) 23.27
30. If the radius of a sphere (c) 21.45 (d) 17.64
A

22
= ) increases by 10%, then what
7 34. The ratio of the total surface area
would be the change in the and volume of a sphere is 2 : 7.
( 1 "& - 1 surface
$ T %area of; the sphere?
Its radius is:
U ; < # ’
+BK OA U
22 !"2 # 1 #Q’ 8 ) ( !"
#= ! " 6 3 4
7 &; # ! 6 &
SSC CPO 09/11/2022 (Shift-03) SSC CPO 10/11/2022 (Shift-03) SSC CPO 25/11/2020 (Shift-2)
(a) 560 cm2 (b) 660 cm2 (a) 20% (b) 21% (a) 7 cm (b) 10 cm
(c) 850 cm 2
(d) 760 cm 2 (c) 31% (d) 25% (c) 10.5 cm (d) 7.5 cm

Aditya Ranjan (Excise Inspector) Selected Selection 168

Downloaded by Kunal Chauhan (kc6970929@gmail.com)


lOMoARcPSD|39315497

Mensuration-3D

35. A solid lead sphere of radius 11 # & 14 cm ! " 42. A wheel has diameter 84 cm, then
cm is melted and recast into small "& 6160 cm³
# !" how far does the wheel go (in
solid spheres of radius 2 cm each. meters) in 16 revolutions?
;9 (cm²) 1 &
How many maximum number (in
integer) of such spheres can be  22 
 22   Take   
made?  =   7 
 7 
11 cm ; T%
SSC CPO 25/11/2020 (Shift-1) ! C -, 1 !" +
5 ! "2 "& HM
cm ; > 7 (a) 880 T O (b) 940 1 ! # & 31 7&
!" O (c) 778 1(d) 660
 22 
G 3 1 4 39.# If the
! volume of! " sphere
a 6 is 4851  1 #  
cm³, then its surface area (in cm²)  7 
SSC CPO 24/11/2020 (Shift-1)
SSC CPO 09/12/2019 (Shift-01)
(a) 30 (b) 166  22 
is:  Taken    (a) 42.24 (b) 21.12
(c) 100 (d) 125 7 
36. A solid metallic cube of side 9 cm (c) 36.28 (d) 27.48
4851 cm³# ! "2
and a solid metallic cuboid having

r
dimensions 5 cm, 13 cm 31 cm F cm² 1 34 43. &The length of a right circular cone
are melted to form a single cube. is 35 cm and the area of its curved
 22 

si
How much (in Rs.) is the cost to =  surface is four times the area of
7
polish the new cube at a rate of its base. What is the volume of
Rs.10 per cm2?
9 cm : ’
an by
; M(a)
SSC CPO 25/11/2020 (Shift-2)
; 1399
5 # (b) 1268
the cone (in 10–3 m3 and correct
up to three decimal places)?

n
M 5Scm, 13 cm, 31 cm ;1 ; (c) 1386 (d) 1427
U ; < = ’ $ %
M; 5 # : 5 &
40. Let A and B be two cylinders such
ja "& ;9
5 # # !" #that5the#capacity of A is the same
R s
V = & # 1 cm W
2
& + B as the
H capacity of B. The ratio of % & ’# !"
the diameters of A and B is 1 : 4. Q ! "X . 31. + 1B "& = 1 ;
a th

# 3W 1 4 What is6 the ratio of the heights


SSC CPO 24/11/2020 (Shift-1) of A and B? RS # ! 46
(a) 8,650 (b) 27,440 1 #A B
"& A @ & SSC! " CPO 09/12/2019 (Shift-01)
(c) 11,760 (d) 13,620
ty a

B
& & A& & (a)! "3.316 (b) 2.994
37. The internal measures of a "B& C 1 :#4’ ! "A
cuboidal room are with length as (c) 2.625 (d) 3.384
di M

"B& $T% #’ &


12 m, breadth as 8 m and height
as 10 m. The total cost (in Rs.) SSC CPO 25/11/2020 (Shift-2) 44. 60 discs each of diameter 21 cm
of whitewashing all four walls of 1
(a) 3 : 16 (b) 1 : 16
the room and also the ceiling of and thickness cm are stacked
3
the room, if the cost of (c) 16 : 1 (d) 16 : 3
whitewashing is Rs.25 per m2 is: one above the other to form a right
41. The internal length, breadth and
circular cylinder. What is its
5 # : & ; height
1 & of a cuboidal room are 12
& 1 12 m, % "8@ A m, 8 m and 10 m, respectively.  22 
The total cost (in Rs.) of volume in m3  Take   
m 10Tm% ! "
"& $ 7 
whitewashing only all four walls
W 8m²* ! "H2 1 & of the room at the cost of Rs.25
: % & ; & S m², >is:
per : /B @R 2 #1 HM
& # ’ 3 W 5 #1 : 4 1
A

& ; 1& " & 13 &71 ! " 2 $


& 12 m, % "8 @
m A " & 10 $T%
SSC CPO 24/11/2020 (Shift-2) m !" & # & W & G8 * !"2
(a) 12,400 (b) 12,000 Hm² ! " 2 1& ; % & U ; ;< & # # 1
(c) 18,000 (d) 13,600 & # ’ 3W 1 . 4 1# 1 # ! 6
38. The radius of the base of a cylinder &
is 14 cm and its volume is 6160 SSC CPO 09/12/2019 (Shift-01)
cm3. The curved surface area (in SSC CPO 25/11/2020 (Shift-2)
(a) 7.62 × 10–2 (b) 8.05 × 10–3
 22  (a) 12,600 (b) 11,400
cm²) is:  Taken    (c) 6.93 × 10–3 (d) 4.25 × 10–2
 7  (c) 10,000 (d) 18,000

Aditya Ranjan (Excise Inspector) Selected Selection 169

Downloaded by Kunal Chauhan (kc6970929@gmail.com)


lOMoARcPSD|39315497

Mensuration-3D

45. A cylinder 84 cm long is made of 8.4 cm ; ’ 6.31 ;


steel. Its external and internal 5 & U ; < # # 5 25.2 ! "
diameters are 10 cm and 8 cm re- !"2 12 cm ! " # 1 F T% ; U
spectively. What is the volume of
F T% # !"6 3 F <# & = 1 !; " = ’ &
the steel in the cylinder (in 10–3 F F T % # ’ Q
m3 and correct up to three deci- RS # ! ! # % ! 4
mal places)? SSC CPO 09/12/2019 (Shift-02) SSC CPO 11/12/2019 (Shift-02)
(a) 2 : 1 (b) 3 : 2
-, 1 # 1 (a) 7.0Rcm7
# (b) 6.0 cm
(c) 1 : 2 (d) 2 : 3
!" !& "(c)
& 6.5 cm & (d) 5.5 cm
C 91= ) +B 1 "49.
& -The volume
1 ! "of a solid#right circu- 53. The ratio of radius of the base
and the height of a soild right
" &+larBcone is 600  cm , and the di-
3
X. .
1 R7 # 1Q1 ! " 3 circular cylinder is 2 : 3. If its
ameters of its base is 30 cm. The
= 1 ; # RS # ! surface
! " 4 6 area (in cm2) of the volume is 202.125 cm³, then its
total total surface area is: (Take
SSC CPO 09/12/2019 (Shift-01) cone is :
22
U ; < = ’   # )/ B B
(a) 2.112 (b) 9.504 7
.
1!"2 "& & C .B 1

r
(c) 2.376 (d) 4.752 !" = ’ ’ 8
1 4 3 1 U ; < #
"& F T% #’

si
46. Total surface area of a right # ! 6
202.125
# cm³ ! " 2
circular cylinder is 1848 cm². The SSC CPO 11/12/2019 (Shift-01) ’ # !
ratio of its total surface area to
an by (a) 496  (b) 480 
the curved surface area is 3 : 1. 22
(c) 255  (d) 472   4
The volume of the cylinder is: 7
50. The total surface area of a solid

n
22 hemisphere is 1039.5 cm2. The SSC CPO 11/12/2019 (Shift-02)
(Take   ) volume (in cm3) of the hemisphere
7 (a) 192.5 cm² (b) 154 cm²
ja
R s
 22  (c) 168 cm² (d) 115.5 cm²
U ; < # ’ is:  Take   
 7 
a th

1848 cm2 ! " ’ 54. The curved surface area of a right


;9 #’ . ) + ’ circular cone is 156 and the
.
+B.P0* 1 !" # its3 base
radius of 1 is 12 cm. What
!" # # # ! 6
22  is the volume of the cone in cm³?

ty a

22 1 4 # 1 ! # 6  7 
   ; <U = ’ ;9
31 # 7
4
SSC CPO 11/12/2019 (Shift-01) 156 "& &
di M

SSC CPO 09/12/2019 (Shift-02) (a) 2225.5 (b) 2530.6 !" = ’ (cm³ 1) # # ! 6
(a) 3696 cm 3
(b) 4312 cm 3 (c) 2425.5 (d) 2525.6
SSC CPO 11/12/2019 (Shift-02)
51. A solid metallic sphere of radius x (a) 192 (b) 210
(c) 4002 cm 3
(d) 4851 cm 3
cm is melted and then drawn into
126 cones each of radius 3.5 cm and (c) 240 (d) 180
47. The radius of the base of a solid
right circular is 8 cm and its height 3 cm. There is no wastage of 55. A right circular cone of largest
material in this process. What is volume is cut out from a solid
height is 15 cm. The total surface
the value of x? wooden hemisphere. The
area of the cone is:
x 1 ; ’ remaining material is what
U ; < = ’ 5 & !" "& & percentage
F . 0 *of the1volume of the
- 1 !" "& F % +" *& . 1 1 original hemisphere?
$ % ; +8/ = ’
!" = ’ ’ # Q ! ! "6 H 9 1 1 H @A J
A

SSC CPO 09/12/2019 (Shift-02) C # ! x! 1 ! #" : ; # ; U ; <


SSC CPO 11/12/2019 (Shift-01)
& # !" H 9
(a) 128  (b) 200 
(a) 10.5 (b) 7 % @A 1 J
(c) 136  (d) 120  # H = !"6
(c) 35 (d) 21
48. A solid metallic sphere of radius SSC CPO 12/12/2019 (Shift-01)
52. A solid metallic sphere of radius
8.4 cm is melted and recast into 1
6.3 cm is melted and recast into
a right circular cylinder of radius (a) 50% (b) 33 %
a right circular cone of height 3
12 cm. What is the height of the 25.2 cm. What is the ratio of the
cylinder? (Your answer should be 2
diameter of the base to the height (c) 75% (d) 66 %
correct to one decimal place.) of the cone? 3

Aditya Ranjan (Excise Inspector) Selected Selection 170

Downloaded by Kunal Chauhan (kc6970929@gmail.com)


lOMoARcPSD|39315497

Mensuration-3D
56. The volume of a solid right & = 7 -- 1 2
circular cylinder is 5236 cm3, and 63. The height of a cylinder is of its
"& ++ 1 % "@A !"2 7 & 3
its height is 34 cm. What is its
curved surface area (in cm²)? # # !"2 F diameters.
T% Its volume is equal to
the volume of a sphere whose
% "@A & & ! " radius & ! is #4 cm.
1 What# is the
22 curved surface area (in cm²) of the
(Take   ) # # ! 6
7 cylinder?
SSC CPO 12/12/2019 (Shift-02)
U ; < 5236
# #
(a) 7676 cm³ (b) 6776 cm³ 2
cm³ !"2 " & 34 cmF ! "T % # F T% !" C
3
;9 (cm2 1) (c) 6546 cm³ (d) 6786 cm³
4 cm
# ;
22 60. Twelve solid hemisphere of the
31 #  4 # & & !" #
7 same size are melted and recast
to in a right circular cylinder of (cm² 1) # !"6
SSC CPO 12/12/2019 (Shift-01)
diameter 7 cm and height 28 cm.
(a) 1496 (b) 1573 SSC CPO 13/12/2019 (Shift-01)
What is the radius of the
(c) 1650 (d) 1804 hemisphere? 112
57. A conical vessel whose internal (a)  (b) 32 
1 # & ; &! 3
base radius is 18 cm and height
5 & ( 1 C "& 8- 1 F T%

r
60 cm is full of a liquid. The entire
liquid of the vessel is emptied ; U ; < # # (c) 128  ! " (d) 40 

si
into a cylindrical vessel with 3
1
internal radius 15 cm. The height 64. The ratio of the volumes of two
an by
(in cm) to which the liquid rises SSC CPO 12/12/2019 (Shift-02)
right circular cylinder A and B is
in the cylindrical vessel is: (a) 4.5 cm (b) 3.5 cm
x

n
= Q; & #2 (c) 3 cm
& (d) 3.8 cm
y and the ratio of their height is
& 18 cm T % 61.
60 cm
"& F A right circular cylinder of
a : b. What is the ratio of the radii
!"2 & ja S : & ! maximum
’ ! " possible
# size is cut out of A and B?
R s
from a solid wooden cube. The
& & S # & #
remaining material of the cube is U ; <A "B& # #
1 : & !"2 &
a th

what percentage of the original


15 cm ! " # & # 1 cube?
& S x
#’ y !" "& F# F T%
# F T% F & F 6  = 3.14)
(Take
SSC CPO 12/12/2019 (Shift-01) @A 5 # a : b1 ! "A : ;"B& #
ty a

(a) 30.2 cm (b) 28.8 cm & U ; < # 7 &


(c) 27 cm (d) 24 cm # !" 5 # = SSC 1 H 13/12/2019
CPO ; R ; (Shift-02)
di M

58. A cuboidal tank has 25000 litres 5 # # H = !"6


of water. If the depth of the xb xb
1 31 #  = 3.14 4 (a) (b) ya
ya
cuboid is of its length and
5 SSC CPO 13/12/2019 (Shift-01)
1 (a) 22.4 (b) 21.5 xa yb
breadth is of its length, then (c) (d)
8 (c) 22.8 (d) 21.8 yb xa
the length of the tank is:
62. The radius and height of a right 65. The length and breadth of a
5 # & 7 1 8 * B B Bcircular
7& # in! the
cone are " ratio 1 :
cuboid store are in the ratio 2 : 1
1 (2.4). If its curved surface area is and its height is 3.5 metres. If
!& 5
!" 2502.5 cm², then what is its the area of its four walls
22 (including doors) is 210 m², then
"& % "@A + D(Take
volume? - ! " 2= ) 7"
its volume is.
A

1 7
5 # : & ; R7 &
SSC CPO 12/12/2019 (Shift-02) U ; < = ’ "& F T%
% "@A #’ 8 ) + !
(a) 12 m (b) 10 m # ’ 1 : (2.4) ! " ;9
3.5 1 7 & ! " % & ;
(c) 15 m (d) 18 m 2502.5 cm² !"2 #
! 4 210m² ! " 2
59. A rectangular sheet of paper 22
which is 88 cm long and 11 cm # ! 6 3
1 #7 4 # !
wide is rolled to form a cylinder SSC CPO 13/12/2019 (Shift-02)
of height equal to its width of the SSC CPO 13/12/2019 (Shift-01)
(a) 700 m³ (b) 679 m³
paper. What is the volume of the (a) 8085 cm³ (b) 8820 cm³
cylinder so formed? (c) 567 m³ (d) 1050 m³
(c) 11550 cm³ (d) 13475 cm³

Aditya Ranjan (Excise Inspector) Selected Selection 171

Downloaded by Kunal Chauhan (kc6970929@gmail.com)


lOMoARcPSD|39315497

Mensuration-3D

ANSWER KEY
1.(a) 2.(a) 3.(c) 4.(d) 5.(b) 6.(d) 7.(a) 8.(c) 9.(a) 10.(c)

11.(c) 12.(c) 13.(d) 14.(d) 15.(a) 16.(c) 17.(d) 18.(b) 19.(d) 20.(c)

21.(d) 22.(a) 23.(a) 24.(a) 25.(b) 26.(a) 27.(c) 28.(a) 29.(d) 30.(b)

31.(d) 32.(c) 33.(a) 34.(c) 35.(b) 36.(c) 37.(a) 38.(a) 39.(c) 40.(c)

41.(c) 42.(a) 43.(b) 44.(c) 45.(c) 46.(b) 47.(b) 48.(d) 49.(b) 50.(c)

51.(a) 52.(c) 53.(a) 54.(c) 55.(a) 56.(a) 57.(b) 58.(b) 59.(b) 60.(b)

61.(b) 62.(d) 63.(c) 64.(a) 65.(a)

r
si
an by SOLUTIONS
SOLUTIONS

n
1. (a)
ja 4. (d) 6. (d)
R s
Total surface area of hemisphere Given,
2 3
r
a th

= 3r2 Volume of hemisphere = Speed of flowing water


3
 25 
2 = 18 km/h
25
 3     3  2 22 Diameter of canal = 7 m
 = 75 m
2
   =   21  21  21 = 19404 cm3
3 7 7
ty a

2. (a) Radius of canal = cm


Hints: divisibility rule by 9 2
Given, Hence, only option (d) is correct. Distance covered in 30 minutes
di M

Ratio of height and base radius 5. (b) (9 km) will be considered as


=7:5 height.
Given,
Volume of cylinder = r2h h = 9000 m
Height of frustum, h = 18 cm
14836.5 = 3.14 × (5x)2 × 7x Volume of water flowing through
Large base radius of frustum, R
14836.5 = 25 cm the canal in 30 minutes.
 x3  = 27 = r2h
3.14  25  7 Small base radius frustum, r =
20 cm 22 7 7
x = 3 =    9000 = 346500m3
Diameter of sphere = d = 2cm 7 2 2
Hence, height = 21 cm and base
Radius of sphere, rsphere = 1 cm 7. (a)
radius = 15 cm
Let the numbers of sphere be Given,
Total surface area = 2r(r + h)
formed = n. Diameter of hemisphere = 42 cm
= 2 × 3.14 × 15(15 + 21)
 Volume of frustum = n ×
42
A

= 2 × 3.14 × 15 × 36 = 3391.2 cm2 volume of spheres radius, r =  21cm


3. (c) 2
1 4 Curved surface area of hemisphere
Given, surface area of cube  h  R  r  R.r  = n  (rsphere )3
2 2

3 3 = 2r2
= 54 cm2
Surface area of cube = 6 × (side)2 18 (252 + 202 + 25 × 20) = n × 22
4 × (1)3 = 2  21  21 = 2772 cm2
54 = 6 × (side) 2 7
18 (625 + 400 + 500) = 4n Total surface area of hemisphere
9 = (side)2
n = 6862.5 = 3r2
side = 3 cm
Required numbers of sphere 22
Now,
= 6862 = 3  21  21 = 4158 cm2
Volume of cube = (side) = 27 cm 3 3 7

Aditya Ranjan (Excise Inspector) Selected Selection 172

Downloaded by Kunal Chauhan (kc6970929@gmail.com)


lOMoARcPSD|39315497

Mensuration-3D

8. (c) 13. (d)


25  1 5
C.S.A of Hemisphere = 2r2 Let breadth = x = =
95 9
then length = 5x
22
 2  21  21 = 2772 cm2 Area of four walls = 2(l + b)h Thus, ratio of their volume
7
 2(5x + x)8 = 720 m =5:9
2772  75 17. (d)
75% of CSA = 96x = 720 m
100
 x = 7.5 m 1 2
= 2080 cm2 (Approx) Volume of cone = r h
 Breadth = 7.5 m and Length 3
9. (a)
= 37.5 m
Total Surface area of hemisphere 1
TSA of cold Storage = 2(lb +bh + =  3.14  7  7  9
= 3r2 3
hl)
22 2 = 461.58 cubic feet
 462  3  r = 2(37.5 × 7.5 + 7.5 × 8 + 8 × 37.5)
7 18. (b)
= 2(281.25 + 60 + 300)
 462 × 7 = 22 × 3 × r2 = 2 × 641.25 Capacity of the tank = 64 liter
462  7 = 1282.5 m² Volume = 64 × 1000 cm3
 r2  = 49
22  3 14. (d)  Volume of tank = 2h

r
 r = 7 cm Volume of cone = Volume of all  64 × 1000 = r2h
three sphere

si
A.T.Q,  64000 = r2h
The hemisphere is converted into 1 4 As, r = h
 R 2h    r13  r23  r33 
cones. an by 3 3  r3 = 64000
Volume of hemisphere = volume
 R h  4  r1  r2  r3 
2 3 3 3  r = 40 cm
of Cone

n
19. (d)
2 3 1 2  122h = 4(103 + 83 + 63)
 r  r h Radius of Hemispherical Bowl
3 3 144h = 4(1000 + 512 + 216)
ja
R s
18
2 3 1 2 4  1728 R =  9 cm
 7  7 h  h  = 48 2
3 3 144
a th

Radius of Cylinderical Bottle


 14 = h
Slant height, l = 122  482
Height of the cone = 14 cm. 6
r=  3 cm
10. (c) = 144  2304 2
ty a

Number of Metallic Spheres Height of Cylinderical Bottle


= 2448 = 49.48
Volume of Cylinder R H 2
CSA of cone = Rl h = 3 cm

di M


Volume of Sphere 4 3 = 3.14 × 12 × 49.48 = 1864.4064 No. of required Bottles
r
3 = 1864.41cm2 Volume of bowl
2
15. (a) = Volume of bottle
3R H 3  4  4  18
  =8 Here, The cylinder is converted
4r 3
4333 into 3 cones of the same height 2
11. (c) radius. R 3 2R 3 2 9 9 9
= 3 = = = 18
Let the breadth of the wall be x, So their, volume will be equal. 2
r h 3r 2
h 3 333
height be 6x and length be 3x. Volume of cylinder = 3 × volume 20. (c)
Volume = Length × Breadth × of cones
Area of the base of cone = 616
Height
l  62  82  r2 = 616
 23958 = 3x × x × 6x
= 36  64 22 2
3 23958   r  616
 x 
A

= 1331 7
18 = 100 = 10 cm2
 x = 11 616  7
CSA of all three cones = 3 × rl 2
 r 
Thus, breadth of the wall is 11 cm. 22
= 3 ×  × 6 × 10 = 180 cm2
12. (c) 16. (c)  r = 14 cm
TSA of cuboid h1 : h 2 = 1 : 5 Total surface area of the cone
= 2(lb + bh + hl) P1 : P2 = 5 : 3 = r  l  r 
= 2 (7 × 5 + 5 × 9 + 9 × 7)
r1 : r2 = P1 : P2 = 5 : 3
= 2 (35 + 45 + 63) 22
=  14  20  14  = 44 × 34
= 2 × 143 V1 r12h1 7

= 286 cm2 V2 r22h2 = 1496 cm2

Aditya Ranjan (Excise Inspector) Selected Selection 173

Downloaded by Kunal Chauhan (kc6970929@gmail.com)


lOMoARcPSD|39315497

Mensuration-3D

21. (d) = 252 – 72 = 625 – 49 = 576 22


Given, = × 14 × 14 × 10
= 24 cm 7
Radius of hemispherical bowl
1 = 6160 cm3
R = 3 cm Volume of cone =  r 2h
Radius of cylindrical bottle 3 32. (c)
Volume of cube = a3 = 729 cm3
2 1 22
r  1cm =   7  7  24 = 1232 cm3 Volume of cuboid
2 3 7
= 5 × 13 × 31 = 2015 cm3
Height of cylindrical bottle 25. (b) Total volume
h = 4 cm Total surface area of cylinder = = 2015 + 729 = 2744 cm3
No. of required bottles 2r(r +h)
Volume of new cube (A3) = 2744
Volume of bowl 22 A = 14 cm
= 2  7  7  8  = 660 cm2
= Volume of bottle 7 Total surface area of new cube
26. (a) = 6A2
2 Surface Area of sphere = 4r2 = 6 × 142
R 3 2R 3
= 3 = = 1176 cm2
 4  5   100  cm2
2
r 2h 3r 2h
33. (a)

r
27. (c) One revolution of wheel = 2r
2666
= = 36 Given,

si
3 111  4 12 revolution of wheel = 12 × 2
Curved surface area of cylinder
22. (a) = 2200 cm2 22
an by
Let the other parallel sides be l. × × 21 (  2r = 42 cm)
Perimeter of base, 2r = 110 cm 7
Area of trapezium ATQ, CSA of cylinder = 2rh = 72 × 22

n
1  2200 = 110 × h = 1584 cm
(Some of parallel sides) × h  h = 20 cm = 15.84 m
2
ja Thus, height of the cylinder = 20 34. (c)
R s
1 cm
 1785    42  l   35
2 28. (a) 4 r 2 2
 
a th

1785  2 CSA of cylinder = 2rh 4 3 7


 42  l  r
35 3
22
 1386  2   r  21
 42  l  102 7 21
r=
ty a

 l  102 – 42 = 60 feet 1386  7 2


r = 10.5 cm
Thus, the length of the other 44  21 r = 10.5 cm
di M

parallel sides = 60 feet. 29. (d) 35. (b)


23. (a) CSA of cylinder = 2rh 4 4
TSA of cone = rl + r2  70 = 2r × 7 x×  × (2)3 = × (11)3
3 3
CSA of cone = rl r=5
 TSA – CSA = r2 TSA of cylinder = 2r(r + h) 113
= 2× 5(5 + 7) x=
22 23
  35  35 = 3850 cm2
7 = 120 cm2 = 166
24. (a) 30. (b) 36. (c)
base area of cone = r2 Net change in area Volume of cube = 93 = 729 cm3
22 2 10  10  Volume of cuboid

 154  r = 10  10   % = 21% = 5 × 13 × 31 = 2015 cm3
7  100 
Taken = 729 + 2015
A

 r = 7 cm 31. (d) = 2744 cm3


Again, curved surface area of Radius = 14cm Volume of new cube (a3)
cone = 550
Curved surface area = 880 cm2 = 2744
 rl  550  2rh = 880 (h = height of a = 14 cm
22 cylinder) Total surface area of new cube
  7  l  550
7 = 6a2
880  7
550 = = 6 × 196
l  = 25 cm 2  22  14
22 = 1176 cm2
h = 10 cm Total cost of polish of a new cube
h  l 2 – r2 Volume = r2h = 1176 × 10 = Rs. 11760

Aditya Ranjan (Excise Inspector) Selected Selection 174

Downloaded by Kunal Chauhan (kc6970929@gmail.com)


lOMoARcPSD|39315497

Mensuration-3D

37. (a) 41. (c) 8


Total area of room Total area of 4 walls = = 4cm
2
= 2(l + b) ×h + lb = 2(l + b)×h
We know,
= 2(12 + 8) × 10 + 12 × 8 = 2(12 + 8) × 10
Volume of the cylinder
= 400 + 96 = 496 cm2 = 400 cm2 = (R2 – r2)h
Total cost Total cost = (5² – 4²) × 84
= 496 × 25 = Rs. 12400 = 400 × 25 = Rs. 10000 =  × 9 × 84 = 2376 cm³
38. (a) 42. (a)
= 2.376 × 10–³ m³
Volume of cylinder = r2h Diameter = 84cm (given)
Radius = 42 46. (b)
22 One revolution = 2r T.S.A = 2r(r + h)
 14  14  h  6160
7 Wheel go in 16 revolutions C.S.A = 2rh
Given,
[  r = 14 cm] 22
= 2  42  16
7 2r(r  h) 3
6160  7 
h= = 4224 cm = 42.24m 2rh 1
14  14  22
43. (b) = r + h = 3h
h = 10 cm h = 35 cm (given) = r = 2h

r
Curved surface area = 2rh rl = 4r2
l = 4r r
=h=

si
22 l2 = h2 + r2
=2× × 14 × 10 = 880 cm2 2
7 (4r)2 = (35)2 + r2
= 2r(r + h) = 1848
39. (c) an by 4 3
16r2 = 1225 + r2
15r2 = 1225
=2×
22
7
×r


r
r  
 2
 = 1848

n
Volume of a sphere = r 1225
3 r2 =
15
22 3r
4 3 Volume of cone =2× r = 1848
r = 4851 ja 7 2
R s
3
1 2 1 22 1225 = r2 = 196
4851  7  3  r h     35
r =
3 3 3 7 15 = r = 14 cm
a th

4  22
2994 r 14
441  21 = 2994 cm3 = 
r3 = 106 m3 h= = 7 cm
8 2 2
= 2994 × 10 – 6 m3
21 Volume of cylinder
ty a

r= cm = 2.994 × 10–3 m3
2 44. (c) 22
Area of a sphere = 4r2 60 disc each of diameter 21cm = r2h = × 14 × 14 × 7
7
di M

22 21 21 1 = 4312 cm3
=4×   Thickness = cm
7 2 2 3 47. (b)
R = 8cm
= 1386 cm2 21
Radius = = 10.5 cm h = 15cm
40. (c) 2 l2 = r2 + h2
Both cylinders capacity volume 1 l2 = 64 + 225
equal. Height = 60  = 20cm l = 17 cm
3
2r1 : 2r2 = 1 : 4 Total surface area of cone
Volume of cylinder = r2h
= r(r + l)
1 22
r1 : r2 = :2 =  × 8(8 + 17)
2 = × 10.5 × 10.5 × 20
7 =  × 8 × 25
 r12h1 1 = 693 cm3 = 6.93 × 10–3 m³ = 200

A

 r22h 2 1 Note : In this question, 48. (d)


We can check the divisibiling of A.T.Q,
(11 and 7)
h1 r22 4 3
 So, its only (6.93 × 10–3 m³) is r = r²h
h 2 r12 divisible by both (11 and 7) 3
45. (c)
4 84  84  84
2
2
16 Height of cylinder = 84 cm   12  12  h
= 2
 External Radius of cylinder 3 1000
1 1
  10 49  84
2 = = 5cm =h=
2 250
h1 : h2 = 16 : 1 = h = 5.488  5.5 cm
Internal radius of cylinder

Aditya Ranjan (Excise Inspector) Selected Selection 175

Downloaded by Kunal Chauhan (kc6970929@gmail.com)


lOMoARcPSD|39315497

Mensuration-3D

49. (b) Ratio of radius of base and height 2 3 1 3


= 2x : 3x Remaining volume = r - r
30 3 3
Radius = = 15cm r2h = 202.125 cm3
2 1 3
22 = r
3
1 2 × 2x × 2x × 3x = 202.125
r h = 600 7
3 1 3
r
202.125  7 3 100 = 50%
1 Required % = 2
= x3 = r 3
× 15 × 15 × h = 600 2  2  3  22
3 3
x = 1.75 56. (a)
h=8
R = 15 Radius of cylinder = 2 × 1.75 = 3.5 Volume of cylinder = r2h
Height of cylinder = 3 × 1.75 = 5.25
l=+ (15)2 (8)2  225  64  289 22
Total surface area of cylinder  × r2 × 34 =5236
= 17 cm 7
T.S.A of cone = r(r + l) 22 r=7
=  × 15 (15 + 17) = 2r(r + h) = 2 × × 3.5 (3.5 +
7 C.S.A of cylinder = 2rh
=  × 15 × 32
= 480cm2 5.25)
22

r
50. (c) 2× × 7 × 34 = 1496 cm²
22 7
3r2 = 1039.5 =2× × 3.5 × 8.75 = 192.5 cm²

si
7 57. (b)
22 54. (c)
=3× × r² = 1039.5 1 2
7
21
an by Radius of cone = R = 12 cm
Curved surface area of cone = rl
ATQ,
3
r h = r2h

n
r= = 10.5 cm = rl = 156 1
2   × 18 × 18 × 60 =  × 15 × 15 ×h
= 12 × l = 156 3
Volume of Hemisphere
ja = l = 13 1
R s
2 22  × 18 × 18 × 60 = 15 × 15 × h
=  × 10.5 × 10.5 × 10.5 l2 = r2 + h2 3
3 7
a th

169 = 144 + h2  h = 28.8 cm


= 2425.5 cm³
h=5 58. (b)
51. (a)
1 Volume of tank = 25000 litres
4 1 Volume of cone = ×  × r² × h
x3 = 126 ×  × 3.5 × 3.5 × 3 3 As we know,
ty a

3 3 1000 litres = 1m3


63  49  3 9261 1 Let the length of the tank be x m
= ×  × 12 × 12 × 5
di M

= x3 =  3
8 8 1 x
= 240cm³ Depth of tank = x × 
5 5
21 55. (a)
x= = 10.5 cm
2 1 x
Breadth of tank = x × 
52. (c) 8 8
4 Volume of cuboid = L × B × H
Volume of sphere = r3
3 x x
x× × = 25
1 5 8
Volume of cone = r2h
3 x3 = 53 × 23
1 2  x = 10m (length of tank)
4 Volume of cone = r h
= ×  × 6.3 × 6.3 × 6.3 3 59. (b)
3
A

2 3 Length = 88 cm
1 Volume of hemisphere = r Width = 11 cm
=  × r2 × 25.2 3
3 Height of cylinder = 11 cm
The radius of the cone and
63 hemisphere be r cm Length will be the circuler base of
=r= = 6.3, Diameter cylinder, So-
10 Height of cone = r
2r = 88
= 6.3 × 2 = 12.6 cm
1 2 1 3 r = 14 cm
Required ratio = 12.6 : 25.2 Volume of cone = r × r = r
=1:2 3 3 Volume of cylinder = r2h
53. (a) 2 3 22
Volume of cylinder = r2h Volume of hemisphere = r  × 14 × 14 × 11 = 6776 cm3
3 7

Aditya Ranjan (Excise Inspector) Selected Selection 176

Downloaded by Kunal Chauhan (kc6970929@gmail.com)


lOMoARcPSD|39315497

Mensuration-3D
60. (b) 64. (a)
l = (12x )2  (5x )2  169x 2 = 13x
Formula Used:
Curved surface area = rl Volume of cylinder = r2h
2 3 Let the radius ratio of two cylinder
Volume of hemisphere = r 22
3  × 5x × 13x = 2502.5
7 r1
Volume of cylinder = r2h be
Given, 2502.5  7 r2 , Volume ratio of two
 x2 = = 12.25
Diameter = 7 cm 22  5  13
Height = 28 cm 7 V1 x
cylinder 
ATQ, x = 3.5 =
2
cm V2 y & Height ratio of
12 × The volume of a hemisphere
1 h1 a
 The volume of a cylinder We know, Volume of cone = r2h
3 two cylinder 
2 h2 b .
12 × r 3 =  × R2h 1 22
3 =  × 5x × 5x × 12x
3 7
V1 (r1 )2 h1
7 7 1 1 22 7 7 7 Now, 
r³ = × × 28 × =   5   5   12  V2 (r2 )2 h2
2 2 8 3 7 2 2 2
= 13475 cm3
7  7  7 7 
3
63. (c) x (r1 )2 a
r = 
   

r
3
 Given, y (r2 )2 b
8 2  Radius of the Sphere = 4 cm.

si
7 Let the radius of the cylinder be 3x.
r= = 3.5 cm Height of the cylinder r1 xb
2  
an by 2 r1 ya
61. (b) = 2×3x × = 4x.
3
Volume of cube = a3 65. (a)
A.T.Q,
Radius of cylinder = a/2, Height =

n
Volume of cylinder = Volume of Length and breadth of a cuboidal
a sphere store = 2x : x
Volume of cylinder = r2h
ja 4 Height of the cuboidal store (h)
  r²h = r³
R s
a2 a3 3
=×  a  3.14  = 3.5m
4 4 4
Area of four wall = 2(l + b) × h
a th

= 0.785 a 3 3x × 3x × 4x = × 4 × 4 × 4
3
Remaining volume of cube 2(l + b) × h = 210
4
= a3 – 0.785 a3 = 0.215 a3 x =  2(2x + x) × 3.5 = 210
3
0.215a 3
We know, 3x = 30
ty a

Required % = ×100% Curved surface of cylinder = 2rh


a3 x = 10
= 2 ×  × (3x) ×(4x)
= 21.5% Volume of cuboidal store = l × b × h
di M

4 4
62. (d) =2××3× ×4×
3 3 = 2x × x × 3.5
Radius : Height
10x: 24x 128 = 7 × 10 × 10
=  cm²
5x : 12x 3 = 700 m³


A

Aditya Ranjan (Excise Inspector) Selected Selection 177

Downloaded by Kunal Chauhan (kc6970929@gmail.com)


lOMoARcPSD|39315497

Data Interpretation

DATA INTERPRETATION
23

1. Study the given graph and answer the question that


follows. 50
45
40
35
Production of cement (in lakh tonnes) by A, B & C 30

r
70 25
20

si
60
50 15

40
30
an by 10
5

n
20 0
2016 2017 2018 2019 2020
10
ja X Y Z
R s
0
2015 2016 2017 2018 2019 2020
a th

A B C 2. The average production for 5 years was maximum


for which company/companies?

What is the difference between the production of ./ 2 # 3


ty a

company C in 2015 and the production of company # 4 %


B in 2018?
SSC CPO 09/11/2022 (Shift-01)
di M

C !B (a) X and Z (b) Y


" # # $ %
(c) X and Y (d) X and Z
SSC CPO 09/11/2022 (Shift-01) 3. For which of the following years is the percentage
(a) 15,00,000 tonnes rise/fall in the production from the previous year
the maximum for company Y?
(b) 1,00,000 tonnes
5 * ) # Y ./ 0* 6* ./ 0
(c) 10,000 tonnes #7* # # .8 93 .:
(d) 10,00,000 tonnes SSC CPO 10/11/2022 (Shift-02)
Direction (2-3): Study the given bar-graph and (a) 2019 (b) 2018
answer the questions based on it.
A

(c) 2016 (d) 2017


& 4. ’ following
The # bar graph shows the production of
various models of mobiles in the year 2019 and 2020.
The bar graph given here shows the data of the The total production is 40 lakhs during 2019 and
production of cars by three different companies X, Y, Z 65 lakhs in 2020.
over the years.
5 * ) # ;( ) ./ 0 <
$ ( ) # X,* Y,& Z * + . , -- >(* 0#
, - & , - ./ 0 7* (1 ? * )0 # #
$
* ) $

Aditya Ranjan (Excise Inspector) Selected Selection 178

Downloaded by Kunal Chauhan (kc6970929@gmail.com)


lOMoARcPSD|39315497

Data Interpretation

100 F
Student P1 P2 P3
F
90 E Mohit 79 78 77
80 E D Shivam 73 87 80
D C
In Percentage

70
Vishal 82 74 83
60
C
B
Viraj 77 89 86
50
40 B SSC CPO 09/11/2022 (Shift-01)
30 (a) Viraj, Vishal, Mohit (b) Vishal, Viraj, Viraj
A
20 A (c) Shivam, Viraj, Vishal
10 (d) Mohit, Viraj, Vishal
0 2019 2020 Direction (7-9): Study the given table and answer
the questions based on it.
What is the total production of model A mobiles in # * ’
2020 and model E mobiles in 2019?
A >(* =* E
< =>
* (*

r
The percentage marks obtained by 6 students in
7* # $ % different subjects are given below. The maximum marks

si
SSC CPO 09/11/2022 (Shift-01) for each subject have been indicated in the table.
(a) 39,00,000 (b) 32,00,000 . , - ./ @ 6 E + J F#

5.
an by
(c) 36,00,000 (d) 21,00,000
The given graph shows the number (in hundreds)
# * J ./ * ’ #

n
of trees axed in four cities during the period 2016- Subject Phy. Maths Hindi Geo. Eng. His.
2020. Students
80 150 100 75 120 50
ja @& . ’ Max. " $
R s
: (1 ) A (1 B Marks 0# $
P 70 44 88 88 70 38
a th

30
Q 90 40 54 92 65 40
25 24 24.4
25 23 23 22.5 R 85 32 70 64 55 30
21
20 20 19.5 20
18 18.5
18 18.4 17.6 19 17.5 S 75 70 58 80 60 35
ty a

16
15 T 64 60 45 88 50 42
10 U 60 50 60 72 25 48
di M

5 7. What is the average marks obtained by all students


0 in Geography?
2016 2017 2018 2019 2020
, K * , 6 E + F#
Chandigarh Ahmedabad Pune Kolkata SSC CPO 09/11/2022 (Shift-01)
(a) 65.5 (b) 63.8
Find the year in which the least number of trees (c) 60.5 (d) 58.2
are axed. 8. The total marks obtained by student Q in Physics
and Hindi are what percentage (rounded off to the
.$ ./ 0 C # (1 :
nearest integer) more than the total marks obtained
SSC CPO 09/11/2022 (Shift-01) by student T in Geography and History?
6 EQ + , # .C $
(a) 2016 (b) 2020
= #$ T + 6 E F# 7*
(c) 2019 (d) 2017
A

A :# K; 2 # K; 2 #B ’ $
6. There are three papers in an examination. Marks SSC CPO 10/11/2022 (Shift-02)
obtained by four students have been tabulated below: (a) 45 (b) 49
Who are the students who got the highest marks (c) 56 (d) 38
9. What are the approximate percentage marks obtained
in P1, P2 and P3 respectively.
by student T in all subject together?
D # $ " 6 E + ET +
6 F# , ./ " * F
; 9 $ G # # # $ %
SSC CPO 11/11/2022 (Shift-03)
P1, P2 P3 H I 6 E ’
(a) 65 (b) 58
* $ % (c) 47 (d) 52

Aditya Ranjan (Excise Inspector) Selected Selection 179

Downloaded by Kunal Chauhan (kc6970929@gmail.com)


lOMoARcPSD|39315497

Data Interpretation

10. The following table shows the percentage of marks Marks obtained
obtained by five students - V, W, X, Y and Z - in Subjects
out of 100
five subjects.
English 55
5 # * V,
" W,
6 X,E Y Z
& & + "
./ J F# J # # 0# $ Hindi 90
Mathematics 75
Subject Maths Physics Chemistry Botany Hindi Science 80
Maximum 300 300 200 100 100 Foreign language 90
Marks Five marks are to be deducted from each subject due
to shortage of attendance. The net average of the mean,
Students
median and mode of the marks obtained is ______.
 R4 # $ ; ./
V 60 80 90 100 95 $ $ F# M
W 90 70 80 70 40 #SSSSSSSS$
X 70 90 95 80 75 SSC CPO 09/11/2022 (Shift-01)
Y 80 60 70 75 80 (a) 79.33 (b) 77.67

r
Z 90 80 80 85 85 (c) 82.66 (d) 73
What are the average marks, in percentage, obtained 13. The following pie chart shows the contribution (in

si
`) of some investors in the corpus fund of a company.
by X?
If total contribution by these investor is 100%, then
X + an by F# #
SSC CPO 09/11/2022 (Shift-01)
# # what is# the $percentage
the corpus fund?
% contribution of Rakesh in

n
5 =0 " :0 > 0
(a) 76.5% (b) 80.5%
ATS B 0 $
(c) 82.5% (d) 85.5% 7* U $ M # > 0 (
ja
R s
11. The following table shows the number of students # # $ %
of seven colleges participating in extra co-curricular
a th

activities. Contribution in corpus fund


5 * ) # # * # L# NO # # . , *
. * # >* 6 E ) 0#
67.52 $ Sahil
74.00
Rakesh
ty a

Read the table and answer the question given.


70.00 75.00 Sonia
# * P1
di M

Kapil
College  69.05 77.50
Extra co  Preeti
Currilar A B C D E F G
activities
 SSC CPO 09/11/2022 (Shift-02)
I 150 250 450 50 350 250 150 (a) 17.30% (b) 18.12%
II 50 150 150 50 50 50 50 (c) 17.95% (d) 16.95%
III 100 50 400 125 100 500 150 Direction (14-15): Study the given pie-chart and
IV 250 125 350 275 250 150 150 answer the questions based on it.
Percentage of the number of students in activity II =0&" :0
to that of IV is (consider up to two decimals)
# .II ’ .Q 4 0 IV ) .Q #4 .0 The population of 7 villages is shown in the following
pie-chart.
A

) # # C # A *. R4 #
5 =0&" :0 V . )
$
SSC CPO 09/11/2022 (Shift-01) G A
(a) 45.48% (b) 35.48% 15% 13%
F B
(c) 34.25% (d) 39.18% 16%
13%
12. Marks scored by a student in different subjects are
E C
given in the table belew
18% D 8%
6 E + . , - ./ F# " # * 17%
$

Aditya Ranjan (Excise Inspector) Selected Selection 180

Downloaded by Kunal Chauhan (kc6970929@gmail.com)


lOMoARcPSD|39315497

Data Interpretation

14. What is the difference beween the population of Direction (17-18): Study the given bar-graph and
village D & E, if the total population of the all the answer the questions based on it.
villages is 60000? &
, . 7* ) D @E $ M # .
) " # # $ % The bar-graph shows the export of rice (in ` lakh)
during five years.
SSC CPO 09/11/2022 (Shift-02)
(a) 400 (b) 800
( ) " ./ 2 " .* 0
(c) 500 (d) 600
15. If the total population of all the villages together 25
21.6
is 50,000 what is the population of village D? 20 18.8
, . 7* ) D $ 15 # . 15.6
13
) # $ % 10.4
10
SSC CPO 11/11/2022 (Shift-01)
(a) 10000 (b) 9000 5
(c) 8500 (d) 9500 0
16. The following bar graph displays the highest, average 2010 2011 2012 2013 2014
and lowest marks obtained in Accounts, Business

r
studies, Economics and English by students of Class 17. The average export of rice for the years 2010 to 2014
XII in an examination. The maximum marks for each

si
(in ` Lakh) was:
subject is 100. Study the graph carefully and answer
the question that follows:
./ 0 ? # " .* # 0#
5
* )
an by(
M W .
) D
M 4 0 RE
$.
4 %
D 6 E
F#
+
SSC CPO 09/11/2022 (Shift-02)
XX # M

n
# 5 # 0# # $ (a) 13.28 ./ (b) 14.78
* K; 2 $ (c) 15.88 (d) 16.56
) ja K.0 18. " In how many years was the export of rice less than
R s
the average export for the five-year period?
# ./ 2 " .* 0#M "
a th

The highest, Avg. and the lowest marks obtained in


accounts, business studies, economics & english 0# 4 %
by students of Class XII in an examination SSC CPO 11/11/2022 (Shift-02)
120 (a) 4 (b) 1
ty a

100
100 100 (c) 3 (d) 2
96 96
78.6 80.94 86.42 Direction (19-20): Refer to the following table. Read
80 74.3
di M

72 the table and answer the questions.


60 60
46 5 # * , 0 * # *
40 34

20 Food grain production in a country in 1999. (in lakh tons)


0 <<< ) Q - A* )
Accounts Business Economics English
Studies
State Rice Wheat Jowar Pulses Other
Highest Average Lowest
P 55 113 – 37 39
Which of the two highest score is better and by how Q 58 96 83 29 25
much in the combined highest score of accounts and R 69 42 77 24 41
business studies or the highest score of economics
S 51 47 69 31 25
and English taken together.
T 47 32 51 23 21
* ) W . 4 * X"#
A

U 78 25 22 – 28
4 . 4 0 RE 4 * X"#
V 67 18 17 22 20
’ $ # ’ $ %
W 48 38 41 32 55
SSC CPO 09/11/2022 (Shift-02)
(a) Economics and English by 10 19. State P alone produc how much percentage of wheat
4 0 RE out of the total wheat production in the country?
(b) Accounts and Business Studies by 9 YP * 7* $KZ
* ) W .R < # $ %
(c) Accounts and Business Studies by 8 SSC CPO 09/11/2022 (Shift-02)
* ) W .R !
(d) Economics and English by 8 (a) 22.50% (b) 27.50%
4 0 RE ! (c) 25.50% (d) 23.50%

Aditya Ranjan (Excise Inspector) Selected Selection 181

Downloaded by Kunal Chauhan (kc6970929@gmail.com)


lOMoARcPSD|39315497

Data Interpretation

20. What was the proportion of rice production to wheat 23. What is the average population of village D during
production in country? all the 4 years?

" .* $KZ , ?7 . /# 2# D 4 % # . ) #
SSC CPO 10/11/2022 (Shift-01)
SSC CPO 11/11/2022 (Shift-01)
(a) 5295 (b) 5300
(a) 411 : 422 (b) 473 : 411
(c) 5307 (d) 5315
(c) 418 : 422 (d) 322 : 311
24. The following table represents the number of
21. The following table shows the percentage of
computers (numbers in thousands) manufactured
population of three states below poverty line and
by four companies from period 2014 to 2016. Study
the proportion of boys and girls.
the table carefully and answer the following question
5 * ) # ; # Y ) 5 # *" ? ) @ . ’ "
# # *(1 *(1 70 # # 5 F K : 0# $ ) A ) $
State Percentage Proportion of K.0 5 * ) #
population boys and girls year A B C D
below *(1 *(1
poverty line 2014 1350 750 630 810
7 #
) " 2015 570 520 1280 990

r
) Below Above 2016 420 1170 650 580
poverty line poverty

si
# # For the year 2014 to 2016,what is the difference
) line
) (numbers in thousands) between the combind
"
an by
A 25 3:4 4:3
production of computers by companies A and D and
the production of computers by companies B and C
taken together?

n
B 60 2:4 1:2 ./ 0 ? @A # D + 5F K:
7L# B C + 4 5F K
ja
C 15 4:5 3:1
R s
" # A ) $
If the total population of state B is 3000, then what
SSC CPO 09/11/2022 (Shift-02)
a th

is the number of girls below poverty line in state B?


(a) 460 (b) 280
BY 7* ) [ $B M # Y
(c) 370 (d) 310
) " *(1 ) #
25.
$ %
The following table shows the number of candidates
SSC CPO 09/11/2022 (Shift-02) from different locations, who appeared and passed
ty a

(a) 1200 (b) 700 in a competitive examination over the yeears.


(c) 650 (d) 720 5 # * . , -- R4 5
di M

Direction (22-23): Study the given table and answer


the questions based on it. $ M ./ 2 #
# * ’ $ 7# ; 0 $7
Year Rural Semi-urban State capitas Metropolises
The population of four village in 4 years is presented
in the following table. App. Passed App. Passed App. Passed App. Passed

5 # * ? ./ 2 " . 1990 1652 ) 208 2513


R# 7# 1468 3214
$ 1991 1839 317 2933 3248 4018
Village/year 2018 2019 2020 2021 1992 2153 932 2468 3159 3038
A 12381 12800 13045 13256 1993 5032 1798 3528 3628 5158
B 10089 11478 11502 11931
1994 4915 1658 4015 4311 6328
C 7896 7942 8069 8275
1995 528 2392 4263 4526 6419
A

D 5152 5230 5346 5500


22. What is the percentage increment in the population What approximate value was the percentage drop
of village A in 2021 as compared to the population in the no. of semi urban candidates, who appeared
in the year 2018? from 1991 to 1992?
A. ./ 0 ! ) #7* . /. / 00 < < << # R4 # $7 ’0
) # # # . 8 9 $ 7 = 0 $ %) A *. # # R4 .: * , L
# $ B SSC CPO 09/11/2022 (Shift-02)
SSC CPO 09/11/2022 (Shift-02)
(a) 15% (b) 8%
(a) 8.07 (b) 7.07
(c) 6.89 (d) 9.63 (c) 5% (d) 10%

Aditya Ranjan (Excise Inspector) Selected Selection 182

Downloaded by Kunal Chauhan (kc6970929@gmail.com)


lOMoARcPSD|39315497

Data Interpretation

26. The given pie chart shows the percentage of students 28. Study the given bar-graph and answer the question
who speak different languages in a hostel. that follows. The bar graph shows the number of
employees recruited (in lakhs) by three different
The total number of students in the hostel is 120.
companies in five different years.
=0 " :0 6 E . . , - , / ( )* . * "
6 E # # 0# $ 6 E . 7* 6 E( ) )" * & * ./
$ + , # 0 0" )
Number of Students Number of employees recruited (in lakhs) in three
different companies in five different years
Bhojpuri
10 38
35
30
Bengali 25
24 23
8 21
18 17
Hindi 14 15 13
Tamil 45 1012 12

5 Punjabi
17

r
English
A B C

si
15
The number of employees recruited in company B
What is the ratio of students speak English and
an by in the year 2019 was what percentage of the number
Bhopuri to those who speak Tamil? of employees recruited in company C in the year 2021
, 7 * . * 6 E (correct up
# to* 2 decimal
* places)?
. *

n
7 # L $ % ./ 0 < B , # 0 0"
./ 0 C , # 0 0"
SSC CPO 09/11/2022 (Shift-03)
ja # # # A *. N R4
R s
(a) 10 : 3 (b) 1 : 1
SSC CPO 09/11/2022 (Shift-03)
(c) 20 : 3 (d) 5 : 1
a th

(a) 88.85% (b) 85.25%


27. The following pie chart shows the number of workers
(c) 105.88% (d) 102.35%
of different categories A, B, C, D, E, F, G and H of
29. Study the given bar-graph and answer the question
a factory in 1995. that follows. The bar graph shows the length (in
5 =0 " :0 << A,
) B, thousand
. , - kilometers)
\ ; of mountains, forests and
ty a

rivers in different states.


C, D, E, F, G H \ ) 0# $
5 * ) # *
di M

H . , - Y $ (1
A
G 9% 10% R
A$ B 0# $
6% 15%
F 45
40 40
15% 35 35 35
C 35
30 30
32
30 30
28
E 12% 30
Length

25 25
8% D 20
25% 15
10
5
If the number of workers in category B is increased 0
by 10% and the number of workers in category D
a

ab
r

a
P

ha

n
sh
U

ya

nj
Bi

is decreased by 5% in 1996, then what is the total


di

Pu
ar
A

no. of workers in categories B and D in 1996, if Mountains Forests Rivers


number of workers in B was 1500 in the year 1995?
<<@ B \ ;\ ) U .8 9
The length of mountains in Punjab is what
# $ D \ ; \ ) U percentage more than the length of rivers in Odisha?
# $ M # B< < @ D \ ; 7* ) $ (1 * =0 (1
L $ M 1995 . / 0 B
\ ; \ ) 4# %# ’ $ %
SSC CPO 09/11/2022 (Shift-03) SSC CPO 09/11/2022 (Shift-03)
(a) 4025 (b) 4500 (a) 400% (b) 300%
(c) 5000 (d) 4000 (c) 200% (d) 100%

Aditya Ranjan (Excise Inspector) Selected Selection 183

Downloaded by Kunal Chauhan (kc6970929@gmail.com)


lOMoARcPSD|39315497

Data Interpretation

30. Total number of candidates who appeared and qualified 32. Study the given table and answer the question that follows.
in a competitive examination from different cities are The table shows the consumption of various grains
given in the following bar graph. Study the bar graph by a total of four families of four members each from
carefully and answer the following question. 2002 to 2005.
. , - $ # D = 0* # * ; 0 $ "
. * 5 . 7* ) 5 =0 $
# * # " R
K.0 +" . , - ) #
Wheat Rice Oats Quinoa
Year
4500 kg  kg  kg  kg 
5000
2002 162 62 131 61
3900
4500 2003 196 80 116 64
4000 2004 187 116 103 46
3000
3500 2005 189 122 101 45
2500 2500
3000 For the total of these four families what is the
1350 difference (in kg) between the average consumption

r
2500 per year of wheat and that of oats?
1200
2000 = , " . * M #./ 0

si
1500 850 ) # " # A B # $
1000

500
an by (a) 129.5
SSC CPO 09/11/2022 (Shift-03)
(b) 90.67

n
(c) 70.75 (d) 63.5
0 Direction (33-34): Study the given table and answer
Agra Pune Mumbai Jaipur
the questions that follow.
ja
R s
Appeared Qualified =0 # *
a th

What is the difference between the average number


of appeared candidates and the average number of School Total No. Percentage Ratio of male to
qualified candidates in the exam from all the cities? Name of students of enrolled female students
, $ D * $7 5 . Enrolled students
# ) who opted Biology
.)Qwho4opted
ty a

D ; 0 $7 5 . ## 0
Biology " .C 6 E .
# # $ % 7* ) 6 E
# .Q 4 0
7 #
di M

SSC CPO 09/11/2022 (Shift-03) # # -$


(a) 525 (b) 550 . .C "7
(c) 650 (d) 625
31. Study the following table and answer the questions. A 900 30 7:8
5 * ) # # * B 400 38 9 : 10

Student Subject
C 1000 24 5 : 19
Chemistry Mathematics Physics Hindi English
Maximum Maximum Maximum Maximum Maximum D 800 20 5:7
Marks Marks Marks Marks Marks
300 150 300 200
Raju 60 85 90 80 65 33. Find the number of students who opted Biology in school D.
Shyamu 65 70 60 75 65 D*
R K . .C "7 . * .Q 4 0
Mohan 70 75 80 65 85 SSC CPO 09/11/2022 (Shift-03)
A

Shobha 60 65 60 85 80
(a) 20 (b) 160
Sushil 65 75 70 60 75
(c) 120 (d) 800
Percentage of marks obtained by five students in
34. Find the ratio of the number of students who opted
different subjects is given. What are the average
marks in percentage obtained by Mohan? biology in school B and school D.
. , - ./ " 6 E + R FKB#* RD K * # . #. C "7 .
$ $ + F# # # # ) L $ 7% # C #
SSC CPO 09/11/2022 (Shift-03) SSC CPO 10/11/2022 (Shift-02)
(a) 70.6% (b) 74.6% (a) 1 : 4 (b) 1 : 2
(c) 73.2% (d) 73.6% (c) 1 : 1 (d) 2 : 1

Aditya Ranjan (Excise Inspector) Selected Selection 184

Downloaded by Kunal Chauhan (kc6970929@gmail.com)


lOMoARcPSD|39315497

Data Interpretation

35. Study the following table carefully and answer the 38. Number of books issued by a library for 6 months
questions based on it. has been represented in the following bar graph.
5 * ) # # * K.0 7R# * + @ $ *
’ # ) 5 * ) # 0
The following table shows the domestic sales of
vehicles of four manufacturers from 2010 to 2015 Number of books
5 * ) # # * # " 0# . $
] *K H 0# $ 8000

Manufacturer 2010 2011 2012 2013 2014 2015 6000


A 560000 700000 600000 620000 650000 680000
B 540000 590000 570000 630000 710000 650000
4000
C 610000 580000 620000 680000 690000 630000
D 630000 570000 700000 690000 700000 640000
2000
With respect to which of the following combinations,
is the sales of vechicles highest over the given period? 0
5 * ) # ’ M Jan =Feb
0 March
. ’ April May June

r
. $ H ’ $ %
In which month the maximum number of books were
SSC CPO 09/11/2022 (Shift-03) issued?

si
(a) A, 2011 (b) B, 2014 $ . 0 ’ 7R# =
(c) D, 2012 (d) D, 2014
an by
Direction (36-37): Study the given pie-chart and
answer the questions based on it.
(a) Jan
(c) May
(b) April
(d) February
SSC CPO 10/11/2022 (Shift-01)

n
=0&" :0 ’ # (39-40): Study the given bar-graph and
Direction
answer the questions based on it.
ja
R s
The percentage distribution of Anil's expenditures &
is presented in the given pie-chart.
a th

* W # # .# ; = 0 & The
" : given
0 0 $
table represents the population of different
Miscellaneous villages.
8%
=0 # * . , - . )
ty a

Name of Number of Number of


Rent
12% the village males females
Power
di M

A 1500 1220
and fuel
B 1460 1320
25%
C 1105 1180
D 1305 1170
Studies
Food 40% 39. J : denotes the average number of males in villages
25% A, B and C.
K : denotes the average number of females in village
B, C and D.
What is the average of J and K?
36. If Anil's monthly expenditure is ` 65,000, how much J: A,
. B # 4C 7T/ # )
money does he spend on food and rent?
K: B,
. C # 4D $* # )
* W TS @ M $ M # .$ ,
J # 4K # L $ %
# W # $ %
A

SSC CPO 10/11/2022 (Shift-01)


SSC CPO 10/11/2022 (Shift-01)
(a) ` 24,050 (b) ` 25,040 (a) 1289.17 (b) 1279.62
(c) ` 24,500 (d) ` 25,400 (c) 1278.36 (d) 1288.62
37. What is the corresponding central angle for Anil's 40. Find the ratio of males to females in all villages.
expense on power and fuel? , . 7T/ $*
* =2’ * W * # ; L $ %
SSC CPO 11/11/2022 (Shift-03) SSC CPO 10/11/2022 (Shift-02)
(a) 50° (b) 54° (a) 537 : 389 (b) 163 : 179
(c) 55° (d) 52° (c) 179 : 163 (d) 389 : 537

Aditya Ranjan (Excise Inspector) Selected Selection 185

Downloaded by Kunal Chauhan (kc6970929@gmail.com)


lOMoARcPSD|39315497

Data Interpretation
41. The following table shows different types of geysers
Age group Percentage
sold by a company over 6 years (numbers in thousands)
Up to15 15.00
5 # # * @ ./ 2 + "
1625 20.25
, - & , - R A ) $ B 0# $
2635 16.75
Year Types 3645 26.75
4655 15.00
A B C D E 5665 5.50
2007 75 122 103 98 95 66 and above 1.25

2008 100 102 103 112 102 If there are 30 million people below 36 years, then
find the number of people (in millions) in the age
2009 105 108 112 109 108 group (56-65) (consider up to two decimals).
2010 100 189 123 102 122 [@ ./ 0 7 . * *
2011 95 123 102 124 124 # A @ & @ B 7 . 0 . * *
A *. R4 # K; 2
2012 85 145 134 134 145
SSC CPO 10/11/2022 (Shift-01)
(a) 1.16 million (b) 3.17 million

r
Answer the following question based on table:
# * ’ 5 * ) # (c) 1.87 million
I (d) 2.17 million

si
The total sales of all the six years are the minimum 44. Study the following table and answer the question.
for which geyser? 5 * ) # # *
an by @ ./ 0 7*
SSC CPO 10/11/2022 (Shift-01)
.H - K # $ Number of
Students
Percentage
of students
Number of

n
scoring
(a) D (b) A School scoring students
marks
marks less appeared
(c) C (d) B more than
ja then 50%
R s
42. The average marks of students in five subjects in 50%
class X in a school is given in the table. A 270 55 600
B 120 40 400
a th

5 # * R K* . D C
" 300
. / 20 375
6 E # $ D 220 10 350
E 200 25 300
Year History Geography Mathematics Science English
(out of (out of (out of (out of (out of The ratio of total number of students scoring marks less
ty a

200 marks) 100 marks) 200 marks) 200 marks) 150 marks) than 50% to that of scoring marks exactly 50% is:
1984 88 46 98 98 60 U F# . * 6 E
di M

1985 90 35 120 120 90 N U F# . * 6 E


1986 100 45 102 102 66 L $ %
1987 120 38 112 112 45 SSC CPO 10/11/2022 (Shift-01)
1988 124 54 96 96 57 (a) 111 : 12 (b) 111 : 24
The average percentage marks of which of the (c) 101 : 24 (d) 101 : 12
following pairs of subjects was the same in 1985? 45. Annual production of four different types of cold
drinks has been tabulatd below. Which of these cold-
<! 5 * ) # 7^ .drinks
/ has encountered
# maximum
# # percentage
4 % increment in production?
" . , -- _( (‘ .
SSC CPO 10/11/2022 (Shift-01)
; 9 $= _
(a) Geography, Mathematics ’ # # # .8 9 $7=0 $ %
A

(b) Science, Geography 2020 2021


Cold drink
(c) Science, English units  units 
Sprite 5400 5800
(d) English, Mathematics
Cocacola 5300 5600
43. The table shows the percentage of total population Pepsi 6000 6800
of a city in different age groups. Study the table and
Fanta 1800 2700
answer the question.
SSC CPO 10/11/2022 (Shift-01)
# * . , - 7 . 2 $ 7* )
(a) Coca-cola (b) Pepsi
# # 0# $ # *
(c) Fanta (d) Sprite

Aditya Ranjan (Excise Inspector) Selected Selection 186

Downloaded by Kunal Chauhan (kc6970929@gmail.com)


lOMoARcPSD|39315497

Data Interpretation
46. The following bar graph shows the sales (in thousand 49. The total number of centuries scored by which two
numbers) of scooters from six different companies players in the mentioned years are the same?
in 2011 and 2012.
_* ) # ./ 2 ) * (1
5 * ) # ( ) 76* $ )* & * $ %
R K: H A$ ) B 0# $
SSC CPO 10/11/2022 (Shift-03)

Sale of scooters from six companies (a) D and E (b) D and B


in 2011 and 2012 (c) A and C (d) A and E
Sale (in thousands in numbers)

30 50. The following table shows the marks (in percentage)


25 obtained by six students in six different subjects
25
21 21 19 in an examination. The maximum marks in each
18 18
20 subject are 100.
17
15
15
12
5 # # * D 6$ ,
11 10 12
10
6$ 6 E F# A # #
’ # $
5

0
A B C D E F Students Subject

r
Different Companies
2011 2012 Cs. Chem. Maths His. Geo.

si
The average sales of companies A and E in 2011 are A 95 65 90 70 80
what percentage of the average sales of companies
an by
B and D in 2012?
A E # H MB
B

C
90

80
70

80
75

65
85

90
85

95

n
D # H # # # $ %
SSC CPO 10/11/2022 (Shift-02)
D 85 90 85 90 95
ja
R s
(a) 65.89% (b) 72.87% E 80 95 95 90 95
(c) 70.89% (d) 74.36%
F 90 90 90 90 90
a th

Direction (47-49): Study the given table and answer


the questions based on it. Answer the following question based on the table:
# * ’ ## * ’ # 5 * ) #
ty a

In which of the following subjects is the overall


The following table shows the number of centuries percentage highest?
scored by given five cricketers in the past 5 years.
di M

5 * ) # ./ # # .
5 # * 6* ./ 0 " H : +
SSC CPO 10/11/2022 (Shift-02)
# ) 0# $
(a) Chemistry (b) Geography
Years
2021 2020 2019 2018 2017 (c) History (d) CS
Cricketers
A 1 0 1 0 1
51. Study the following data and answer the question
below.
B 1 0 2 1 1
C 1 2 0 0 1 5 * ) # (1
D 0 0 1 1 0
E 0 1 1 0 1
Year 2016 2017 2018 2019 2020
47. In which year the maximum number of centuries
Company's%
were scored? 30 20 25 15 10
A

Profit
. 0 ’ # ./ 0 4 %
SSC CPO 10/11/2022 (Shift-02) What was the revenue of the company if its
(a) 2020 (b) 2018 expenditure was ` 50 lakhs in the year when its
(c) 2019 (d) 2021 profit percentage was the highest?
48. Who scored the maximum number of centuries? W ./ 0 TS *
. 0 ’ # % * , # # XX # 4 M #
SSC CPO 10/11/2022 (Shift-02) SSC CPO 10/11/2022 (Shift-02)
(a) A (b) C (a) ` 55 lakhs (b) ` 70 lakhs
(c) B (d) E (c) ` 65 lakhs (d) ` 60 lakhs

Aditya Ranjan (Excise Inspector) Selected Selection 187

Downloaded by Kunal Chauhan (kc6970929@gmail.com)


lOMoARcPSD|39315497

Data Interpretation
52. Study the given bar-graph and answer the question 54. Study the given table and answer the question that
that follows. follows.
The bar graph shows the exports and imports (in `
lakh) of a company from 2000-2004. =0 # *
& "
Year Rice Wheat Barley
& ? # 0 # 2016 6800 #2700 2500
ATS * ) B 0# $ 2017 8000 4200 4400
Exports & Imports (In Rs. Lakh) 2018 6800 3500 5200
160 150
2019 7200 4000 5800
140
140 130 2020 8200 5600 6100
120 120 120
110
100 100 100 Which of the cereals show continuous increase in
80 its production over the years?
60
60 50
40
./ 2
20 0# $ %
0
2000 2001 2002 2003 2004 SSC CPO 10/11/2022 (Shift-03)

r
Exports Imports (a) Barley (b) Others
(c) Wheat (d) Rice

si
What is the average export (in ` lakh) from the year
55. Study the given table and answer the question that follows.
2000 to 2004?
an by Production of masks (in million) by a company during
? # # 0# # A* )the firstBhalf$ of%2020 is given.
SSC CPO 10/11/2022 (Shift-03)
=0 # *

n
(a) 100 (b) 125
$* 6 $
(c) 112 (d) 130
53. ja
The wages of workers (W1, W2, W3, W4, W5, W6) + R A * B
R s
for different tasks (T1, T2, T3) are given
Month
. , - (T1, T2,2T3) * \ (W1, W2, W3, April May June July August September
a th

Type
W4, W6) K =0 $ G
Surgical 250 250 180 400 200 150
Wages in Hundreds N95 250 270 500 450 300 200
FFP1 250 270 150 250 300 200
ty a

W-6
Activated
250 260 250 330 300 350
Carbon
di M

W-5 Cloths &


250 260 260 350 320 240
Spong
W-4 T-3 Total 1250 1310 1340 1750 1420 1140

T-2 In the case of Activated Carbon Masks, in which


W-3
T-1
pair of months was the production equal?
H # R * M $
W-2 SSC CPO 10/11/2022 (Shift-03)
(a) April, June
W-1 (b) April, July
(c) July, September
A

0 1 2 3 4 5 6 (d) April, May


What is the difference in average wages per worker 56. The following table gives the number of patients
for task T1 and task T3? visiting a dental clinic for the issues related to their
oral health during the months May, July and August
T1
0 T3 0 * # 0" # . #
in 2015. Study the table carefully and answer the
# # $ %
question that follows.
SSC CPO 10/11/2022 (Shift-03)
5 # * =0M K M 7* =
1 1
(a) (b) ) R. R4 ’# 7
3 2
L* . * )
2 1
(c) (d) # * K.0 "
4 4

Aditya Ranjan (Excise Inspector) Selected Selection 188

Downloaded by Kunal Chauhan (kc6970929@gmail.com)


lOMoARcPSD|39315497

Data Interpretation
58. In which subject are the maximum number of teacher
Month Cleaning and Fixing crowns Orthodontic Tccth
(2015) Polishing of Teeth and bridging Treatement Whitening males?

May 410 880 680 360 ./ D ’ # )


June 160 970 790 270 SSC CPO 11/11/2022 (Shift-02)
July 520 680 460 530 (a) Mathematics (b) Hindi
August 440 590 1020 280
(c) Commerce (d) English
For the given months in 2015, the average number 59. Study the given table and answer the question.
of patients visiting the dental clinic for cleaning and
The marks obtained by four students A,B,C and D
polishing of teeth. Fixing crowns and bridging and
in Mathematics, Statistics and Computers are given
teeth whitening is what percentage (rounded off to
in the following table.
1 decimal place) more/less than the average number
of patients visiting the dental clinic for Orthodontic =0 # *
treatment?
$ * M # =0 > * M
" 6 A,B,C
E D + ; #M )
H L # " * # "
F# 5 # * $
L* . * # ) M >4 0( :

r
" * # " * . * Students # B
A C D

si
) # # # 3 $ Subjects
%
A *. R4 # K; 2 #B Mathematics 60 50 70 75
an by
(a) More by 31.2%
SSC CPO 10/11/2022 (Shift-03) Statistics 70 45 60 80
Computers 80 55 55 75

n
(b) Less by 31.2% Who among the students has ranked first on the
(c) Less by 38.9% basis of the total marks obtained as per the given
(d) More by 38.9%
ja table?
R s
Direction (57-58): Study the given table and answer =0 # * 7 M F# 7*
a th

the questions based on it. 4 R4 F# $ %


# * ’ #
SSC CPO 10/11/2022 (Shift-03)

The total number of teachers in different subjects (a) C (b) D


ty a

and the respective percentage of females among them in (c) B (d) A


a school are given in the following table. Study the table
60. Calculate the mean from the following table
di M

and answer the following question.


R K* . , - ./ D 57 * # )* ;
$* ’# # # 5 # * $
Scores # *
Frequencies
5 * ) # 0 -10 2
Subject 0 - 20 4
Total number of teacher % of female
20 - 30 12
Hindi 50 80
30 - 40 21
English 40 90
40 - 50 6
Mathematics 60 35 50 - 60 3
Science 55 40 60 - 70 2
Commerce 30 50
A

SSC CPO 10/11/2022 (Shift-03)


Social science 40 75
(a) 33.4 (b) 32.6
57. What is the ratio of the number of male teachers (c) 35.8 (d) 34.2
to that of female teachers in Social Science?
61. The given pie chart represents the percentage of
.C 7T/ D )people in a city
$ *who are D
interested to buy the laptop
7 # # $ % of five different companies - A, B, C, D and E.
SSC CPO 10/11/2022 (Shift-03) =0 " :0 $ *
(a) 3 : 1 (b) 2 : 3 $ " * & *A, B, C, D E * : >
(c) 1 : 3 (d) 3 : 2 ) T " ) # $

Aditya Ranjan (Excise Inspector) Selected Selection 189

Downloaded by Kunal Chauhan (kc6970929@gmail.com)


lOMoARcPSD|39315497

Data Interpretation

Total People = 5200

E A
11.5% 31%

D
20%

C B
25% 12.5%

The average of the number of students playing rugby,


The number of people interested to buy company hockey, badminton and squash is what percentage
A's laptop is what percentage to the number of of the average number of students playing tennis
people interested to buy company D's laptop? and football?
5 A * : > ) =X67 * ^ M $) >M M5 ( : R . > )

r
D * : > ) =X67 * ) M
) : # 7: >* ) * . *

si
# # $ % # # # $ %
SSC CPO 11/11/2022 (Shift-01) SSC CPO 11/11/2022 (Shift-01)
(a) 135
(c) 150
an by (d) 155
(b) 125 (a) 60%
(c) 50% (d) 45%
(b) 30%

n
62. The given pie chart shows the percentage of Direction (64-65): Study the given bar-graph and
students who enrolled in different hobby classes answer the questions based on it.
in a school. The number of students who enrolled
ja &
R s
in cooking classes is what percentage of those
enrolled in dancing classes? (Rounded off 2 digits
The following bar diagram shows the number of
a th

after decimal)
students who opted for the different subjects in the year
=0 " :0 R K* . , - $ > L* Y #
2020 and 2021
.Q 4 0 # # 0# $ * D #
6 E ) M 8 * D 5 () . / #0 6 E . , - .
ty a

) # # # $ A 6 E
*. R4 )# K; 2 0 ## $
di M

Cooking Singing
22% 18%

Drama Dancing
13% 21%

Stiching Painting
11% 15%
A

SSC CPO 11/11/2022 (Shift-01)


(a) 92.65% (b) 125.25%
(c) 105.56% (d) 104.76%
64. Find the ratio of students opting Economics to the
63. The following bar chart shows the number of students opting Physical Education in 2020.
students playing various games on a given day. Study
the bar chart carefully and answer the question that 4 0 RE " . * 6
follows. " . * 6 E 7 # C #
5 " :0 . , - ) * ) * . * 6 ESSC CPO 11/11/2022 (Shift-01)
) 0# $ " :0 (a). 03 : 7
K (b) 3 : 5
" (c) 5 : 9 (d) 5 : 7

Aditya Ranjan (Excise Inspector) Selected Selection 190

Downloaded by Kunal Chauhan (kc6970929@gmail.com)


lOMoARcPSD|39315497

Data Interpretation
65. What is the sum of the total number of students Direction (68-69): Study the given pie-charts and
who opted for different subjects in 2020 and only answer the questions based on it.
Geography in 2021? =0&" :0
. , - ./ .* , K * "7
. * 6 E 7* ) # $ %
The following pie-charts show the number of start-
SSC CPO 11/11/2022 (Shift-01) ups started in various sectors since 2016 and the number
(a) 345 (b) 510 of successful start-ups in those sectors respectively.
(c) 150 (d) 435 5 =0&" :0 @ . , - D E
66. Study the given table and answer the question ) D E H I * R: :0
follows: Number of start-ups (since 2016)
=0 # * K6

The distance travelled by two online delivery Entertainment Ed-Tech


employees during a week. 560 720
F# $ > * = (* . 0" + #
Travel
=0 K

r
324 Fin-tech
256
Day Distance travailed (in km.)

si
Food
First Second 650
an by
Monday

Tuesday
200

300
250

400 Number of successful start-ups

n
Wednesday 200 350

Thursday 150 250 Entertainment


ja 175
R s
Friday 250 350
Ed-Tech
Travel 320
a th

Saturday 350 100 108


Find the ratio of the total distance covered by the
first employee during first three days and the last Food Fin-tech
three days by the second employee. 260 104
ty a

$* 0" + $* # K 0" +
# # # =0 7* K68. 7 # C #
Find the ratio of the number of succssful start-ups
di M

SSC CPO 11/11/2022 (Shift-01) in entertainment sector to the total number of start-
(a) 2 : 1 (b) 1 : 1 ups in entertainment since 2016.
(c) 1 : 2 (d) 3 : 1 @ D E * R: :0&
67. The following table shows the data of different collges R: :0& 7* ) 7 # C
whose students participated in the olympiad. SSC CPO 11/11/2022 (Shift-02)
5 # * . , - >* * (a) (5 : 16 , * (b). 5* : 17
6 E (1 0# $ (c) 7 : 16 (d) 25 : 81
Subjects CollegeA CollegeB CollegeC CollegeD CollegeE
Hindi 110 100 125 103 112 69. What is the ratio of the number of successful start-
English 98 120 80 122 105 ups to that of unsuccessful start-ups in Fin-tech
Maths 130 110 250 160 180
sector?
Science 100 100 150 200 80
GK 182 200 120 130 183 &: D E * R: :0& )
A

What is the percentage increase the number of ) 7 # # $ %


students who participated in the GK Olympiad to
those who participated in the Science Olympaid from SSC CPO 11/11/2022 (Shift-03)
all colleges? (Correct to two decimals places) (a) 3 : 4 (b) 13 : 18
, >* - C * ((c) 23 : ,32 * . *(d) 613 E
: 19
) M .C * ( , * . * 6 E
70. The following pie chart shows ) the distribution of
# # # ’ $ % A *. R4 income# fromKdifferent
; 2 #taxes. Study the B chart and
SSC CPO 11/11/2022 (Shift-01) answer the question.
(a) 29.37% (b) 29.89% 5 * ) # =0 " :0 . , -
(c) 29.74% (d) 29.01% " :0

Aditya Ranjan (Excise Inspector) Selected Selection 191

Downloaded by Kunal Chauhan (kc6970929@gmail.com)


lOMoARcPSD|39315497

Data Interpretation

Excise duty Class x marks of a Student


Property tax
12% (Values are in Degrees)
16%

English
Custom duty Science 60%
6% 82%

Hindi
Mathematics 62%
Income tax 84%
40% Market tax Social
26% Science
72%

If the total marks are 1800, then find the difference


If the income from the market tax in a year is ` between the marks scored in science and matematics.
260 crore, then the total income from other sources 7* ! $ M # .C ;
is: (in ` Crore) " C # #
./ 0 TS @ ( 1 $ M SSC
# CPO- 11/11/2022 (Shift-02)
RE # 7* TS (1 B (a)
# 2 $ % (b) 10

r
(c) 1 (d) 5
SSC CPO 11/11/2022 (Shift-02) 73. The following bar chart shows the vends of foreign

si
(a) 540 (b) 560 direct investment (FDI) into India all over the world.
5 * ) # " :0 7 , , #
71.
(c) 740 an by (d) 760
The circle graph given here shows the spending of
a country on various sports during a year. Study,
(FDI) .H 0#
India’s FDI (Millions of Euros)
$

n
the graph carefully to answer the question.
35 31.36
$ ja .8 ) ./ 0 . 30, - ) *
24.23
R s
) "0 0# $ 25 *
20
) K.0 15 12.16
a th

10.15 10.22
10 5.7
Percentage of money spent on various sports for one
year. 5
0
./ 0 * . , - ) * ) "0 ’ # #
1992 1993 1994 1995 1996 1997
ty a

Other Tennis For which year is the percent increase in FDI over
6% the preivious year the highest?
4%
./ 0 6* . / 0FDI # 7 * # # .8 9
di M

’ $ %
Golf SSC CPO 11/11/2022 (Shift-02)
Football 10%
15% (a) 1997 (b) 1994
(c) 1996 (d) 1993
Hockey 74. The following bar graph describes the population of
10% Basket ball
25% a town (in Lakhs) from 2001 to 2007)

Cricket
5 * ) # ( ) V #
30% A* ) B 0# $

If the total amount spent during the year was `


12000000 then how much was spent on Basket Ball?
A

./ 0 ) "0 =0 7* TS 4 M #
R : >* # ) "0 4 %
SSC CPO 11/11/2022 (Shift-02)
(a) 2500000 (b) 1000000
(c) 4000000 (d) 3000000 Find the percentage increase in population from 2006
to 2007.
72. The given chart shows the marks scored by a class
X student in different subjects.
@ V # ) # # .8
SSC CPO 11/11/2022 (Shift-02)
" :0 . D 6 E + . ,15%
(a) - ./ (b) 20%F #
0# $ (c) 16% (d) 18%

Aditya Ranjan (Excise Inspector) Selected Selection 192

Downloaded by Kunal Chauhan (kc6970929@gmail.com)


lOMoARcPSD|39315497

Data Interpretation
75. The following table shows the production of different 77. Number of units manufactured (M) and number of
types of two wheelers from 1993 to 1998. (No. of units sold (S) (in hundred by a company over the
two wheelers is in 1000s) years.
5 # * <<[ <<! # . , - $
. , - ./ 2 + (M) 0#
. $ 0# $ A $ . $ )
$ B .H =0 = (S)
= A (1 ) B $

Year Type 1993 1994 1995 1996 1997 1998 Year M S


A 36 34 40 35 37.5 40 2011 2.6 1.7
B 20 22 25 23 19.5 18
2012 2.2 1.5
C 14 22 16 25 29 35
D 60 62 67.5 75 76 80 2013 2.1 1.0
E 40 45 48 50 80 105 2014 2.8 1.4
F 45 52 55 60 57.5 56 2015 2.6 2.1
Total 215 237 251.5 268 299.5 334
What is the average number of units sold by the
What is the approximate percentage increase in company per year?

r
the total production of all types of two wheelers in
1998 in comparison to 1994? + .H =0 = =

si
<<? #7* <<! , # $ $% . $
7* * , # # .8 9 # $ %

(a) 42
an by SSC CPO 11/11/2022 (Shift-02)
(b) 45 (a) 140
SSC CPO 11/11/2022 (Shift-03)

(b) 150

n
(c) 40 (d) 41 (c) 154 (d) 156
76. The following pie chart shows the monthly household 78. The following table gives quantity of various items
ja
expenditure of Family A under various heads. The
R s
used by a restaurant during four months of the year
monthly expenditure incurred for Family A is Rs
(in kg)
50,000. Study the chart carefully and answer the
a th

question that follows. 5 # # * ./ 0 " $


5 =0&" :0 . , - A 2 # ]$ # * K . $ . * . , - ) Q 4
W 0 #A $ ` 50,000 $
. W a ; # $
ty a

" :0 K.0 "


Food
March April May June
di M

Item
Misc A 220 180 270 320
11%
Food B 255 320 390 420
Medicines 30% C 280 295 280 315
15%
D 350 310 250 280
Travel Education E 308 340 350 365
10% 15%
The quantity of E used in the month of March is
what percentage of the total quantity of food items
used in the May?
Electricity
9% "0 $ E E7 L #M =0 $
Entertainment
A

10% , ) Q 4 2 7* E #
What is the ratio of the combined monthly SSC CPO 11/11/2022 (Shift-03)
expenditure on Entertainment, Travel and (a) 20% (b) 14%
Miscellaneous to the average expenditure on Food,
Education, Electricity and Medicine? (c) 12.3% (d) 17.8%
M E . . .’ 7L# 79. The
W following table
, showsM the different
D M types of cars
* " # W 7 # sold #by a company
$ % over the given years (numbers
in thousands).
SSC CPO 11/11/2022 (Shift-03)
(a) 69 : 125 (b) 69 : 124
5 # * + ./ 2
(c) 124 : 69 (d) 31 : 69
. , - ) 0#

Aditya Ranjan (Excise Inspector) Selected Selection 193

Downloaded by Kunal Chauhan (kc6970929@gmail.com)


lOMoARcPSD|39315497

Data Interpretation

./ 0 ! # # 6 E ; # ,
Types
Year . .C . R # .C ; 0
A B C D E
SSC CPO 11/11/2022 (Shift-03)
2007 15 12 10 8 9
(a) 112 (b) 97
2008 10 10 13 12 12
(c) 100 (d) 99
2009 25 10 12 19 18
Direction (82-87): The given bar graph shows the
2010 12 18 13 12 12
number of students of two schools over a period of six
2011 19 12 12 14 14 years.
2012 15 14 14 14 15
6$ ./ 0 . ’ .
The percentage of type E cars sold out of the total ) 0# $
number of cars sold was minimum in which year?
H =0 E7 * ) School A School B
1000
H # # ./ 0 - K # 4 % 900
900
SSC CPO 11/11/2022 (Shift-03) 800 820
800 750 750
(a) 2011 (b) 2007 700
700 640
(c) 2010 (d) 2008 600
600 550

r
500 500 480
80. The following table shows the number of delivery 500
partners (in thousands) who joined five different

si
400
companies during six different years.
300
5 #
an by * @ * & * ./ 2 200
" * & *
* $ . * , 100 A$ B )
0# $

n
0
2008 2009 2010 2011 2012 2013
Companies
Year 82. In the bar graph, in which year is the sum of the
ja
Emozon Clipkart Twiggy Tomato Pyntra
R s
students from schools A and B taken together, the
2016 2.4 4.5 1.2 0.9 4.2
minimum?
2017 1.8 5.4 1.5 1.2 5.6
a th

2018 3.2 7.2 2.4 2.1 6.3 M A . / B0 . Q. Q* b 4


2019 3.9 5.6 2.8 2.7 6.5 * - K # $ %
2020 4.2 6.4 3.2 3.3 7.0 SSC CPO 23/11/2020 (Shift-1)
ty a

2021 5.0 7.2 3.6 3.6 7.2


(a) 2012 (b) 2010
Find the average number of delivery partners who
(c) 2013 (d) 2011
di M

joined Twiggy in the last 6 years.


83.
6* @ ./ 2 :O. * $ In the bar
. *graph,( *what
. is the
, ratio of the students
taken for the years 2009, 2011, 2013 together from
# ) C # school A to the students taken for the years 2008,
SSC CPO 11/11/2022 (Shift-03) 2012, 2013 together from school B?
(a) 2545 (b) 2450 2009,M
2011,
. / 2013
0 . AQ * .Q b4
(c) 2540 (d) 2455 # ) 2008,
M 2012, 2013
./ 0 .B
Q *
81. The numbers of students who passed with different .Q b4 # ) 7
honours from a college during 2015 to 2018 are
depicted in the given table. Study the table and SSC CPO 23/11/2020 (Shift-1)
answer the following question.
(a) 17 : 25 (b) 25 : 17
=0 # * ! >*
. , - ./ ; 0 $ . * (c)
6 E 18 : 25 ) (d) 250: 18
=0 =0
A

$ # * 5 84.
* ) In
# the bar graph, what is the ratio of the average of
the total students from school A to the average of
the total students from school B?
Year Maths Phy. Chem. Zoo. Bot.
2015 10 5 8 7 5 AM . Q . *Q b 4 7*
2016 4 8 3 4 1 B *
.Q .Q b4 7* )
2017 15 10 8 5 3 C #
2018 9 3 1 2 1
SSC CPO 23/11/2020 (Shift-1)
How many students passed with mathematics,
(a) 370 : 429 (b) 429 : 799
Physics, Chemistry, Zoology and Botany from the
year 2015 to 2018? (c) 799 : 429 (d) 429 : 370

Aditya Ranjan (Excise Inspector) Selected Selection 194

Downloaded by Kunal Chauhan (kc6970929@gmail.com)


lOMoARcPSD|39315497

Data Interpretation
85. In the bar graph, in which year is the maximum 89. What is the percentage of students whose height is
difference of the students from school A to B taken in the class interval 160-170? (correct to the nearest
together? integer)
M A . / B0 . Q. Q
* b4 )6 E # # C # 160-
M
# ’ # 4 % 170 $ %
SSC CPO 25/11/2020 (Shift-1) A :# K; 0 # 79B
(a) 2010 (b) 2013 SSC CPO 23/11/2020 (Shift-1)
(c) 2012 (d) 2011 (a) 39 (b) 25
86. In the bar graph, what is the ratio of the students (c) 34 (d) 51
taken for years the 2008, 2012, 2013 together from Direction (90): The given histogram shows the height
school A to the students taken for the years 2009, of the students:
2010, 2011 together from school B?
$R: 6 E cZ" =0
M 2008, 2012,
4 2013
./ 0 .A
Q *
.Q b4 7* ) 2009,
M 2010, 2011 4 ./ 0

Number of St udents
B *
.Q .Q b4 7* ) " 20 7 # C #
15 14
15 13

r
12
10
SSC CPO 25/11/2020 (Shift-1) 10
6

si
(a) 217 : 229 (b) 118 : 251 5
0
(c) 229 : 217
an by (d) 251 : 118 5 0 5 0 5 0
5 6 6 7 7 8
87. In the bar graph, what is the ratio of the average of 0 -1 5 -1 0 -1 5 -1 0 -1 5 -1
15 15 16 16 17 17
total students from school B to the average of total

n
Height of the student in cm
students from school A?

ja F B M . Q. Q* b 4 7 90.
* The
) number of students
#M whose height is in the class
interval 170-175 is what
" percent less than the number
R s
. QA * .Q b4 7* ) #
of students whose height is in the interval 165-1707?
7 # C #
a th

(correct to one decimal place)


SSC CPO 25/11/2020 (Shift-1)
. 0 & 170-175
# * cZ" =0 . * .Q b4
(a) 799 : 429 (b) 429 : 370
. 0 & 165-170
# * cZ" =0 . * .Q b4
(c) 370 : 429 (d) 429 : 799 # # # $ %
ty a

Direction (88-89):The given histogram shows the


height of the students.
A *. R4 # 79B
di M

SSC CPO 23/11/2020 (Shift-1)


$R: 6 E cZ" =0 0# $
(a) 17.3% (b) 11.5%
(c) 14.3% (d) 19.5%
Number of St udents

20 91. What is the percentage of students whose height is


15 14 in the class interval 165-175?
15 13 12
10 6 E # # C 165-175
# M. 0
10
6
5 # * $ % A : # K; 0 #
0 SSC CPO 25/11/2020 (Shift-1)

55 60 65 70 75 80 (a) 34 (b) 25
0 -1 5 -1 0 -1 5 -1 0 -1 5 -1
15 15 16 16 17 17 (c) 39 (d) 37
Height of the student in cm 92. The number of students whose height is in the class
interval 165–170 is what percentage less than the
A

88. The difference between the number of students whose number of students whose height is in the interval
height is between 150-155 cm and the number of 150–155?
students whose height lies between 175-180 cm is: (correct to one decimal place)
150-155 cm " c Z " = 0 . * 6175-
E 165-170
) . 0 & # * c Z " = 0 . *150- . Q b
180 cm " cZ" =0 . * 6 E 155
) . 0 &" # # * C #c Z " = 0 . * .Q b4
# # $ % A *. R4 #
SSC CPO 23/11/2020 (Shift-1) SSC CPO 25/11/2020 (Shift-1)
(a) 3 (b) 8 (a) 6.7% (b) 5.5%
(c) 9 (d) 7 (c) 4.8% (d) 1.9%

Aditya Ranjan (Excise Inspector) Selected Selection 195

Downloaded by Kunal Chauhan (kc6970929@gmail.com)


lOMoARcPSD|39315497

Data Interpretation
Direction (93 - 98): The given pie chart represents 98. If a percentage point shift results in annul additional
the popularity of ice-cream flavours in the years 2015. sales of Rs.5,000, how much (in Rs.), did the combined
=0 " :0 ./ 0 = H annual
d * strawberry
. 0 *and butterscotch
# sales increase
0# $ from 2005 to 2015?
2005 2015 # # - 7 "H T M . b
Other Other
* # $ M # # R:‘ >
Vanilla
15.7%
Vanilla
20.6% 7L# . b/ H # .8 9 AT
21.9% 16.5% SSC CPO 25/11/2020 (Shift-1)
Strawberry Strawberry
18.9% 20.5% (a) 74,000 (b) 10,000
Chocolate Chocolate
32.7%
Butter
25.8%
Butter (c) 37,000 (d) 65,000
scotch scotch
10.8% 16.6% Direction (99 - 104): The given bar graph represents
the number of boys and girls in five different schools. Study
93. In 2015, if the total sale of vanilla flavour is for Rs. the graph and answer the question the follows.
3,300, then the total sale (in Rs.) for chocolate flavour is:
" . , - .Q *
. * d* . 7* H T [M[ 4 # " > * :
) 0# $
d* . 7* H AT B C #

r
SSC CPO 23/11/2020 (Shift-1)
(a) Rs. 4,120 (b) Rs. 5,160

si
Number of Boys and Girls in Different Schools
(c) Rs. 3,320 (d) Rs. 4,100 Boys Girls 800
850
94. If a percentage point shift results in annual
an by 800 750
750 700
additional sales of Rs.10,000, how much (in Rs.), 700

Number of Students
650 600 650 600 650
d id the c om bine d an nu al s tr awbe rr y an d 600 550

n
550
butterscotch sales increase from 2005 to 2015? 500 500
450
450
# # - 7 "H T M H . b/ 400
350
# L#
*# ja
$ 2005
M # 2015 # R:‘ > : R 300
250 >"
R s
200
7L# . b/ H # .8 9 ATF 150
100
B $ %
a th

50
SSC CPO 23/11/2022 (Shift-1) 0
A B C D E
(a) Rs.65,000 (b) Rs.74,000
Schools
(c) Rs.37,000 (d) Rs.10,000
95. In 2005, if 10% of the ‘other’ category is mix fruit 99. What is the average number of boys in schools A, B,
ty a

flavour and 1570 people surveyed preferred mix fruit C, D and E?


flavour, then how many people were surveyed? . Q A,
* B, C, D E *(1 # )
di M

2005 e - 10%f . 0 L gK: d* . $ SSC CPO 23/11/2020 (Shift-2)


. 0 $ L g K : d * . (a) 569 (b) 596
. *
) 1570 $ M # $ . 0 # * (c) 660 (d) 616 %
SSC CPO 23/11/2020 (Shift-1) 100. What is the ratio of girls and boys from all the schools
(a) 1,75,000 (b) 1,00,000 taken together?
(c) 1,50,000 (d) 4,00,000 4 , .Q * *(1
96. In 2015, if the total sale of chocolate flavour is for Rs.5,160, 7 # C #
then the total sale (in Rs.) for vanilla flavour is: SSC CPO 23/11/2020 (Shift-2)
2015 " > * : d* . 7* H T (a) 59 :M66 @ 4 M (b)
# 61 : 59
. * d* . 7* H AT B C #(c) 59 : 61 (d) 66 : 59
SSC CPO 25/11/2020 (Shift-1) 101. In which school is the percentage of girls the lowest?
(a) 4,100 (b) 5,160 . O * *(1 # # -
A

(c) 4,120 (d) 3,300 SSC CPO 23/11/2020 (Shift-2)


97. In 2005, if 40% of the 'other' category is mix fruit (a) B (b) E
flavour and 1570 people surveyed preferred mix fruit (c) A (d) C
flavour, then how many people were surveyed? 102. What is the average number of student (girls and
2005 e - f . 0 40%L $g KM: d * . boys) in schools A, B, C, D and E?
. 0 L gK: d* . . O . A,** B, C, D )E . O b4 A*
1570 $ M # $ .0 # * *(1 % B # ) C #
SSC CPO 25/11/2020 (Shift-1) SSC CPO 24/11/2020 (Shift-1)
(a) 50,000 (b) 75,000 (a) 625 (b) 596
(c) 25,000 (d) 10,000 (c) 660 (d) 1250

Aditya Ranjan (Excise Inspector) Selected Selection 196

Downloaded by Kunal Chauhan (kc6970929@gmail.com)


lOMoARcPSD|39315497

Data Interpretation

103. In which school is the percentage of boys more than 55%? 108. What is the central angle of the sector representing
the number of employees in department E?
. O * *(1 # # U ’ $ %
SSC CPO 24/11/2020 (Shift-1) E ., 0" )
(a) B (b) A ; C #
(c) C (d) E SSC CPO 24/11/2020 (Shift-1)
104. What is the ratio of the girls from schools A, B and C (a) 36º (b) 108º
taken together to the boys from all schools taken together? (c) 72º (d) 90º
A, B C .Q * *(1 ,109. What
. Q *is the number
* ( 1 of employees working in
" 7 # L $ % department F?
SSC CPO 24/11/2020 (Shift-1) F ., 0 # 0" )
(a) 59 : 66 (b) 17 : 33
SSC CPO 24/11/2020 (Shift-1)
(c) 33 : 17 (d) 66 : 59
(a) 63 (b) 99
Direction (105 - 110): The given pie chart shows
the percentage distribution of 450 employees in an (c) 36 (d) 72
organisation. Study the pie chart and answer the question 110. If 75% of the employees in department D are males,

r
the follows. how many female employees are there in that
=0 " :0 N ? 0 " department? # # .# ;

si
0# $ =0 " :0 D . ’, # 75% 0" 7T/ $ M #
an by
Percentage of Employees in different Dept.
(Total 450 Employees)
0" # $ %
SSC CPO 24/11/2020 (Shift-1)

n
(a) 18 (b) 27
F A (c) 9 (d) 36
ja 22% 20%
Direction (111 - 116): The given histogram shows
R s
B the frequency distribution of the speed of cars passing
E 14%
a th

through at a particular spot on a highway. Study the graph


20%
C and answer the question that follows.
D 16%
8% $R: 0 .
ty a

" * (frequency
# distribution)
: 0#
105. What is the number of employees working in $ ’#
department B?
di M

B ., 0 . * 0" 100 ) C #
SSC CPO 23/11/2020 (Shift-2) 90
(a) 36 (b) 72 80
Number of Cars

70
(c) 63 (d) 90
60
106. What is the central angle of the sector representing 50
the number of employees in department A? 40
., A 0" ) T # 30 . * .8 ) (
- ; C # 20
SSC CPO 23/11/2020 (Shift-2) 10
0
(a) 72º (b) 36º 45-50 50-60 60-70 70-80 80-90 90-100
A

(c) 108º (d) 90º Speed in km/h


107. If 60% of the employees in department E are females, 111. What is the ratio of the number of cars with speed
how many male employees are there in that less than 60 km/h to the number of cars with speed
department? more than 80 km/h?
E ., 60% 0" $* $ M # 60. km/h
, 7 "T /* . * 80 km/h
)
0" ) C # ’ " * . * ) 7 # C
SSC CPO 23/11/2020 (Shift-2)
SSC CPO 23/11/2020 (Shift-2)
(a) 72 (b) 54 (a) 8 : 7 (b) 4 : 5
(c) 18 (d) 36 (c) 5 : 4 (d) 7 : 8

Aditya Ranjan (Excise Inspector) Selected Selection 197

Downloaded by Kunal Chauhan (kc6970929@gmail.com)


lOMoARcPSD|39315497

Data Interpretation
112. The number of cars with speed between 70 km/h
and 80 km/h is what percentage more than the number 120
110
of the cars with speed between 50 km/h and 60 110 105
km/h? (correct to one decimal place) 100 95 95 95
90

Number of Cars sold


70 km/h 80 km/h " " * . * ) M 85
80 80
80 75 75
50 km/h 60 km/h " " * . * ) 70
70 65
# # # ’ $ % A *. R604 # $ B
SSC CPO 23/11/2020 (Shift-2) 50
(a) 22.2% (b) 29.7% 40
(c) 28.6% (d) 15.5% 30
113. What percentage of cars were running with speed 20
less than 70 km/h? 10

70 km/h " * "* . * # 0# CB1 # B2 B3 B4 B5 B6


SSC CPO 23/11/2020 (Shift-2) Branches

(a) 51.25% (b) 30% 117. The average sale of cars (from all branches) for the

r
(c) 45.75% (d) 60% year 2018 is:
114. What is the ratio of the number of cars with speed M ./ 0 ! $7=0 #

si
less than 60 km/h to the number of cars with speed SSC CPO 24/11/2020 (Shift-2)
more than 70 km/h? (a) 80 (b) 78
60 km/h

an by
" * . *
" * . *
)
70 km/h
)
7
(c) 85 (d) 90
118. The total sale of cars from all the branches in the
# Cyear # 2018 is what percentage less than the total

n
SSC CPO 24/11/2020 (Shift-1) sales of cars from all the branches in the year 2019?
(a) 8 : 13 (b) 5 : 8 (correct to one decimal place)
ja
R s
(c) 8 : 5 (d) 13 : 8 . / 2018
0 M , ) 2019
$7=0 ,7 *
115. The number of cars with speed 60 – 70 km/h is what ) $7=0 7* H #
a th

percentage less than the number of cars with speed R4 # $ B


(km/h) in the interval 70 – 80 km/h? (correct to one SSC CPO 24/11/2020 (Shift-2)
decimal place) (a) 12.7% (b) 14.3%
(c) 11.9% (d) 13.5%
ty a

60 &70 km/h " " * . * 70 – 80 ) M


km/h " " * . * ) 119. #In
7* the bar graph,
# the ratio of the total sales from
branches B1, B2 and B3 taken together for the year
# # $ % AN *. R4 # # B
di M

2018 to the sales from the branches B6, B4 and B5


SSC CPO 24/11/2020 (Shift-1) for the year 2019, is:
(a) 5.9% (b) 8.6% 2018M . / 0 B1, ) B2 B3 $7=0 7
(c) 5.6% (d) 7.5% H 2019
M ./ 0 B6,
) B4 B5 $7=0
116. What percentage of cars were running with the speed 7 # C #
of 90 km/h and above? SSC CPO 24/11/2020 (Shift-2)

90 km/h ’ " * "* . * (a) 25 : 29 # # (b) 25 : 27


(c) 29 : 25 (d) 27 : 25
C #
Direction (120-125): Study the given histogram that
SSC CPO 24/11/2020 (Shift-1) shows the marks obtained by students in an examination
(a) 10% (b) 15.75% and answer the question that follows.
5 # $R: M
(c) 11.25% (d) 21.25%
A

0# $
Direction (117 - 119): The given bar graph shows
the sales of cars from six branches of a dealer B1, B2. B3,
70 60
B4, B5, B6, during two consecutive years 2018 and 2019.
Number of Students

60
Blue colour corresponds to the year 2018 and red colour 50 45
40
corresponds to the year 2019. 40
30
35 35
30
M # ./ 0H ! 20 < M
( * 6 I B1,) B2, B3, B4, B5, B6 $7=0 107 *
0
H 0# $ * ./ 0 ! ’# $ 150 200 * 250
* 300 . 350
/ 0 400 450
< 500
Marks
5 -’# $

Aditya Ranjan (Excise Inspector) Selected Selection 198

Downloaded by Kunal Chauhan (kc6970929@gmail.com)


lOMoARcPSD|39315497

Data Interpretation
120. The number of students who obtained less than 250 Direction (126-131): The following pie chart shows
marks is: percentage expenditure of a country on different heads.
250 F# . * 6 E The total expenditure
) C # is Rs.1,680 (in billions), Study the
chart and answer the question.
SSC CPO 24/11/2020 (Shift-2)
(a) 135 (b) 75 5 # =0 " :0 # # $ $
(c) 30 (d) 45 $ 7* ) "0 T @! A B $ " :0
121. If the total marks obtained by students be represented
as a pie chart, then the central angle corresponding Misc:1
Employees: 7
to marks 250 or more but less than 300 is:
(correct to the nearest degree) Defence: 12
6 E + F# 7* =0 " :0 a
Subsidy
b # # ’ M * [ 21
. * .8 ) ( - ; # $ A :# # B
Proceed’s
SSC CPO 24/11/2020 (Shift-2) Interest
to states
34 Payment: 15
(a) 188º (b) 88º
(c) 68º (d) 128º Health: 4

r
122. The number of students who obtained less than 350
Education: 6
marks is what percent more than the number of

si
students who obtained 400 or more marks? 126. The amount of expenditure of Defence is what
(correct to one decimal place) percentage more than the expenditure of Education?

350

an by F#
F#
. *
. * 400
6 E
6 E =
) $
) (Defence)
D
7 =
M
0 #
) " 0 $ 7 = 0(Education)
# # # # #
M

)D " 0
$ %

n
’ $ % SSC CPO 24/11/2020 (Shift-2)

A *. R4 # $ B (a) 50% (b) 125%


ja
R s
SSC CPO 24/11/2020 (Shift-2) (c) 150% (d) 100%
(a) 350% (b) 100% 127. The central angle of the sector representing
a th

(c) 385.7% (d) 375.8% expenditure on Subsidy is:


123. The number of students who obtained less than 250 h ((Subsidy) $7 W 0 . *
marks is what percent more than the number of
students who obtained 400 or more marks? - ; C #
(correct to one decimal place)
ty a

SSC CPO 24/11/2020 (Shift-2)


250 F# . 400
* 6 E= ’ ) 54.9º
(a) M (b) 90º
F# . * 6 E ) # # # ’ (d)$ 75.6º
%
di M

(c) 108.3º
A *. R4 # $ B 128. The total amount of expenditure for education and
SSC CPO 25/11/2020 (Shift-2) Health (in billions Rs.) is:
(a) 150% (b) 114.3% (Education)
D R .(Health)
R4 ) "0 $7=0
(c) 175.8% (d) 100% AT B C #
124. The number of students who obtained less than 200 SSC CPO 24/11/2020 (Shift-2)
marks is:
(a) 168 (b) 126
200 F# . * 6 E )
(c) 186 C # (d) 84
SSC CPO 25/11/2020 (Shift-2) 129. The expenditure on Education is what percentage
(a) 75 (b) 30 less than the expenditure on Defence?
(c) 40 (d) 135 (Education)
D ) " 0 $ 7 =(Defence)
0 M ) D" 0
125. If the total marks obtained by students be represents $7=0 # # # $ %
A

as a pie chart, then the central angle of the sector SSC CPO 25/11/2020 (Shift-2)
representing marks 200 or more but less than 300, is:
(a) 50% (b) 100%
(correct to the nearest degree)
(c) 150% (d) 125%
6 E + F# 7* =0 " :0 a
130. The expenditure for Proceeds to state (in billions
b # 200 =M # 300*
’ M Rs.) is:
0 . * .8 ) ( - Y ; C (Proceds
.# A
to state)
:# ) "0 $7=0
( # $ B B C #
SSC CPO 25/11/2020 (Shift-2)
SSC CPO 25/11/2020 (Shift-2)
(a) 154º (b) 128º (a) 126 (b) 586.50
(c) 88º (d) 68º (c) 684 (d) 571.20

Aditya Ranjan (Excise Inspector) Selected Selection 199

Downloaded by Kunal Chauhan (kc6970929@gmail.com)


lOMoARcPSD|39315497

Data Interpretation
131. The central angle of the sector representing 5 $R: . / X .Q
expenditure on interest payment is: 6 E . X 0# $ D
# 6 E
h , (Interest
7 # payment) ’# .8 ) ( $
- ; C #
SSC CPO 25/11/2020 (Shift-2) Y

Numbers of Students
80
(a) 54º (b) 90º 70
60
60
(c) 72º (d) 108º 50 45
40
The number of students in Class XI in Science, Arts 40
30
and Commerce streams of a school over a period of 4 years 30
20
20
2011 – 2014 has been depicted through the bar graph given 10 5
below. 0 x
40 45 50 55 60 65 70
4 . / 20110- 2014 . ’ . O * .C M * Weight (in kg)
. ; Y . XI 0 6 E D ) "
The number of students weighting less than 55 kg
0=0 =0 $ is what percentage less than the number of students
weighting 55 kg to 65 kg?
60

r
55 kg . . * 6 E 55 kg 65 kg) . M
50
. * 6 E ) # # #

si
40 Science SSC CPO 25/11/2020 (Shift-1)

30 an by (a) 5% (b) 10%


Arts
(c) 8% (d) 7
20 Commerce

n
Direction (136-137) : The given pie-chart represents
10 the distribution of the percentage of sales of a particular
brand of car from five showrooms A, B, C, D and E during
0 ja 2018. The total number of cars sold during that year from
R s
2011 2012 2013 2014
the five showrooms is 5000.
132. What is the difference between the averages of the
a th

=0&" :0 ! A, B, C, D E "
number of Science and Commerce students in the
given class over the given period of 4 years? . /: ( H # #
./ 0 " a " =0
4 ./ 0 0=0 =0 . ’ M b /: D .C . ; Y
ty a

6 E ) # # # $ %
E
SSC CPO 25/11/2020 (Shift-2)
D 7%
di M

(a) 12 (b) 8 9%
(c) 5 (d) 10
C A
133. What is the average of the number of Arts students 48%
14%
in the given class over the given period of 4 years?
4 ./ 0 0=0 =0 . ’ M b /: D B *
22%
6 E ) # L $ %
SSC CPO 25/11/2020 (Shift-2)
(a) 30 (b) 35 136. What is the total number of cars sold from the show-
(c) 25 (d) 32 rooms B and D?

134. During which week was the fasting blood sugar level Ba D " =0 7* )
of the patient approximately 115.4% of the upper SSC CPO 09/12/2019 (Shift - 01)
A

limit of normal PP blood sugar level? (a) 1750 (b) 1550


F# $ PP (c)
R :h * ( h * ( 7 1650 * .*M (d) 1450
7 * .* - c 115.4% 4* ,% 137. What is the central angle (nearest to 0.1 degree) of
the sector corresponding to the sales from the
SSC CPO 25/11/2020 (Shift-2) showroom C?
(a) W4 (b) W1 C
a H # ) ( i
(c) W3 (d) W2 :# B C #
135. The following histogram shows the weights of SSC CPO 09/12/2019 (Shift - 01)
students of class X in a school on a particular day. (a) 48.6 (b) 60.5
Total number of students enrolled in class X = 200. (c) 56.7 (d) 50.4

Aditya Ranjan (Excise Inspector) Selected Selection 200

Downloaded by Kunal Chauhan (kc6970929@gmail.com)


lOMoARcPSD|39315497

Data Interpretation

138. The following pie-chart and table show the percentage YC . . * E


distribution of famers in 5 states A, B, C, D and E YA 7* ) A -$
of a country who claimed insurance due to drought ) ! ’E $ Y .
in the year 2017 and ratio of males to females of the
# # L $ %
number of farmers in states respectively.
SSC CPO 09/12/2019 (Shift - 01)
5 * ) # =0&" :0 A,
# B, C,
* D (a) 10% Y (b) 15%
E 7(1 # # : 0 #
(c) 40% $ M(d) 25% -$
./ 0 V K) ; Direction
4 (139-140) : The
# * bar graph represents
given
)* 7T/ $* the average7marks# , obtained in English
0# (E) and Vernacular
(V) by the students of five district (D1, D2, D3, D4, D5) in a
$ state at the secondary level examination of a particular year
Study the pie-chart and the table carefully and (marks secured out of a total of 100)
answer the following question. . / ./ 0
=0&" :0 # * K.0 Y (D51, D"2*, D)3, *D
#4, D5) 6 E (E)+
#(V)8 , / F# # A
Note : Some data are missing in the pie-chart and table, 0# # $
if required in any question, find the missing data first

r
Average Marks in Exam
and then answer the question 100 E V 90
90

si
85
:I =0&" :0 # * 76 (1 * 7 80
F # $80 M 75
. # $ M # $* F# 70 70 70
60 60

./ 0
= 2,64,000
an by V . . *
60
50
40
7 * ) 50
55

n
30
Total number of farmers who claimed insurance in 20
10
the year 2017 = 2,64,000
ja 0 01 02 03 04 05
R s
./ 0 V . . * 7* )
= 2,64,000 139. What is the average percentage of marks in English
a th

of the five district?


" * 6 E +
E # # C #
A
SSC CPO 09/12/2019 (Shift - 01)
24%
ty a

(a) 73% (b) 67%


(c) 75% (d) 68%
D
di M

140. What is the difference the average percentage of


20%
marks in vernacular of the districts (D1, D2, D4) and
B (D3, D5)?
C * (D1, D2, D4) (D3, D5) 6 E + #8
36%
# # # # C
SSC CPO 09/12/2019 (Shift - 01)
(a) 1.7% (b) 11.3%
(c) 22.5% (d) 3.5%
States Ratio of male to female in
Direction (141-143): The given histogram represents
the farmers who claimed
the frequency distribution of average runs scored by 50
insurance in 2017
selected players from a district in a local cricket tournament.
A Male : Females # "E R4 H : :K 0
B 05 : 03 ) * (1 + #
A

C ............
18
D 5 : ______ 16 16

14
E 03 : 02 12 12
Frequency

10
10
8
The number of farmers who claimed insurance in 6
6
state C exceeds the total number of farmers (who 4
4

2 2
claimed insurance) in state E and state A together
0
by 5280. What is the percentage of farmers who 10.5 20.5 30.5 40.5 50.5 60.5 70.5

claimed insurance in state E? Runs

Aditya Ranjan (Excise Inspector) Selected Selection 201

Downloaded by Kunal Chauhan (kc6970929@gmail.com)


lOMoARcPSD|39315497

Data Interpretation
141. How many players scored more than 30.5 on average? <<V X" * .
# ) * (1 # [ S ’ < <$V % = . ’ #
SSC CPO 09/12/2019 (Shift - 01) # K_ # L 4 %
(a) 28 (b) 34 SSC CPO 09/12/2019 (Shift - 01)
(c) 12 (d) 16 (a) 77.5 Lakhs (b) 70.5 Lakhs
142. Which class of boundaries does the frequency of ‘10’
correspond to? (c) 76.5 Lakhs (d) 87.5 Lakhs

'10' # (boundaries)
( \ ; ’Direction
# $ %(145-147): Study the pie-chart and answer
the question.
SSC CPO 09/12/2019 (Shift - 01)
=0&" :0
(a) 20.5 to 30.5 (b) 10.5 to 20.5
Distribution (degree-wise) of the number of employees
(c) 40.5 to 50.5 (d) 30.5 to 40.5 of a company working in 5 departments A, B, C, D and E.
143. What is the overall average score of the 50 players? A,. B,
, C, D E . * 0
) * (1 7* # R C # ) : A 7 B $
Total number of employees = 3200
SSC CPO 09/12/2019 (Shift - 01)

r
(a) 29.3 (b) 38.5 E
A

si
(c) 32.6 (d) 37.1 36°
79.2°
144. Out of the two bar graphs provided below, one shows D
an by
the amounts (Rs. in Lakhs) invested by a company
in purchasing raw materials over the years and the
64.8°
B

n
other shows the values (Rs. in Lakhs) of finished 57.6°
goods sold by the company over the years. C
" ja M $* + 6* ./ 2 122.4°
76
R s
X" * H . =0 ’ ATS * ) B 0#
# $ K 6* 7* ./ 2 145. The number
+ of employees
" working
# in department
* C
a th

is what percentage more than the total number of


K_ ATS * ) B 0# # $
employees working in D and E? (Your answer should
Amount invested in Raw Material (Rs. in Lakhs) be correct to one decimal place.)
., C . * 0 "D E )
ty a

600 . * 0" 7* ) #
525
A *. R4 # $ $
di M

450 420 SSC CPO 09/12/2019 (Shift - 02)


Rs. In Lakhs

375
300 330
(a) 23.2 (b) 20.6
225
(c) 22.8 (d) 21.4
150 120 146. If the total number of employees working in departments
A and B exceeds the number of employees in department
0
1995 1996 1997 1998 1999 2000
C by x, then x lies between.
Years . ,A B . * 0"
., C 0" x ’ $) xM # =
Value of sales of Finished Goods (Rs. in Lakhs) R4 # $ %
SSC CPO 09/12/2019 (Shift - 02)
800 (a) 120 and 140 (b) 100 and 120
600 (c) 80 and 100 (d) 140 and 160
600
147. If the ratio of male and female employees working in
A

460
Rs. in Lakhs

500
400 400 department D is 4 : 5 and that in department E is 9 :
300 11, then what is the ratio of the total male employees
120
200 in D and E to the number of employees in B?
. ,D 0 # 7a/ 0"
0
1995 1996 1997 1998 1999 2000 7 # ? I $E $. , 7 # D< I $
Years
E , 7a/ 0" B 0) # M
0"
What was the difference between the average amount ) 7 # L $ %
invested in raw materials during 1997 to 2000 and SSC CPO 09/12/2019 (Shift - 02)
the average value of sales of finished goods during (a) 31 : 32 (b) 25 : 32
the same period 1997 to 2000? (c) 7 : 8 (d) 15 : 16

Aditya Ranjan (Excise Inspector) Selected Selection 202

Downloaded by Kunal Chauhan (kc6970929@gmail.com)


lOMoARcPSD|39315497

Data Interpretation
Direction (148-150) : Study the graph and answer Wheat
Rice
the question.
700 Production (in lakh tonnes) of rice and wheat in five
states of a country in 2018

Production (in lakhs tonnes)


" 600 575 560 525
500
500 450 425 475
Daily wages (in Rs.) of workers in a factory 400
80
400 350 375
Number of workers

70 72
60
300
60 58 55
50 50 48 200
40
40 37
100
30
20 0
A B C D E
10
0 States
400 450 500 550 600 650 700 750 800
Daily Wages (in Rs.) 151. Total production of wheat in states A, B and E is what
percentage less than the total production of rice in
148. The number of workers in the factory whose daily states C, D and E?
wages are Rs. 450 or more but less than Rs. 600 is YA, B E $KZ 7C,
* D E M Y
__________
" .* 7* # # # $
) \ ) # $ M .SSC
# T S09/12/2019
?

r
CPO (Shift - 02)
’ $ M * TS @ $ %(a) 10 (b) 12

si
SSC CPO 09/12/2019 (Shift - 02) (c) 12.2 (d) 11.1
152. The ratio of the total production of rice in states C
(a) 148 an by (b) 180
and E to the total production of wheat in states B
(c) 190 (d) 170 and D is ______.

n
YC E " .* 7* B D M Y$ K
149. The number of workers in the factory whose daily
wages are Rs. 500 or more but less than Rs. 650 is 7* 7 # SSSSSSSSSSS
ja
what percentage more than the number of workers SSC CPO 09/12/2019 (Shift - 02)
R s
whose daily wages are Rs. 650 or more but less than (a) 11 : 10 (b) 22 : 19
Rs. 750?
a th

(c) 20 : 17 (d) 13 : 12
(Your answer should be correct to one decimal 153. The number of states in which the production of
wheat is more than 20% of the total production of
place.)
rice in all states is _________.
ty a

) \ ) M . #YTS ) # $ M $K

$ M * TS @ $ M \ " .* ) 7* # U ’ $ %
di M

# # ’ $ . # TS @ ’ $ SSC
M *CPO 09/12/2019 (Shift - 02)
TS V $ % (a) 4 (b) 3

A *. R4 # $ $ (c) 2 " $ B (d) 1


Direction (154-156) : Study the given pie chart and
SSC CPO 09/12/2019 (Shift - 02) answer the question that follows.
(a) 84.5 (b) 45.8 5 =0&" :0
(c) 75.4 (d) 76.6 Break-up (degree-wise) of the number of students in
150. The ratio of the number of workers in the factory five schools (A, B, C, D and E) in a city.
whose daily wages are below Rs. 600 to the number $ "(A, B,
. QC, *D E) .Q 4 0
of workers whose daily wages are more than Rs. 650 .. ; A 7 B $
but less than Rs. 800 is ________.
) TS @ . # . * \ ) M
A

T
TS @ ’ * TS ! . # . * \
57.6° A
) 7 # L $ % 86.4°
D
SSC CPO 09/12/2019 (Shift - 02)
64.8°
(a) 2 : 1 (b) 11 : 10 B
(c) 17 : 14 (d) 11 : 7 C 72°

Direction (151-153): Study the graph and answer 79.2°


the question.
Total number of students = 5200
" = 5200

Aditya Ranjan (Excise Inspector) Selected Selection 203

Downloaded by Kunal Chauhan (kc6970929@gmail.com)


lOMoARcPSD|39315497

Data Interpretation
154. The number of students in school C is what 158. In how many years was the revenue of the company
percentage more than the number of students in more than 1.2 times the average expenditure over the
school B ? given the years?
. Q C* .Q 4 0 B .) Q M 4 .0Q * ) Z" ./ 2 # ./ 0 #
S 7 4 %
# # # ’ $ %
SSC CPO 11/12/2019 (Shift - 01)
SSC CPO 11/12/2019 (Shift - 01) (a) 2 (b) 1
(a) 7.2% (b) 10% (c) 3 (d) 4
(c) 11% (d) 9.5% 159. Total expenditure of the company in 2016, 2017 and
2018 is what percentage less than the total revenue
155. If the ratio of the number of boys to that of girls in school B
in the given five years (correct to one decimal place)?
is 7 : 6 and the ratio of the number of boys to that of girls in
school D is 4 : 5, then what is the ratio of the number of
@M V ! 7* W
boys in B to that of girls in D? 7* # # # $ A *.
SSC CPO 11/12/2019 (Shift - 01)
. QB * *(1 ) *(1 )
(a) 38.4%
7 # V I(b)@34.5%
$ D. Q ** ( 1 ) *(1 )
(c) 36.2% 7 # (d) 36.8%
? I $ M B# *. (Q1 * )D Direction
* ( 1. Q * (160-162): Study the given graph and answer
) 7 # L $ % the question that follows.

r
SSC CPO 11/12/2019 (Shift - 01) 5
Break-up age-wise (in years) of workers in a company work.

si
(a) 14 : 13 (b) 12 : 13
5 \ 7 .. ; A./
(c) 13 : 12 (d) 13 : 14
an by
156. It the total number of students in schools D and E
exceeds the number of students in school A by x,
100
90
80
90 85
80

Number of workers
75 75
then x lies between.

n
70
70 65

. QD * E .Q 4 0 A7 * M ) . Q60
50
* 55

.Q 4 0 ja x ’ $) xM # R4 # $ %
40
R s
30 24
SSC CPO 11/12/2019 (Shift - 01) 20
(a) 475 and 500 (b) 550 and 575 10
a th

(c) 525 and 550 (d) 500 and 525 0


20 24 28 32 36 40 44 48 52 56
Direction (157- 159): Study the given graph and Age (in years)

answer the question that follows 160. What is the ratio of the number of workers in the
5 company whose age is 28 years or more but less than
ty a

Revenue and Expenditure (In Rs. Crore) of a company 40 years to the number of workers whose age is 40
in five years years or more but less than 52 years?
di M

Z" ./ 2 5 7* W A (1 T ! ./ 0 B $ ’ * ? ./ 0
) M ? ./ 0 ’ * .
Revenue and Expenditure (In Rs. Crore)
of a company in five years \ ) 7 # C #
Revenue Expenditure SSC CPO 11/12/2019 (Shift - 01)
400 380 (a) 7 : 6 (b) 9 : 8
350 320 325 (c) 10 : 7 (d) 23 : 21
300
300 275
161. The number of workers whose age is 32 years or more
250 250
210
but less than 44 years is:
200
200 [ ./ 0 ’ M * ?? ./ 0
150
150
) # $ %
100
SSC CPO 11/12/2019 (Shift - 01)
50
(a) 220 (b) 215
A

0
2014 2015 2016 2017 2018 (c) 225 (d) 230
Years 162. The number of workers whose age is 36 years or more
157. What is the ratio of the total expenditure in 2015 but less than 48 years is what percentage more than
and 2016 to the total revenue of the company in 2016 the number of workers whose age is less than 28
and 2018? years (correct to one decimal point)?
[@ ./ 0 ’ * ?! ./ 0
@ 7* W @) M A ! !. / 0 7 . * \
7* 7 # C # ’ $ % A *. R4 # $ B
SSC CPO 11/12/2019 (Shift - 01) SSC CPO 11/12/2019 (Shift - 01)
(a) 4 : 5 (b) 19 : 20 (a) 60.7% (b) 61.2%
(c) 6 : 7 (d) 11 : 20 (c) 62.8% (d) 59.4%

Aditya Ranjan (Excise Inspector) Selected Selection 204

Downloaded by Kunal Chauhan (kc6970929@gmail.com)


lOMoARcPSD|39315497

Data Interpretation
Direction (163-165): Study the pie chart and answer
Number of trees planted by an oragnisation
the question. in states A and B during 2014 to 2018

=0&" :0 State- A State- B


140

No. of trees planted (in hu ndered


Distribution (degree wise) of the number of computers 120
120
sold by a shopkeeper during five months
100
100 90 90 90
Total number of computers sold = 5400 80
80 80
70 70
7 + " $ " 5 F K60: ) :
A ( 7 B 40
20
0 2014 2015 2016 2017 2018
May Years
January
77°
85° 166. In how many years was the number of trees planted in
April state B more than the average number of trees planted
in state A over the years?
54°
February # ./ B2 * Y (1 ) M

r
March 45° YA * (1 # )
99° SSC CPO 11/12/2019 (Shift - 02)

si
(a) 1 (b) 3
an by
163. If the difference between the number of computers sold
in March and the number of computers sold in January
(c) 4
167. Percentage decrease in the number of trees planted
in 2016 in state A as compared to that in 2015 in
(d) 2

n
is x, then x lies between.
the same state was
"0 " 5F K: ) . "
5F K: ja ) x x" #
$ M # YA M @ * (1
R s
$ % * (1 ) #7*
SSC CPO 11/12/2019 (Shift - 02)
a th

SSC CPO 11/12/2019 (Shift - 02)


5 2
(a) 200 and 250 (b) 300 and 350 (a) 15 (b) 15
13 3
(c) 250 and 300 (d) 150 and 200
ty a

164. The number of months, in which the number of 1 1


(c) 11 (d) 13
computer sold was above 20% of the total number of 9 3
computers sold in 5 months was:
di M

168. The total number of trees planted in state B in 2015 and


$ ) # $ M " 2017 was5 what
F Kpercentage
: ) the
less than M total number of
$ " 5F K: 7* ) trees plantedUin state A ’ 4 % and 2017? (Your
in 2014, 2016
answer should be correct to one decimal place.)
SSC CPO 11/12/2019 (Shift - 02)
YB V * (1
(a) 3 (b) 4 A ?M @ V *
(c) 1 (d) 2 # # $ % A *. R4
165. The total number of computers sold in February and SSC CPO 11/12/2019 (Shift - 02)
April is what percentage more than the number of (a) 19.8 (b) 21.4
computers sold in May? (Your answer should be (c) 20.8 (d) 20.1
correct to one decimal place.) Direction (169-171): Study the bar graph and
. * " 5F K: answer the
7 *question.
) M =0 "
# # 5 ’ * )$ #% A
A

5F K: ) #
*. R4 # $ $ " $ B 100
Life time (in hours) of neon lamps

90 90
SSC CPO 11/12/2019 (Shift - 02) 80
80
75
Number of neon lamps

70
70
(a) 25.8 (b) 28.6 60
60 55
(c) 30.2 (d) 26.4 50 45
40 35
Direction (166-168): Study the bar graph and 30
answer the question. 20
10
5 * ) # 0
400 500 600 700 800 900 1000 1100 1200

Hours

Aditya Ranjan (Excise Inspector) Selected Selection 205

Downloaded by Kunal Chauhan (kc6970929@gmail.com)


lOMoARcPSD|39315497

Data Interpretation
169. The number of lamps having a life of 600 hours or 173. If the ratio of the number of boys to that of girls in
more but less than 800 hours is what percentage of school C is 6 : 5 and that in school B is 7 : 11, then
the total number of neon lamps having a life of 800 what is the ratio of the number of boys in school C
hours or more? to the number of girls in school B?
@ ] : ’ M * ! ] : 0 . QC** . * ’( 1. * ) *(1
> * ) M ! ] : ’ @ 0I *$ . B’ . .* Q. $ * >7 # V IC $ *M( 1#
* ) # # # $ % ) M B . Q ** ( 1 ) 7
SSC CPO 11/12/2019 (Shift - 02) SSC CPO 12/12/2019 (Shift - 01)
(a) 66.67 (b) 67.50 (a) 10 : 11 (b) 11 : 10
(c) 69.25 (d) 68.75 (c) 12 : 11 (d) 11 : 12
170. The ratio of the number of neon lamps having a life 174. The number of students in school E exceeds the total
of 900 hours or more to the number of lamps having number of students in schools A and B by x, then x
life 700 hours or less. lies between.
900 ] : ’ 0 * . ’ . * . Q E*> * . Q 4 0 A 7 *B ) M
.Q .Q
4 0*
) V ] : 0 * . ’ . * ) x* ’ $ xM #) = "
7 # L $ % SSC CPO 12/12/2019 (Shift - 01)
SSC CPO 11/12/2019 (Shift - 02) (a) 420 and 440 (b) 440 and 460

r
(a) 25 : 21 (b) 8 : 9 (c) 460 and 480 (d) 400 and 420
(c) 6 : 7 (d) 7 : 8 Direction (175-177): Study the given graph and

si
171. The number of neon lamps having a life of 500 hours
answer the question that follows.
or more but less than 800 hours is:
an by >

*
$ M * !
)
]
#
:
$ M
$ %
5 0 *
Distribution of marks obtained by students in an
examination
. ’ ] :

n
SSC CPO 11/12/2019 (Shift - 02)
D 6 E + F#
(a) 180 (b) 305
(c) 270 ja (d) 225 160 150
R s
Direction (172-174): Study the given pie chart and 140
142

answer the question that follows:


Number of Students
120 110
a th

100
5 =0&" :0 100
80
80 78
Distribution (degree wise) of students studying in 60 45 55
schools A, B, C, D and E 40
ty a

. Q A,
* B, C, D E (1 . * 6 E : A 20 7 B
0 10 20 30 40 50 60 70 80 90 100
Total number of students = 9000/ 6 E = 9000
7* )
di M

Marks

175. The number of students who obtained 20 marks or


A more but less than 60 marks is:
36° .Q 4 0 ) # $
E $ M * @ $ %
118.8° B
SSC CPO 12/12/2019 (Shift - 01)
64.8° (a) 402 (b) 600
(c) 480 (d) 522
D C 176. If 60% of the total number of students obtaining 50
79.2° marks or more but less than 70 marks and 55% of
61.2°
the total number of students obtaining 70 marks or
more but less than 90 marks are also girls, then the
A

total number of girls obtaining 50 marks or more but


172. The total number of students in schools D and E is
less than 90 marks is:
what percentage more than the total number of
students in schools B and C? ’ M * V
.Q 4 0 7* ) @ U *(1
. Q D* E .Q 4 0 B 7 *C ) M . Q ** < F# . *
.Q 4 0 7* ) # # ) # U$ , % * ( 1 $ M #
SSC CPO 12/12/2019 (Shift - 01) < F# . * *(1
(a) 20 (b) 25 SSC CPO 12/12/2019 (Shift - 01)
(a) 261 (b) 207
(c) 30 (d) 35 (c) 236 (d) 257

Aditya Ranjan (Excise Inspector) Selected Selection 206

Downloaded by Kunal Chauhan (kc6970929@gmail.com)


lOMoARcPSD|39315497

Data Interpretation

177. The total number of students obtaining less than 50 Direction (181-183): The pie-chart given here shows
marks is what percentage more than the total number the country’s expenditure on various sports during a
of students obtaining 60 marks or more (correct to particular year. Study the pie-chart and answer the
one decimal place). question given below.
50 F# . * . Q60 4 0 7 * = )0 & " : 0 + . / .
= ’ F# . * .Q 4 0 7* )W # 0# # #
# $ =0&
’ $ A *. R4 # $ "$ " $ B
SSC CPO 12/12/2019 (Shift - 01)
(a) 6.8 (b) 4.4 Hockey
Cricket 65°
(c) 5.6 (d) 6.2
80° Foot ball
Direction (178-180): Study the given graph and
answer the question that follows: 52°
5 Tennis Others
48° 30°
Daily sales (in thousands) of newspapers
Basket Golf
P and Q in five cities
Ball 35°
Newspaper P Newspaper Q 50°

r
800 750 181. If the total amount spent on sports in the year was
700 680

si
600 620 Rs. 1,80,00,000, then the amount spent on tennis
600 560 550
640 is more than the expenditure on golf by:
500 450 480
an by
400
300 #
K
:
./ 0 ) * 1,80,00,000
W
W $
=4 0
. * ’ M
M

_

n
200 # ’ $ %
100 SSC CPO 12/12/2019 (Shift - 02)
0
ja A B C D E
(a) Rs. 2,40,000 (b) Rs. 4,75,000
R s
Cities (c) Rs. 5,50,000 (d) Rs. 6,50,000
182. The expenditure on hockey is what percent more than
a th

178. In which city the daily sales of newspapers P is 1.25 that of basketball?
times the average daily sales of newspaper Q in
$ > W M R : >*
cities A, B, C, D and E?
# # ’ $ %
$ P " E A, B,
H C, D E $ SSC CPO 12/12/2019 (Shift - 02)
ty a

"Q E # 1.25H 7 $ % (a) 40% (b) 25%


(c) 30% (d) 35%
SSC CPO 12/12/2019 (Shift - 01)
di M

183. If the total amount spent on sports in the year was


(a) B (b) D Rs. 2,40,00,000, then the amount spent on cricket
(c) C (d) A and football together was:
K ./ 0 ) * W TS
179. The total daily sales of newspaper P in cities B, D
and E is what percentage less than that of newspaper 77 : >* 7* * # W
Q in cities A, C, D and E? SSC CPO 12/12/2019 (Shift - 02)
(a) Rs. 86,00,000 (b) Rs. 88,00,000
$B, D E "P E7 * A,
H M $
(c) Rs. 78,00,000 (d) Rs. 44,00,000
C, D E "Q E# 7 * # # # $ Study
Direction(184-186): % the graph and answer the question.
SSC CPO 12/12/2019 (Shift - 01) 5
(a) 24.4 (b) 22.5 Percentage distribution of Total Expenditure of a
company in 2012.
(c) 20.2 (d) 20.8
A

7* W # #
180. What is the ratio of the total daily sales of
25
newspaper P in cities A and C to the total daily sales
20
20
of newspaper Q in cities B and D? 17.5

15
15 12.5
$A C "P E7* HB M $ 10
10
D "Q E 7* H 7 # C # 5
5

SSC CPO 12/12/2019 (Shift - 01) 0


In

Tr

Ta

Sa

In
Lo res

(a) 16 : 17 (b) 7 : 8
dv
fr

t
an

an

la
xe

e
an t o
as

er

ry
d
sp

s n
ti
tr

D
or

se
uc

m
tu

(c) 15 : 16 (d) 4 : 5
en
re

Aditya Ranjan (Excise Inspector) Selected Selection 207

Downloaded by Kunal Chauhan (kc6970929@gmail.com)


lOMoARcPSD|39315497

Data Interpretation

184. If the total amount of expenditure of the company is x times 187. A sum of Rs. 3.5 lakhs was invested in company Q
the expenditure on transport, then the value of x is: in the year 2000 for one year. How much more simple
W 7 * (Transport)
’ .W
$ interest will accrue if the amount was invested in
company P?
x 7 x
$ M # L $ %
SSC CPO 12/12/2019 (Shift - 02) ./ 0 ./ Q
0 3.5
*T S
* )
(a) 6 (b) 8 . = ’P .
(c) 12 (d) 5 $ # M # ’ ; h ’ #
185. What is the ratio of the expenditure on salary and SSC CPO 12/12/2019 (Shift - 02)
interest on loans to the total expenditure on (a) Rs. 5,500 (b) Rs. 5,000
infrastructure and transport?
(c) Rs. 5,200 (d) Rs. 5,250
. #(salary) j; (interest
h on loans) $ #7 188. If two sums in the ratio 9 : 10 are invested in
7* W M (infrastructure)
’ , K# P " (trans-
.$ companies P and Q respectively in 1999, then what
port) 7* W 7 # L $ %is the ratio of the simple interest received from
SSC CPO 12/12/2019 (Shift - 02) companies P and Q respectively after one year?
(a) 15 : 13 (b) 15 : 11 <<< ’ < IP
(c) 14 : 11 (d) 13 : 12 Q . MP # Q ./ 0 F

r
186. If the expenditure on salary is Rs. 2.8 crores, then ; h 7 # L $ %
the difference between the expenditure (in Rs.) on

si
SSC CPO 12/12/2019 (Shift - 02)
advertisements and taxes is?
(a) 7 : 8 (b) 8 : 9
. (salary)
#
an by 7* W T S S ! (adver- (1 $ M # .C
(c) 9 : 8 (d) 8 : 7
tisements) (tases) . * W AT#S B
189. An investor invested a sum of Rs. 4 lakhs in company

n
# $ % Q in the year 2000. After one year, the entire amount
SSC CPO 12/12/2019 (Shift - 02) was transferred to company P in 2001 as an investment
(a) 70 Lakhs (b) 60 Lakhs for one year. What amount will the investor receive from
ja company P?
R s
(c) 30 Lakhs (d) 75 Lakhs
Direction (187-189): Two different finance . ./ 0 Q TS ? * ) ’
a th

companies declare a fixed annual rate of interest on the amount . ./ 0 M P K


invested by investors with them. The interest rate declared by
these companies can vary from year to year depending on the ./ 0 * . a R4 # #
variation in the country’s economy and the interest rate of the P # ’ * %
banks. The annual rate of interest offered by the two companies
ty a

P and Q is shown in the graph below. SSC CPO 12/12/2019 (Shift - 02)
* & * . $ . (a) Rs.. 4,54,420
* . (b) Rs. 4,64,340
di M

’ "# . / 0 h ] (c)/ Rs.; 4,64,430


# $ =(d) Rs. 4,56,320
+ . * h 4 0 W . R 4 Direction (190-192): Study
h the pie-chart’ and answer
. / 0 & & . / 0 , - P$ Q +
# $ . *
the question.
h . / 0 " 0# = 0= &0 " $: 0
Study the graph and answer the question.
Distribution (degree wise) of the total number of
5
employees of a company in its five officers A, B, C, D and E
Annual percentage rate of interest offered by two
finance companies P and Q over the years. 0" 7* )
6* 76 ./ 2 P Q + . R# . #7 B = " ’ A, B, C, D E
. / 0 h # # Total Number of employees = 9200
0" = 9200
7* )
A

E
64.8°
A
D 100.8%
57.6°

C B
1 54° 82.8°

Years

Aditya Ranjan (Excise Inspector) Selected Selection 208

Downloaded by Kunal Chauhan (kc6970929@gmail.com)


lOMoARcPSD|39315497

Data Interpretation

190. If 50% of the employees in office E are transferred to 194. What percentage of total number of workers in the factory
office D, then what percentage of the total number of is the number of workers whose weekly earning are Rs.
employees of the company is the number of employees 6,500 or above but less than Rs. 8,500? (Your answer
in office D? should be correct to one decimal place)
0E * U 0" D R4 0* # #) \ 7* ) #
M # 7* 0" ) F# #$ . ## T#S @ M
0" D 0* $ % $ % A *. R4 #
SSC CPO 13/12/2019 (Shift - 01) SSC CPO 13/12/2019 (Shift - 01)
(a) 28 (b) 25 (a) 50.2 (b) 42.6
(c) 35.8 (d) 49.3
(c) 26 (d) 30
195. The number of workers in the factory whose weekly
191. Total number of employees of the company in offices earnings are Rs. 5,500 or more but less than Rs. 7,500 is:
C and D exceeds the number of employees in office B ) \ ) # $
by x, where x lies between
M ’ $ M . # TS VM
0C* D 0" 7* M ) 0SSC
* CPO 13/12/2019 (Shift - 01)
B 0" x ’ $) Mx $ Z " R4 # $(a) 375 (b) 285
SSC CPO 13/12/2019 (Shift - 01) (c) 345 (d) 240

r
(a) 720 and 740 (b) 680 and 700 Direction (196-198): Study the graph and answer
(c) 700 and 720 (d) 740 and 760 the question.

si
192. The total number of employees of the company in
offices D and E is what percentage more than the Exports (in Rs. millions) of cars of Type A and Type
an by
number of employees in office A? (Your answer should
be correct to one decimal place.)
8 over the years
K ./ 0 A :B =: 0= 0
& & 0# AT

n
D 0 *E 0" 7A * ) M 0* Type- A Type- B
400
0" ) # # # ’ $ % 350
A *320.

Exports (in Rs. million)


350 340

ja R4 # $ $ " $ B 300
300
280 270
R s
275 260
250
250 225
SSC CPO 13/12/2019 (Shift - 01) 200
a th

(a) 21.8 (b) 20.6 150


100
(c) 21.4 (d) 22.2 50
Direction (193-195): Study the graph and answer 0
2012 2013 2014 2015 2016
the question. Years
ty a

196. In which year, were the exports of type B can exactly 20%
Weekly earnings (in Rs.) of workers in a factory more than the exports of type A cars in 2014?
di M

. / B0 A 0#M ?
5 \ F# $ . # AT B
0# K U ’ 4 %
120 SSC CPO 13/12/2019 (Shift - 01)
105
100
95
90 (a) 2015 (b) 2016
85
(c) 2012 (d) 2013
Number of workers

80
80 70
65 197. In how many years were the exports of type B cars
60
60 50 more than the average exports of type A can during
40 2012 to 2016?
20
@ M B # . / 0 4
0
7* A 0#M # 0#
50 55 60 65 70 75 80 85 90 95
00 00 00 00 00 00 00 00 00 00 SSC CPO 13/12/2019 (Shift - 01)
Weekly Earnings (in Rs.) (a) 2 (b) 3
A

193. The number of workers in the factory whose weekly (c) 1 (d) 4
earnings are below Rs. 7,000 is what percentage more 198. The export of type A cars in 2016 are what percentage less
than the number of workers whose earnings are Rs. than the total exports of type B cars in 2014 and 2015?
7,000 or above but less than Rs. 8,500? @A 0#M B ?
) TS VM F# $ . # . * \ 7* ) 0 #M # # #
\ ) # # # ’ $ M FSSC
# CPO
$ 13/12/2019 (Shift - 01)

. # TS VM ’ $ M * M2
T S(a) ! 26 $ %(b) 32
SSC CPO 13/12/2019 (Shift - 01)
3
(a) 43.75 (b) 42.50 1
(c) 45 (d) 43
(c) 44.25 (d) 45.25 3

Aditya Ranjan (Excise Inspector) Selected Selection 209

Downloaded by Kunal Chauhan (kc6970929@gmail.com)


lOMoARcPSD|39315497

Data Interpretation

Direction (199-201): Study the pie-chart and answer


80
the question. 70
70 65
=0&" :0 60 55

Number of families
52
Distribution (degree-wise) of the number of employ- 50 47 43
ees of a company working in five departments 40 35
30
5 Z" ., . * 0" 30
23 : A
20
7 B 10
0 15 2 2 3 3 4 4 5 5 6
HR 00 000 500 000 500 000 500 000 500 000
Accounts
57.6° 32.4° Expenditure (in Rs.)

202. The number of families whose monthly expenditure


Marketing Production on education are Rs. 2,500 or more but below Rs.
86.4° 111.6° 4,000 is what percentage more than the number of
IT families whose monthly expenditure on education
72° are Rs. 4,500 or more but below Rs. 6,000?
D TS M ’ * TS ?M
Total number of Employees = 1400 . * . ) M D TS

r
= 1400 @M W . * .
199. If 80% of the number of employees working in the IT
# # $ %

si
department and 40% of the number of employees
working in both the HR and Accounts department are SSC CPO 13/12/2019 (Shift - 02)
an by
females, then total number of female employees (a) 37.6 (b) 36.4
working in these three departments is. (c) 29.8 (d) 27.3
=(IT)
0 S :. , . * 0 " 203. What is the )ratio of the number of families whose

n
! U . (HR) ’ *(Accounts)
) . , monthly expenditure on education are below Rs.
3,000 to the number of families whose monthly
. * 0" ) ?expenditure
U $ * on education
$ M #are Rs. = 4,000 or above but
ja
R s
# ., . * $* less
0 " than Rs. 5,500?7 * ) $
SSC CPO 13/12/2019 (Shift - 02) D TS [M W
a th

(a) 364 (b) 312 M D TS ?M ’ * TS


(c) 344 (d) 332 W . * . ) 7 #
200. The total number of employees of the company SSC CPO 13/12/2019 (Shift - 02)
working in production and IT department exceeds the (a) 23 : 35 (b) 21 : 31
ty a

total number of employees working in the Marketing (c) 16 : 23 (d) 21 : 32


and Accounts departments by. 204. The number of families whose monthly expenditures
(Production) (IT)
= 0 S .: , S
di M

on education are Rs. 2,000 or more but less than Rs.


. * 0" (Marketing)
7* ) M . ; 4,000 is __________ .
* ) (Accounts) ., . * 0" . 7* ) # $ D
) ’ $ ) "0 TS M ’ $ M * TS
SSC CPO 13/12/2019 (Shift - 02) SSC CPO 13/12/2019 (Shift - 02)
(a) 164 (b) 143 (a) 207 (b) 197
(c) 158 (d) 154 (c) 230 (d) 237
201. The number of employees of the company working in Direction (205-207) : Study the graph and answer
the Marketing department is what percentage more than question.
the number of employees working in the IT department?
(Marketing)
. ; ., . * Import
0 " and Export of a country over the period 2014
to 2018 (in Rs. crores)
) M (IT) = 0. ,S : S . * 0" )
? ! #
A

# # # ’ $ %
Import Export
SSC CPO 13/12/2019 (Shift - 02) 600
560 550
(a) 16.67 (b) 22 500
540
500
(c) 20 (d) 18.33 410
450 425 440
400
400
Direction (202-204):Study the graph and answer the 375

question. 300

200

100
Distribution of monthly expenditure (in Rs.) of
different families on education. 0
2014 2015 2016 2017 2018
. D W : AT B Years

Aditya Ranjan (Excise Inspector) Selected Selection 210

Downloaded by Kunal Chauhan (kc6970929@gmail.com)


lOMoARcPSD|39315497

Data Interpretation

205. The ratio of total imports in 2014, 2016 and 2017 of @ V 7* #M


the country to the total exports in 2015 and 2016 is: 7* 0# # # # $ % A
?M @ #4 V 7* #
R4 # 4 # $ $ . @
" $ B
7* 0# 7 # L $ % SSC CPO 13/12/2019 (Shift - 02)
SSC CPO 13/12/2019 (Shift - 02) (a) 84.8 (b) 46.7
(a) 11 : 10 (c) 87.5 (d) 48.4
207. In how many years were the exports of the country
(b) 8 : 11
more than the average imports during the given
(c) 10 : 11 years?
(d) 12 : 11 ./ 2 # #
206. By what percentage are the total imports of the 4 %
country in 2016 and 2017 less than the total exports SSC CPO 13/12/2019 (Shift - 02)
in 2014, 2015 and 2018? (Your answer should be
(a) 2 (b) 3
correct to one decimal place)
(c) 4 (d) 1

r
ANSWER KEY

si
1.(d) 2.(d) 3.(d) 4.(b) 5.(c) 6.(b) 7.(c) 8.(a) 9.(b) 10.(c)

11.(b)
an by
12.(b) 13.(a) 14.(d) 15.(c) 16.(d) 17.(c) 18.(c) 19.(b) 20.(b)

n
21.(a) 22.(b) 23.(c) 24.(b) 25.(c) 26.(d) 27.(a) 28.(c) 29.(b) 30.(c)
ja
R s
31.(d) 32.(c) 33.(b) 34.(c) 35.(b) 36.(a) 37.(b) 38.(c) 39.(a) 40.(c)

41.(b) 42.(c) 43.(b) 44.(b) 45.(c) 46.(d) 47.(c) 48.(c) 49.(d) 50.(b)
a th

51.(c) 52.(c) 53.(a) 54.(a) 55.(a) 56.(b) 57.(c) 58.(a) 59.(b) 60.(a)

61.(d) 62.(d) 63.(a) 64.(b) 65.(b) 66.(b) 67.(a) 68.(a) 69.(d) 70.(c)
ty a

71.(d) 72.(b) 73.(c) 74.(b) 75.(d) 76.(c) 77.(c) 78.(a) 79.(b) 80.(b)
di M

81.(a) 82.(b) 83.(b) 84.(d) 85.(c) 86.(c) 87.(c) 88.(c) 89.(c) 90.(c)

91.(d) 92.(a) 93.(b) 94.(b) 95.(b) 96.(d) 97.(c) 98.(c) 99.(c) 100.(a)

101.(b) 102.(d) 103.(d) 104.(b) 105.(c) 106.(a) 107.(d) 108.(c) 109.(b) 110.(c)

111.(a) 112.(c) 113.(a) 114.(a) 115.(c) 116.(c) 117.(a) 118.(a) 119.(b) 120.(b)

121.(b) 122.(c) 123.(b) 124.(b) 125.(a) 126.(d) 127.(d) 128.(a) 129.(a) 130.(d)

131.(a) 132.(c) 133.(a) 134.(d) 135.(a) 136.(b) 137.(d) 138.(a) 139.(a) 140.(c)

141.(b) 142.(a) 143.(d) 144.(a) 145.(d) 146.(a) 147.(b) 148.(b) 149.(a) 150.(d)
A

151.(a) 152.(c) 153.(b) 154.(b) 155.(a) 156.(d) 157.(a) 158.(a) 159.(b) 160.(a)

161.(d) 162.(a) 163.(a) 164.(a) 165.(b) 166.(b) 167.(c) 168.(c) 169.(d) 170.(b)

171.(d) 172.(b) 173.(c) 174.(b) 175.(c) 176.(a) 177.(c) 178.(b) 179.(b) 180.(b)

181.(d) 182.(c) 183.(b) 184.(b) 185.(a) 186.(a) 187.(d) 188.(c) 189.(b) 190.(b)

191.(a) 192.(c) 193.(a) 194.(d) 195.(a) 196.(b) 197.(d) 198.(d) 199.(a) 200.(d)

201.(c) 202.(a) 203.(d) 204.(a) 205.(a) 206.(b) 207.(b)

Aditya Ranjan (Excise Inspector) Selected Selection 211

Downloaded by Kunal Chauhan (kc6970929@gmail.com)


lOMoARcPSD|39315497

Data Interpretation

SOLUTIONS
1. (d) The student who got the highest marks in subject
Required difference = Production of company & in 2018 P2 = Viraj(89)
– Production of company C in 2015 The student who got the highest marks in subject
= 55,00,000 – 45,00,000 = 10,00,000 tonnes P3 = Viraj(86)
2. (d) Hence, Required Answer = (b) Vishal, Viraj, Viraj
Average production of company X 7. (c)
Average Percentage marks obtained by all the students
30  45  25  50  40 in Geography
=  38
5
88  92  64  80  88  72
Average production of company Y =
6

r
25  35  35  40  50
=  37 484
5 = = 80.66%

si
6
Average production of company Z
Average Marks = 80.66% of 75
=
an by
35  40  45  35  35
5
 38 = 60.495
= 60.5

n
Hence, Required Answer = X and Z 8. (a)
3. (d) Total Marks of student Q in Physics and Hindi
ja
From 2016 to 2017, = 90% of 80 + 54% of 100
R s
Required percentage increment = 72 + 54
= 126
a th

10
=  100% = 40% Total Marks of student T in Geography and History
25
= 88% of 75 + 42% of 50
Rise in the productin of cars for
= 66 + 21 = 87
company Y from 2016 to 2017 is 40 percent which is
ty a

highest. 39
Required percentage =  100%
4. (b) 87
di M

Total production of model A mobiles in 2020 and model = 44.82%


E mobiles in 2019 = 45% (Approx)
= 40% of 6500000 + 15% of 4000000 9. (b)
= 2600000 + 600000 Total Maximum Marks
= 3200000 = 80 + 150 + 100 + 75 + 120 + 50 = 575
Marks obtained by student T
5. (c)
= 65% of 80 + 60% of 150 + 45% of 100 + 88% of 75
No. of trees are axed in 2016 + 50% of 120 + 42% of 50
= 18 + 25 + 18 + 20 = 81 = 52 + 90 + 45 + 66 + 60 + 21 = 334
No. of trees are axed in 2017 Required percentage
= 25 + 23 + 18.5 + 18 = 84.5 334
No. of trees are axed in 2018 =  100% = 58%
575
A

= 23 + 24 + 18.4 + 17.6 = 83
10. (c)
No. of trees are axed in 2019
Maximum marks
= 24.4 + 19.5 + 19 + 16 = 78.9
= 300 + 300 + 200 + 100 + 100 = 1000
No. of trees are axed in 2020
Marks scored by X = (70% of 300) + (90% of 300) +
= 22.5 + 21 + 20 + 17.5 = 81 (95% of 200) + (80% of 100) + (75% of 100)
Hence, The least number of trees are axed in 2019 = 210 + 270 + 190 + 80 + 75 = 825
6. (b) Percentage marks obtained by X
The student who got the highest marks in subject
825
P1 = Vishal(82) =  100% = 82.5%
1000

Aditya Ranjan (Excise Inspector) Selected Selection 212

Downloaded by Kunal Chauhan (kc6970929@gmail.com)


lOMoARcPSD|39315497

Data Interpretation

11. (b) 17. (c)


Total number of students in Activity II Average Export of rice
= 50 + 150 + 150 + 50 + 50 + 50 + 50 = 550 10.4  13  15.6  21.6  18.8 79.4
= = = 15.88
Total number of students in Activity IV 5 5
= 250 + 125 + 350 + 275 + 250 + 150 + 150 18. (c)
= 1550 Average Export of Rice

550 10.4  13  15.6  21.6  18.8


Required percent =  100% =
1550 5

= 35.48% 79.4
=
12. (b) 5
Marks of students after deduction of 5 marks, = 15.88
50, 85, 70, 75, 85 In 2010, 2011 and 2012 there was less export than
the average export.
Mean
19. (b)
50  85  70  75  85
=  73 Total Production of wheat
5
= 113 + 96 + 42 + 47 + 32 + 25 + 18 + 38

r
Mode = 85
= 411
Median = 75

si
% Production of P's Alone
Net Average
113
=
an by
Mean  Mode  Median
3
=
411
= 27.49%
 100

n
73  85  75 233 = 27.50% (Approx)
= = =77.67
3 3 20. (b)
13. (a) ja Total production of rice in the country–
R s
Total contribution in corpus fund, = 55 + 58 + 69 + 51 + 47 + 78 + 67 + 48 = 473
Total production of wheat in the country–
a th

= 74.30 + 75.00 + 77.50 + 69.05


+ 70.00 + 67.52 = 113 + 96 + 42 + 47 + 32 + 25 + 18 + 38 = 411
= 433.37 Hence Required ratio = 473 : 411
Contribution of Rakesh in Corpus fund = 75.00 21. (a)
ty a

Percentage contribution of Rakesh Given,


Total population of State B = 3000
75
=  100 = 17.30% Population below poverty line
di M

433.37
= 60% of 3000
14. (d) = 1800
Total population of village = 60000 Number of girls below poverty line
In percentage, population of village D = 17%
In percentage, population of village E = 18% 4
 1800   1200
Difference between population of village D and E 6
= 1% of Total population 22. (b)
Population of village A in 2018 = 12381
1
= 60000  = 600 Population of village A in 2021 = 13256
100
Increament = 13256 – 12381= 875
15. (c) Percentage Increment
Total population = 50000
Increment
A

Population of Village D
= Population in 2018  100%
17
= 50000  = 8500
100 875
=  100%
16. (d) 12381
Combined highest score in Accounts and Business = 7.07%
studies 23. (c)
= 96 + 96 = 192 Average Population of village D
Combined highest score in Economics and English
5152  5230  5346  5500 21228
= 100 + 100 = 200 = = = 5307
4 4
Hence, Answer (d) Economics and English by 8

Aditya Ranjan (Excise Inspector) Selected Selection 213

Downloaded by Kunal Chauhan (kc6970929@gmail.com)


lOMoARcPSD|39315497

Data Interpretation
24. (b) 30. (c)
Combined production of A and D for the year 2014 The average of appeared students
to 2016. 2500  3000  4500  1200
= 1350 + 570 + 420 + 810 + 990 + 580 =
4
= 4720
Combined production of B and C for the year 2014 11200
= = 2800
to 2016, 4
= 750 + 520 + 1170 + 630 + 1280 + 650 The average of passed students
= 5000 1350  2500  3900  850
Required Difference =
4
= 5000 – 4720 = 280
8600
25. (c) = = 2150
4
Appeared in 1991 = 8562
Hence, difference between average
Appeared in 1992 = 8139
= 2800 – 2150 = 650
Reduction = 8562 – 8139 = 423
31. (d)
423 Total marks = 300 + 300 + 150 + 300 + 200 = 1250
Required Drop % =  100%
8562

r
Marks obtained by Mohan:
= 4.940%
In Chemistry – 70% of 300 = 210

si
= 5% (Approx)
26. (d) In Mathematics – 75% of 300 = 225
an by
The number of person who speaks English and Bhojpuri In Physics – 80% of 150 = 120
= (15 + 10)% of 120 In Hindi – 65% of 300 = 195
= 25% of 120 = 30

n
In English – 85% of 200 = 170
and, The number of person wh speaks Tamil = 5%
Total obtained marks = 210 + 225 + 120 + 195 + 170
of 120 = 6
ja = 920
Required Ratio = 30 : 6
R s
Average marks percentage
=5:1
a th

27. (a) Obtained marks


In 1995   100%
Total marks
No. of worker in category B is 15% = 1500
Therefore, No. of worker in Category D is 25% = 2500 920
  100%  73.6%
ty a

In 1996 1250
No. of worker is increased by 10% in category B
32. (c)
di M

= 110% of 1500 = 1650


No. of worker is decreased by 5% in category D Average consumption of wheat
= 95% of 2500 = 2375 162  196  187  189
Hence, Required Sum =  183.5
4
= 1650 + 2375
Average consumption of oats
= 4025
28. (c) 131  116  103  101
=  112.75
The number of empolyee of company B in 2019 = 18 4
lakh
Difference = 183.5 – 112.75 = 70.75
The number of empolyee of company C in 2021 = 17
lakh 33. (b)
Required percentage No. of students who opted Biology in School D = 20%
of 800 =160
A

18 34. (c)
=  100% = 105.88%
17 Number of students in school B who opted biology
29. (b)
36
The length of mountains in Punjab = 40 400   144
100
The length of rivers in odisha = 10
Number of students in school D who opted biology
Difference between the mountains and rivers lengths
= 40 – 10 = 30 18
800   144
100
30
Required percentage =  100% = 300%
10 Required Ratio = 1 : 1

Aditya Ranjan (Excise Inspector) Selected Selection 214

Downloaded by Kunal Chauhan (kc6970929@gmail.com)


lOMoARcPSD|39315497

Data Interpretation
35. (b) 42. (c)
By option– Average % of History
No. of vehicles sold in the following combinations:
in A, 2011 = 700000 0
=  100  45%
in B, 2014 = 710000 200
in C, 2014 = 690000 Average % of Geography
in D, 2012 = 700000
Maximum in combination –B, 2014 35
=  100  35%
Hence, Required answer is (b) 100
36. (a) Average % of Mathematics
Money spent on Food and Rent
88
= (25 + 12)% of Total Expenditure =  100  44%
200
= 37% of 65000
= 24050 Average % of Science
37. (b)
120
Central angle for Anil's expenses on Power and fuel– =  100  60%
200
15

r
  360 Average % of English
100

si
= 54° 90
=  100  60%
38. (c) 150
an by
Check By option:
Number of Books issued in January = 4500
Hence, Required answer is (c) Science and English.
43. (b)

n
Number of Books issued in April = 4800
Given,
Number of Books issued in May = 6800
Number of Books issued in February = 6000 People below the age of 36 years
ja
R s
Maximum number of books were issued in May = 30 million
 (15.00 + 20.25 + 16.75)%
a th

39. (a) = 30 million


1500  1460  1105 4065  52% = 30 million
J  = 1355
3 3
30
ty a

132011801170 3670  1% = million


K   52
3 3
The number of people to the age group 56 – 65 is:
di M

Average of J and K
1355  1223.33 2578.33 30
  5.50% =  5.5
= = 1289.165 = 1289.17 52
2 2
40. (c) = 3.17 million
Total number of male in all village 44. (b)
= 1500 +1460 + 1105 + 1305 Number of students scoring less than 50% marks
= 5370
= 270 + 120 + 300 + 220 + 200
Total number of female in all village
= 1110
= 1220 + 1320 + 1180 + 1170
= 4890 Number of students scoring more than 50% marks
Required ratio = 5370 : 4890 = 55% of 600 + 40% of 400 + 20% of 375 + 10% of
= 179 : 163 350 + 25% of 300
A

41. (b) = 330 + 160 + 75 + 35 + 75 = 675


Total sale of Type D geyser Total number of students
= 98 + 112 + 109 + 102 + 124 + 134 = 679
= 600 + 400 + 375 + 350 + 300
Total sale of Type A geyser
= 75 + 100 + 105 + 100 + 95 + 85 = 560 = 2025
Total sale of Type C geyser Exactly 50% scoring students
= 103 + 103 + 112 + 123 + 102 + 134 = 677 = 2025 – (1110 + 675) = 240
Total sale of Type B geyser
Required Ratio = 1110 : 240
= 122 + 102 + 108 + 189 + 123 + 145 = 789
Hence Required Answer is (b). = 111 : 24

Aditya Ranjan (Excise Inspector) Selected Selection 215

Downloaded by Kunal Chauhan (kc6970929@gmail.com)


lOMoARcPSD|39315497

Data Interpretation
45. (c)
515
Increment% production in sprite =  85.83%
6
5800 – 5400 Overall Average % of CS
  100% = 7.40%
5400
95  90  80  85  80  90
Increment% production in Cocacloa =
6
5600 – 5300
  100% 520
5300 =  86.67%
6
= 5.66%
Hence, Answer (b) Geography.
Increment% production in Pepsi
51. (c)
6800 – 6000 Revenue in year 2016
  100%
6000 = 50 lakhs + 30% of 50 lakhs
=13.33% = 50 lakhs + 15 lakhs
Increment% production in Fanta = 65 lakhs
2700 – 1800 52. (c)
  100% Average Export from the year 2000 to 2004–
1800

r
= 50% 60  110  120  120  150
=
Thus maximum % increment in 'Fanta'. 5

si
560
46. (d) an by = = 112
5
Average sales of companies B and D in 2012
53. (a)

n
18  21 39 Average wages for task T1
   19.5
2 2 4  2  3  4  5  3 21
= 
ja
Average sales of companies A and E in 2011 6 6
R s
11  18 29 Average wages for task T3
   14.5
a th

2 2 3  4  5  4  3  4 23
Required percentage = 
6 6
14.5 23 21 2 1
  100% –
19.5 Difference = = =
ty a

6 6 6 3
= 74.36% 54. (a)
47. (c)
di M

In the given table, you can see that the production


Maximum number of centruies scored in 2019 = 5 of barley is continuously increasing.
48. (c) 55. (a)
We can see in the figure Production of Activated Carbon Masks in month of
Cricketer B has scored maximum number of centuries = 5 April and June = 250
49. (d) Hence Answer (a) April and June
Number of centuries scored by A = 3 56. (b)
Number of centruies scored by E = 3 The average number of patients visiting the dental
clinic for Cleaning and polishing of teeth, Fixing
Hence, Answer (d). crowns and bridging and Teeth whitening.
50. (b)
 410  880  360   160  970  270 
Overall Average % of Chemistry
(520  680  530)   440  590  280 

90  75  65  85  95  90 500 12
= =  83.33%
A

6 6 1650  1400  1730  1310 6090


Overall Average % of Geography = = = 507.5
12 12
80  85  95  95  95  90 The average number of patients visiting the dental
= clinic or Orthodontic treatment
6
680  790  460  1020 2950
540 = = = 737.5
=  90% = 90% 4 4
6 Now, Difference = 737.5 – 507.5 = 230
Overall Average % of History 230
The percentage of patients =  100% = 31.2%
70  85  90  90  90  90 737.5
=
6 Answer (b) Less by 31.2%

Aditya Ranjan (Excise Inspector) Selected Selection 216

Downloaded by Kunal Chauhan (kc6970929@gmail.com)


lOMoARcPSD|39315497

Data Interpretation
57. (c) 63. (a)
3 Female Average of students playing rugby, hockey, badminton
75% = and squash –
4 Total
Male : Female = 1 : 3 30  60  20  10

58. (a) 4
Number of Male Teachers in Hindi
120
= 20% of 50 = 10 = = 30
4
Number of Male Teachers in English
Average of students playing tennis and football–
= 10% of 40 = 4
Number of Male Teachers in Mathematics 20  80 100
= = = 50
= 65% of 60 = 39 2 2
Number of Male Teachers in Science
30
= 50% of 30 = 15 Required percentage =  100%
50
Number of Male Teachers in Social Science
= 25% of 40 = 10 = 60%
Hence, the maximum number of male teachers are 64. (b)
in Mathematics. Number of students opting Economics = 75

r
59. (b) Number of students opting Physical Education = 125
Hence, Required Ratio = 75 : 125

si
Total Marks of student A
= 60 + 70 + 80 = 210 =3:5
an by
Total Marks of student B 65. (b)
= 50 + 45 + 55 = 150 Total number of students of all subjects in 2020 and
Total Marks of student C only Geography in 2021

n
= 70 + 60 + 55 = 185 = 75 + 100 + 60 + 125 + 75 +75
Total Marks of student D = 510
ja
= 75 + 80 + 75 = 230 66. (b)
R s
Hence, D has secured first rank. Total distance covered by the first employee during
the first three days
a th

60. (a)
= 200 + 300 + 200 = 700
Frequency Total distance covered by the second employee during
Scores x fx
(f ) the last three days–
0  10 2 5 10 = 250 + 350 + 100
ty a

10  20 4 15 60 = 700
20  30 12 25 300 Hence, Required Ratio = 1:1
di M

30  40 21 35 735 67. (a)


The number of students participated in science
40  50 6 45 270
olympiad–
50  60 3 55 165 = 100 + 100 + 150 + 200 + 80
60  70 2 65 130 = 630
Total 50 1670 The number of students participated in GK olympiad–
= 182 + 200 + 120 + 130 + 183
fx
Mean  = 815
f
Increment = 815 – 630 = 185
1670 Required Increment Percentage
=
50 185
=  100% = 29.365% i.e. 29.37%
= 33.4 630
A

68. (a)
61. (d) In Entertainment Sector–
Required percentage Successful Startup : Total Startup
31 = 175 : 560
=  100% = 155%
20 = 5 : 16
62. (d) 69. (d)
The ratio of the number of successful start-ups to
22 that of unsuccessful start-ups in Fintech sector
Required percentage =  100%
21 = 104 : (256 – 104)
= 104.76% = 104 : 152 = 13 : 19

Aditya Ranjan (Excise Inspector) Selected Selection 217

Downloaded by Kunal Chauhan (kc6970929@gmail.com)


lOMoARcPSD|39315497

Data Interpretation
70. (c) 78. (a)
Income from Market Tax, 26% The quantity of E used in month of March = 308
= 260 Crore Total quantity used in month of May
Income from other Taxes = 270 + 390 + 280 + 250 + 350
= (40 + 12 + 16 + 6)% = 1540
= 74% Required percentage
Hence, Required Income – 74%
308
= 740 Crore =  100%
1540
71. (d)
= 20%
Money spent on Basketball
79. (b)
25 In 2007, the percentage of type E cars sold out of
= 12000000 × = 3000000
100 the total numbers of cars sold was minimum.
72. (b) 80. (b)
Given, The average number of delivery partners who joined
Total Marks = 1800 Twiggy in the last 6 years
Difference b/w the marks of Maths and Science 1.2  1.5  2.4  2.8  3.2  3.6
=

r
84 – 82 6
= 1800  = 2.45
360

si
Since, the given data is in Thousands.
2
= 1800  = 10 So,

73. (c)
an by
360 Required Average = 2.45 × 1000
= 2450

n
In 1996, the percentage increase in FDI over the 81. (a)
previous year was the highest.
Total number of passed students = 112
74. (b) ja 82. (b) In 2010
R s
Population in 2006 = 9.5
Population in 2007 = 11.4 Sum of A and B = 500 + 600 = 1100
a th

Percentage increase 83. (b) A = 800 + 700 + 750


1.9 = 2250
=  100%
9.5 B = 550 + 500 + 480
ty a

= 20% = 1530
75. (d) A : B = 2250 : 1530
di M

Total Production in 1994 = 237 = 25 : 17


Total Production in 1998 = 334 84. (d) A = 640 + 800 + 500 + 700 + 900 + 750
Increase Percentage = 4290
97 B = 550 + 820 + 600 + 750 + 500 + 480
=  100%
237 = 3700
= 40.92% 4290 3700
A:B= :
= 41% 6 6
76. (c)
= 429 : 370
69 85. (c)
Required Ratio = 31% : %
4
(a) 2010 – Difference = 600 – 500 = 100
= 124 : 69
A

(b) 2013 – Difference = 750 – 480 = 270


77. (c)
Average number of units sold by the company per year (c) 2012 – Difference = 900 – 500 = 400
(d) 2011 – Difference = 750 – 700 = 50
1.7  1.5  1.0  1.4  2.1
= 86. (c) A = 640 + 900 + 750
5
= 2290
7.7
= = 1.54 B = 820 + 600 + 750
5
= 2170
Since, The given data is in hundreds.
Required Answer = 1.54 × 100 A : B = 2290 : 2170
= 154 = 229 : 217

Aditya Ranjan (Excise Inspector) Selected Selection 218

Downloaded by Kunal Chauhan (kc6970929@gmail.com)


lOMoARcPSD|39315497

Data Interpretation

87. (c) 95. (b)


Average of A = Given that
640  800  500  700  900  750 4290 1
= 15.7% × = 1570
6 6 10
Average of B = 15.7% = 15700
550  820  600  750  500  480 3700 1% = 1000
=
6 6 Then
3700 4290 100% = 100000
Ratio (B : A) = :
6 6 96. (d)
= 370 : 429  25.8% = 5160
88. (c) 5160
150 – 155  15 Vamilla 16.5% =  16.5
25.8
175 – 180  6
= 3300
Difference = 15 – 6
=9 97. (c)
89. (c)  15.7% × 40% = 1570

r
Number of student in 160 – 170 = 10 + 14 = 24 1570 100  100
Total numbers of students = 15 + 13 + 10 + 14 + 12 100% =  = 25000

si
15.7 40
+ 6 = 70
98. (c)
24
an by
Required % =

= 34%
70
 100% Strawberry and butter seotch in 2005 –
= 18.9% + 10.8%

n
= 29.7%
90. (c)
Strawberry and butter scotch in 2015 –
170 – 175 = 12
ja
165 – 170 = 14 = 20.5% + 16.6%
R s
= 37.1%
14 – 12
Less% =  100% % Increment = 37.1 – 29.7
a th

14
= 7.4%
= 14.3%
Total sales = 7.4 × 5000
91. (d)
Total = 15 + 13 + 10 + 14 + 12 + 6 = 70 = 37000
99. (c) Average
ty a

(165 – 175) class interval = 14 + 12 = 26


26 600  450  750  700  800 3300
Required %  × 100% =   660
di M

70 5 5
= 37.1%  37% 100. (a)
92. (a) Boys = 600 + 450 + 750 + 700 + 800
Students in (165 – 170) = 14 = 3300
Students in (150 – 155) = 15 Girls = 500 + 550 + 650 + 600 + 650
15 – 14 = 2950
Less% =  100%
15 Girls : Boys
1 Ratio 2950 : 3300
= × 100% = 6.66% = 6.7% 59 : 66
15
93. (b) 101. (b)
 16.5% = 3300 500
(A) ___ % =  100%  45.5%
A

3300 1100
25.8% =  25.8 = 5160
16.5 550
94. (b) (B) ___% =  100%  55%
1000
Total % of strawberry and butter scotch in 2005
= 18.9% + 10.7 = 29.7% 650
(C) ____% =  100%  46.42%
Total % of strawberry and butter scotch in 2015 1400
= 20.5% + 16.6%
= 37.1% 650
(E) ____% =  100%  44.82%
% Increment = 37.1% – 29.7% 1450
= 7.4% Clearly
Total sales = 7.4 × 10000 = 74000 Minimum percentage of girls in school E

Aditya Ranjan (Excise Inspector) Selected Selection 219

Downloaded by Kunal Chauhan (kc6970929@gmail.com)


lOMoARcPSD|39315497

Data Interpretation

102. (d) 109. (b)


22
 600  500    450  550  F = 450 × = 99
100
  750  650    700  600 
110. (c)
  800  650 
Average = 8
5 Employee in D = 450 × = 36
100
1100  1000  1400  1300  1450

5 25
Women in D = 36 × =9
100
6250
 = 1250 111. (a)
5
Ratio = (50 + 70) : (60 + 45)
103. (d)
= 120 : 105
600 =8:7
(A) ___ % =  100%  54.5%
1100 112. (c)
450 70 – 80 km/h  90
(B) ___% =  100%  45%

r
1000 50 – 60 km/h  70

si
750 90 – 70
(C) ____% =  100%  53.58% Required % =  100%
1400 70

(E) ____% =
an by
800
1450
 100%  55.18%
= 28.6%
113. (a)

n
Total Number of Cars = 50 + 70 + 85 + 90 + 60 + 45
Clearly
= 400
Maximum percentage of Boys in school E
ja Total cars which speed ago than 70 km/h = 50 + 70
R s
104. (b)
+ 85 = 205
Girls in A, B and C = 500 + 550 + 650
a th

= 1700 205
%= × 100% = 51.25%
Total Boys = 600 + 450 + 750 + 700 + 800 400
= 3300 114. (a)
Ratio = 1700 : 3300
ty a

Ratio = (50 + 70) : (90 + 60 + 45)


= 17 : 33 = 120 : 195
105. (c) = 8 : 13
di M

Total = 450 115. (c)


B = 14%
60 – 70 km/h = 85
14 70 – 80 km/h = 90
B = 450 × = 63
100
90 – 85
106. (a) Less% =  100%
90
 100% = 360°
Department A = 20% 100
= %
18
360
20% =  20 = 72° = 5.6%
100
116. (c)
107. (d) Total Cars = 50 + 70 + 85 + 90 + 60 + 45 = 400
A

Total employee = 450 Number of cars which are running 90 km/h and above
20 = 45
Total employee in department E = 450 × = 90
100 45
Required % =  100%
400
40
Total male in department E = 90 × = 36 = 11.25%
100
117. (a)
108. (c)
Average sales in 2018
E = 20%
 100% = 360° 80  75  95  85  75  70 480
= =  80
20% = 72° 6 6

Aditya Ranjan (Excise Inspector) Selected Selection 220

Downloaded by Kunal Chauhan (kc6970929@gmail.com)


lOMoARcPSD|39315497

Data Interpretation

118. (a) 126. (d)


2018 = 80 + 75 + 95 + 85 + 75 + 70 = 480 Defence = 12
2019 = 105 + 65 + 110 + 95 + 95 + 80 = 550 Education = 6
550 – 480 12 – 6
Less% =  100% More% = × 100% = 100%
550 6
127. (d)
70
= × 100% Total = 1 + 21 + 15 + 4 + 6 + 34 + 12 + 7 = 100
550
Subsidy = 21
= 12.7%
 100 = 360°
119. (b)
B1, B2 and B3 in 2018 = 80 + 75 + 95 = 250 360  21 756
21 = =
B4, B5 and B6 in 2019 = 95 + 95 + 80 = 270 100 10
Ratio = 250 : 270 = 25 : 27 = 75.6°
120. (b) 128. (a)  100%  1680
Total No. of students who obtained less than 250 (Education + Health) = 6 + 4  10%
marks = 30 + 45 = 75
10%  168

r
121. (b)
129. (a)
Total students obtained 250 or more but less 300

si
Expenditure on education = 6
= 60
Expenditure on defence = 12
Total marks = 30 + 45 + 60 + 35 + 40 + 35 = 245
 245 = 360°
360
an by Loss% =
12 – 6
12
 100%

n
60 = × 60 = 88.16° = 88° = 50%
245
130. (d)
122. (c)
ja Expenditure for proceds to state (in billion) = 34
R s
Students who obtained loss than 350 = 60 + 45 +
30 + 35 = 170  Total = 100%
a th

Students who obtained 400 or more than 400 = 35  100% = 1680


then
170 – 35
Required % =  100% 1680
35 34%   34
100
ty a

135
=  100%  385.7% 57120
35 =
di M

100
123. (b)
= 571.20
Number of students who's marks less than 250 =
30 + 45 = 75 131. (a)
Number of students who's marks 400 or more than = 35 Total expenditure = 100%
Expenditure on interest payment = 15%
75 – 35
Required % =  100%  100% = 360°
35
then
40  100 360
=  100% 15% = × 15 = 54°
35 100
= 114.3% 132. (c)
124. (b)
20  30  40  10
A

Number of students who obtained less Average of science students =


4
Than 200 marks = 30
125. (a) 100
= = 25
Number of boys who obtained 200 or more than 200 4
and less than 300 = 45 + 60 = 105 40  30  20  30
Total = 30 + 45 + 60 + 35 + 40 + 35 = 245 Average of commerce students =
4
 245 = 360°
120
360 = = 30
105 =  105 = 154.2  154 4
245 Difference = 30 – 25 = 5

Aditya Ranjan (Excise Inspector) Selected Selection 221

Downloaded by Kunal Chauhan (kc6970929@gmail.com)


lOMoARcPSD|39315497

Data Interpretation

133. (a) 140. (c)


Total work of D1, D2 and D4 = 80 + 70 + 75 = = 225
30  50  20  20
Average of Arts students = 225
4
Average marks of D1, D2 and D4 = = 75
3
120
= = 30 Each subject are 100
4
75 means 75%
134. (d) Total marks of D3 and D5 = 50 + 55 = 105
 Normal range of fasting blood sugar level
F = 70 – 100 105
Average marks of D3 and D5 = = 52.5
Normal range of fast prandial blood sugar level (PP) 2
= 100 – 130 Average percentage of D3 and D5 = 52.5%
Upper limit of Normal PP blood sugar level = 130 Required difference = 75% – 52.5% = 22.5%
 115.4  115% 141. (b)
Players who scored more than 30.5 on average = (12 +
115 16 + 4 + 2) = 34
So,  130   149.5
100 142. (a)
Frequency of '10' correspond to = (20.5 to 30.5)

r
 150
150 which is in week W2 143. (d)

si
135. (a) Sum of run score by all 50 players
Number of students who's weight less than 55kg = 31 51 71 91 111 131
an by
20 + 45 + 30 = 95
Number of students who's weight between 55 kg to
=
2
6
2
 10 
2
 12 
2
× 16 

= 93 + 255 + 426 + 728 + 222 + 131 = 1855


2
4
2
2

n
65 kg
= 60kg + 40kg = 100kg 1855
Avg score of the 50 players = = 37.1
ja 50
R s
 100 – 95 
Less% =    100% = 5% 144. (a)
 100 
Avg amount invested in Raw material during 1997 to
a th

136. (b)
Total number of cars = 5000 (375  330  525  420 1650
2000 = 
No of cars sold from the showroom B and D 4 4
5000 Avg value of sales of finished good during 1997 to 2000
ty a

=  (22  9)%
100% (500  400  600  460) 1960
= =
4 4
di M

5000
=  31%  1550
100%
 1960 1650  310
137. (d) Difference =  –  = 77.5 Lakh
 4 4  4
360
 14%  50.4 145. (d)
100%
C = 122.4°
138. (a)
A.T.Q, D + E = (64.8° + 36°) = 100.8°

5280 (122.4 – 100.8)


Difference = 100 = 2% % change = 100%
264000 100.8
Let,
21.6
E = x% then = 100% = 21.4%
Given that, 100.8
A

x% + 24% + 2% = 36% 146. (a)


x% + 26% = 36% (A + B) = (79.2° + 57.6°) = 1368°
x = 10% C = 122.4°
139. (a) No. of employee in A and B is more than that of C
Total marks obtained in english in all five districts
= (136.8° – 122.4°) = 14.4°
= (70 + 60 + 60 + 85 + 90) = 365
3200
365 = 14.4  128
Avg. = = 73 360
5
Each subject is 100 marks As we know,
 Avg.% = 73% 128 lies between (120 and 140)

Aditya Ranjan (Excise Inspector) Selected Selection 222

Downloaded by Kunal Chauhan (kc6970929@gmail.com)


lOMoARcPSD|39315497

Data Interpretation

147. (b) 153. (b)


Total production of rice in all states
3200
Employees in D = × 64.8° = 576 = (350 + 450 + 575 + 400 + 525) = 2300
360 20% of the total of rice = 2300 × 20% = 460
D States in which the production of wheat is 20% more
than that of rice is = 3
M F 154. (b)
4 : 5 Student in school 'C' = 79.2°
Student in school 'B' = 72°
576
× 4 = 256 (Male) (79.2 – 72)
9
% change = × 100%
3200 72
Employees in E =  36 = 320
360 7.2
E = 100% = 10%
72
155. (a)
M F School (B)
9 : 11 Boys : Girls
320 7 : 6

r
=  9 = 144 (male)
20 5200
=  72 = 1040

si
Total male in (D and E) = (256 + 144) = 400 360
3200
an by
employees in B =  57.6 = 512 1040
360 = × 7 = 560 (Boys)
So, rate of total employees male in (D and E) to the
13

n
number of employees in B School (D)
= 400 : 512 Boys : Girls
= 25 : 32 4 : 5
148. (b)
ja
R s
5200
Daily wages Rs. 450 or more but less than Rs. 600 × 64.8° = 936
(50 + 58 + 72) = 180 360
a th

149. (a)
936
The number of workers whose daily wages are Rs. 500 =  5 = 104 × 5 = 520 (girls)
or more but less than Rs. 650 9
= 58 + 72 + 60 = 190 Boys in (B) : Girls in (D)
ty a

Number of workers whose daily wages are Rs. 650 or 560 : 520
more but less than Rs. 750 14 : 13
di M

= 55 + 48 = 103 156. (d)


Total (D and E) = (64.8° + 57.6°) = 122.4
(190 – 103)
% Change = 100% = 84.5% School (A) = 86.4°
103
Exceed (x) = (122.4 – 86.4) = 36
150. (d)
The number of workers in the factory whose daily wages
5200
are below Rs. 600 = (40 + 50 + 58 + 72) = 220 So, × 36 = 520
The number of workers in the factory whose dauily 360
wages are more Rs. 650 and less then Rs. 800 = (55 + x lies between = (500 and 525)
48 + 37) = 140
Ratio = 220 : 140 = 11 : 7 157. (a)
151. (a) Total expenditure (2015 and 2016) = (210 + 350) = 560
Total production of wheat in states A, B and E Total revenue (2016 and 2018) = (320 + 380) = 700
= (500 + 375 + 475) = 1350
Ratio = 560 : 700 = 4 : 5
A

Total production of rice in states C, D and E = (575 +


400 + 525) = 1500 158. (a)
150 Total expenditure from (2014 to 2018)
% change = 100% = 10%
1500 = (150 + 210 + 350 + 275 + 325) = 1310
152. (c)
Total production of rice in state C and E = (575 + 525) 1310
Avg expenditure from (2014 to 2018) = = 262
= 1100 5
Total production of wheat in state B and D
= (375 + 560) 1.2 times of expenditure = 262 × 1.2 = 314.4
= 935 So, in 2016 and 2018 the revenue of the company more
Ratio = 1100 : 935 = 20 : 17 than 1.2 times the average expenditure over the five years

Aditya Ranjan (Excise Inspector) Selected Selection 223

Downloaded by Kunal Chauhan (kc6970929@gmail.com)


lOMoARcPSD|39315497

Data Interpretation
159. (b) 167. (c)
Total expenditure (2016, 2017 and 2018) Trees planted in state A in 2016 = 80
Trees planted in state A in 2015 = 90
= (350 + 275 + 325) = 950
(90 – 80) 10 1
Total revenue in five years % Change = × 100% = × 100% = 11
90 90 9
= (200 + 250 + 320 + 300 + 380) = 1450
168. (c)
(1450 – 950) Trees planted in state B in 2015 and 2017
% Change = × 100% = (120 + 70) = 190
1450 Trees planted in state A in 2014, 2016 and 2017
500 = (70 + 80 + 90) = 240
= 100% = 34.5%
1450 (240 – 190)
% Change = 100%
160. (a) 240

Workers in company whose age is 28 years or more but 50


= × 100% = 20.8%
less than 40 years = (90 + 85 + 70) = 245 240
Workers whose age is 40 years or more but less than 169. (d)
52 years = (75 + 80 + 55) = 210 Lamp which live more then 600 hours but less then
800 hours

r
= Ratio = 245 : 210 = 7 : 6
= 75 + 90 = 165
161. (d) Lamp which live more then 800 hours 80 + 70 + 55 + 35

si
The number of worker whose age is 32 years or more = 240
but less than 44 years is = 85 + 70 + 75 = 230 Required percentage
162. (a)
an by
Workers whose age is 36 years or more but less than

165
240
 100 = 68.75%

n
48 years = (70 + 75 + 80) = 225 170. (b)
Workers whose age is less than 28 years Lamp which live more then 900 hours
ja 70 + 55 + 35 = 160
R s
= (65 + 75) = 140 Lamp which live less then 900 hours
45 + 60 + 75 = 180
(225 – 140)
a th

% Change = × 100% Required ratio = 160 : 180 = 8 : 9


140 171. (d)
According to question
85 required lamp
= × 100% = 60.7%
ty a

140 60 + 75 + 90 = 225

172. (b)
di M

163. (a) Total students in school D and E = 61.2° + 118.8° = 180°


5400 Total students in school B and C = 64.8° + 79.2° = 144°
x= × (99 – 85)
360 (180 – 144)
% change = × 100%
144
5400
= 14 = 210
360 36
= 100 = 25%
x lies between = (200 and 250) 144
164. (a) 173. (c)
20 6
360 × = 72 Total boys of college C = 79.2 × = 43.2
100 11
Months which is greater than 72 is = 3
165. (b) 11
Total girls of college B = 64.8 × = 39.6
18
A

No of computer sold in (Feb and April) = (45° + 54°) =


99° Required ratio = 43.2 : 39.6 = 12 : 11
Computer sold in May = 77° 174. (b)
(99 – 77) Total students in school E exceed the total students
% Change = × 100% = 28.6% in school A and B by
77
166. (b) = 118.8 – (36 + 64.8) = 18
Avg no of trees planted in state So,Required number x
(70  90  80  90  100) 430 18
A=  = 86 = × 9000 = 450
5 5 360
In state B above the 86 trees planted are = 3 450 lies between 440 and 460

Aditya Ranjan (Excise Inspector) Selected Selection 224

Downloaded by Kunal Chauhan (kc6970929@gmail.com)


lOMoARcPSD|39315497

Data Interpretation
175. (c) 183. (b)
Number of students who obtained 20 marks or more Total spent on sports = 2,40,00,000
but less than 60 marks = 78 + 110 + 142 + 150 = 480 Total spent on cricket and football together
176. (a)
(80  52)
Total number of girls who obtained 50 marks or more = 2,40,00,000 ×
but less than 70 360

60 132
= (150 + 120) × = 162 = 2,40,00,000 × = Rs. 88,00,000
100 360
Total number of girls who obtained 70 marks or more 184. (b)
but less than 90 marks Total expenditure = 100%
ATQ,
55 12.5% of x = 100%
= (100 + 80) × = 99
100 x = 8
Total number of girls = 162 + 99 = 261 185. (a)
177. (c) Total expenditure on salary and interest on loans
Total number of students obtaining less than 50 marks = 20% + 17.5% = 37.5%
= 142 + 110 + 78 + 45 = 375 Total expendituere on infrastru-cture and transpotrt
Total number of students obtaining 60 marks or more = 20% + 12.5% = 32.5%
= 120 + 100 + 80 + 55 = 355 Required ratio = 37.5 : 32.5 = 15 : 13

r
186. (a)
375 – 355
100 = 5.6% Total expendiuture = 100%

si
% change =
355 expendiiture on salary = 20%
178. (b) 20% = 2.8 crores
an by
Total sales of newspaper Q
= 450 + 600 +650 + 680 + 620 = 3000
Avg sales of newspaper Q = 600 × 1.25 = 750
Difference between the expenditure on advertisements
and taxes 15% – 10% = 5%

n
Sales of newspaper P in D city is 1.25 times of average 2.8
Required Difference= × 5 = 70 lakhs
sales of the newspaper 20
179. (b) ja 187. (d)
R s
Total sales of newspaper P in cities B, D and E The difference of interest rate in 2000 between company
= 560 + 750 + 550 = 1860 P and Q
a th

Total sales of newspaper Q in cities A, C, D and E  9% – 7.5% = 1.5%


= 450 + 650 + 680 + 620 = 2400  3,50,000 × 1.5% = 5250
188. (c)
2400 – 1860 100%
% change = Ratio of the sum = 9 : 10
2400
ty a

PRT
540 SI =
= × 100% = 22.5% 100
di M

2400 Required ratio of simple interest


180. (b)
10 8
Total sales of newspaper P in cities A and C = 640 + = 9 : 10  =9:8
480 = 1120 100 100
Total sales of newspaper Q in cities B and D = 600 + 189. (b)
680 = 1280 The sum invested in company Q in the year 2000
Required ratio = 1120 : 1280 = 7 : 8 = 4,00,000
The interest rate in the year 2000 for investors in
181. (d) company Q = 9%
Total spent on sports = 1,80,00,000 The interest rate in the year 2001 for investors in
The amount spent on tennis is more than the company P = 6.5%
Amount received by the investors from company P
expenditure on golf by
109 106.5
48 – 35 = Rs. 4,00,000 × × = Rs. 4,64,340
= × 1,80,00,000 100 100
A

360
190. (b)
13 Total employees = 360°
= × 1,80,00,000 = 6,50,000
360 Number of employees in company E = 64.8°
182. (c) Number of employees in company D = 57.6°
Expenditure on Hockey = 65° If 50% employees of company E transferred to company
Expenditure on Bassketball = 50° D, then number of employees in company D = 57.6° +
Expenditure on Hockey is more than that of basketball 64.8
by = 90°
2
65 – 50 90
= × 100% = 30% % Change = 100 = 25%
50 360

Aditya Ranjan (Excise Inspector) Selected Selection 225

Downloaded by Kunal Chauhan (kc6970929@gmail.com)


lOMoARcPSD|39315497

Data Interpretation
191. (a) 199. (a)
Total employees in office D = 57.6° Number of female employees in IT department
Total employees in office C = 54° 72 80
Total employees in office C and D = 57.6° + 54° = 111.6° =  × 1400 = 224
Total employees in office B = 82.8° 360 100
The total number of employees of the copmpany in Number of female employees in HR and Account
offices C and D exceeds the number of employees B by department

(111.6 – 82.8) 28.8 (57.6  32.4) 40


= × 9200 = × 9200 = 736 =  1400 = 140
360 360 360 100
So, 736 lies between the 720 and 740. Total female employees = 224 + 140 = 364
192. (c) 200. (d)
Number of Production and IT department exceed than
Total number of employees of the company in offices D
that of marketing and Accounts department by
and E = 57.6° + 64.8° = 122.4°
Total number of employees of the company in offices A [(111.6  72) – (57.6  86.4)]
= 100.8° = ×1400 = 154
360
(122.4 – 100.8) 201. (c)
Required percentage = × 100 = 21.4% Number of employees in the Marketing department is = 86.4°
100.8
Number of employees in the IT department = 72°
193. (a)

r
The number of workers whose weekly earnings are below 86.4 – 72
Required % = × 100% = 20%
Rs 7000 = 105 + 95 + 85 + 60 = 345 72

si
The number of workers whose weekly earnings are Rs 202. (a)
7000 or above but less than Rs 8500 Number of families whose monthly expenditure on
an by
= 90 + 80 + 70 = 240
Difference = 345 – 240 = 105
education are Rs 2500 or more but below Rs 4000 is
= 47 + 55 + 70 = 172

n
Number of families whose monthly expenditure on
105 education are Rs 4500 or more but below Rs 6000 is =
Required percentage = × 100 = 43.75%
240 52 + 43 + 30 = 125
194. (d) ja 172 – 125
R s
Total number of workers in the factory are = (60 + 85 + Required % = × 100% = 37.6%
95 + 105 + 90 + 80 + 70 + 65 + 50) = 700 125
a th

Number of workers whose weekly earning are Rs. 6500 203. (d)
or above but less than Rs. 6500 The number of families whose monthly expenditure
= (105 + 90 + 80 + 70) = 345 on education is below Rs. 3000 is
= 23 + 35 + 47 = 105
345 The number of families whose monthly expenditure
ty a

Required % = × 100% = 49.3% on education is Rs. 4000 or above but less than Rs.
700
195. (a) 5,500 is
di M

= 65 + 52 + 43 = 160
The number of workers in the factory whose weekly
Required ratio = 105 : 160 = 21 : 32
earning are Rs. 5,500 or more but less than Rs. 7500 204. (a)
is = 85 + 95 + 105 + 90 = 375 The number of families whose monthly expenditure on
196. (b) education are Rs. 2000 or more is
Exports of type A cars in 2014 are = 225 = 35 + 47 + 55 + 70 = 207
205. (a)
120 Total imports in 2014, 2016 and 2017 of the country
120% of 225 = 225 × = 270
100 are = (410 + 375 + 425) = 1210
In 2016 the exports of type B cars exactly 20% more Total exports of the country of the country in 2015 and
than the exports of type A cars in 2014 2016 = 560 + 540 = 1100
197. (d) Required Ratio = 1210 : 1100 = 11 : 10
206. (b)
Total exports of type A cars during 2012 to 2016
Total imports of the country in 2016 and 2017 are =
= (275 + 250 + 225 + 260 + 340) = 1350 375 + 425 = 800
A

1350 Total exports in 2014, 2015 and 2018 are = 500 + 560 +
Avg exports of type A cars = = 270 440 = 1500
5
In 2012, 2013, 2014 and 2015 the exports of type B 1500 – 800
cars more than the average exports of type A cars during Required% = × 100% = 46.7%
1500
2012 to 2016
207. (b)
198. (d)
Avg imports during the years are
Exports of type A cars in 2016 are = 340
Total exports of type B cars in 2014 and 2015 are = 280 410 + 450 + 375 + 425 + 550
= = 442
+ 320 = 600 5
600 – 340 1 In 2014, 2015 and 2016 the exports of the country more
Required % = ×100% = 43 % than the avg imports.
600 3

Aditya Ranjan (Excise Inspector) Selected Selection 226

Downloaded by Kunal Chauhan (kc6970929@gmail.com)


lOMoARcPSD|39315497

Downloaded by Kunal Chauhan (kc6970929@gmail.com)


lOMoARcPSD|39315497

Downloaded by Kunal Chauhan (kc6970929@gmail.com)

You might also like